'타고났다'는 가정에 도전하기 - 모든 것을 확인하라 (Med Educ, 2019)

Challenging assumptions of innateness – leave nothing unturned

Jason J Han1 & Neha Vapiwala2



개인이 어떤 근본적인 능력이나 직관을 가지고 있다고 가정하는 것이 여러 학계에서 한때 보편화되었다(예: 경제학 및 사회과학 분야의 합리성 가정). 그러나 복잡한 기계로서 인간의 뇌는 체계적인 오류에 취약하다.

It was once common in various academic fields to assume that individuals possess certain fundamental abilities or intuitions (e.g. the assumption of rationality in the fields of economics and social sciences).1 However, the human brain as complex machinery is vulnerable to systematic errors.


Daniel Kahneman과 Amos Tversky는 이것을 두 가지 처리 메커니즘의 공동 존재 탓으로 돌렸다.2 그들은 

  • 첫 번째, 적절한 이름을 가진 시스템 1을 빠르고, 자동적이고, 직관적이며, 무의식적인 접근법

  • 번째(시스템 2)를 더 느리고, 더 고의적이고, 분석적이고, 의식적인 접근방식이라고 묘사했다.

Daniel Kahneman and Amos Tversky, attributed this to the co- existence of two processing mechanisms.2 They described the first, aptly named System 1, as the fast, automatic, intuitive, unconscious approach and the second (System 2) as the slower, more deliberate, analytical and conscious mode.


이 분류의 목적은 계층을 할당하는 것이 아니라, 두 시스템 모두 직무에 따라 각각의 장단점이 있음을 인정하는 것이었다. 

  • 시스템 1은 효율적이지만 오류가 발생하기 쉽다. 

  • 시스템 2는 더 철저하지만 더 많은 자원을 필요로 하며 작업 메모리와 주의를 빠르게 고갈시켜 오류를 일으키기 쉽다.

The purpose of this categorisation was not to assign a hierarchy, but rather to acknowledge that both systems have their respective pros and cons depending on the task. 

  • System 1 is efficient but more error-prone. 

  • System 2 is more thorough but requires greater resources and quickly drains our working memory and attention, thereby making it too susceptible to error.


편중성 또는 좌우식별(LRD)하는 우리의 능력. LRD의 오류는 잘못된 진단과 개입, 그리고 궁극적으로 환자의 위해를 초래할 수 있기 때문에 이러한 능력은 의학에서 특히 중요하다.

our ability to discern laterality or left–right discrimination (LRD). This ability is particularly critical in medicine, as errors in LRD can lead to wrong diagnoses and interventions, and ultimately patient harm.


LRD는 흔히 인간발달 초기 단계에서 선천적이거나 획득된 것으로 가정되지만, 실제로는 LRD는 여성의 17%, 남성의 9%가 어려움을 보고한 복잡한 신경심리학적 과정이다.3 의대생들은 이 도전으로부터 면제되지 않는다. 한 연구에서, 대부분의 학생들이 객관적 LRD 시험에서 77% 미만의 점수를 받았다.4

although LRD is often assumed to be innate or acquired during early stages of human development, in reality LRD is a complex neuropsychological process with which 17% of women and 9% of men have reported difficulty.3 Medical students are not exempt from this challenge. In one study, a majority of students scored less than 77% on an objective LRD test.4


이러한 연구결과는 의학 교육계에 중요한 의미를 가지며, 이는 LRD가 선천적인 인간의 기술이라는 가정을 뒤집고 교과과정에 있어서의 편중성 교육의 중요성을 높일 필요성을 시사한다.5

these findings have important implications for the medical education community, suggesting the need to overthrow assumptions that LRD is an innate human skill and to raise the importance of laterality training in the curriculum.5


우리가 선천적이거나 직관적이거나 쉬운 것으로 가정하는 다른 것은 무엇이 있는가?

What else are we assuming is easy, innate or intuitive?


더 나아가, 우리는 [개인의 인지적 편견을 인식]하거나 [공감과 명확한 소통 같은 '소프트' 기술을 개발할 수 있는 능력]등의 다른 역량에도 동일한 질문을 적용할 수 있었다. 우리가 효과적으로 그리고 전문적으로 나쁜 소식을 깼다고 가정하고, 실수를 폭로하거나, 정보에 입각한 동의를 얻었다고 가정하는 것은 의심의 여지가 없는 상황이지만, 환자의 눈에는 우리의 수행능력이 부족하게 보이는 상황이 있다.

Extrapolating further, we could apply the same line of questioning to other competencies in medical education, such as our ability to recognise personal cognitive biases or develop ‘soft’ skills such as empathy and clarity of communication. There are undoubtedly circumstances in which we assume we effectively and expertly broke bad news, disclosed error or obtained informed consent, but in the eyes of the patient our performance was lacking.


의학 교육에서 [선천성에 대한 암묵적인 가정]은 배제되어서는 안 된다. 여기에는 LRD뿐만 아니라 커뮤니케이션, 공감, 청렴과 같은 다른 기술도 포함된다.

No tacit assumption of innateness should go unchecked in medical education. This includes LRD but also other skills such as communication, empathy and integrity.


마지막으로 [무엇인가가 선천적으로 타고나는 것이라는 가정]을 버려야만 자신의 능력에 대한 중요한 통찰력을 얻고 창의적인 해결책을 내놓을 수 있다.

Lastly, only when we discard assumptions of innateness, can we begin to gain important insights into our own abilities and come up with creative solutions.


이 연구의 세 가지 중요한 통찰력은 이 노력에 중요하다. 

  • 첫째는 아무리 직관적으로 보일지라도 현실은 항상 더 분산되어 있다는 것이다. 많은 사람들이 LRD를 기본적인 읽고 쓰는 능력만큼 의과대학에서 입학해야 하는 필수 조건이라고 생각하겠지만, 다양한 연구들은 이제 이러한 가정과 모순되어 오류를 줄일 수 있는 가능성을 가진 새로운 연구 분야를 만들어낸다.

Three critical insights from this study are important to this endeavour. The first is that no matter how intuitive a skill may seem, the reality is always more distributed. Although LRD would be considered by many to be as much of a prerequisite to matriculating in medical school as basic literacy, various studies now contradict this assumption, thus giving rise to a new field of study with the potential to reduce errors.


  • 둘째로, 선천성을 의심함으로써 사람이 스스로 인식한 능력과 실제 능력 사이의 차이를 완화시킬 수 있다. 더닝 크루거 효과에 따르면, 사람들은 자신의 능력을 평균보다 더 나은 것으로 과대평가하는 경향이 있다.8

Secondly, challenging assumptions of innateness can mitigate the gap between people’s perceived abilities and actual abilities. According to the Dunning-Kruger effect, people tend to overestimate their own abilities as being better than average.8


흥미롭게도, 그들의 실적이 나쁠수록, 과대평가 정도가 더 높다. 이러한 편견으로부터 보호되고, 사실 자신의 능력을 과소평가한 유일한 집단은 [최고수준의 4중주단]이었는데, 이는 그 주제를 잘 알고 있는 사람들 역시 더 자기 인식적인 사람들이었음을 암시한다.

Interestingly, the worse their performance, the higher the degree of overestimation. The only group that was protected fromthis bias, and in fact underestimated their ability, was the top-performing quartile, suggesting that those who knew the subject well were also more self-aware.


더닝-크루거 효과의 심리적 메커니즘은 자기중심적으로 유리한 방식으로 증거를 선택하거나 해석하려는 우리의 경향과 같은 다요소적인 것으로 생각된다. 다른 사람들은 우리가 체계적으로 비교의 낮은 점들을 선택하는 경향이 있다고 제안했다.

The psychological mechanism of the Dunning-Kruger effect is believed to be multifactorial, such as our tendency to select for or to interpret evidence in an egocentrically favourable manner (‘egocentrism’). Others have suggested that we tend to systematically choose inferior points of comparison (‘selective recruitment’).


  • 세 번째 통찰은 도전적인 선천성이 탈-낙인, 적응, 강력한 문제 해결로 이어질 수 있다는 것이다. 저자들이 밝힌 바와 같이, 자동적으로 편중성을 결정할 수 없었던 학생들은 다양한 적응 전략의 개발을 보고했다. 여기에는 자신의 몸에 단서(예: 시계 착용)를 사용하여 laterality을 결정하고 다른 맥락에 project하는 것뿐만 아니라 '안전망'을 사용하여 오류가 발생하지 않았는지 반복적으로 작업을 점검하는 것도 포함된다.

The third insight is that challenging innateness can lead to de-stigmatisation, adaptation and robust problem solving. As the authors reveal, students who could not automatically determine laterality reported development of various adaptive strategies. These include using cues on their own bodies (e.g. wearing a watch) to determine laterality and projecting it to other contexts, as well as ‘safety netting’, checking their work repeatedly to ensure no error has been made.


LRD를 2진법 또는 고정된 기술로 간주하는 대신, 그들은 LRD를 신중하게 연습하면 개선할 수 있는 플라스틱으로 간주하기 시작한다. 이러한 성장 지향적인 사고방식은 단지 [자신의 부족함을 인식한 사람만을 돕는 것]에서 [모든 사람들이 현재의 단점을 더 자각하고 그것들을 극복하기 위해 노력하도록 격려하는 것]으로 네러티브의 변화를 가져온다.

Instead of regarding LRD as a binary or fixed skill level, they begin to regard it as plastic, something that can be improved with deliberate practice. This growth-oriented mindset changes the narrative from only helping those with a perceived deficit to encouraging everyone to become more self-aware of current shortcomings and work towards overcoming them.


의학에는 직관적이라고 가정하는 다른 많은 기술들이 있지만, 실제로는 더 분산된 스펙트럼에 존재한다. Dunning-Kruger 효과에 따라 이러한 영역에서 인식된 능력과 실제 능력 사이에는 상당한 차이가 있을 수 있다. 우리는 이러한 기술들을 선천적인 것이 아니라 가변적인 것malleable으로 보는 데 힘써야 한다. 그래서 우리는 [deficit의 낙인]을 제거하고 전반적인 성과를 증대시키기 위한 보다 창의적이고 의도적인 전략을 장려할 수 있다.

There are many other skills in medicine that are assumed to be intuitive, but in reality, exist on a more distributed spectrum. There may be significant gaps between perceived and actual abilities in these domains according to the Dunning-Kruger effect. We must work towards viewing these skills as malleable, not innate, so that we can destigmatise the deficit and encourage more creative and deliberate strategies to augment overall performance.





 2019 May;53(5):423-425. doi: 10.1111/medu.13824. Epub 2019 Mar 3.

Challenging assumptions of innateness - leave nothing unturned.

Author information

1
Division of Cardiovascular Surgery, Department of Surgery, University of Pennsylvania, Philadelphia, Pennsylvania, USA.
2
Division of Radiation Oncology, Perelman School of Medicine, University of Pennsylvania, Philadelphia, Pennsylvania, USA.
PMID:
 
30828859
 
DOI:
 
10.1111/medu.13824


효과적인 멘토 개발을 위한 열두가지 팁(Med Teach, 2006)

Twelve tips for developing effective mentors

SUBHA RAMANI1, LARRY GRUPPEN2 & ELIZABETH KRAJIC KACHUR3

1Boston University School of Medicine, USA; 2University of Michigan Medical School, USA; 3New York, USA



Introduction


많은 전문가들은 멘토링 관계를 정치, 비즈니스 및 학계에서 성공을 달성하기위한 필수 단계라고 본다(Roche, 1979). 
  • Professional transition 과정에서 멘토링의 중요성이 강조되었다 (Bligh, 1999; Freeman, 2000; Grainger, 2002; Levy et al., 2004). 
  • 연구에 따르면 멘토를 찾은 교직원은 자신감이 더 높았고, 생산적인 연구 경력을 갖게 되었고, 더 큰 경력 만족도를 보였을 가능성이 더 높다(Palepu et al., 1998; Ramanan et al., 2002; Levy et al., 2004) . 
  • 멘티에 대한 다른보고 된 혜택은 다음과 같습니다. 
    • 직업에 대한 사회화
    • 경력 경로에서 선택과 성취에 도움 
    • 의미있는 참여 학술 활동; 
    • 긴밀한 협력 관계의 발전 (Morzinski et al., 1996; Pololi et al., 2002). 
  • 멘토에 대한 자기보고 된 혜택에는 
    • 차세대 육성에 대한 자부심
    • 기관 내 전문적인 협력자 네트워크를 구축
    • 멘티 그룹에게 자신의 전문 지식과 기술을 전파. 
  • 멘토링 프로그램 관점에서 교수들의 retention에 긍정적 효과가 있다(Benson et al., 2002). 
  • 이러한 이점에도 불구하고 많은 초기 경력의 임상의와 연구자는 적절한 멘토를 찾는 데 어려움이 있습니다. 특히 여성과 임상가-교육자는 부적절한 멘토링 관계의 위험에 처해있다 (Chew et al., 2003).
Many professionals identify a mentoring relationship as an essential step for achieving success in politics, business and academia (Roche, 1979). 
  • The importance of mentoring throughout one’s career has been emphasized, especially during professional transitions (Bligh, 1999; Freeman, 2000; Grainger, 2002; Levy et al., 2004). 
  • Studies have shown that faculty members who identified a mentor felt more confident, were more likely to have a productive research career and reported greater career satisfaction (Palepu et al., 1998; Ramanan et al., 2002; Levy et al., 2004). 
  • Other reported benefits for mentees include: 
    • socialization into the profession; 
    • help with choice and fulfillment in of career path; 
    • meaningful involvement academic activities; and 
    • the development of close collabora- tive relationships (Morzinski et al., 1996; Pololi et al., 2002). 
  • Self-reported benefits for mentors include 
    • pride in develop- ing the next generation, 
    • building a network of professional collaborators within an institution and 
    • being able to disseminate their expertise and skills to a group of mentees. 
  • From a mentoring program perspective faculty retention has been reported as a positive outcome (Benson et al., 2002). 
  • Despite these benefits, many early career clinicians and investigators have difficulty in finding appropriate mentors. Women and clinician-educator faculty in particular are at risk of inadequate mentoring relationships (Chew et al., 2003).

멘토링 관계는 보통 나이 든 전문가 인 '멘토'와 젊은 동료 '멘티'(Grainger, 2002) 사이에서 발생합니다. 오딧세이에서 멘토는 오딧세우스의 믿을 수있는 친구였습니다. 오디세우스는 멘토에게 자신의 집을 돌보고, 아들 텔레마쿠스 (Telemachus)의 교육을 맡기고 트로이 전쟁으로 떠났다. 이 서사시에서 멘토라는 단어를 지혜롭고 충실한 카운셀러로 사용하게되었습니다. 오늘날 멘토는 카운셀러이자 교사인 누군가이며, 성공을 달성하기 위해 주니어 연수생이나 동료를 지도하고 조언하고 돕습니다.

The mentoring relationship usually develops between an older professional, the ‘mentor,’ and a younger colleague, the ‘mentee’ (Grainger, 2002). In the Odyssey, Mentor was a trusted friend of Odysseus, who entrusted Mentor with the care of his house and the education of his son, Telemachus, when he set out for the Trojan War. From this epic arose the use of the word mentor as a wise and faithful counselor. Today, a mentor is someone who is a counselor and a teacher and instructs, admonishes and assists a junior trainee or colleague in attaining success.



팁 1 : 멘토는 역할에 대한 명확한 기대와 경청능력, 피드백 기술이 필요합니다.
멘토는 타고나는 것이 아니라 개발되는 것이다.

Tip 1: Mentors need clear expectations of their roles and enhanced listening and feedback skills

Mentors are not born but developed


훌륭한 멘토는 
    • 자신의 분야에서 지식과 존경을 받고, 
    • 멘티에게 잘 반응하며 멘티가 만날 수 있고, 
    • 멘토링의 관계에 대한 관심을 가지며, 
    • 멘티의 역량과 잠재력에 대해 잘 알고 있고, 
    • 멘티가 스스로에게 적절하게 도전하도록 동기부여하고
    • 멘티의 옹호자 역할 하는 
..등의 귀중한 특징이 있습니다. 
valuable characteristics of include 
    • being knowledgeable and respected in their field, 

    • being responsive and available to their mentees, 

    • interest in the mentoring relationship, 

    • being knowledgeable of the mentee’s capabilities and potential, 

    • motivating mentees to appropriately challenge themselves and 

    • acting as advocates for their mentees. 

그 외에도 멘토링 할 때 듣기와 긍정적 인 피드백뿐만 아니라 부정적인 피드백을 줄 때 필요한 핵심 기술 등이 필요하다.

Some key skills required when mentoring others include listening and the ability to give positive as well as negative feedback.


많은 교육자들이 이러한 기술을 가지고 태어나지 않으며 멘토링 기술에 대한 기관 차원의 교수개발 프로그램의 혜택을받을 것입니다. 그러한 프로그램은 
    • 멘토의 책임, 
    • 효과적인 멘토링 관계에 필요한 기술 
    • 관계에서의 문제점을 인식하는 전략
...을 핵심으로 강조 할 수 있습니다 (Benson et al., 2002). 

Many educators are not born with these skills and would benefit from institutional staff development programs on mentoring skills. Such programs could highlight key 

    • the responsibilities of a mentor, 

    • skills required for an effective mentoring relationship and 

    • strategies to recognize problems in a relationship (Benson et al., 2002). 


이 워크숍은 예비 멘토가 비디오 촬영 시나리오 및 역할극 시청과 같은 실제 연습에 참여하도록 허용하는 교육 전략의 조합을 사용하는 경우 가장 효과적입니다 (Connor et al., 2000). 그러나 이러한 교수개발 프로그램의 실제 성과는 실제 멘토링 환경에서 측정되어야한다는 점을 강조해야합니다.

These workshops would be most effective if they used a combination of educational strategies that allowed prospective mentors to engage in practical exercises such as watching videotaped scenarios and role-plays (Connor et al., 2000).  It is to be emphasized, however,that the actual outcomes of such staff development programs should be measured in real mentoring settings.


팁 2 : 멘토는 문화 및 성 문제에 대한 인식을 갖추어야 한다.
멘토와 멘티 간 반드시 성별과 문화가 반드시 일치해야 하는 것은 아니나, 그러한 것을 원할 경우에는 가능해야 한다.

Tip 2: Mentors need awareness of culture and gender issues 

Mentor and mentee matching by gender and culture should not be mandatory, but available for those who desire it 


젠더와 문화의 차이가 효과적인 관계에 대한 상대적인 장벽으로 고려되었지만, 문헌에서 대부분의 멘티는 이것을 진정한 장벽으로 보지 않았다고 기록했다 (Jackson et al., 2003). 사실, 워크샵 참가자들은 멘토가 차별을 전혀 용인하지 않음으로써, 다양한 문화와 성별의 멘티를 지원할 수 있다고 생각했습니다. 성별 및 문화적 차이는 서로의 문화에 대한 지식을 습득하게 함으로써 멘토와 멘티의보다 큰 동반 성장을 촉진 할 수 있습니다. 이러한 지식을 습득함으로써 사람들이 타고난 편견을 극복하는 데 도움이 될 수 있으므로, 멘토는 자신이 가진 성별이나 문화에 대한 편향을 인식하는 것이 좋습니다.

 Although differences in gender and culture have been considered relative barriers to an effective relationship, literature reports have documented that these have not been viewed by most mentees as real barriers ( Jackson et al., 2003). In fact, our workshop participants thought that mentors can support mentees of different cultures and gender by having zero tolerance for discrimination. Gender and cultural differences can foster greater mutual growth of the mentor and mentee as they gain knowledge of each other’s cultures. It has been recommended that mentors be aware of their own gender and culture biases as this knowledge could possibly help people overcome innate prejudices. 


교차-성별 멘토링에 관한 워크샵에서 두 가지 이슈가 제기되었습니다. 
    • 첫 번째는 개인적인 경계의 문제 였고 
    • 두 번째는 다른 성별의 책임에 대한 이해 부족이었습니다. 
이러한 우려에도 불구하고, 대부분의 멘티들은 멘토의 성별이 꼭 동일해야 할 필요성을 느끼지 못했습니다. 그들은 문화와 성을 넘나 드는 관계가 차이점을 더 수용하고 편견을 낮추게 될 것이라고 생각했습니다. 학생들은 기관이 적극적으로 성별이나 문화에 기반하여 멘토와 멘티를 매칭해서는 안되며, 멘토는 멘티의 성별 및 인종과 관련된 문제를 이해하는 데 필요한 기술을 갖추어야한다고 말했다. 그러나 개별 연수생이 이러한 특성 / 변수에 근거하여 의미있는 멘토링에 대한 차별이나 중대한 어려움을 보고하는 경우, 기관은 그들이 더 편하게 느낄 수 있고, 멘토링 요구를 더 잘 수행 할 수 있는 멘토를 찾아야합니다.

Two issues were raised at the workshops in relation to cross-gender mentoring. The first was that of personal boundaries and the second, lack of understanding of the other gender’s domestic responsibilities. Despite these concerns, most mentees did not feel the need for having a same gender mentor. They felt that relationships across cultures and gender would promote more acceptance of differences and lowering of biases. They stated that institutions should not actively try to pair mentors and mentees based on gender and culture and mentors should be equipped with the skills required to understand issues related to their mentees’ gender and ethnicity. However, if individual trainees report discrimination or significant barriers to meaningful mentoring based on these characteristics/variables, the institution should find them mentors who can put them more at ease and better fulfill their mentoring needs. 


팁 3 : 멘토는 멘티를 지원해주면서, 동시에 도전정신을 갖게 해야 합니다.
지원과 도전의 균형

Tip 3: Mentors need to support their mentees, but challenge them too

Balance support and challenge


Daloz (1986)는 효과적인 멘토 - 프로 테지 멘션 관계 선박은 세 가지 요소, 즉 지원, 도전, 미래의 비전에 대한 균형을 맞춰야한다고 주장했다. 멘토가 멘티에 도전하지 않고 과도하게 지지한다면 멘티는 전문적으로 성장하지 못합니다. 다른 한편으로는 멘티가 자신의 전문성 개발에서 후퇴 할 수 있도록 지원하지 않으면 서 도전한다 (그림 1).

Daloz (1986) states that effective mentor–prote´ge´(e) relation-ships should balance three elements: support, challenge and a vision of the prote´ge´(e)’s future. If mentors are overly supportive without challenging mentees, the mentees do not grow professionally; on the other hand, challenging without supporting causes mentees to regress in their professional development (Figure 1).



팁 4 : 멘토에게도 불확실성과 문제점을 표현할 수있는 포럼이 필요합니다.
멘토도 문제를 겪는다.

Tip 4: Mentors need a forum to express their uncertainties and problems 

Mentors have problems too


일단 교수진이 멘토가되면, 그들은 모든 것을 알게될 것이며, 프로그램에서 더 이상 관심을 기울일 필요가 없다고 생각하는 경우가 있다. 그러나 많은 멘토들은 멘토링 관계에서 문제를 토론하고 조언을 구할 수있는 메커니즘이 필요하다고 표명했습니다. 멘토가 담당하는 ​​멘티는 종종 두 명 이상이고, 각 대인 관계가 다를 수 있으므로 한 쪽에서 효과가있는 기술은 다른 쪽에서 효과적이지 않을 수 있습니다.

It is often assumed that once faculty become mentors, they become all-knowing and do not need any further attention from the program. However, many mentors expressed the need to have a mechanism by which they could discuss problems in their mentoring relationships and get advice. Given that mentors often have more than one mentee and each interpersonal relationship is likely to be different, skills that are effective in one may be ineffective in the other.


해결책을 논의하는 동안, 특정 개인에 관한 세부 사항은 기밀로 유지되어야한다는 점을 기억해야합니다 (Freeman, 1997). 교육 기관은 멘토링 관계의 문제를 해결하는 방법에 대해 잘 알고있는 외부 컨설턴트와 함께 수석 교육자가 진행하는 정기 멘토 회의 일정을 수립 할 수 있습니다. 이러한 회의는 멘토가 성공과 실패를보고하고 동료 및 전문가로부터 피드백을받을 수있는 포럼을 제공 할 수 있습니다. 이러한 토론에는 멘토링 문제 및 멘티 이름에 대한 필수 세부 사항 만 포함되어야하며 기타 세부 사항은 기밀로 유지되어야합니다.

While discussing challenges solutions, in it their must mentoring relationships and seeking be remembered that details regarding specific individuals must remain confidential (Freeman, 1997). Institutions can schedule periodic mentor meetings led by senior educators along with external consultants who are knowledgeable about methods for troubleshooting problems in mentoring relationships. Such meetings could provide a forum for mentors to report their successes and failures, and to receive feedback from their peers and the experts. These discussions should include only essential details of the mentoring issues and mentee names and other details must remain confidential. 




팁 5 : 멘토는 전문적인 경계를 알아야합니다.
멘토는 멘토링에 충실해야합니다.

Tip 5: Mentors need to be aware of professional boundaries 

Mentors should stick to mentoring


멘토링 관계에서 유념해야 할 몇 가지 유형의 경계가 있습니다. 멘토와 멘티 간의 개인 접촉은 학생과 교사/어드바이저/롤모델 사이의 관계보다 훨씬 가깝습니다. 더욱이, 멘티가 종종 개인적인 이슈와 문제를 들고올 수 있는데, 이로 인해서 둘 중 한명 또는 두 명 모두가 부적절하게 친밀한 감정을 품게 될 수 있다 (Palepu et al., 1996; Jackson et al., 2003).

There are several types of boundaries that need to be considered in a mentoring relationship where the personal contact between mentors and mentees is much closer than in other professional relationships such as a student with teacher, advisor or role model. Moreover, personal issues and problems may be discussed by a mentee, which could lead to one or both of them harboring inappropriately intimate emotions towards the other (Palepu et al., 1996;Jackson et al., 2003). 


참가자 중 한 명이 다음과 같이 언급했습니다. "나는 훈련 기간 동안 나에게 어머니 역할을 해줄 기대한 멘티가있었고 이로 인해 무척 감정적으로 고갈되었습니다." 멘토 교육에는 professional boundaries에 대한 지식을 전달해야 하며, 심리학자 또는 카운슬러와 같은 전문가에게 referral이 필요한 정신 사회적 문제에 대한 인식이 포함되어야합니다.

As one of our participants stated: ‘‘I had a mentee who expected me to mother him throughout his training period and that was emotionally exhausting.’’ Mentor training should include knowledge of professional boundaries, and recognition of psychosocial problems that need referral to professionals such as psychologists or counselors. 



팁 6 : 멘토도 멘토링이 필요합니다.
멘토를 위한 멘토

Tip 6: Mentors also need mentoring

Mentors for mentors


교육 기관에서는 종종 시니어 교사에게 멘토를 제공하지 않습니다. 그러나 고위 교육자라도 이미 자신의 조직 내에서 높은 직책을 유지하고있는 동안 자신의 경력 초점이나 전문직 경로를 바꿀 수 있기 때문에 멘토링을 받아야합니다.

Educational institutions often do not provide mentors for senior teachers. even the most senior educators need to be mentored as they may wish to change their career focus or professional path while they already hold high positions within their organizations. 



팁 7 : 멘토는 인정이 필요합니다.

멘토링의 가치를 높이십시오.

Tip 7: Mentors need recognition

Raise the value of mentoring


전세계의 대부분의 교육 기관에서 멘토는 멘토링 업무를 수행하지만, 일반적으로 이것은 보상이 있기 때문이 아니라 보람이 있기 때문입니다. 전체 학교에 멘토링이 의과 대학에서 가장 중요한 임무 중 하나라는 것을 납득시키기 위해, 기관의 리더는 멘토 그룹이 수행하는 일에 대해 가치를 인정해주고, 감사를 받는 '엘리트'그룹으로 공개적으로 인정해야합니다 (Palepu 외, 1998).

At most educational institutions around the world, mentors usually perform their mentoring duties not because they are reimbursed for it but because they consider it a rewarding aspect of their profession. To convince the entire institution that mentoring is one of the most important duties at medical schools, institutional leaders should publicly recognize their group of mentors as an ‘elite’ group of faculty who are highly valued and appreciated for their work (Palepu et al., 1998).



팁 8 : 멘토는 보상을 받아야합니다.

멘토는 다양한 방식으로 보상받을 수 있습니다.

Tip 8: Mentors need to be rewarded

Mentors can be rewarded in different ways


교육 기관은 몇 가지 혁신적인 방법으로 핵심 멘토 그룹에게 보상 할 수 있습니다. 

    • 멘토 피정 또는 저녁 식사는 정기적으로 개최 할 수 있습니다. Retreat이나 종종 있는 저녁식사에서 멘토는 동료와 상호 작용하며, 효과적이거나 비효과적이었던 기술을 공유 할 수 있습니다. 

    • 멘토에게 보상하는 또 다른 방법은 그들에게 컨퍼런스 기금을 추가로 제공하는 것입니다. 

    • 또한 멘토링은 승진의 기준이 될 수있다 (Benson et al., 2002).

Educational institutions can reward their core group of mentors in several innovative ways. Mentor retreats or dinners can be held periodically. At retreats or even just occasional dinners mentors can interact with their colleagues, share their experiences and techniques, both effective and ineffective. Another method to reward special mentors would be to give them extra conference funding. Additionally mentoring can become a criterion for promotion (Benson et al., 2002). 



팁 9 : 멘토링은 보호받는 시간이 필요합니다.

멘토링은 '그때그때 대충' 수행 할 수 없습니다.

Tip 9: Mentoring needs protected time

Mentoring cannot be done ‘on the fly’


몇 명의 연수생을지도하는 교수진은이 중요한 의무를 효과적으로 수행하기 위해 어느 정도의 보호 시간을 할당 받아야합니다. 기존의 업무량에 이토록 중요한 멘토링 업무를 단순히 추가하기만 한다면, 멘토링은 실패할 것이다.ㄴ

Faculty who mentor several trainees should be allocated some degree of protected time to perform this important duty effectively. Just adding this important duty to the existing workload is a recipe for poor mentoring relationships. 



팁 10 : 멘토는 지원이 필요합니다.

멘토는 개인적 또는 심리적 문제를 다루지 않아야합니다.

Tip 10: Mentors need support

Mentors should not be expected to tackle personal or psychological problems


일부 멘티의 문제는 일반적인 멘토와 멘티 사이의 관계나 토론의 경계를 넘을 수 있습니다. 멘티는 임상 적으로 우울하거나, 성격 문제가 있거나, 약물 남용 문제가 있거나 학업 문제가있을 수 있습니다. 멘토는 이러한 문제를 해결할 수 없다고 생각할 때 이를 인식 할 수 있어야하며, 학습 상담사 및 심리학자와 같은 전문가 네트워크를 통해 멘티를 refer 할 수 있어야합니다. 멘토에게 멘토가 갖추지 못한 기술을 필요로하는 역할이 강요되어서는 안됩니다.

Some mentees’ problems may overstep the boundaries of the usual mentor–mentee relationships and discussions. Mentees may be clinically depressed, have personality problems, have substance abuse problems or just academic problems. Mentors should be able to recognize when they feel unable to resolve such problems and should be supported by a network of specialists such as study counselors and psychologists to whom they can refer their mentee. The mentors should not be forced to take on roles in which they do not have expert skills.



팁 11 : 동료 멘토링 장려

멘토링의 피라미드 모델

Tip 11: Encourage peer mentoring

A pyramidal model of mentoring


피어 (또는 동료 피어) 멘토링을 연구 한 의학 교육자는 이것이 전통적 dyadic 멘토링 접근법에 대한 실현 가능하고 아마도 바람직한 대안이라고 제안합니다 (Woessner et al., 1998; Pololi et al., 2002). 참가자들은 공유 된 통찰력, 경험, 아이디어, 지침, 문제 해결 및 지원을 구하면서 자신의 동료를 '협력자'또는 '동료'(비 계급적 관계를 암시 함)로 식별했습니다.

 Medical educators who have studied peer (or near-peer) mentoring suggest that it is a feasible and perhaps more desirable alternative to traditional dyadic mentoring approaches (Woessner et al., 1998; Pololi et al., 2002). Participants identified their peers as ‘collaborators’ or ‘colleagues’ (implying a non-hierarchical relationship), while seeking shared insights, experiences, ideas, guidance, problem-solving and support from them.  


교수 시간에 대한 압력은 피라미드 멘토링 시스템을 통해서 어느 정도 완화 될 수 있습니다. 그러한 모델은 피라미드의 맨 아래에있는 멘티 집단을 수반하며 피라미드의 작은 그룹으로부터 조금 더 높은 조언을 구할 수 있습니다. 경험 많은 수석 멘토가 피라미드의 꼭대기에서 모든 것을 감독하고 안내합니다 .

Pressures on faculty time could be alleviated to a certain extent by creating a pyramidal system of mentoring. Such a model would entail a group of mentees at the bottom of the pyramid who can seek advice from a small group of peers a little higher in the pyramid with the more experienced, senior mentors overseeing and guiding all of them at the top of the pyramid.



팁 12 : 멘토링 프로그램의 효과를 지속적으로 평가하십시오.

멘토링은 지속적으로 진행되는 작업입니다.

Tip 12: Continuously evaluate the effectiveness of the mentoring programs

Mentoring is a work in progress



멘토링 프로그램이 성공하기 위해서는 기관들이 멘티와 멘토가 주기적으로 프로그램을 평가하고 현재 문제를보고하고 멘토링 또는 기존 프로그램의 변경에 대한 새로운 접근법을 제안해야합니다.

For mentoring programs to succeed, institutions need to have the mentees and mentors evaluate the program periodically, report the current problems and suggest new approaches to mentoring or changes to the existing program.


  • . 방법

    • 명확한 목표

    • 정기적이고 목적이있는 미팅

  • . 내용

    • 피드백

    • 멘티가 쟁점을 제기하고 멘토에 도전 할 수 있습니다.

  •  . 결과

    • 진도 및 경력 개발

    • 네트워킹


. Process 

Clear objectives   

Regular, purposeful meetings   

. Content 

Feedback   

Mentee could raise issues and challenge mentor  

 . Outcome 

Progress and career development   

Networking  


주어진 기관의 모든 멘티와 멘토는 적어도 멘토링 관계를 최소 3-4 회 평가해야합니다. 다음 항목은 평가할 수있는 멘토링 관계에있는 영역의 예입니다.

All mentees and mentors at a given institution should be asked to evaluate their mentoring relationships at least 3–4 times a year. The following items are examples of areas in a mentoring relationship that could be evaluated:


. 직업적인 목표에 합치;

. 멘토 (들)의 가용성;

. 멘티에게 책임과 기회를 주는 멘토;

. 위원회 및 기타 전문 활동에 대한 멘티와 관련된 멘토;

. 내외부 교수와의 네트워킹을 촉진시키는 멘토;

. 멘토가 일과 개인 생활을 통합하는 것을 돕는 멘토;

. 제자에 대한 존경을 보여주는 멘토.

. 멘토링의 개인적 혜택.


. congruence on professional goals; 

. availability of mentor(s); 

. mentor giving mentee responsibilities and opportunities; 

. mentor involving mentee on committees and other professional activities; 

. mentor facilitating networking with internal and external faculty; 

. mentor helping mentee integrate work and personal life; 

. mentor showing respect for the mentee as a person; 

. personal benefits from mentoring.





Conclusions








 2006 Aug;28(5):404-8.

Twelve tips for developing effective mentors.

Author information

1
Department of Medicine, Section of General Internal Medicine, Boston University School of Medicine, MA 02118, USA. sramani@bu.edu

Abstract

Mentoring is often identified as a crucial step in achieving career success. However, not all medical trainees or educators recognize the value of a mentoring relationship. Since medical educators rarely receive training on the mentoring process, they are often ill equipped to face challenges when taking on major mentoring responsibilities. This article is based on half-day workshops presented at the 11th Ottawa International Conference on Medical Education in Barcelona on 5 July 2004 and the annual meeting of the Association of American Medical Colleges in Boston on 10 November 2004 as well as a review of literature. Thirteen medical faculty participated in the former and 30 in the latter. Most participants held leadership positions at their institutions and mentored trainees as well as supervised mentoring programs. The workshops reviewed skills of mentoring and strategies for designing effective mentoring programs. Participants engaged in brainstorming and interactive discussions to: (a) review different types of mentoring programs; (b) discuss measures of success and failure of mentoring relationships and programs; and (c) examine the influence of gender and cultural differences on mentoring. Participants were also asked to develop an implementation plan for a mentoring program for medical students and faculty. They had to identify student and faculty mentoring needs, and describe methods to recruit mentors as well as institutional reward systems to encourage and support mentoring.

PMID:
 
16973451
 
DOI:
 
10.1080/01421590600825326


경험은 어떻게 맥락적으로 학습되는가* (교육인류학연구, 2017)

조 현 영

인하대학교

손 민 호**

인하대학교




1. 상황주의 패러다임과 실천: 실천공동체? 공동체적 실천!

상황학습론이 교수학습 연구에서 대두된 이래 꽤 세월이 흘렀다. 상황학습론은 70년대 이후 아동발달연구에서 출발하였다. 아동발달연구에서는 사회문화적 맥락이 아이가 무엇을 할 수 있고 무엇을 할 수 없는지에 직접적으로 연관되어 있는 내생적 인 변인이라는 점을 여러 연구들을 통해 밝히고자 하였다. 여기에 당연히 주역은 비 고츠키학자들이었다. 그들은 생태심리학이나 문화심리학을 배경으로 하여 피아제의 연구에 전제되어 있는 구조주의적인 가정들을 정면으로 반박하고자 하였다. 그들은 아이의 반응이나 생각 등으로 표출되는 인지구조는 일관적이지만 상황적이고 역동적 이라는 점을 보여주고자 하였다. 이후 많은 발달 연구들은 정태적이고 선형적으로 변화해가는 인지구조의 특징보다는 아이가 살아가는 상황들 안에서 미시적이고 다양 한 인지 양상의 모습을 보여주었다. 


상황학습에 관한 많은 연구들은 1980년대 이후부터 미국 서부지역 대학과 연구소를 중심으로 활발히 논의가 전개되었다. 가장 큰 이유는 비고츠키주의 심리학자들이 이 지 역 대학들을 연고로 해서 서로 교류하였기 때문이었다. 그들은 소위 미시적인 사회이론 들과 융합하여 활동이론(activity theory)을 만들어 내기도 하였다.


특히 미국 서부지역을 중심으로 한 상황인지 또는 상황학습 연구의 커뮤니티의 형성은 소위 미시사회학과 같은 히피 사회학의 영향도 무시할 수 없다. 전통적으로 심리학을 배경으로 하는 교수 학습연구에서는 상황 맥락을 학습이나 발달의 내생 변 인으로 보는 데 한계가 있을 수밖에 없었다. 한편 실리콘 밸리를 중심으로 인지과학 등 학제 간 연구들에서는 상호작용과 맥락에 관한 연구가 한창이었고 미시사회학 연 구들이 여기에 가세하였다. 여기서 히피사회학이라 함은 정향성과 규범을 토대로 하 는 기존 사회학과는 거꾸로 ‘일상’을 현실의 토대라고 보았다는 점, 그래서 세상을 뒤집어 보려는 성향 때문에 붙여진 이름이다. 


이러한 미시적 접근 특히 상징적 상호작용과 민속방법론은 방법론적 상황주의 그리고 일상생활의 실천의 복원이라는 기치 하에 미시적인 상황 분석을 통해 사회나 조직, 일 등이 어떻게 존속되어 가는지 보여주었다. 그들에 따르면 실천의 맥락은 그 자체로 이미 구조화된 사회고 조직이고 업무의 처리과정이다. 그리고 맥락과 일, 맥 락과 사회적 질서는 서로 분리된 것이 아니라 맥락을 만들어가는 것 자체가 곧 그 일이고 업무고 사회 질서다. 여기서 맥락은 실천에 의해 만들어지고 실천 안에서 일 궈가는 시간과 공간의 엮음새를 말한다. 학습이론에서 상황주의 패러다임은 사회적 상호작용과 맥락의 실제적인 엮음새에 관한 이들 연구의 통찰에 힘입은 바가 크다. 


상황학습과 실천공동체론은 이러한 배경에서 나왔다.1) 체화인지(embodied cognition), 분산인지(distributed cognition) 등 상황인지(situated cognition)에 관한 연 구들이 쏟아져 나왔으나 정작 ‘맥락적 경험은 언제 학습이라는 지위를 얻게 되는가’에 관해서는 여전히 모호하였다. 왜냐하면 우리의 사고나 경험이 상황 맥락에 기반해 있 다는 점에 대해서는 충분히 동의할 수 있지만, 그렇다고 해서 모든 맥락적 경험이 곧 학습이라고 볼 수는 없었기 때문이다. 물론 우리의 경험이 어떻게 맥락적인가 하는 것 또한 명쾌하게 해명된 것은 아니었다. 더욱이 모든 경험을 학습이라고 해석하는 것 또 한 일상어법에도 맞지 않는다. 대부분의 학습관에는 바람직한 변화라고 하는 모종의 규범적인 의미가 함축되어 있기 때문이다.  


1) 실천공동체라는 아이디어는 사실 50년대 이후 상징적 상호작용 부류의 질적 연구들에서 비롯되었다는 점은 언급될 필요가 있다. 웽거(Wenger, 1998)는 이를 실천이론을 중심으로 하나의 학습이론으로 체계화시켰다. 당시 실천공동체론이 새롭게 제기된 배경에는 학습에 있어서 상황성이 왜 중요한지에 관해 설득력있는 논증이 필요했다는 사실을 염두에 둘 필 요가 있다. 웽거를 포함한 여러 사회과학자들은 인공지능 연구자들과 함께 인공지능의 상 황인지기술 문제를 다루어야만 했다. 웽거는 그의 저서에서도 언급하고 있듯이 실천공동 체에 관한 설명이 인공지능은 역량을 가질 수 있는가라는 질문으로 시작하였다는 점을 밝 히고 있는데 이는 그가 몸담고 있었던 연구소의 지적 배경에 관한 이야기이기도 하다. 상 황주의 관점에서 보면 규칙따름을 기반으로 하는 인공지능은 정보를 처리하는 용량 (capacity)만 가지고 있을 뿐 규칙따름과는 전혀 상반되는 애매모호한 상황적 경험은 할 수 없으며 바로 이런 연유에서 ‘인격적인’ 의미에서의 역량은 결코 생기지 않으며 역량체 인 아이덴티티 또한 생성되지 않는다.


상황학습 연구자들 사이에서 부각된 과제는 우리의 경험이 어떻게 가변적이고 즉흥적인 상황 맥락을 넘어서서 일관성을 찾아나가는가에 대한 해답을 모색하는 것 이었다. 여기서 우리의 경험은 어떻게 일관성을 찾아나가는가 하는 질문은 거꾸로 뒤집어 보면 우리 경험은 어떻게 변화해가는가 라는 질문이기도 하다. 모든 경험은 상황적인 경험이지만 우리의 경험은 그러한 가운데에서도 항상성을 유지해 나가기 때문이다. 상황주의 관점은 개인과 같은 일관된 항상성이 선행한다는 규범주의적인 관점과 학습을 바라보는 데 있어서 서로 대척점을 이루고 있다. 우리 경험의 맥락적 속성에 관해 살펴보는 가운데 경험의 항상성, 즉 정체성(identity)의 존재가 문제시된 것이다. 그리고 경험의 항상성을 생성, 유지, 복원해가기기 위해 주변과 어떻게 협상 (negotiation, transaction)해 가는가 그리고 그 결과 우리의 경험은 어떻게 변화해가는 가 하는 문제로 다루기 시작했다. 이러한 관점에 따르면 협상된 경험을 기반으로 하 여 참여의 정체성, 즉 역량이 생겨난다. 


오늘날 실천공동체(CoP)는 네트워크 학습이나 조직학습의 모형을 의미하는 것으 로도 알려져 있고 학습의 생태계로 통용되기도 한다. 더욱이 CoP와 같은 상황학습의 ‘이론’화는 교육이론에서는 성공적인 모형으로 자리 잡아 왔다. 오늘날 상황주의 패 러다임 안에서조차도 실천공동체에의 참여과정은 상황학습의 전형으로 받아들여졌으 며, 여기서는 상황학습과 실천공동체론을 동의어로 간주하기도 한다. 상황주의 패러 다임의 관점에서 보면 CoP를 이론적 모형으로 상정하는 이들은 실천(practice)에 관 해 면밀히 살펴보는 것은 배제시킨 채 공동체에서의 학습 또는 지식조직에서의 학습 이라는 전략만을 취하고자 하였다. 


경험과 상호작용의 어떤 양상을 가리켜 공동체라고 한다는 점에서 모든 공동체 는 무형의 공동체(intangible community), 또는 상상의 공동체다. 무형이고 상상의 산물이지만 구성원들로 하여금 공유하는 객체로서 실재한다고 믿게끔 하는 장치이기 도 하다. 이러한 실재감은 참여자들 사이에 경험이 정체성 수준에서 함께 생겨나야 만 가능하다. 예를 들어 어느 학교에서 교사들 사이에 전문학습공동체를 조직했다고 곧바로 학습공동체가 조성되는 것은 아니다. 어떤 집단이 공동체 수준으로 자리 잡 았다면 조직은 별도의 시간 별도의 프로그램을 통해서가 아니라 일상에 널리 편재되 어 당사자들을 옭아맬 수 있는 힘을 발휘해야 한다. 학습공동체의 존재 확인은 쉬는 시간이 될 수도 있고 점심시간에 모여 잡담하는 시간도 될 수 있다. 긴 시간일 수도 있고 찰나의 시간일 수도 있다. 


상황주의 패러다임에서 보면 실천공동체에서의 경험의 형성은 ‘공동체 실천을 통 한 학습’으로 보는 것이 더 적절하다. 실천공동체라는 개념은 모든 개별적인 행위나 경험을 자칫 집단 층위의 활동으로 환원시키는 한계를 갖고 있기 때문이다. 물론 공 동체로의 환원은 학습행위를 바라보는 스펙도 무척이나 협소하게 만드는 약점을 안 고 있다. 뒷부분에서 살펴보겠지만 모든 실천에는 다른 사람과의 상호작용이 관여되__ 어 있는 만큼 개개의 실천을 면밀히 살펴보는 것으로도 충분히 공동체에 관한 의미 를 함축한다. 그보다 상황주의 패러다임은 그 문제 인식이 실천(practice)을 살펴보는 것으로 시작해서 실천으로 끝난다고 보는 것이 더 적절하다. 


실천공동체론은 상황학습에 관한 한 이론일 뿐 상황주의 학습이론 전체를 대표 하지 않는다. 상황주의 학습이론은 하나의 이론이 아닌 하나의 패러다임이라고 할 정도로 그 스펙트럼이 광범위하고 그 관심은 훨씬 근원적인 질문에서 출발한다. 물 론 상황주의 패러다임에 해당되는 연구 전체 지형도를 정리하기란 그리 어렵지 않 다. 

  • 심리학에서는 비고츠키 발달이론과 생태심리학이 그리고 

  • 사회학에서는 상징적 상호작용과 민속방법론이, 그리고 

  • 실천에 관한 여러 철학적 배경과 사회인류학이 만 나 학습연구에서 상황주의 패러다임을 일궈왔다(Nicolini, 2012). 

최근에는 위에서 언 급한 지적인 뿌리들이 혼종되어 나온 활동이론(activity theory)도 실천공동체론만큼 이나 상황주의 학습이론 패러다임에서 큰 비중을 차지하고 있다. 상황주의 패러다임 은 활동이론은 갈등론의 입장도 취하긴 하지만 기본적으로 협력 또는 조화론의 입장 을 전제로 하고 있다. 


상황주의 패러다임에서 경험과 지식 그리고 학습을 어떻게 보는지 대체로 다음 과 같이 정리해 볼 수 있다. 

  • 첫째, 경험은 그 지식이 다루고 있는 객체뿐만 아니라 맥락으로 분산되어 있다. 따라서 모든 경험은 우선적으로 개념이나 표상으로 매개되지 않은 직접적인 경험이 고 따라서 그 경험들은 즉흥적이고 생성적인 성격을 띤다. 

  • 둘째, 맥락은 우리가 대면하는 대상이면서 또한 그 안에 거주하는 공간이다. 따라 서 직접 경험으로서 지적인 경험은 객체 또는 이를 둘러싼 타자들, 즉 맥락과 직접 적으로 상호작용하는 가운데 비롯된다. 동시에 그것은 맥락의 흐름으로 임베디드되 어(embedded) 있다. 

  • 셋째, 문제해결과정에서 실제로 작동하는 추론은 맥락적인 단서와 자원을 동원하 는 만큼 상황 맥락적이다. 

  • 넷째, 지각과 행위 그리고 정서, 지적 경험은 개념이나 지식에 매개되지 않은 직 접적인 경험인 만큼 학습은 이해나 사고의 문제가 아니라 지각과 행위 더 나아가 정 서나 감성의 문제다. 

  • 다섯째, 모든 경험은 비매개적인 직접 경험이며, 동시에 타자들과의 공유 및 재생 산 가능성을 전제로 하는 만큼 그것의 학습에는 타자와의 조율과 타협, 복종과 모델 링 등이 관여된다.

  • 여섯째, 우리 경험을 이끄는 활동 시스템으로서의 맥락, 즉 즉각적이면서 미시적 인 환경을 구성하기 위해서는 자신을 어떤 맥락에 처하게(positioning)하는 행위 즉 참여 행위가 요구된다. 

  • 일곱째, 학습은 개인 내적인 구조의 변화가 아니라 개인과 맥락 간의 관계 양상 의 흐름이다. 


그리고 이상의 내용보다 더 상위의 전제는 다음과 같다. 즉 모든 행위는 협상과 조율의 맥락적 행위이고, 모든 사고는 협상과 조율의 맥락적 사고이며, 따라서 모든 학습 또한 협상과 조율의 상황학습이다. 바로 이 점이 상황주의 학습이론으로서는 곤혹스러운 지점이다. 교육이론에서 학습은 규범적인 차원에서 그 위상에 걸맞은 어 떤 경험 또는 행위의 양식을 의미하는 것이기 때문이다. 예를 들어보자. 국문학과 학 생들이 시를 반복해서 외우고 있는 상황을 가정해보자. 암기에 의한 학습인가 아니 면 실천에 기반한 상황학습을 하고 있는 것인가? 


또 하나의 상위 전제는 모든 경험은 실천을 기반으로 한다는 점이다(Gherardi, 2008; Schatzki, 2001). 위에서 제시된 상황주의 속성들은 다름 아닌 실천의 속성이기 도 하다. 맥락과 실천은 동전의 양면과도 같아서 의미가 성립하기 위해서 없어선 안 될 두 축이라고 볼 수 있다. 실천과 맥락은 서로를 구성하는 것으로 볼 수 있기 때 문이다. 독서를 한다는 것은 이야기의 맥락, 즉 행간의 의미를 발견한다는 것을 말하 며 여기에는 독서 행위가 수행되지 않으면 안 된다. 대체로 우리가 어떤 행위를 실 천이라 할 때 그것이 다음과 같은 특징을 함축하고 있다. 


첫째, 실제적인 행위가 관여되어 있고 그 만큼 맥락 변용적이다. 

둘째, 일회적 경험이 아닌 반복되는 만큼 패턴화되어 있으며 일상적(routine)이다. 

셋째, 누군가와 공유가능 한 방식으로 재생산되는 만큼 사적(private)이 아닌 공 적(public)이다. 

넷째, ‘그 일에 관여하는 동안’ 에이전트와 에이전트 에이전트와 사물을 한데 엮 어가는 만큼 구체적이다. 


2. 경험은 어떻게 상황으로 분산되는가?

2.1. 지식의 도구성과 객체의 어포던스


 상황주의에 따르면 모든 지식은 도구의 속성을 지닌다. 여기서 도구라 함은 어떠 어떠한 일을 하기 위해 동원되는 수단이라는 의미다. 지식은 그것이 명시적 지식이 든 암묵적 지식이든 도구로 볼 수 있다. 개념적인 지식조차도 우리가 그 지식을 활 용하는 것은 마치 도구처럼 부리는 것과 다름없다. 우리는 실용주의적 의미에서 지 식은 도구라는 말을 자주 사용한다. 상황주의에서 말하는 도구성은 우리가 어떤 일 을 도모해 나가는 데 있어서 닥치는 상황에 대하여 ‘실제적인’ 방식으로 행위를 가한 다는 의미를 함축하고 있다. 


지식의 도구성이 무엇을 의미하는지 이해한다면 맥락을 학습한다는 것이 무엇을 의미하는지 좀 더 쉽게 파악할 수 있다. 지식이 어떻게 도구성을 띠게 되는지에 관 해서 이해하기 위해서는 지식은 일단 익히고 나면 그 지식이 지향하고자 하는 객체 대상과 구분되지 않는 방식으로 우리에게 경험된다는 점에 관해서 되짚어볼 필요가 있다. 


메를로 퐁티(Merleau-Ponty, 1962)는 장님의 지팡이 예를 들어 도구의 비가시 성에 관해 논증한 바 있다. 어느 한 장님에게 지팡이를 주면서 그것이 어떤지 이야 기해 보도록 하자. 무겁다든지, 너무 길다든지, 표면이 매끄럽다는 등 그 지팡이 생 김새에 관해 다양하게 이야기할 것이다. 그러나 그가 그것을 자신의 도구로 전유화 (appropriation), 즉 자신 몸의 연장(extension)처럼 마음대로 사용하기 시작하면서부 터는 그 자체에 대해서는 지각하지 못한다. 이 때 지팡이는 그에게 손으로 전달되는 대상의 느낌, 예컨대 커브 길이나 길 앞의 장애물 등의 대상으로 느껴진다. 이와 마 찬가지로 어떠한 도구도 일단 그 사용법이 터득되기 시작하면 그것의 존재는 마치 없는 것처럼 투명하게 된다. 


우리가 문제를 해결할 때 동원하는 지식이나 기술도 하나의 도구로 볼 수 있다. 지식이나 기술은 우리가 그것들을 익히기 전에는 가시적인 존재이지만 일단 체득하 고 그것들을 자유자재로 사용하기 시작하면 그것들의 존재성을 따로 의식하지 못한 다. 어떤 지식이나 기술을 자유자재로 사용할 만큼 터득한 상황을 보면, 사용자가 그 지식이나 기술을 있는 그대로 적용하기에 급급하거나 얽매이지 않고 상황에 맞게 유 연하게 그것들을 부릴 수 있게 되었다는 것을 쉽게 떠올릴 수 있다. 아이러니하게도 지식이나 기술을 터득했다는 것은 그것에 의해 규제를 받는다는 의미와 동시에 그로 부터 자유롭게 되었다는 이중적인 의미를 함축하고 있다. 


아무리 고차원적인 지식 내지는 기술이라도 일단 그것이 행위자에 의해 터득되 면 그것은 당사자에게 상황 안에서 직접 경험되는 대상이 된다. 

    • 예컨대 터득된 상태 에서 바이올린과 연주곡은 연주자에게 즉각적인 반응을 불러일으키는 손끝 느낌의 대상이다. 

    • 익숙해진 말은 상황 즉각적으로 구사된다. 

    • 또한 목판에 못을 박을 때 목수 의 손놀림은 사실상 망치와 하나가 된 듯 현란하다. 

    • 전문의들은 흑백 필름 속에서 폐렴 증상이 얼마나 진척되었는지 본다. 

대상과 머릿속간의 상호작용이 아닌 실제 맥락과 그에로 열려져 있는 지각 또는 행위의 양태로 그 관계 양상이 변화한 것이 다. 여기서 사물은 사유되는 대상이 아니라 보고 듣고 말하고 사용하는 등 행위의 대상이 된다. 


상황주의 패러다임 가운데 한 이론인 활동이론(activity theory)에 따르면 지식을 체득화한다는 것은 그것을 도구로 활용할 수 있도록 전유화(appropriation)하는 것이 면서 동시에 객체, 즉 사물, 말, 문제 상황이 띠고 있는 질(quality) 또는 맥락의 결에 따라 지각과 행위가 이끌려 간다는 의미를 담고 있다. 폴라니(Polanyi, 1958)는 객체 에 의해 이끌려지는 내재적 경험을 가리켜 ‘실천 안에의 거주’(dwelling in a practice)라고 보았다. 내부자적 관점에서 보았을 때 그가 참여하고 있는 실천은 지 적일 뿐만 아니라 심미적이고 윤리적인 애착의 대상이 되기도 한다. 여기에서 지식 을 체득화한다는 것, 다시 말해서 학습이란 객체의 질을 따라 지각하고 반응할 수 있게 됨, 즉 역량(competence)의 발달을 뜻한다. 


이 때 상호작용은 직접적인 경험과 행위로 전개되는 만큼 그것은 비매개화된 지 각과 실천적, 실제적 행위 그리고 정서의 양태로 표출된다. 여기서 비매개화된 지각 이라 함은 실천적 행위라 함은 객체를 향하는 우리의 지각이 지식이나 표상에 의해 매개되지 않고 즉각적으로 이루어진다는 것을 뜻한다. 그리고 객체든 상황이든 그것 와의 상호작용이 실제적인 행위의 양태를 띤다고 함은 어느 경우에도 우리의 기획과__ 행위는 결코 규칙을 따르지 않다는 것을 의미한다. 규칙을 따르지 않고 객체의 질성 을 따른다는 말이다. 예컨대 문제 상황에 봉착했을 때 우리는 일단 어떤 조치를 취하 고 그 결과로 비롯되는 상황을 봐서 다음의 행위를 선택하고 결정한다. 물론 이러한 과정은 하나, 하나의 분절된 절차가 아닌 순환적이고 총체적인 흐름으로 일어난다. 일상의 실천은 습속이나 전통과도 같이 그 안에 거주하는 사용자에 대해 자동적으로 행위를 이끄는 처방전과도 같은 역할을 한다. 이러한 습속이나 전통으로 인해 우리는 매번 맞닥뜨리는 상황이나 객체에 대해서 새롭게 규정하고 계획세울 필요 없이 안정 된 방식으로 살아갈 수 있다.



2.2. 일의 선후관계 그리고 경험의 흐름으로서의 맥락


상황주의 패러다임에서 말하는 맥락이란 우리 행위와 추론의 국지적 환경을 말 한다. 이 점과 관련해서 상황주의 학습이론에서 상정하는 질문은 맥락적 관심과 태 도가 맥락적인 변수들을 어떻게 시간적으로 구조화해(temporally structured) 가는가 에 관해서다. 상황주의는 기본적으로 초월자의 관점이 아닌 상호주관적이라는 의미 에서의 당사자적 관점을 취한다. 맥락에 몰입되어 있는 당사자라면 맥락초월적이 아 니라 맥락적으로 생각하고 행위를 할 수밖에 없다. 이를 도식으로 표현하면 다음과 같다. 



이 점에서 행위를 통해 의미를 형성해 나간다는 것에는 상황적 추론의 또 다른 작동 방식인 ‘애씀(effort)’이 소요되어야 한다. 앞서 설명한 대로 문제를 해결하고 의 미를 창출하는 데에는 시간성의 전개라는 맥락적 흐름이 필요하며, 이것은 ‘여기 지 금’이라는 현재의 무한한 흐름 속에 끝없이 열린 결론을 가지고 전개되는 행위의 예 측 불가능성을 전제로 한다. 우리가 만나는 매 순간의 상황은 그것이 비록 반복된 일상의 일들이라 할지라도 언제나 긴장과 낯설음을 갖고 있다. 이것은 우리가 만나 는 ‘여기 지금’이라는 순간은 과거의 경험과 미래 반영되어 구현된 고유하고 개별화 된 ‘현재’이기 때문이다. 따라서 문제 맥락에 봉착하고 이를 해결하기 위해 추론이 일어나는 경우, 시간적 흐름 가운데 지난 경험을 반추하고 미래의 기대를 반영하며 끝없이 새로운 선택과 결정을 해야 하는 순간에 놓이게 된다. 


우리는 일상의 맥락에서 그러한 긴장과 새로움을 의식하지 못한 채로 대부분의 일들을 처리해 나간다. 그리고 그 가운데 기존의 인지 구조와 위배되는 문제 상황에 부딪히게 되었을 때에 흔히 반성이나 성찰이 일어난다. 듀이(Dewey, 1931)는 바로 이러한 애씀이 동원되는 경험이 바로 학습이 일어나는 순간이라고 보았다. 


학교수업에서 쉽게 찾아볼 수 있는 수업조직방식인 교사와 학생간의 질문-대답- 평가의 순환과정은 맥락의 구성이 어떻게 참여자들의 주의를 한데로 집중시키는지 잘 보여주는 예다. 특정 학생에게 던진 교사의 질문은 그 학생으로 하여금 옴짝달싹 하지 못하게 어떻게 답변을 찾아야 할지 궁리하도록 만드는 등 상황을 돌변시킨다. 질문 이전에 어떤 이야기가 오고 갔는지, 교사가 원하는 대답이 무엇인지, 나의 대답 이 어떻게 하면 전후 맥락에서 벗어나지 않을 수 있는지, 또는 앞에 대답한 학생과 어떻게 차별된 의견의 답변을 할 수 있을지 순간 궁리하게 만든다. 


교사의 질문이 학생의 반응을 맥락적으로 구조화한다는 기능은 개인에 그치지 않는다. 맥락의 돌변이 불러일으키는 영향력은 그 질문을 떠안은 학생에게만 일어난 것에 그치지 않는다. 즉, 영석이에게 한 질문은 그에게만 해당되지 않는다. 나머지 학생들은 자신이 지목당한 것은 아니지만 영석이에게 한 질문은 자신들에게로 온 질 문이기도 하다. 다음 차례가 자신이 될 수도 있다는 것을 알기 때문이다. 한 학생에 게 한 교사의 질문은 학생 개개인을 하나의 정체성으로 엮는 효과를 갖는다. 이처럼 학급에서 우리라는 실천공동체는 매 상황 구성된다. 


매 상황이 어떻게 다음을 추론시키는지는 퍼즐풀이에 비유해보면 쉽게 이해할 수 있다. 퍼즐맞추기 게임은 전체와 부분간의 연관성을 매 상황 견주어가는 탐색의 과정으로 구성된다. 그것은 마치 우리가 책을 읽을 때에 문장 하나하나에 주목할 뿐 만 아니라 그것이 지금까지의 흐름, 즉 문맥과 어떻게 연관되는지를 동시에 탐색하 는 이치와도 같다. 각각의 조각들이 갖는 연관성들이 하나의 맥락적 조건을 만들고,__ 또 그렇게 만들어진 조건은 또 다른 맥락이 되어 다음 퍼즐 조각이 어떤 것이 선택 되어야 할지에 직접적으로 영향을 주는 등의 방식으로 퍼즐풀이의 과정 전체를 형성 해나간다. 예컨대, 퍼즐 조각들 사이의 빈 공간의 그 때 그 때의 즉각적인 출현은 우 리에게 ‘무언가 있어야 할 것’이라는 기대감을 바탕으로 소위 현상학적 장 (phenomenal field)을 가시화시켜 다음 행위를 동기화시키도록 하는 조건으로 작용하 여 전체와 부분의 연관성을 추론하도록 한다(Garfinkel, 1967). 


퍼즐을 맞춰나가는 동안 우리는 퍼즐조각간의 전후관계를 찾아나가면서 즐거움 을 만끽한다. 같은 퍼즐풀이라도 사람들은 그것을 반복해서 즐긴다. 매번 경험되는 다음 상황은 당사자적 관점에서 보면 항상 ‘처음처럼’의 정서를 유발하기 때문이다. 그리고 그러한 예측 불가능한 긴장감이 다음 행위에 대한 추론을 불러 일으킨다. 


문제를 해결해 나가는 동안 그리고 도모한 일을 수행하는 과정에서 다양한 맥락 적 추론이 유발된다. 이러한 추론은 다음 상황에 대한 기대감, 이전의 상황에 따른 행위로 인한 매번 낯선 현장성으로 인한 것으로, 상황 맥락을 성립시키는 기반으로서 시간성(temporality)의 속성을 잘 보여준다. 여기서 시간성이란, 우리 행위에 총체 성을 부여하는 구조화의 원천으로서, 우리가 행위를 해나가는 테두리로서 작용하며 행위를 유발해나간다. 이러한 시간성이 행위의 테두리로서 작용할 수 있는 것은 사 고에 선행하여 경험되는 정서적 기대감을 동반하기 때문이다. 여기에는 이전의 경험 이나 맥락에 대한 예견과 같은 개인의 이해의 지평이 행위 전반에 작용하며 우리의 행위의 방향성으로 작동하게 된다. 이것은 행위자의 맥락이 유발한 이전 경험에서에 서 비롯된 막연한 결과에 대한 우려나 기대와 같은 요인들이 고려된 우연적인 상황 의 산물인 것이다(손민호·조현영, 2014). 이처럼 실천이 엮어가는 시간의 흐름은 일이 나 경험, 학습 등의 과정을 결정지우는 맥락의 속성을 잘 보여준다.


3. 경험은 어떻게 맥락적으로 학습되는가?

3.1. 경험의 학습에서 실천공동체는 어떻게 경험되는가


감각적 지식과 같은 경험의 학습은 통상적으로 암묵지의 생성과 공유로 다루어 진다. 커피 바리스타 기술은 바리스타 교육 프로그램에 한동안 참여하는 과정에서 생겨난다. 고수들이 교수자로 참여해서 수련생들에게 그 기법을 전달해 준다. 그러한 기법은 커피를 추출해내는 기술과 맛을 감별하는 능력 등으로 구성된다. 수련과정에 참여하는 수련생의 경험을 재구성할 수 있다면 커피 바리스타의 기술이 어떻게 해서 생겨날 수 있는 것인지, 즉 감각적 지식이 어떻게 학습되는지 알 수 있을 것이다. 쓰 고 시기만 한 비슷비슷한 맛의 커피들이 어떻게 수십 가지의 커피로 분류될 수 있는 지, 콩이라는 자연의 식물성 자재가 어떻게 커피라는 사회문화적 소비재를 둘러싼 문화적 기호의 대상로 둔갑될 수 있는지 그 과정에 관여되는 수많은 실천들에 관한 우리의 이해를 새롭게 해 줄 것이다. 


일반적으로 암묵지를 습득하기 위한 부단한 연마와 연습은 흔히 무맥락적이고 개인적인 상황으로 생각할 수 있다. 그러나 모든 연습에는 일정 부분 시연의 성격이 있다. 여기서 시연이라 함은 누군가를 염두에 둔 연습이라는 의미다. 예컨대 다가올 연주회를 준비하면서 연습에 임하는 경우가 그렇다. 도자기 만드는 법을 마스터를 따라 열심히 연습하는 도제는 어떠한가? 도자기 흙을 빚는 연습도 시연에 해당하는 가? 모든 연습에는 타자의 현존(co-presence)이 직접 또는 간접적으로 결부된다는 점에 관해 되짚어볼 필요가 있다(Wenger, 1998). 상징적 상호작용주의자들에 따르면 개인의 머릿속 사고조차 실제 대화의 시뮬레이션이다. 우리 안의 사고의 흐름은 독 백체(monologue)라고 생각되지만 훨씬 대화체(dialogue)의 성격을 갖는다는 것이다. 어떤 사안을 두고 우리가 머릿속으로 어떤 생각을 정리해 나가는 데 말들을 만들면 서 이어나가는 경우를 떠올려 보면 이 지적이 의미하는 바를 충분히 이해할 수 있 다. 비록 혼자 생각을 하는 것이지만 거기에는 주체적 자아(I)와 객체적 자아(Me)간 의 대화체가 스며들어 있다. 이런 점에서 비고츠키주의자들은 우리의 사고는 어떤 개념이나 의미의 내면화가 아니라 상호작용의 내면화로 나타나는 양상이라고 주장하 였다. 


상황학습, 즉 감각적 지식이 맥락적으로 습득된다는 것은 다음과 같은 의미에서 다. 이 모든 감각적 지식은 ‘동료 등 다른 사람들과의 상호작용이 가능한 방식으로’ 구성된다는 점이다. 다소 추상적으로 정리하자면 이 모든 과정에는 실천이 관여되어 야 하고 실천은 상호행위적인(interactive) 속성을 기반으로 하기 때문이다. 상호행위 적인 측면들은 대개 상황적이고 주변적이어서 당연시되어 간과되기 마련이다. 왜냐 하면 커피의 맛과 향에 관해 익힐 때 학습자의 주의는 온통 객체 대상, 즉 커피 자 체에만 집중된다고 생각하기 때문이다. 따라서 커피 맛의 감별이라는 암묵지가 생겨 나는 조건으로 학습자와 객체 대상간의 상호작용만을 생각하기 쉽다. 


한편 여러 연구 결과에 따르면 상호작용은 대상 객체와의 상호작용에 국한되지 않는다. 일례로 시카고 사회학자 베커(Becker)는 ‘마리화나 흡연자 되기’라는 연구를 통해서 마리화나 상용자가 되는 것은 하나의 학습의 과정을 통해서라는 점을 설득력 있게 간파하였다. 마리화나의 소위 ‘뿅가는’ 기분을 맛보게 되는 것은 화학 물질 때 문만은 아니다. 그는 초심자가 노련한 흡연자 등 동료들과 그 경험을 암묵적으로 맞 춰가는 가운데 생겨난다는 점을 발견하였다. 사실상 우리 경험은 사물 객체가 아닌 경험 객체와의 상호작용의 결과다. 


그러한 객체를 마주하는 이 일과 연관되어 있는 사람들과의 상호작용 또한 객체 에 관한 지식이나 그것을 다루는 기술에 스며들어 있다. 우선 ‘누가’ 그렇게 하느냐 의 문제가 암묵적으로 개입된다는 것이다. 

    • 신참의 경우 교수자는 고참으로 이미 공 동체 안에서 상황적으로 설정되어 있다. 덕분에 고참이 하는 말이나 행동양식들은 지식의 전형인 것으로 받아들여 그를 모방하고 따르려고 한다. 이 때 초심자는 어떠 한 의심도 유보한 채 고참의 행태를 따르고자 한다. 고참의 지시나 경험이 설령 당 장 따르기도 그리고 자신의 경험으로 받아들이기 힘든 부분들이 있더라도 신참은 일 단 자신의 판단을 유보한 채 그를 믿고 따르려는 태도를 취한다. 실천공동체가 학습 의 기제로 작동할 수 있는 데에는 집단 정서, 즉 에토스(ethos)가 기반이 되기 때문 이다. 

웽거(Wenger, 1998)가 그의 저서에서 실천공동체론이 네트워크 학습론과는 다 르다고 역설하는 것도 바로 학습의 정서적 토대 때문이다. 이러한 정서는 자신과 자 신이 동일시하고자 하는 공동체의 전형간의 차이를 서로 맞춰 나가고자 하는 태도에 서 시작된다. 참여를 통해 형성하는 아이덴티티는 동일시(identification)의 기제가 작동하면서 비롯되는 산물이다. 


상호작용의 객체, 즉 커피의 맛을 감별해내는 능력에는 혀끝에 체화된 암묵지 뿐 만 아니라 공동체가 공유하는 방식으로 객체화하는 능력도 동시에 요구된다. 물론 그 양자는 분리될 수 없는 하나지만 설명의 편의상 구분하자면 그렇다. 같은 맥락에 서 윈치(Winch)는 기계공이 기계를 수리할 수 있게 되었다는 것은 정확히 말하자면, 기계를 수리할 수 있게 되었다가 아니라 다른 기계공이 기계 수리하는 것을 할 줄 알게 되었다는 것을 의미한다고 말한 바 있다. 학습 경험이 어떻게 생겨나는 것인가 에 시사점을 제시해 준 많은 질적인 사회학 연구들은 학습 경험에서의 당연시된 측 면을 그러한 사회적 상호작용에서 발견하고 있다. 암묵지를 개인적인 지식이 아닌 인격적인 지식이라고 지칭하는 것도 바로 이러한 연유, 개인적(personal)이면서도 공 적인(public) 성격 때문이다. 


정체성의 범주와 그것의 형성 역시 암묵지와 그의 학습의 양상과 유사하다고 한 다면 암묵지 학습의 이중구속에 관한 위와 같은 해석은 이에도 동일하게 적용해 볼 수 있다. 모든 멤버십 범주는 특정한 공동체가 아니라 실천의 성운(constellation)으 로서 성립한다고 말하는 것이 더 정확하다(Wenger, 1998). 익명의 다수로 된 공동체 를 전제로 해서 성립한 것이다. 예컨대 아버지들의 실천공동체는 존재한다고 말하는 것은 어색한 표현이다. 그러나 아버지라는 아이덴티티 범주는 비슷한 처지에 놓은 익명의 다수가 공유한다고 보는 실천의 양상과의 연관성 하에서 성립한다. 마치 밤 에 육안으로 비치는 일곱 개의 서로 인접한 별자리를 하나의 북두칠성으로 보듯이, 실체로서 존재하지는 않지만 의미로서 존재한다는 말이다. 


마찬가지로 암 환자라는 실천공동체는 형식적으로 존재하지 않지만, 물론 암환자 협회 같은 제도화된 조직이 있을지는 모르겠지만, 암환자라는 정체성 범주는 실제로 존재한다. 누군가 암환자가 된다는 것은 몸에 암에 걸렸다는 것 이상의 의미를 갖는 다. 자신의 질병이나 세상을 어떻게 바라보고, 암환자로서 자신의 삶을 어떻게 살아 가야 하는지, 자신이 걸린 질병과 관련해서 어떻게 먹고 어떻게 대응해 가야하는지 등 자신이 겪어보지 못한 사람들의 경험들과 함께 서서히 익혀간다. 그들의 전형을 암묵적으로 받아들인 것이고 그런 만큼 암환자라는 범주는 그의 정체성 일부로 자리 잡게 된다. 암환자로서의 정체성은 질병에 대한 대응전략의 일환으로 형성되는 것이 기도 하고 사회적 대응전략으로 생겨나는 것이기도 하다. 이러한 아이덴티티는 실제 적으로 우리가 그 실천에 참여하는 만큼만 상황화되어 나타나는 전형이다. 존재의 임(being)은 되기(doing)에의 참여 결과 나타나는 기투된 양상(being in the world)이 다.


3.2. 연습은 어떻게 맥락의존적인가

대부분의 학습은 지식이나 경험의 숙달과 관련되어 되어 있다. 아이들이 구구셈 을 익히기 위해서 셈법의 숫자들을 무한히 반복해서 읊는다. 시험을 앞두고 교과서 에 나오는 개념을 익히기 위해서 암기도 하고 그 개념이 응용된 문제를 풀어보기도 하는 공부에서도 반복 연습은 필수다. K팝 악보를 보고 이를 피아노를 연주하기 위 해서는 피아노 건반을 두드리는 연습을 수없이 반복해야 한다. 전문 목공이나 전문 연주자의 기량에 도달하기 위해서는 1만 시간의 연습이 필요하다고 한다(Sennett, 2008). 


연습이 어떻게 맥락적으로 경험되는 것인지 짚어보기에 앞서 전문가들의 실천이 어떻게 맥락의 운영(situated management of knowledge)과 같은 특징을 보이는지 살펴볼 필요가 있다. 다음은 실제 진료상황의 시뮬레이션 상황인 가상의 진료상황에 서의 문진 장면이다. 여기에서 의사는 환자의 개별적이고 모호한 신체증상을 환자와 의 상호작용 과정에서 자연스럽게 하나의 의학적 사실로 연관시켜 진단내리고 있다.


<사례 1>

의사(SD): 전혀 없으시구요. 예, 음… 그 검은색 변. 검은색 말고는 뭐… 점액

질이라던지 그런거는 섞여 나온다 그런 건 없었나요?

환자(P)S: 점액질이요?

SD : 네. 뭐 하얗게 뭐 다른 물질이 섞여나온 건 아니에요?

PD : 글쎄요. 그런 건 잘 모르겠네요.

SD : 그런 건 모르시겠구요. 설사 같은 건 하세요?

PD : 아니요

SD : 안하시구요. 예. 배가 혹시 아프세요?

PD : 아. 가끔.

SD : 어디가 아프세요?

PD : 글쎄요. 명치 근처에서 아픈가? 그런 게 좀 있었어요.

SD : 명치 근처요. 어… 언제부터 그러셨어요?

PD : 글쎄요. 정확하게는 잘 모르겠구요. 몇 달 된 거 같아요.

SD : 몇 달 전부터요. 어떻게 아프세요? 쓰린 느낌이에요? 아니면은…

PD : 쓰릴 때도 있어요.

SD : 아. 쓰린 느낌이세요?

PD : 예. 쓰릴 때도 있어요.

SD : 아. 그게 식사랑 연관이 있는거 같나요? 밥을 먹으면 괜찮다던지 뭐 밥 을 먹으면 더 아프다던지…

PD : 예. 쓰린거는… 에… 그런… 밥먹기 전엔 좀 쓰리다가 먹고 나면 좀 괜찮아지는 거 같기도하고… 예…

(중략)

SD : 예. 알겠습니다. 통증이 명치 부위가 아프다고 하셨잖아요. 통증이 뭐.…

다른 쪽으로 뻗친다던지 그런 느낌은 없으세요?

PD : 뻗친다구요?

SD : 예. 뭐… 여기가 아프다가 등이 또 아프다던지…

PD : 아니요. 그런 거 모르겠는데….

SD : 그런 건 모르시겠구요. 예. 메스꺼우세요, 좀?

PD : 가끔 그럴 때가 좀 있었던 거 같아요. (조현영, 2015)


위 상황에서 의사는 환자에게 질문을 하고 이에 대한 환자의 대답을 다시 한번 반복하며 확인한다. 그것은 환자의 대답을 스스로 재확인하는 혼잣말처럼 보이지만, 그 공간 안에 있는 누구든지 들을 수 있다. 그런데 한 가지 흥미로운 사실은 의사가 환자의 애매하고 불확실한 대답을 다시 말하는 과정에서 의미를 한정시키는 방식으 로 재구성하여 말하고 있다는 사실이다. 

    • 예컨대, 통증의 양상을 묻는 질문에 “쓰릴 때도 있어요.”라고 말하는 환자의 대답에 대해서 의사는 “쓰린 느낌이세요.”라고 환 자의 통증이 쓰린 느낌이라고 단정지어 말한다. 환자가 “쓰릴 때도 있다.”라고 말한 것에는 문법상 그렇지 않은 느낌일 때도 있다는 의미를 내포하고 있을 것이다. 그런 데 의사는 이에 대해 더 이상 구체적으로 묻지 않고, 쓰린 느낌이라고 통증의 양상 을 한정지어버린다. 그런데 환자는 의사의 이러한 말에 어떠한 반박도 하지 않는다. 

    • 마찬가지로 통증의 부위도 환자는 “명치 근처에서 아픈가?”라고 대답하며 통증 부위 의 경계를 애매모호하고 불확실하게 대답한다. 그런데 의사는 잠시 뒤 신체 진찰이 이루어지는 상황에서 통증의 부위를 이야기할 때, “통증이 명치 부위가 아프다고 하 셨잖아요.”라고 단정짓고 다음 질문을 이어간다. 마찬가지로 환자는 이에 대해 어떠 한 반박도 하지 않으며, 오히려 환자는 다음의 신체진찰 상황에서는 ‘정확히 명치부 위’에서 강한 통증을 호소한다. <상황>에서 나타난 환자의 애매모호한 설명방식과는 달리 이어지는 상황에서 환자의 복부 통증은 매우 자명한 증상인 것처럼 보인다. 


의사가 환자의 말을 재구성하여 말하는 방식은 단순한 혼잣말로 보이지만, 이것 은 환자의 대답에 대한 의미를 확정짓는 중요한 역할을 한다. 환자가 ‘명치 근처’라 고 통증의 경계 부위를 모호하게 말한 뒤, 다음 대화에서 의사가 ‘명치 부위’라고 경 계를 한정지어 말했을 경우, 위 대화에서는 환자가 어떠한 언급도 없이 다음 대화가 이어진다. 이것은 직접적으로 대답을 하지는 않았지만 의사가 명치 부위로 한정한 것에 큰 이견이 없다는 것을 암묵적으로 인정한 것이 된다. 


의사의 지나가는 듯한 말들에 대한 환자의 인정 혹은 거절의 입장 표명은 다음 대화에서 이 문제에 대한 협상을 계속 할 것인지 다음의 문제로 넘어가도 되는지에 대한 국면의 전환 여부를 결정한다. 또한 이러한 방식의 발화는 환자의 대답을 다시 한번 상기시키면서 다음 발화의 내용을 떠올리기 위한 시간적 여유를 제공하기도 한 다. 자연스러운 진료 상황에서 대화와 대화 사이, 행위와 행위 사이의 공백이 너무 길다면 이것은 매우 어색하거나 혹은 그 또한 어떠한 의미를 갖는 것으로 해석되기 쉽다. 


따라서 공백이 다른 의미를 갖는 것이 아니라면, 다음 행위를 탐색하는 중에도 그 상황을 자연스럽게 이어가는 말이나 행동이 이루어져야만 한다. 특히 임상수행평 가의 가상의 진료 상황에서 학생 의사는 표준화 환자의 말을 재구성하여 되묻는 방 식을 통하여 진단에 필요한 증상들의 의미를 한정시켜 문제해결의 실마리를 찾아낼 뿐만 아니라, 다음 발화 행위를 준비할 수 있는 시간을 확보해 나간다. 이러한 상호 작용의 방식은 전문가와 비전문가 사이의 비대칭적 상호작용 상황에서의 특징을 잘 보여준다. 전문가는 환자의 진료라는 개별적이고 유사한 상황들 가운데서 어떻게든 진단과 처방이라는 방식으로 매우 그럴듯하게 문제를 해결해야하는 처지에 놓인다. 따라서 의사는 어떠한 방식으로든 애매모호한 증상들을 자신이 처리할 수 있고, 설 명가능한 문제로 재상정해야 하며, 이것은 상호간의 상호작용이라는 담화의 과정을 통해 위와 같이 이루어질 수 있다. 


다시 말해서, 전문가는 문제 상황에서 자신을 둘러싼 상황 속에 놓여진 다양한 자원들, 즉 여기서는 환자의 말이나 환자와의 불평등한 파워 관계 등을 적절히 활용 해가면서 상황을 의도에 맞게 이끌어낸다. 즉, 차이의 간극을 좁혀가기 위해 상황적 요인들을 적재적소에 배치하는 전략을 통해 맥락의 유사함을 동일함으로 바꾸어가는 것이다. 


하이데거(Heidegger)는 ‘거리제거’와 ‘방향잡기’라는 두 가지 성격의 존재 양식을 통해 상황 맥락의 운용전략에 관해 설명한 바 있다(박찬국, 2014). 어떤 것을 하기 위해 필요한 도구를 사용하기 편리하게 가까이 두는 것을 ‘거리제거’에 해당한다. 이 는 익숙하고 능숙하다는 것을 뜻한다. 거리가 가까울수록 대상에 대한 의존도가 커 짐에 따라 그 존재 자체를 인식하지 못할 정도로 비가시화되어 편안하고 익숙함을 느끼게 된다. 


여기에서 거리 혹은 가시성은 결코 물리적 사실을 의미하는 것이 아니다. 이는 도구의 용도에 따라 매우 상황적이고 상대적으로 존재한다. 

    • 예를 들어 우리가 못을 박고 있을 때 망치가 나에게서 2미터밖에 떨어져 있지 않더라도 손을 뻗으면 닿을 수 있는 거리가 아니기 때문에 멀리 있는 것으로 여겨지는 것과 같은 이치이다. 뿐 만 아니라, 너무나 가깝고 익숙해서 그것이 가시화되지 않는 대상의 경우, 오히려 가 시화된 다른 대상들의 비해 더욱 원거리로 느껴질 수밖에 없다. 예를 들어, 내가 착 용하고 있는 안경의 존재는 대화중인 눈앞의 누군가보다 더욱 멀리 느껴질 수도 있 다. 


익숙하고 편안한 존재란 거리제거와 동시에 방향을 잡는다는 성격을 지니고 있 다. 도구의 거리를 제거하고 방향을 잡는 행위들은 상황의 흐름 속에서 연속적으로 발생한다. 

    • 벽에 못을 박기 위해서는 망치와 플라이어(일명, 펜치라고 불리는 집게)가 필요하다. 망치와 플라이어는 행위자가 망치질을 하는 방식에 따라 서로 번갈아가며 사용되기도 하고 동시에 사용된다. 따라서 망치와 플라이어는 다음에 이어질 행위에 따라 때론 망치가 더 잡기 쉬운 방식으로, 때론 플라이어가 더 잡기 쉬운 방식으로 거리를 두고 놓일 때, 막힘없이 못 박는 행위가 전개될 수 있다. 여기서 방향잡기란 일의 전후맥락 속에서 도구와 행위자 사이에 거리를 조절하는 행위를 가리킨다. 


익숙하지 않은 길에서 자전거 타는 법을 익히는 과정을 예로 들어보자. 평탄하고 고른 길을 자전거로 달릴 경우, 핸들을 잡는 방식과 페달을 밟는 방식과 같은 자전 거 타기의 기능들은 매우 익숙하고 편안하여 크게 의식하고 애써야 하는 고려대상이 아니다. 그런데 요철을 넘어야 하는 상황처럼 새로운 상황에 놓일 경우, 핸들을 느슨 하게 잡기도 하고 세게 잡기도 하며, 페달을 천천히 혹은 빠르게 밟기도 하면서 적 절하게 조절해가야만 한다. 또한 브레이크를 잡음과 동시에 요철을 넘어가면 펼쳐질 평탄한 길을 예측하고 브레이크를 서서히 풀며 페달을 힘차게 밟는 다음 행위를 도 모하기도 한다. 여기서 자전거를 타는데 필요한 기술이란 상황적 도구들을 때론 어 떤 것은 상황에 가깝게 끌어오고, 때론 조금 멀리 둠으로써 상황에 적절하게 거리를 조절한다는 것을 의미한다. 또한 다음에 펼쳐질 상황과 이전에 일어난 상황들 가운 데서 다음에 필요한 기능들을 활용하기 용이하도록 도구들의 활용 방향을 예측하고 준비시키는 과정이기도 하다. 


한편, 여기서 방향을 잡는다는 것은 잔재주나 잔기술과 같은 변형된 행위를 의미 함과 동시에 도구 자체가 갖는 의미를 확장해나간다는 의미를 함축한다. 좌측과 우 측이라는 방향이 존재한다는 것은 정방향이라는 그것의 기준이 존재할 때에만 가능 한 것이기 때문이다. 그리고 차이를 얼마만큼 알고 있는지는 결국 정방향이 어디인 지를 더 정확히 숙지하게 되었음을 의미하는 것이기 때문이다. 우리가 그 대상에 변 형된 다양한 행위를 가한다는 것은 대상 자체에 가하는 힘이라고 생각하기 쉽지만 사실은 그것을 둘러싼 맥락들을 조율하는 방식을 취한다. 이는 본인이 하고자 하는 일에 걸림돌이 되는 저항하는 요인들을 재배치하는 활동이기도 하다. 이 과정은 끊 임없이 모호함 속에서 해를 찾아가는 과정이기도 하여, 이 과정에서 겪는 실패의 경 험들은 다양한 상황에의 경험이며 맥락적 자원들을 활용해보는 교육적 경험이라 할 수 있다. 이러한 경험들은 모두 그 객체를 둘러싼 맥락의 정서를 풍부하게 해주는 의미들로 남게 된다. 시행착오의 과정에서 느낀 그 어떤 사소한 경험들도 의미 없는 순간은 없으며, 맥락적인 경험은 또 다른 실용적인 목적에 따라 동원될 수 있는 가 능성을 잠재한 자원으로 남는다. 


이러한 점에서 연습이란 완벽함에의 갈망 그리고 결코 성취될 수 없는 완벽함을 향한 긴장 가운데 수행되는 반복이자 동시에 부단히 쇄신하는 실천을 의미한다. 완 벽하지 못한 상태(imperfection)가 만들어내는 반복 연습과 변주는 그 상태로의 멈춤 을 의미하기도 하지만 때로는 자기 안에서 새로운 스타일의 창출로 이끌기도 한다. 국 지식이나 기술을 숙달하기 위해 임하는 연습은 맥락 간 차이와 동일성을 반복 적으로 경험하는 상황학습이다.


3.3. 경험의 공유는 어떻게 맥락 참여에 의존하는가: 실천으로서의 시각화 사례


가구뿐만 아니라 직접 조립하여 사용하도록 판매하는 물건들의 경우, 제조사는 소비자들이 조립을 돕기 위한 조립도를 함께 제공하기 마련이다. 특히, 최근 인터넷 을 통한 정보의 공유방식이 대중화되면서 조립도 이외에도 직접 조립하는 과정을 사 진으로 보여주거나 동영상 자료들을 올리는 방식으로 조립의 과정이나 전략들에 대 한 정보를 공유하고 전달할 수 있는 방법이 다양해졌다. 이러한 자료들은 대체로 언 어로 전달하는 것에 어려움이 있는 정보들을 시각화(visualization) 함으로써 사용자 가 보다 용이하게 제작과정을 따라할 수 있도록 한다. 그렇다면 이러한 시각화 자료 들은 전달하고자 하는 지식이나 기술에 대한 정보를 얼마만큼 정확하고 풍부하게 제 공할 수 있을까? 다음은 조립식 가구로 유명한 I회사의 가구조립도이다. 




위와 같은 가구 조립도에서는 순서대로 따라 하기만 하면 완성된 형태의 가구 조립이 가능하다고 설명한다. 조립도와 실제 조립과정은 얼마나 서로 매칭될 수 있 을까? 분명 조립도는 언어의 정보전달력이 갖는 한계를 보완하고 좀 더 구체적인 정 보를 제공하고 있음에는 틀림없다. 각 단계의 그림은 과정상 필요한 조작의 방식을 가급적 시각화하여 전달하는 것에 목적을 둔다. 그럼에도 불구하고 조립도의 그림이 무엇을 의미하는지 이를 따라 하고자 하는 사용자에게는 항상 애매모호함이 뒤따른 다. 위 조립도에서 각 그림이 보여주는 화살표의 방향이나 나사의 조립순서는 분명 치 않다. 


실제 조립과정에서 조립도에 나와 있는 그림들은 실제로 대상과 그림을 견주어 보고, 행위를 가하고, 조율하는 방식을 통해서 그 의미를 찾아가는 실천이 동반되어 있을 때에만 비로소 그 의미가 성취된다. 예컨대, 물건을 다루는데 필요한 정교함이 나 힘의 강도, 나사를 조이는 방향이나 각도 등은 사실 그림의 표현된 화살표만으로 는 정보가 부족하다. 그리고 그것을 다루는 개인들의 조작능력의 차이나 신체적 조 건의 차이들도 그 정보를 활용하고 해석하는데 차이를 빚어낸다. 물론 이전 경험이 나 배경지식들 역시 조립도 그림을 파악하는 데 차이를 가져온다. 다시 말해서, 어떤 기술이나 경험을 담는 이러한 시각 자료는 그 자체로 어떠한 정보를 전달해주지는 못한다. 사실상 각 단계의 시각화된 정보는 각 단계에서 필요로 하는 정보의 일부를 보여줄 뿐, 그것이 어떤 의미를 갖는지는 순전히 사용자의 역량에 달려있다. 시각자 료를 통해 정보나 기술이 어떻게 전달되는지 이해하기 위해서는 사용자가 어떻게 실 제 상황에서 무엇을 하는지 면밀히 살펴보아야 한다. 


시각화 자료가 정보를 전달하는 데 제한적일 수밖에 없는 이유는 사람들은 통상 현상이나 대상 사물을 있는 그대로 보지 않는다는 데에서 찾아볼 수 있다. 누군가가 무엇을 보는지는 그의 관점이나 의도 나아가 맥락에 따라 실용적으로 결정된다. 단 순하게 말하자면 사람들은 대상을 보고 싶은 대로 본다. 이러한 점에서 보면 시각화 자료가 이해나 경험의 변화로서 학습에 직접적으로 영향을 미친다는 것은 제한적인 범위 안에서 가능하다. 새로운 정보를 받아들인 학습의 결과 애초에 자신이 본 것과 는 다른 것을 보게 되기 위해서는 시지각의 작동만으로는 충분치 않다. 여기에는 자 신을 상황지우는 다른 지각행위가 동반되어야만 한다.


다음은 5세 어린아이가 새로운 글자를 배우는 상황에서 관찰한 내용이다.


<사례 2>

엄마 : (알파벳 'P'를 그리며) 이렇게 ‘1’을 쓰고.. 이렇게 ‘⊃’를 그리면 되는거야.

아이 : 알았어. 알았어. (‘1’을 그리고 거기에 ‘○’를 붙인 형태의 그림을 그린다)

엄마 : 아니, (‘1’을 그리고 우측 반원 형태의 모양을 반복적으로 그리면서) 이게 아니고 이거라고.

아이 : 알았다고. (다시 ‘1’을 그리고 거기에 ‘○’를 붙인 형태의 그림을 그린다.

엄마 그림과 비교하며 갸우뚱한 표정을 지으며 몇 개의 그림을 더 그려본다)

엄마 : 아니. 잘 봐. 동그라미까지 그리지 말고 여기서 멈추라고. 엄마 손잡아봐.(손을 잡고 ‘P'를 함께 그려준다.)

아이 : 아... 멈추라고? (다시 혼자서 반원을 주춤거리며 그려 보이며 엄마의 그림과 비교해본다)


위 상황은 본다는 것이 어떻게 대상을 있는 그대로 보기 보다는 자신에게 익숙 한 대로 이루어지는 일인지 잘 보여주는 예다. 위 상황에서 아이의 반응을 보면 아 이에게 알파벳 ‘P’는 기존에 자신에게 익숙한 숫자 ‘1’과 ‘동그라미’와의 조합으로 보 였을 것이라는 점을 유추해 볼 수 있다. ‘아마도’ 반원 형태의 표상을 한 번도 그려 본 적 없는 아이에게 그것은 인지하기조차 쉽지 않은 대상이다. 위 예에서 아이가 원으로 본 것을 반원으로 보기 시작하는 것은 엄마의 손에 안내를 받으며 자신의 손 으로 직접 반원을 그리면서부터다. 아이가 마침내 도달한 ‘P’의 형상은 이해를 바꿈 으로써 도달한 결과가 아니라 누군가와 호흡을 맞추며 직접 자신의 손을 움직여 차 이를 발견하는 실천의 산물이다. 즉, 손을 엄마와 함께 맞춰보면서 아이가 다른 것을 보게 된 것이다. 엄마와의 협력 행위가 어떻게 아이의 시각을 다르게 상황지우고 그 리고 이에 따라 아이가 다른 이해를 성취하게 되는지 잘 보여주는 예다. 이처럼 상 호작용의 행위는 주체와 객체 사이의 매 순간 적절한 이해의 가능성을 열어주는 마 중물을 제공한다. 


어떤 경험의 전달에는 시연뿐만 아니라 말이 관여하기도 한다. 말을 통한 경험의 전달은 일상 어디에서나 목격되는 가장 전형적인 예이다. 이 때 말은 경험의 내용을 전달하는 표상보다는 주의를 어디에 집중시켜야 할지 행위를 지시하거나 촉구하는 등 행위에 가깝다. 여기서 말은 추상적이고 문어체적인 원거리어(distant language) 보다는 구체적이고 대화체적이며 상황적인 근접어(proximal language) 속성을 띤다. 실천으로서 근접어를 사용하고 있다는 점을 잘 보여주는 대표적인 예는 맥락지시어 (indexicality)다. 맥락지시어의 활용은 우리의 사고와 행위가 얼마나 대상 맥락에 몰__ 입(immersion)되어 그와 구분하기 어렵게 전개되는지를 잘 보여주는 예다. 가장 간 단한 상황지시어는 ‘이’, ‘그’, ‘저’와 같은 형용사로 이러한 말들은 발화의 상황을 참 조해서만 그 의미를 알 수 있다. 상황학습의 문제의식 또한 학습에서 맥락지시어 문 제를 다루는 것으로부터 출발하였다. 


일상에서 맥락지시어와 같은 근접어의 사용은 매우 광범위하다. 화학 실험실의 한 상황에서 연구자가 조교에게 “이제 그 물 충분히 끓었어.”라고 말을 한 경우를 생 각해보자. 예컨대 이 말을 객관적인 표현으로 바꾸면 “한국 표준 시각 13시 57분 H2O가 섭씨 97.7도로 가열되었다.”라고 할 수 있다. 그러나 사람들은 그들의 생활공 간에서 혹은 일을 해 나가는 과정에서 실지로는 그러한 객관적인 표현은 사용하지는 않는다. 사람들은 구체적인 상황 안에 있는 사람들이라면 이미 맥락지시적인 표현으 로 충분히 의미를 통용할 수 있다는 점을 알고 있다. 맥락지시적인 표현들은 실제 상황에서 오히려 정확하고 객관적인 표현의 사용보다 안정성과 실용성을 보장해 준 다. 예컨대, 사람들은 일의 진행을 잠시 멈추고 맥락지시적인 표현들에 대해 객관적 으로 풀어 설명하거나 이것이 상대방에게 전달되었는지 확인하고 다시 진행되던 일 로 돌아오는 방식을 취하지 않는다. 만약 그런 방식으로 사람들이 말을 해야 한다면 매 상황 하고자 하는 일이나 나누고자 하는 대화는 매끄럽게 전개되지 못할 것이다. 또한 참여자들 사이에 맥락을 서로 공유했다고 당연시하고 따라서 근접어를 자유자 재로 사용하게 되면 다루고자 하는 대상에 집중하게 되는 주의의 초점이 좀 더 분명 해질 수 있다. 


구체적인 삶에 더욱 가까운 말이 구사되었을 때 그 말은 활성화된 기능을 갖게 되면서, 즉 더욱 상황에 대한 조작 가능성이 커지면서 하고자 하는 일을 성공적으로 이끈다. 즉, 삶에 더 가까운 언어가 공유된 지식이나 공감을 이끌어낼 수 있다고 볼 수 있다. 이러한 삶에 가까운 언어의 사용은 일의 흐름을 훨씬 더 원활하게 한다. 이 를 두고 세넷(Sennett, 2008)의 말을 빌려 표현하면, “표현 자체가 중요한 게 아니라 그렇게 상상을 동원하는 행위가 어떤 목적에 쓰이느냐 하는 것”이 중요하다(p. 309). 


세넷(Sennett, 2008)은 그의 저서 ‘장인’에서 손재주와 같은 기술의 숙달에 관해 상세히 설명하였다. 경험의 전달에서 근접어의 사용이 학습자로 하여금 어떻게 공감 을 불러일으키는지 세넷의 예를 좀 더 길게 인용해보겠다. 


은유와 같은 근접어, 삶에 가까운 말, 즉 상황지시적인 말들이 유사한 것을 동일 시하게 한다는 힘으로서의 상상력을 어떻게 유발시키는지(Lakoff & Johnson, 1980), 세넷(Sennett)은 자신의 경험을 예로 들어 설명한 바 있다. 고참을 따라 무심코 따라 하면서 배우는 행위에는 자신의 행위와 고참의 행위를 동일시시키는 상상력의 언어 가 동원된다는 것이다. 한 예가 유럽에서 건너온 영어에 서툰 할머니가 그녀의 전통 적인 요리법을 전달하고 그가 따라하는 상황에 관한 자신의 경험 이야기다. 그의 설 명은 조리법에 관한 정확한 서술(예컨대, 들어가는 양념의 양을 나타내는 수치, 몇 도의 불에 그 고기를 익히는 정도를 정확히 표현한 서술)보다 은유적인 표현들로 제 시된 닭요리 레시피가 이를 따라 요리를 배우는 참여자로 하여금 어떻게 상상을 일 으키고 따라서 좀 더 정확히 그 기술을 전달하는지 보여준다. 


“네 아이가 여기 죽어 있다[닭을 가리킴], 그 아이를 새 생명으로 준비시킨다 [뼈를 발라냄], 흙으로 그를 채운다[재료를 채워넣음], 조심하라! 그 아이가 너 무 많이 먹으면 안된다[재료를 가볍게 넣음], 금빛 외투를 입힌다[익히기 전 노 릇하게 그을림], 목욕을 시킨다[삶을 국물을 준비함]. 이제 아이를 데우는데, 주 의해야 한다! 어린아이는 햇볕을 너무 많이 쐬면 죽는다[가열온도는 섭씨 130 도]. 아이에게 보석을 달아준다[조리가 끝나면 부드러운 후추소스를 뿌림]. 이게 내 조리법이다.”(Sennett, 2008, 김홍식 역, p. 308) 


위 인용에서 보면 할머니의 요리 레시피는 비유와 그 비유가 지시하고자 하는 객관적인 표현([ ]안)으로 되어 있다. 세넷은 객관적이고 정확한 전달방식보다 비유 와 같은 근접어가 어떻게 경험을 더 정확히 전달할 수 있는지 다음과 같이 설명하고 있다.


공감하는 방식으로 이야기를 구성하다보면 비유가 많이 동원된다. 여기서의 비유는 정확한 비유가 아닌 느슨한 비유다. 비유가 느슨한 데는 이유가 있다. 닭의 힘줄을 자르는 것은 기술적으로 끈을 자르는 것과 비슷한데, 그렇다고 느 낌이 아주 비슷한 것도 아니다. 이렇게 느슨한 비유를 쓰면, 읽는 이들에게 배 움의 순간을 열어준다. ‘똑같다’가 아닌 ‘비슷하다’고 하면, 힘줄을 자르는 바로 그 행위에 신경을 집중하도록 유도한다. 조리할 사람의 손과 뇌가 활발히 교류 할 장이 열린다. 느슨한 비유는 정서적인 작용도 해서, 무슨 동작이나 행동을 처음으로 접하는 사람이 전에 해봤던 것과 비슷하다는 말을 들으면 자신감이 생긴다(Sennett, 2008, 김홍식 역, pp. 299).  


무엇인가를 설명할 때 대상을 추상적인 것으로 두는 것이 아니라 대상을 보고자 하는 것으로 보는 상상의 맥락을 조정한다. 일단 현상학적 장이 ‘만들어지면’ 상상이 가능해지고 결국 그에 따라 과정 전체를 볼 수 있게 된다. 즉 맥락을 공유가능하게 만들어 상상을 돕는다. 이렇게 근접어의 사용은 과정의 전체상을 먼저 상상하고 실 천을 행하는 순서는 참여자가 목표해야하는 것이나 앞으로 나아갈 과정을 미리 알게 하여, 실제로 행하게 되는 과정 속에서 목표지향적인 실천적 참여를 이끌어낼 수 있 다. 


처음 해보는 일에 대해 기존의 방법과 ‘똑같이’ 잘해야 한다고 생각하면 나의 행 위를 바라보고 있는 지시자의 눈치를 보게 되고 알 수 없는 기준에만 신경을 쓰게 된다. 결국 행위자가 처음 해보는 행위 그 자체에 집중하지 못하고 다른 외부적, 내 부적 상황에 신경을 쓰게 되고 일을 망치게 된다. 하지만 느슨한 비유를 통해 행위 자의 이전 경험과 연관을 시켜주면 그에 대해서 동일성을 느끼고 정서적으로 이전에 해봤다는 자신감을 가지고 불확실한 결과보다는 행위 그 자체에 집중을 할 수 있게 된다. 이를 느슨한 비유를 통해 처음 해보는 일에 대해 망설이게 되는 사고를 경험 적 동일성의 사고로 바꾸어주며 동시에 행위 그 자체로 집중을 이끌었다는 점에서 사고와 행위의 극간을 해소했다고 표현했다. 


활동이론(activity theory)에 따르면, 이야기는 하나의 사물과 같이 우리 행위의 대상이 된다. 이야기 또는 말도 하나의 객체이며 사물의 대상이 된다는 것이다. 말이 전유화 되기 이전에는 언어적 기능인 의미에 초점이 맞추어지곤 하는데, 전유화가 되고 난 후부터는 말도 사물처럼 객체화되어 사용된다는 것이다. 여기서 말 자체는 일을 가능하게 해주는 매개물이기도 하며 말 자체가 일이기도 하다. 또한 일을 하는 사람 사이에서 말은 목표에 어떻게 도달할 것인가를 위해서만 쓰이는 것이 아니라 그들의 정체성도 만들어 내며, 행위를 이끌거나 배제시키는 장치도 만들어 낸다. 여 기서 이러한 동의를 이끌어낼 수 있는 사물 장치(affordance)는 말이다. 


다음은 소크라테스와 사동이 서로 문답하는 방식으로 피타고라스 정리의 원리에 관해 가르치고 배우는 상황의 한 장면이다.


b40 소크라테스 : 이제 이 구석과 이 구석을 연결하는 선분을 그어 보자. 이러한 선분은 이들 각 정사각형을 반으로 쪼개겠지?

사동 : 예.

b41 소크라테스 : 이러한 선분을 4개 그으면 사각형이 하나(BEHD)생기겠지?

사동 : 그렇습니다.

b42 소크라테스 : 새로 생긴 이 사각형의 넓이는 얼마이겠니, 생각해 보렴.

사동 : 잘 모르겠습니다.

b43 소크라테스 : 정사각형이 4개 있었지. 각 선분은 각 정사각형을 반으로 쪼갰지?

사동 : 예.

(중략)

b49 소크라테스 : 이 4자 넓이 정사각형의 구석 이쪽에서 저쪽으로 그은 선말이냐?

사동 : 예.

b50 소크라테스 : 이러한 선분을 학자들은 ‘대각선’이라고 부르는데, 이 용어를사용하기로 하면, 이 대각선을 한 변으로 하는 정사각형이 처음 정사각형의 두 배가 된다는 것이 너의 생각이란 말이지?

사동 : 그렇습니다, 소크라테스. (강완, 1996에서 발췌)


위 대화에서 ‘이 구석과 이 구석을’, ‘정사각형을 반으로 쪼개겠지’, ‘이러한 반 조 각들이’, ‘정사각형의 구석 이쪽에서 저쪽으로’, ‘그 한 변은?-이것(DB)입니다’의 말들 은 한편으로 보면 수학적 원리에 관한 설명을 담고 있다. 다른 한편으로 이러한 설 명은 도형을 지시하거나 도형을 분할하는 가운데 병행되며 최소한의 지식에 익숙한 사람이라면 볼 수 있는 매우 상식적인 의미들로 구성된다. 즉, 교수자에 의해 유도된 지각의 구조에 따라 보고 확인하고 듣고 말하는 만큼 그 과정은 일상적이며 비매개 적 행위로 진행된다(조현영·손민호, 2015).


여기서 교수자가 말을 통해 상황을 시지각화하는 실제적인 전략을 참여적 전유 (participatory appropriation)로 개념화해볼 수 있다. 참여적 전유는 맥락에 참여함으 로써만 다루고자 하는 지식이 이해되고 활용되게 된다는 점에서 지식을 내면화함으 로써 그것을 다룰 수 있게 된다는 ‘내면화’(internalization)의 논리와는 상반된 의미를 담고 있다. 그것은 현장에서 다루는 정보들이나 지식들이 그 현장에 임베디드되어 있는 상황적인 말과 표현들(indexicality)들로 드러나기 때문에 그 맥락에의 참여를 통해 그 의미를 이야기할 수밖에 없다는 의미이기도 하다(Hall & Nemirovsky, 2012). 


지식을 전달하기 위해 동원된 이러한 전략들은 대개 말에 의해 구사된다는 점에서 담화적 실천(discursive practice)이라 할 수 있다. 담화적 실천이란 말이 표상의 역할 보다는 어떤 행위를 촉구하거나 지시하고 견제 조정하는 역할을 수행한다는 의미를 담고 있다. 수행적인 발화(performative utterance)의 기능은 ‘무엇을 중요한 것으로 보 아야 하는지’, ‘다음번에 비슷한 경우가 생겼을 때 무엇을 명심하면서 어떻게 동일시해 야 하는지’ 등의 수행적인 의미를 암묵적으로 전달해 준다. 


실천행위로서 말의 이러한 기능을 염두에 둔다면 위 예에서 지식이 전달되는 기 제는 시각화된 자료의 활용도, 지식을 전달해주는 말도 아닌 교수자와 학습자간에 질문과 대답을 주고받는 과정에서 유발된 지각의 구조화 과정이라고 할 수 있다. 교 수자의 말 하나 하나는 상황에의 지표 또는 지각행위에의 지시로 작용해 그 의미는 사용자 사이에서 너무도 자명해서 누구라도 접근 가능(learnable)한 일상의 통용어와 크게 다르지 않다. 어느 누구도 위 도형을 있는 그대로의 형상, 즉 객관적으로 볼 리 가 만무하며, 교사의 지시에 따라 돌출되어 지각되는 부분만을 선별적으로 구조화해 볼 것이다. 즉 학습자는 맥락적으로 지각을 이끄는 행위 유도의 상태, 즉 시청각의 상황 안에서 기대되는 것만을 보고 듣는 것이다. 


시각 자료를 통한 경험의 전달이 어떻게 상황학습, 즉 실천에 의한 학습 또는 공 유인지에 관해서 한 가지 예를 더 들어보겠다. 

    • 굿윈(Goodwin, 1994)은 고고학자들이 무에서 유를 어떻게 성립시켜 나가는지 그들의 작업을 미시적으로 분석한 바 있다. 고고학자들은 일반 사람 눈에는 절대 보이기 어렵다는 점에서 '맨땅'에서 뭔가를 발 견하여 선사의 유물과 자취를 찾아낸다. 마치 발견자의 실천이 관여될 여지조차 없 을 정도로 명명백백 취하는 경우 발견의 과정이지만 그 과정을 보면 그냥 발견이라 고 하기가 어려울 정도로 해석과 일 등 전문가들의 실천이 관여되는 작업의 과정이 다. 실제 객체로 놓여져 있는 사물이나 자취에 대한 발견이지만 전문가의 실천에 의 해 재'구성'되는 만큼만 실재화되는 작업이다. 그의 분석은 전문가의 시지각이 실제 적인 전략들을 통해 어떻게 대상을 운용가능 한 상태로 구성해내는가를 보여준다. 특히 그는 맥락을 공유가능하게 시각화하는 다음과 같은 실제적인 전략들에 관해 주 목한다. 


첫째, 코딩행위로서, 현상을 지식의 객체로 변환시키는 전략이다. 

둘째, 하이라이트하는 행위로서, 특정 측면들만을 부각시킴으로써 상황을 의도하 는 현상학적 장으로 전환되도록 하는 전략이다. 

셋째, 사물을 표상화시키는 전략인데, 이는 사물을 그 자체로 지식으로 지각하게 끔 표상화시켜 다루는 것을 말한다.2) 


이러한 전략들은 사물을 그 자체로 지식으로 지각하게끔 표상화시켜 조작할 수 있게 만드는 실천들이다. 이러한 전략들은 어떤 일을 해나가는 데 있어서 어디서나 흔히 사용되는 방법이다. 그것들은 지식을 자유자재로 부리기 위해 객체화하고 객체 화된 대상을 다루는 행위에 대해 작용할 수 있는 조건을 마련해주는 실제적인 전략 이라고 이해해도 무방할 것이다. 이러한 전략들이 지식이나 경험을 전달할 수 있도 록 해주는 것은 그것이 갖는 시각적 기능이 아닌 수행적(performative) 기능으로 인 해서다. 위와 같은 점에서 시각자료가 제 기능을 갖게 되는 것은 사물과 담화적 속 성에 기반해 있을 수밖에 없는 맥락적 실천을 통해서다.


4. 나가는 말


전통적인 학습연구에서는 맥락 변인들을 텍스트를 얻기 위한 주변적인 요인이라 고 간주하고 이를 체계적으로 배제시키고자 하였다. 

  • 행동주의 학습이론으로 잘 알려 진 파블로프 실험실에서 조건적 반사라는 일반화된 원리를 도출해내기 위해 실험을 둘러싼 일상에서 벌어진 것들은 어떻게 상황적인 변수들로 배제되었는지 상상해 볼 수 있다. 종을 울리면 ‘나오도록 되어 있는’ 실험실 개에서 나오는 타액의 양은 실제 연구실의 일상에서는 다양한 예기치 못한 변수로 인해 개의 컨디션, 실험 당시의 상 황 등이 지속적으로 관리될 필요가 있었다. 

  • 인지주의 학습이론으로 잘 알려진 피아 제의 보존실험에서 피실험자인 아이는 실험자의 유도된 질문에 맞춰 ‘적절하게’ 반응 을 보이기도 하였는데 그것은 연구 가설과는 연관 없는 주변적인 상황 변수로 처리 되었다. 


행동주의나 인지주의 패러다임에서 보면 조건적 반사 그리고 보존 개념의 획득 여부는 학습을 이해하는 데 중요한 열쇠라고 할 수 있다. 그러나 그 실험실의 일상 과 그 안에서의 실천이 그들이 자신들이 해야 할 일들을 어떻게 하고 있는지에 어떻 게 작용하고 있는지 또한 학습을 이해하는 데 중요한 열쇠일 수 있다. 유기체 또는 개인의 머릿속을 향해 있던 우리 관찰의 프레임을 그 외부의 맥락으로 돌려보면, 연 구실에서 지식의 생산에는 에이전트와 이에 관여하는 네트워크, 즉 연구실의 장치들 이를 조작하는 연구자의 노하우 그리고 이를 아우르는 일상이 은밀하게 관여하고 있 다는 점을 발견할 수 있다. 상황학습에 관한 관심은 당연시된(taken for granted) 상 황에 대한 이해를 바꾸는 데에서 출발한다. 


교육연구에서 상황주의 패러다임은 교육인류학적인 관심과 문제의식에서 촉발되 었다. 전통적으로 인지심리학을 기반으로 했던 학습 또는 학습자 환경의 디자인 영 역에서도 학습자 맥락에 대한 질적인 이해에 관심을 두기 시작하였다(Abrahamson, 2009; Kirsh, 2013; Nishizaka, 2006; Pea, 1997). 학습자의 경험을 이해하기 위해서 교수자의 관점에서 탈피하여 질적연구에서의 참여자 관점처럼 학습자 당사자의 관점 을 도입하기 시작하였다. 그리고 학습자의 실제 경험을 탐색하기 위해 맥락적인 연 구방법을 활용하기 시작하였다. 상황주의 학습에 관한 이해와 설계는 우리 경험의 맥락성 그리고 삶 안에서의 실천에 관한 좀 더 풍부한 이해를 요구한다. 


상황주의 패러다임에서 보면 경험과 학습이 성립되는 포인트는 텍스트에 있지 않고 사태의 전후관계, 즉 맥락에 있다. 우리가 어떤 일을 성취할 수 있는 것은 그 일에 동원되는 지식이 아닌 일의 선후관계 속에서 ‘다음’을 찾아나가는 ‘늘 현재’의 상황적인 추론 때문이다. 그러한 상황은 눈 깜빡할 사이에 지나치는 순간이지만 두 툼한 의미(thick description)로 되어 있다는 말이다. 어떤 일을 함에 있어서 일의 전 개도가 실제 경험의 과정을 잘 보여줄 것이라고 보는 것은 당사자의 관점이 고려되 지 않은 제3자적 관점에서 비롯된 생각이다. 당사자 즉 참여자 관점에서 보면 맥락 이란 현재와 다음이라는 상황의 연속적인 반복이다. 상황학습론은 일상의 실천에 있 어서 디테일한 부분들에 관해서 어떻게 미시적인 이해를 꾀할 것인가 앞으로의 탐구 과제를 제시해주고 있다.





교육인류학연구

2017, 20(1), pp. 25-58


맥락이 우리가 무엇을 어떻게 경험하고 학습하는가에 내재적으로 관여되어 있다는

점은 상황학습론을 통해 꾸준히 이야기되어 왔다. 교육학에서 상황주의 패러다임으

로의 전환은 교육인류학적인 문제의식에서 촉발되어 새로운 학습과학을 비롯한 학습

환경의 디자인 등에 이르기까지 적지 않은 영향을 미치고 있다. 본고에서는 교육에

서 맥락성의 회복은 경험의 상황에 대한 미시적인 이해가 뒷받침되어야 한다는 점을

짚어보고자 하였다. 즉, 발달과 학습이 일상적으로 편재한 생성적인 경험이라면 여기

에는 실제 경험에 대한 미시적이고 디테일한 분석이 요구된다. 이를 위한 한 가지

방법으로 본고에서는 경험의 과정을 사회적 실천 행위로 관찰, 해석하고자 하였다.

본고에서는 두 가지 전제, 즉 도구가 어떻게 상황으로 분산되어 우리의 참여적 전

유에 의해 비가시화되는지, 그리고 상황은 시간의 흐름에 따라 어떻게 다음 상황을

구조화하여 우리의 실천행위를 이끄는지 살펴보았다. 그리고 이에 터하여 경험 또는

암묵지가 어떻게 맥락적 실천의 일환으로 학습되는지 타자와의 상호작용, 개인의 연

습 그리고 시각자료의 활용 등의 사례를 통해 이해해 보고자 하였다. 경험의 학습

특히 암묵지에 관한 질적인 관찰과 분석을 어떻게 할 것인가 라는 문제의식의 일환

으로 정리하였다.

주요어: 실천, 상황학습, 실천공동체, 도구의 비가시성, 시각화, 암묵지, 경험


【Abstract】

How Experience is to be Situationally Learned?

Cho, Hyun-Young

Assistant professor, Inha University

Shon, Min-Ho

Professor, Inha University

The objective of this study is to outline the situated approach to learning, and to

explain how it contributes to our understanding of learning, and propose and

exemplify how this understanding informs the design of learning environments.

Analysis of how cognition makes use of phenomena distributed in everyday settings

and what theories it has contributed to situated learning.

This article investigates the discursive and material practices used by members of

a profession to shape events in the domain of professional scrutiny they focus their

attention to. The shaping process creates the objects of knowledge that become the

masterfulness of a professional work: knowledge, artifacts and bodies of expertise

that are competency in the communities of practice. Analysis of the methods used by

members of a community to build the work that structure their lifeworld contributes to

the development of a practice-based theory of learning. Demonstration of how

cognition is not located in the mind of a single individual, but instead embedded

within distributed systems including socially differentiated actors, and external

representations embodied in tools.

We argue that all cognition is grounded in bodily experience. Specifically, we

demonstrated case analyses that experience, including conceptual understandings, are

grounded in bodily experience. And if so, learning environments can be made more

effective if they are designed to attune to bodily know-how. Accordingly, instructional

design is committed to the hypothesis that the temporal sequence of situation could

be organized in terms of minimalism.

Key Words: practice, situated learning, communities of practice, visualization, tacit

knowledge, experience

보건전문직교육에서 효과적인 피드백 (Med Educ, 2010)

State of the science in health professional education: effective feedback

Julian C Archer






도입

INTRODUCTION


피드백은 인지, 기술 및 전문성 개발을 지원하는 데있어 핵심입니다. 효과적인 피드백은 긍정적이고 바람직한 발전을 촉진하기 위해 이전 퍼포먼스에 대한 정보가 사용되는 것으로 정의 될 수 있습니다. 

  • 인지이론가들은 피드백을 의도한 성과 수준보다 실제 퍼포먼스에 초점을 맞추는 것으로 본다.1 Recipient가 가지고 있는 지식과 필요로 하는 지식 수준 간의 차이를 강조하면 학습 촉매가됩니다 .2 

  • 행동주의자는 행동을 강화하거나 수정하는 방법으로 피드백을 생각합니다 . 예를 들어, 불확실성은 즐겁지 않은 것이고, 원하는 목표에서 멀어지게 만들기 때문에 불확실성을 줄이면, recipient의 성취도가 높아진다 .4 [4]. 

그러나 건강 관리 교육 문헌에서 피드백은 종종 이론적 근거가 없다.

Feedback is central to supporting cognitive, technical and professional development. Effective feedback may be defined as feedback in which information about previous performance is used to promote positive and desirable development. Cognitive theorists see feedback as focusing on actual performance compared with the intended performance level.1 Highlighting the gap between a recipient’s knowledge and the level of knowledge he or she needs provides a learning catalyst.2 Behaviourists conceive feedback as a way to reinforce or modify behaviour.3 For example, reducing uncertainty supports recipients’ achieve- ment4 as uncertainty is unpleasant and distracts from desired goals.5 Yet in the health care education literature, feedback is often devoid of any theoretical basis.6


의학교육에서 피드백은 문제가된다. 피드백을 주는 것은 피드백이 정직하고 정확하다는 것을 보장하면서, 교육자가 수령인의 심리 사회적 필요를 인정해야하기 때문에 어렵다 .7,8 전문 표준, 학생의 자존심, 그리고 환자의 권리와 안전 모두가 보호되어야한다. 이러한 균형을 유지하는 데 필요한 미묘함은 왜 교사가 정기적으로 피드백을 한다고 믿는 반면 학생들은 항상 그것을 인식하지는 않습니다를 설명해준다.9 피드백은 '효과적인 임상 교육의 초석'으로 묘사되었습니다 .10

Health care education feedback is problematic, Feedback provision is challenging as educators must acknowledge the psychosocial needs of the recipient while ensuring that feedback is both honest and accurate.7,8 The protection of... 

  • professional standards, 

  • the self-esteem of the student and, 

  • in health care education, the rights and safety of the patient, 

...must all be safeguarded. 


The subtleties required to maintain this balance may explain why teachers believe that they give feedback regularly but students do not always recognise it.9 Despite these challenges, feedback has been described as ‘the cornerstone of effective clinical teaching’.10




피드백 제공

THE PROVISION OF FEEDBACK


피드백의 유형

Type of feedback


피드백은 지시 또는 촉진 기능을 가진 것으로 설명됩니다. 

    • 지시적 피드백은 학습자에게 교정이 필요한 것을 알립니다. 

    • 촉진적 피드백에는 수령자가 자체적으로 수정하도록하는 의견 및 제안이 포함됩니다.

Feedback is described as having either a directive or a facilitative function. Directive feedback informs the learner of what requires correction. Facilitative feedback involves the provision of comments and suggestions to facilitate recipients in their own revision.


피드백은 또한 그 구체성 측면에서 다양 할 수있다. 

    • 구체적인 피드백초기 퍼포먼스 변화에 도움이되는 것으로 보입니다. 그러나 수령자가 더 많은 탐사를하지 못하게되어 장기간에 걸친 학습과 독립적인 수행을 훼손 할 수있다 .11 구체적인 피드백은 피드백을 준 업무의 성과를 뒷받침하지만 그 지식의 다른 업무로 tranfer되는 것에는 도움이 되지 않을 수있다 .12 

    • 덜 구체적인 피드백은 불확실성을 초래할 수 있으며, 이는 학습의 감소로 이어진다.

feedback can also vary in its specificity. Specific feedback appears to be beneficial for initial performance change. However, it may discourage recipients from further exploration and therefore undermine subsequent learning and independent performance in the longer term.11 Specific feedback may support performance in that task but not the transfer of knowledge to other tasks.12 Less specific feedback may lead to uncertainty, which in turn leads to a reduction in learning.2



피드백은 단순히 답이 옳은지 또는 틀린지를  나타내거나 (검증수령인이 정답 에 도달하는 것을 용이하게 할 수 있습니다(정교화). 정교한 피드백은 다섯 가지 유형으로 묘사됩니다 .13 이 중 응답-특이적 피드백은 학습자의 성취를 향상시키는 것으로 보입니다.

Feedback may simply indicate whether the answer is right or wrong (verification) or it may facilitate the recipient to reach the correct answer (elaboration). Elaborative feedback is described as being of five types.13 Of these, response-specific feed- back appears to enhance learner achievement.


원칙은 피드백이 구체적이어야 한다는 점이며, 다만 검증 및 정교화 피드백 모두 효과적 일 수 있습니다 .14 촉진적인 피드백은 높은 성취자를위한 학습을 ​​향상 시키지만 초보자에게는 그렇지 않을 수 있습니다.

Guiding principles are that feedback should be specific, but both verifying and elaborative feedback may be effective.14 Facilita- tive feedback enhances learning for high achievers, but may not do so for novices.15


피드백의 구조

Structure of feedback


피드백은 다양한 출처에서 나올 수도 있습니다. MSF (multi-source feedback)라고 알려진 체계적인 접근의 일부로 상호 작용이 발생하여 동료가 피드백을 줄 수 있습니다 .16 의료 전문가에게 고유 한 피드백의 추가 소스는 환자이다. 환자의 피드백은 보건 전문가의 성과 변화에 더 큰 영향을 줄 수 있지만, 그 유효성은 확실하지 않다. 환자의 피드백은 다른 소스의 피드백과 거의 상관 관계가 없습니다 .18


Feedback can also come from a variety of sources. It may be generated from colleagues as and when interactions take place or as part of a systematic approach, known as multi-source feedback (MSF).16 An additional source of feedback unique to the health professions is represented by patients. Patient feedback may be more influential in changing health professionals’ performance,17 but its validity is far from assured. Patient feedback rarely correlates well with feedback from other sources.18


모든 피드백은 훈련 된 촉진자가 뒷받침하는 환경에서, 직접 대면하여 제공되는 것이 이상적입니다 .19 

    • 서면 피드백은 중요하지만 부정적인 의견은 이것을 다시 긍정적인 결과를 만들어내는 지원을 필요로합니다 .21 이러한 서면 피드백에는 도전적인 정보가 포함될 수 있으며 길이와 복잡성으로 인해 더욱 복잡해진다. 

    • 복잡한 피드은 무시되거나 주요 메시지가 손실 될 수 있습니다. 스캐폴딩은 직접적인 지시뿐만 아니라 단서, 프롬프트, 힌트 및 부분 해법을 제공함으로써 잠재적인 미로의 복잡한 피드백을 통해 학습자를 안내하는 데 도움이 될 수 있습니다.

All feedback is ideally discussed face-to-face in a context that is supported by a trained facilitator.19 

    • Written feedback is important, but negative comments again require support to produce positive outcomes.21 Such facilitated written feedback may contain challenging information and this may be further complicated by its length and complexity. 

    • Complex feedback is likely to be ignored or its main messages lost. Scaffolding may help to guide learners through the potential maze of complex feedback by providing cues, prompts, hints and partial solutions, as well as direct instruction.22


피드백의 타이밍

Timing of feedback


피드백의 타이밍은 그것의 효과성에 독립적으로 영향을 줄 수있다 .13 효능과 타이밍이 과제의 초점과 난이도와 관련이 있음을 보여준다 23 

    • 지연된 피드백은 지식 transfer을 지원하는 것에 더 좋을 수 있는 반면, 

    • 즉각적인 피드백은 단기적이며 절차 적 기술의 개발을 지원하기위한 것입니다 .23 복잡한 업무를 수행하는 높은 성취도의 수혜자는 지연된 피드백으로 이익을 얻을 수 있습니다. 

Bangert-Drowns 등 [14]은 피드백이 mindfully 받아 들여지면 학습을 촉진 할 수 있다고 결론 지었다. 무의식적 인Mindless 피드백은 수령자가 생각할 시간을 가지기 전에 제공되는 것이며, 이는 과제가 너무 쉽거나 너무 복잡할 때, 프로세스가 무작위적이거나 일관성이 없을 대 일어난다.


The timing of feedback may independently influence its effectiveness.13 Evidence appears to suggest that efficacy and timing are related to the focus and difficulty of the task.23 Delayed feedback may be better for supporting transfer of knowledge, whereas immediate feedback may be more effective in the short-term and for supporting the development of procedural skills.23 High-achiev- ing recipients undertaking complex tasks may benefit from delayed feedback. It is hypothesised that learn- ers are supported by reducing interruptions that occur during the task.24 Bangert-Drowns et al.14 con- cluded that ‘feedback can promote learning if it is received mindfully’. Mindless feedback might include the provision of answers before the recipient has had time to think, when the challenge is too easy or too complex, or when the process is random or inconsistent.


피드백은 그것을 받는 사람이 응답하든 말든 수동적으로 전해지는 정보가 되어서는 안 된다. 개인이 초점입니다. 피드백은 도구modality이다. 그러므로 피드백 교환의 맥락에서 개인을 이해하는 것이 효과의 핵심입니다.

Feedback must not be seen as passive information passed on to an individual who either does or does not respond. The individual is the focus; the feedback is a modality. Understanding the individual in the context of the feedback exchange is therefore central to its effect.



피드백을 받는 사람 입장에서의 효과

THE INFLUENCE OF THE RECIPIENT


자기자신

The self


보건 전문가의 대부분의 연구는 다른 사람들의 견해와 관련하여 자기 자신의 강점과 약점을 식별 할 수있는 능력으로서 개인 성과 평가를 탐구한다 .25 그러나 이 접근법을 통해 자기 평가는 열악한 것으로 묘사되며, 현실을 보여주기보다는 문화나 젠더에 의해 영향을 받는다.

Most studies in the health professions explore the personal assessment of performance as an ability to identify one’s own strengths and weak- nesses in relation to other peoples’ views.25 However, with this approach self-assessment is described as poor,26 and as shaped by culture27 and gender28 rather than as representative of a shared reality.


사회 심리학자들은 우리의 행동과 행동이 우리의 무의식에 의해 알려지기 때문에 자기 평가에 결함이있는 것으로 간주합니다 .30이 무의식적 자기unconscious self는 자기 보존에 초점을 맞추고 있습니다. 이는 자존감을 위협하거나 무조건적인 칭찬을 하지 않는 피드백이 왜 덜 효과적 일 수 있는지 설명하는 데 도움이됩니다 .2 마찬가지로, 부정적인 피드백을받는 사람은 외부 요인을 탓하고 개인적인 책임을 거부합니다. 이를 기본 귀인 오류fundamental attribution error라고합니다.

Social psychologists view self-assessment as flawed because our behaviour and performance are informed by our unconscious minds.30 This unconscious self is focused on self-preservation. It helps explain why feedback that threatens self-esteem or contains noth- ing more than unconditional praise may be less effective.2 Similarly, recipients of negative feedback blame external factors and reject personal responsi- bility; this is known as fundamental attribution error.31


따라서 '정확한'자기 평가를 계속 추구하는 것은 부적절합니다. 우리는 신뢰할 수 있고 타당한 외부 자료를 대표하는 다른 사람들의 견해를 찾아야합니다 .32 개별화되고 내면화된 자기평가를 벗어나서 "자기주도적 평가 탐색"로 옮겨 갈 수 있도록 외부 피드백을 찾도록 동기 부여해야합니다. 33이 과정에서 피드백은 교육적 활동이되며, 수동적이지 않고 능동적입니다. 우리 자신이 아닌 우리의 능력을 학습함으로써 외부 피드백을 통해보다 나은 자기 모니터링을 할 수 있습니다. 자기 모니터링이란 능력에 대한 전반적인 인식에 의해 통제되기보다는 자신의 능력으로 형성된 상황에 대응할 수있는 능력을 말한다. Eva와 Regehr33은 '건강 전문직 공동체는 일반적으로 능력의 포괄적이고 광범위한 자체 평가의 정확성을 염려하기보다는, 행동을 하는 순간순간에 자기 모니터링 행동에 영향을 미치는 맥락적 요인을 파악하는 데 집중해야 한다'고 주장한다.

It is therefore inappropriate to continue to pursue ‘accurate’ self-assessment. We should each seek the views of others who represent reliable and valid external sources.32 Many individuals will need to be supported in their motivation to seek external feedback as they move from individualised, interna- lised self-assessment to self-directed assessment seeking.33 In this process, feedback becomes a pedagogic activity, which is active, not passive. By learning about our abilities, not ourselves, through external feedback we are then able to better self-monitor. Self-monitoring is the ability to respond to situations shaped by one’s own capability at that moment in that set of circumstances, rather than being governed by an overall perception of ability. Eva and Regehr33 argue that the ‘health professional com- munity should predominantly be concerned with identifying contextual factors that influence self- monitoring behaviours in the moment of action rather than worrying about the accuracy of generic and broader self-assessments of ability’.


그러나 외부에서 오는 피드백에 응답하는 능력은 여전히 ​​학습자의 영향을 받습니다 .34 높은 정서적 안정성을 가진 수신자는 동기 부여가 높고, 책임감이 높은 사람은 피드백을 사용해야 한다고 느끼고, 사회성이 높은 사람은 추가 피드백을 구할 수 있습니다.

However, the ability to respond to external feedback is still influenced by the learner.34 Recipients who have high emotional stability are more likely to be motivated, those with high levels of responsibility feel obligated to use feedback, and those with high sociability are more likely to seek additional feedback.35


이 시점에서 '성찰reflection'이라는 용어가 종종 논문에서 '자기 평가'와 상호 교환 적으로 사용되기 때문에 성찰의 역할을 강조 할 가치가있다. Boud et al. 성찰은 '개인이 새로운 경험과 이해를 이끌어 내기 위해 자신의 경험을 탐구하기 위해 참여하는 지적이고 정서적인 활동을 총칭하는 용어'라고 설명한다. 그러므로 성찰는 사건을 이해하는 데 중점을 둔 의식적이고 신중한 과정이며, 이를 통해 자기 개선을 가져오고 초보자의 지위에서 전문가의 지위로 옮기는 과정이다.37 이것은 분명히 중요하며 자기 모니터링의 일부가 될 수 있다. 이것이 직관적이긴 하나, 그러한 우리가 성찰을 통해 자신을 더 잘 이해하게 된다는 생각을 지지하는 증거는 거의 없다. 

At this juncture it is worth highlighting the role of reflection as the term ‘reflection’ is often used interchangeably with ‘self-assessment’ in the litera- ture. Boud et al. describe reflection as ‘a generic term for those intellectual and affective activities in which individuals engage to explore their experiences in order to lead to a new understanding and apprecia- tion’.36 Reflection is therefore a conscious and deliberate process that focuses on understanding events and processes to bring about self-improvement and to move from the status of novice to that of expert.37 This is clearly important and may be part of self-monitoring,29 but although it is intuitive, there is little evidence to support the idea that by reflecting we come to understand ourselves better,37 as is the preoccupation of the self-assessment literature.




수용도

Acceptability


자신의 능력에 대해 새로운 것을 배우는 것이 없거나, 가치가 없는 출처에서 피드백을 받는 것처럼 보이면 피드백의 효과가 손상 될 수 있습니다. 피드백의 정확성이 입증 될 수 있다면 목표 설정에 따라 수용 가능성이 높아진다.

Appear- ing to learn nothing new about one’s abilities or receiving feedback from sources that one does not value undermines the effectiveness of the feedback. Inconsistent feedback may have a negative effect by making the feedback appear inaccurate.38 Accept- ability is increased if the relevance of the feedback can be demonstrated, namely by goal setting.



목표 설정

Goal setting


목표 설정은 수용 가능성을 뒷받침 할 수 있으므로 관련성을 보여줌으로써 피드백의 영향을받을 수 있습니다 .2 목표는 개인적으로 의미 있고 쉽게 생성되어야합니다. 통제 이론은 개인으로서 우리의 행동을 목표와 표준에 맞추려고 노력한다고 주장한다 .39 따라서 통제이론에 따르면 우리는 우리의 입지를 향상시키기 위해 행동을 수정하고 외부 피드백을 포함하여 훈련을 찾거나 시도하려고 노력한다 .39 목표는 학습 지향 또는 성과 지향적일 수있다. 

    • 학습 지향은 새로운 기술을 개발하고 정보가 전성이 있다는 믿음으로 새로운 상황을 마스터하고자하는 열망과 관련이 있습니다. 

    • 성과 지향은 다른 사람들에게 능력을 보여줘서 긍정적으로 평가받고 싶어하는 열망, 그리고 지성이 타고난 것이라고 믿는 열망을 반영합니다. 

Goal setting can support the acceptability and there- fore the impact of feedback by demonstrating its relevance.2 Goals should be personally meaningful and easily generated. Control theory argues that as individuals we try to match our behaviour to goals and standards.39 We therefore reflect and then try to modify behaviour or seek training, including external feedback, in order to improve our standing.39 Goals may be learning- or performance-orientated. 

    • Learn- ing orientation is associated with a desire to develop new skills and master new situations in the belief that intelligence is malleable. 

    • Performance orientation reflects a desire to demonstrate competence to others and to be positively evaluated by them in the belief that intelligence is innate. 


두 가지 유형의 목표 지향성은받는 사람이 작업의 어려움과 실패에 어떻게 반응하는지에 영향을줍니다. 

    • 학습 지향적 인 사람들은 실패에도 불구하고 계속해서 더 복잡한 학습 전략을 사용하고 도전을 추구합니다. 

    • 성과 지향적 인 사람들은 성공을 거의하지 않을 수있는 어렵고 도전적인 과제에 대해 더 쉽게 포기하고 덜 관심을 가지는 경향이 특징입니다. 

예상할 수 있듯이 학습 오리엔테이션이 긍정적인 결과와 관련이있다. 피드백은 수령자가 학습 지향적 인 자세를 취할 수 있도록 지원하는 한 가지 방법이다.


The two types of goal orientation influence how recipients respond to task difficulty and failure. 

    • Those who are learning-orien- tated continue despite failure, use more complex learning strategies and pursue challenges. 

    • Those who are performance-orientated are characterised by a tendency to give up more easily and have less interest in difficult, challenging tasks in which success is less likely. 

As might be expected, learning orientation is associated with more positive outcomes.13 Feedback is one way of supporting recipients to become more learning-orientated.




피드백의 효과

THE IMPACT OF FEEDBACK


개인

The individual


벨로 스키 (Veloski) 등 40)은 평가, 피드백 및 의사의 임상 성적을 조사한 Best Evidence Medical Education review를 출간했다. 그들은 피드백이 신뢰할 수있는 출처에서 체계적으로 전달 될 때 임상 적 성과를 바꿀 수 있다고 결론지었습니다. 이 연구에 포함되지 않은 다중 소스 피드백은 현재 실질적인 성능 변화를 지원할 수 있다는 확실한 증거가 부족하다.

Veloski et al.40 published a Best Evidence Medical Education review looking at assessment, feedback and doctors’ clinical performance. They concluded that feedback can change clinical performance when it is systematically delivered from credible sources. Multi-source feedback, which was not included in this study, currently lacks robust evidence that it can support substantive performance change.41


조직

The organisation


피드백은 일반적으로 개인에게만 초점을 맞추지 만, 피드백 제공이 조직 내의 문화적 변화의 일부인 경우 교육 프로그램에 대한 정보를 비롯한 기타 이점이있을 수 있습니다. 지지적 피드백은 직원의 사기와 유지에 도움이 될 수 있습니다.

Feedback is normally and understandably focused on the individual, but if the provision of feedback is part of a cultural change within an organisation, it may have other benefits, including the informing of training programmes. Overall sup- portive and supported feedback may be beneficial to staff morale and retention.43



부정적 효과

Negative impact


우리가 보았 듯이, 피드백 제공이 반드시 성과에 긍정적 인 변화를 가져 오는 것은 아닙니다 .2 실제로 부정적인 피드백을 받으면받는 사람이 여러 가지로 피드백을 쓸모없고, 부담스럽고, 비판적이거나 통제하는 것으로 믿을 수 있습니다. , Sargeant, 45)는 부정적인 피드백을받은 사람들은 종종 부정적인 반응을 보였고 그와 같은 상황에서는 피드백이 마비 상태임을 보여 주었다. 이러한 반응은 종종 강력하고 오래 지속되었습니다.

As we have seen, the provision of feedback does not necessarily result in a positive change in performance.2 Indeed, receiving negative feedback may lead the recipient to believe variously that the feedback is useless, burdensome, critical or con- trolling.44 For example, Sargeant,45 found that those who received negative feedback often responded negatively and in such away that demonstrated the feedback to be paralysing. The response was often strong and long-lasting. 



실용적 모델

THE PRACTICAL MODELS


  • '피드백 샌드위치'의 중심에 Personal preservation이 있습니다. 촉진자는 긍정적 인 것 사이에 더 중요한 피드백을 끼워 넣습니다. 이 모델은 인식 된 불의의 균형을 맞추거나 피함으로써 수령인과 공급자를 보호합니다. 그러나 이것은 정당한 비판에 대한 수령인의 잠재적 인 긍정적 반응을 감소시킬 가능성이있다. 촉진자는 피드백과 심리 사회적 요구 사이의 균형을 맞춤으로써 상호 정의를 보장합니다 .46 

  • 펜들턴의 규칙은 수령인이 피드백에 먼저 응답 할 수 있도록 더 양방향 프로세스를 포함하도록 피드백 샌드위치를 ​​만듭니다. 

  • 감정적 인 반응을 수용하는 것이 점점 중요 해지고 있습니다. ECO 모델은 결과Outcome를 확립하기 위해 내용Content을 탐색하기 전에 감정적인Emotional 반응을 조장합니다.

Personal preservation is at the centre of the ‘feedback sandwich’. Facilitators deliver the more critical feedback ‘sandwiched’ between the positive. This model shields both recipient and provider by balancing or avoiding perceived injustice. However, this is likely to reduce any potential positive response by the recipient to justifiable criticism. The facilitator balances feedback with the psychosocial needs of the recipient, ensuring interactional justice.46 Pendleton’s rules47 build on the feedback sandwich to include a more two-way process which allows the recipient to respond to feedback first. Accommodating an emotional response is increasingly recognised as important; the ECO model encourages an Emotional response prior to exploring Content in order to establish Outcomes.48



피드백의 새로운 모델: 문화와 연속체

A NEW MODEL FOR FEEDBACK: CULTURE AND CONTINUUM


모델의 문제점

  • 이러한 실용적인 모델은 환원주의적 접근 방식으로 제한됩니다. 그것들은 의료계의 실증주의적 패러다임의 위계적, '진단적'렌즈에 내장되어있다. 새로운 모델에는 양방향 상호 작용의 중요성을 인정하지만, 

  • 피드백은 교육자 중심의 단방향 프로세스로 남아 있어서, 전문가가 초보자를 지원한다는 고전적 위계 모델에 의해 제약을받습니다. 

  • 피드백은 또한 개인이 완치되기 위한 '문제'를 진단하는 것에 초점을 맞추었지만 치료법은 거의 제공되지 않습니다. 

  • 피드백을 제공하는 것은 종종 사후 고려 사항으로 간주되는데, 시간이 제약된 바쁜 임상상황에서 간과되는 경우가 많습니다.

These practical models are limited by their reduc- tionist approach. They remain embedded in the hierarchical, ‘diagnostic’ lens of the medical profession’s positivistic paradigm. There is an acknowledgement in later models of the impor- tance of two-way interactions, but feedback remains an educator-driven, one-way process. It is constrained by the classic medical hierarchical model, in which the expert supports the novice. Feedback has also focused on diagnosing ‘problems’ for individuals to have cured, yet rarely is a therapy offered. Feedback provision is often seen as an afterthought, which is then frequently overlooked in busy, time-constrained clinical settings.


피드백은 복잡하고 맥락적입니다. 대부분의 최신 모델은 복잡성을 제거 할뿐만 아니라 맥락을 제거합니다. 좋은 연습 프레임 워크는받는 사람의 컨텍스트, 포커스 및 형식에 따라 적용 할 수 있는 방식 일 수 있습니다.

Feedback is complex and contextual. As well as removing complexity, most current models remove context. A good practice framework, may be a more robust approach that can then be applied depending on the context of the recipient, focus and format.


피드백은 개인이 아닌 업무에 초점을 맞추어야합니다 .49 전반적인 피드백은 구체적이어야하며 15 높은 성취자는 단순한 검증verification 피드백 (정확하거나 그렇지 않은 것)만큼 이익을 얻을 수 있습니다. 그것은 개인 목표에 직접적으로 연결되어야합니다. 피드백은 자존심을 저해해서는 안되지만 단순히 칭찬으로 만 구성되어서는 안됩니다 .2 피드백 전달은 구체적인 사건에서 시간적으로 지연 될 수 있지만, 피드백이 근거하는 정보가 후향적으로 기록되어서는 안 된다. 동기부여된 수신자는 외부 소스로부터의 피드백을 쉽게 받아 들일 수 있습니다.

Feedback should focus on the task, not on the individual.49 Overall feedback should be specific15 and high achievers may benefit as much from simple verification feedback (correct or not). It should be directly linked to personal goals. Feedback should not undermine self-esteem, but should not simply consist of praise.2 Its delivery can be delayed from the event to which it refers but the information that informs the feedback should not be recorded retrospectively. Motivated recipients benefit from challenging facilitated feedback from external sources.



피드백의 문화

A CULTURE OF FEEDBACK


효과적인 피드백을 위해서는 수혜자의 reflection in action을 키워야한다 .50 이것은 외부 피드백에 의해 정보가 제공되는 자체 모니터링을 기반으로 구축되어야한다. 외부 피드백에 의해 정보가 제공되는 자가 모니터링에 대한 재 개념화는 pedagogical and external-looking.39이다.

To achieve effective feedback, the health professions must nurture recipient reflection-in-action.50 This needs to be built on self-monitoring informed by external feedback. A re-conceptualisation of self- monitoring informed by external feedback is pedagogical and outward-looking.33


이러한 통합된 방식을 달성하려면, 모든 활동에서 피드백이 암시적, 명시적으로 포함되고 학생들이 교사뿐만 아니라 학생에게 피드백을 피드백 문화로 발전시켜야 합니다. 조기 교육과 동료 피드백과 같은 경험은 시간이 지남에 따라 필요한 문화적 변화를 지원할 수 있습니다.

The way to achieve such an integrated approach must be to develop a feedback culture, in which feedback is embedded implicitly and explicitly in all activities and in which students feed back to teachers as well as teachers to students. Early training and experience such as with peer feedback may over time support the required cultural change.


이미 놓쳐서는 안되는 피드백을 제공 할 수있는 많은 기회가 있습니다. summative와 formative 피드백 사이의 인공적인 이분법은 모든 평가에서 잠재적으로 강력한 피드백을 생성하는 것을 방해합니다. 예를 들어, 우리는 대다수의 지분을 가진 시험에서, 특히 성공한 사람들에게 피드백을 거의주지 않습니다. 환자의 죽음이 항상 환자의 치료를 끝내는 것은 아닙니다. 사후 부검은 현대 서구 사회에서 인기가 없을 수도 있지만, 적절하다면 환자의 보살핌에서 중요한 마지막 단계로 항상 의료계에서 보아왔다. 졸업시험과 같은 고부담 시험에서도 학습자에게 피드백을 제공해야합니다. 사후 부검처럼 평가되는 평가는 학습자의 진전뿐만 아니라 교육에도 도움이되는 기회입니다. 그렇지 않으면 평가는 평생 학습의 연속에서 분리됩니다.

There are already many opportunities to provide feedback which should not be missed. The artificial dichotomy between summative and formative feedback often distracts from generating poten- tially powerful feedback from all assessment. For example, we often give little feedback from high- stakes written examinations, especially to those who are successful. The death of a patient does not always end his or her care. The post-mortem examination may be unpopular in modern western society, but, when appropriate, it has always been seen by the medical profession as an important last stage in the care of the patient. High-stakes assessment even for an exit examination should provide feedback to the learner. Assessment, viewed as if it were a post-mortem examination, represents an opportunity to benefit learners’ education, not only their progression. Assessment otherwise is disassociated from the continuum of lifelong learning.




피드백 연속체

THE FEEDBACK CONTINUUM


최근 Cochrane review51의 저자는 감사 및 성과 피드백가 전문적 행위을 개선하는 데 효과적 일 수 있지만 효과는 일반적으로 작다고 결론지었습니다. 따라서 촉진은 잠재적으로 피해를주는 부정적인 피드백을 얻을 수 있고 긍정적 인 결과를 창출하는 데 활용할 수 있다는 점에서 피드백 성공의 핵심이다. 피드백은 일련의 관련이 없는 사건이 아니라, 지원되는 순차적인 과정으로 개념화되어야한다. 이 지속적인 접근방식 만이 모든 효과를 극대화 할 것입니다.


The authors of a recent Cochrane review51 concluded that the impact on doctors of audit and performance feedback data could be effective in improving pro- fessional practice but that the effects were generally small. Facilitation is therefore central to feedback success in that it can take potentially damaging negative feedback2 and use it to create positive outcomes.21 Feedback must be con- ceptualised as a supported sequential process rather than a series of unrelated events. Only this sustained approach will maximise any effect.


이를 위해서는 확고하고 장기적인 전문직 관계가 필요합니다. Apprenticehip 모델은 더 이상 많은 임상 환경에서 지속 가능하지 않음에도, 대부분의 보건 전문 교육 프로그램은 개인을 지원하기위한 감독 형태를 점점 더 중요시하고있다. 관리자는 개인의 프로필을 제공하기 위해 피드백 소스를 대조하는 것이 좋습니다. 다양한 출처의 프로필은 발전과 의사 결정에 필요한 퍼포먼스에 대한 조직적 기억을 제공합니다. 개인에게는 도전적이지만 위협적이지 않은 퍼실리테이션에 의해 뒷받침되는 성찰에 필요한 자료이다.

This requires established, longer-term professional relationships. An appren- ticeship model is no longer sustainable in many clinical settings,52 but most health professional edu- cation programmes increasingly advocate forms of supervision for supporting individuals. Supervisors appear to be well placed to collate sources of feedback to provide a profile of and for an individual. This profile from many different sources provides an organisational memory of performance for development and, when required, decisions. More importantly for the individual, it becomes a resource to inform reflection supported by challenging but non-threat- ening facilitation.




37 Mann K, Gordon J, McLeod A. Reflection and reflective practice in health professions education: a systematic review. Adv Health Sci Educ 2009;14:595–621.



 2010 Jan;44(1):101-8. doi: 10.1111/j.1365-2923.2009.03546.x.

State of the science in health professional educationeffective feedback.

Author information

1
Institute of Clinical Education, Peninsula College of Medicine and Dentistry, University of Plymouth, Plymouth, UK. julian.archer@pms.ac.uk

Abstract

BACKGROUND:

Effective feedback may be defined as feedback in which information about previous performance is used to promote positive and desirable development. This can be challenging as educators must acknowledge the psychosocial needs of the recipient while ensuring that feedback is both honest and accurate. Current feedback models remain reductionist in their approach. They are embedded in the hierarchical, diagnostic endeavours of the health professions. Even when it acknowledges the importance of two-way interactions, feedback often remains an educator-driven, one-way process.

LESSONS FROM THE LITERATURE:

An understanding of the various types of feedback and an ability to actively seek an appropriate approach may support feedback effectiveness. Facilitative rather than directive feedback enhances learning for high achievers. High-achieving recipients undertaking complex tasks may benefit from delayed feedback. It is hypothesised that such learners are supported by reducing interruptions during the task. If we accept that medical students and doctors are high achievers, we can draw on some guiding principles from a complex and rarely conclusive literature. Feedback should focus on the task rather than the individual and should be specific. It should be directly linked to personal goals. Self-assessment as a means to identify personal learning requirements has no theoretical basis. Motivated recipients benefit from challenging facilitated feedback from external sources.

A NEW MODEL:

To achieve truly effective feedback, the health professions must nurture recipient reflection-in-action. This builds on self-monitoring informed by external feedback. An integrated approach must be developed to support a feedback culture. Early training and experience such as peer feedback may over time support the required cultural change. Opportunities to provide feedback must not be missed, including those to impart potentially powerful feedback from high-stakes assessments. Feedback must be conceptualised as a supported sequential process rather than a series of unrelated events. Only this sustained approach will maximise any effect.

Comment in

PMID:
 
20078761
 
DOI:
 
10.1111/j.1365-2923.2009.03546.x


개인주의를 넘어서: 전문직 문화가 피드백에 미치는 영향(Med Educ, 2013)

Beyond individualism: professional culture and its influence on feedback

Christopher Watling,1 Erik Driessen,2 Cees P M van der Vleuten,2 Meredith Vanstone3 & Lorelei Lingard4






도입

INTRODUCTION


교사로부터 받는 피드백은 교육 과정의 핵심 부분이어야합니다. Kluger와 DeNisi는 다양한 학습 환경에서 연구된 피드백에 대한 메타 분석에서 피드백이 전반적인 성과에 약간의 유익한 영향을 미쳤지만 1/3 정도에서는 실제로 해가된다는 것을 발견했습니다. 피드백은 특히 자존심을 위협하는 것으로 인식 될 때 해로울 수 있습니다.

Feedback from teachers to learners should be a vital part of any educational process. Kluger and DeNisi,1 in a meta-analysis of feedback interventions studied across a range of learning contexts, found that although feedback had a modestly beneficial effect on performance overall, it was actually harm- ful to performance about one third of the time. Feedback was especially likely to be harmful when it was perceived as threatening to self-esteem.


많은 저자들은 개인의 피드백에 대한 개인의 인식이 어떻게 그 피드백을 다루는 지, 그것이 행동이나 성과에 영향을 주는지 여부에 대해 조사했습니다. 직원 성과 평가에 대한 산업 심리학 연구는 프로세스의 공정성, 정확성 및 합법성에 대한 직원의 인식이 효과에 큰 영향을 미칠 수 있음을 보여주었습니다 .2-4 이 연구는 놀랍게도 수행이 부적절하여 피드백이 가장 필요한 직원이 가장 덜 수용적임을 보여주었다
A number of authors have explored how individu- als’ perceptions of the feedback they receive shape how they handle that feedback, and whether or not it influences behaviour or performance. Industrial psychology research on employee performance appraisals has shown that employees’ perceptions of the fairness, accuracy and legitimacy of the process can significantly influence its impact.2–4 Soberingly, this work concluded that the employees who most need feedback because of performance inadequa- cies may be least receptive to it.5 

의학 교육도 피드백에 대한 학습자 인식의 영향을 깨달았다. Sargeant 등은 다중 소스 피드백을 받으면 가정의를 연구하여, 피드백이 부정적이거나 자기 평가와 충돌하는 경우에 피드백을 잘 수용하지 않았으며, 오히려 그러한 피드백을 만들어낸 프로세스에 문제가 있다고 인식했습니다 .9 마찬가지로 의학 학습자는 피드백이 믿을만하지 않다고 판단하면 이를 무시합니다.8
Medical education, too, has recognised the impact of learners’ percep- tions of feedback processes.6–9 Sargeant et al., study- ing family doctors in receipt of multi-source feed- back, showed that those who received feedback that was negative and in conflict with their self-assess- ment tended to be unreceptive to it and, instead, perceived the process that generated the feedback as flawed.9 Similarly, our own work demonstrated that medical learners might discard feedback if they judge it to lack credibility.8

피드백에 대한 개인의 반응에 관한이 연구에서 아직 개인이 피드백을 다루는 방식에 영향을 미칠 수있는 학습 환경 내의 사회적 및 문화적 요인에 대한 조사를 부족하다. 학습과 피드백 교환은 진공상태에서 일어나지 않습니다. 오히려 그들은 특정 환경, 상황 및 전문 문화에서 발생합니다. Shulman은 전문직 종사자들이 자신의 지식을 교육하는 과정을 이해하기 위해 서명 교육학signature pedagogies의 개념을 제안했습니다 .10 이러한 서명 교육은 자신이 포함하는 규칙, 책임 및 구조와 함께 한 전문직에서 지식으로 간주되는 요소를 정의하고 효율성을 높일 수 있습니다. 그러나 이러한 학습의 취약점은 제한된 범위의 학습으로 광범위한 학습을 ​​강요하여 배운 것을 왜곡시킬 수 있다는 것입니다 .10 궁극적으로 signature pedagogies은 선택을 수반하며, 이러한 선택은 필연적으로 특정 결과는 전문가로서 준비과정의 다른 잠재적으로 중요한 측면을 다루지 못하게 된다.10 이러한 문화마다 독특한 교육적 선택이 피드백을 관리하는 방법에 미치는 영향은 조사되지 않았다.

Missing from this growing literature on individual responsiveness to feedback is an examination of the social and cultural factors within the learning envi- ronment that may influence how feedback is han- dled. Learning and the exchange of feedback do not occur in a vacuum of individualism. Rather, they occur in a specific setting, context and profes- sional culture. Shulman has proposed the notion of signature pedagogies to understand the processes by which professions educate their own.10 These signa- ture pedagogies, with the rules, responsibilities and structures they contain, define what counts as knowledge in a profession and can promote effi- cient learning.10 The vulnerability of signature pedagogies is that they may force a wide range of learning into a limited range of teaching, thus dis- torting what is learned.10 Ultimately, signature peda- gogies involve choices, and these choices inevitably support certain outcomes while failing to address other potentially important aspects of professional preparation.10 The impact of these culturally dis- tinct pedagogical choices on how feedback is han- dled has not been examined.


학습을 위한 피드백의 힘을 이용하려면 학습자가 어떻게 반응하고 왜 반응하는지 또는 왜 실패하는지 이해해야합니다. 개개인 학습자를 분석 단위로 삼는 학습에 대한 연구는 "학습은 순수한 내적인지 과정"이라고 가정하는데 문제가 있다. 개별 학습자의 심리학 내에서 존재하는 피드백 반응에 대한 영향은 중요하지만 그 자체로는 설명이 충분하지 않습니다. 이 연구에서는 의학, 교육 및 음악의 세 가지 고유 한 전문 문화에서 피드백을 탐구했습니다.

To harness feedback’s power to shape learning, we must understand how and why learners respond or fail to respond. Studies of learning that take the individual learner as the unit of analysis make an inherently problematic assumption that learning is a purely internal cognitive process.11 The influences on feedback responses that exist within the psychol- ogy of individual learners, although important, are only part of the puzzle and by themselves offer insufficient explanations. In this study, we explored feedback in the three distinct professional cultures of medicine, teaching and music.





방법

METHODS


For this exploratory study, we used a constructivist grounded theory approach, in which the vantage points of participants and researchers alike are accounted for as data are interpreted.12 In the constructivist paradigm, particular attention is paid to reflexivity.13 Researchers must not only reflect on their own backgrounds and how these influence their approach to the subject, but must also share these reflections with readers in order to provide a meaningful interpretation of the work. The lead author (CW) is a doctor; his collaborators represent a range of non-medical disciplines including educa- tion (ED), psychology and psychometrics (CvdV), qualitative health research (MV) and rhetoric (LL). Two of the researchers (CW and LL) have signifi- cant training in music. Although all of the research- ers would consider teaching to be part of their professional identities, none has undergone the type of teacher training that supports the culture we studied.


  • Focus groups were our primary vehicle for data col- lection as we anticipated that the interactions among participants might be usefully revealing of culture. 

  • In addition, we interviewed key informants within music and education in advance of focus groups to obtain necessary background information about how learning was structured in these fields and to understand the language of training. Background interviews were felt to be unnecessary in medicine, given the lead author’s first-hand experi- ence of this training culture and his co-authors’ experience in studying this culture. 

  • Finally, we conducted additional individual interviews later in the research process to elaborate early focus group findings.


Our sampling strategy for both focus groups and interviews was purposive, with all participants recruited from one Canadian university. 

  • For focus groups in medicine, we recruited both residents from a range of specialties and senior medical stu- dents, anticipating that the insights offered might vary depending on the level of training of the learners and the extent to which they had become part of the professional culture. 

  • For focus groups in the context of music, we recruited undergradu- ate students. Because music students need to have a significant background in music in order to be accepted into an undergraduate programme, we reasoned that undergraduate students would be suf- ficiently acquainted with the professional culture of music to inform our research. 

  • Teacher training at our university involves a 1-year programme, entry to which demands the completion of at least an undergraduate degree. Because we wanted to focus on the experience of the ‘practicum’ method of training, in which students go into schools to obtain real teaching experience, we recruited from students on this 1-year programme. 

Initial recruit- ment was by an e-mail invitation sent to all senior medical students, residents, undergraduate music students and teacher training students at this university.



In total, data were derived from 50 participants. 

  • A total of 41 learners (eight medical students, 12 res- idents, 13 music students and eight student teach- ers) participated in 12 focus groups (two with medical students, three with residents, four with music students and three with student teachers). 

  • We conducted three background key informant interviews, including one with a music professor and two with education professors with experience as both teachers and educators of teachers. 

  • We encountered unexpected challenges in recruiting student teachers to focus groups and felt that our data required enrichment beyond the three focus groups conducted; therefore, we conducted individual interviews with three recent graduates of the teacher training programme and one doctor who had completed teacher training and had worked as a teacher prior to medical training. 

  • Finally, we interviewed two doctors with extensive training and professional experience in music to provide additional perspectives. The study received approval from the university’s research ethics board and all participants provided informed consent.


Focus groups and interviews were semi-structured, eliciting discussion and elaboration of the experi- ence of learning and receiving feedback within each professional culture. 

  1. Focus group discussions and interviews were recorded and transcribed verbatim without identifying information. 

  2. Data were analysed using the constant comparative approach customary in grounded theory.13 Analysis occurred alongside and informed data collection: initial transcripts were read in detail by two researchers (CW and MV), who identified emerging themes, some of which were specifically explored in subsequent focus groups and interviews. 

  3. As new data were collected, the same two researchers read each new transcript and re-examined earlier transcripts, developing in the process coding schemes for organising and classifying data. 

  4. By comparing and discussing their cod- ing approaches, these two researchers reached consensus on a robust coding system that could be applied to the entire dataset. 

  5. Periodic discussion of emerging themes with the entire research team informed the coding process and refined the approach to data collection. 

  6. Consistent with a theo- retical sampling approach, data collection continued until thematic saturation was achieved.13,14 As saturation is based on theoretical rather than statisti- cal considerations, the resulting numbers of partici- pants varied modestly across the three fields studied. 

  7. Once the complete dataset had been classi- fied using the refined coding scheme, the level of analysis was raised from the categorical to the conceptual by the examination and elaboration of the relationships among the concepts. 

  8. Finally, we con- sidered our conceptual analysis in light of existing theories of learning, exploring how our findings aligned with, elaborated or challenged these constructs.








RESULTS


우리의 분석은 전문적인 문화를 통해 피드백이 관리되는 방식에 있어 변수와 상수를 모두 보여주었습니다. 각 전문문화는 세팅, 학습자 역할, 교사 역할을 통해서 독특한 학습 맥락을 만들었고, 이것은 피드백이 처리되는 방식에 영향을주는 주었다. 이러한 차이에도 불구하고 신뢰성과 구성성은 상수로 나타났으며, 이것들은 모든 문화에서 학습자들이 피드백을 의미있는 것으로 인식하기 위해서는 필요했다. 그러나 신뢰와 구성성의 정의는 각 직업 문화마다 뚜렷했으며, 피드백이 이러한 특성을 갖도록 하는 지원은 문화에 따라 상당히 다양했다.

Our analysis revealed both variations and constants in how feedback is managed across professional cul- tures. Each professional culture, through the set- tings and opportunities for feedback it provided, and the roles it expected teachers and learners to play, created a distinct context for learning that influ- enced how feedback was handled. Despite these dif- ferences, credibility and constructiveness emerged as constants, identified by learners across cultures as essential for feedback to be perceived as meaning- ful. The very definitions of credibility and construc- tiveness, however, were distinct to each professional culture, and support for the occurrence of feedback with these critical characteristics varied considerably across cultures.


맥락

Context


학습 세팅

Settings for learning


의학 및 교사 훈련은 직장 환경에서 확고하게 학습을 배웠습니다. 의료 학습자는 순간적 피드백이 산발적으로 발생하는 까다로운 임상 환경에 몰입하여 학습함으로써 독립적으로 기능 할 수있는 기회를 중요하게 생각했습니다. 언제 피드백을 줄 수 있을지 예측할 수 없었기에, 의학은 피드백이 정기적으로 제공되도록 순환 종료 평가와 같은 공식 기회를 제공했습니다. 그러나 많은 학습자들은 이 공식화 된 피드백이 실질적으로 도움이 되지 못한다고 하였는데, 왜냐하면 이벤트에서 (시간적으로) 너무 멀리 떨어져 의미가 없고, 너무 구체적이지 못하고, 감독관이 상요하는 피드백 양식이 충분한 정보조차 가지고 있지 않은 학습자의 수행능력 측면에 대해 의견을 제시하도록 강제했기 때문이다.

Medicine and teacher training placed learning firmly in the workplace setting. Medical learners valued opportunities to function independently, learning by immersion in unruly clinical environ- ments in which in-the-moment feedback occurred only sporadically. Perhaps in response to the unpredictable nature of in-the-moment feedback, medicine provided formal opportunities, such as end-of-rotation evaluations, to ensure that feedback was given regularly. Many learners complained, however, that this formalised feedback lacked substance, either because it was non-specific or too far removed from the event to be meaningful, or because the feedback forms supervisors were required to use forced them to comment on aspects of learner performance about which they had insufficient information.


의료 훈련에서 의무화 된 형식적 피드백으로 인해, 실질성substance이 부족했고, 이는 일반적으로 피드백에 대해 학습자가 느끼는 가치를 시간이 지남에 따라 감소시키기도 했다.

This lack of substance, characteristic of much of the mandatory formalised feedback in medical training, could, over time, diminish the value that learners placed on feedback in general,


음악 내에서 학습 설정에는 일주일에 1 : 1 레슨뿐만 아니라 오랜 시간 동안 연습실에서 혼자 일하는 것이 포함되었습니다. 수업은 반복되는 성과, 피드백 및 교정 과정을 포함했습니다.

Within music, learning settings included weekly one-to-one lessons as well as long hours working alone in the practice room. Lessons involved a repeating process of performance, feedback and correction:


"..."아니, 충분하지 않아. ""네가 충분히 좋게하기 위해해야 ​​할 일이야. "(음악 교수, 인터뷰 3)

‘…a combination of “No, that’s not good enough” [and] “This is what you need to do to make it good enough.”’ (Music professor, inter- view 3)




선생님의 역할

Role of the teacher


의학적 학습자는 감독자가 교사와 임상의로서의 이중 역할을 하고 있음을 인정했지만, 이러한 역할의 위계에 대해 거의 의문을 품지 않았다.

Medical learners acknowledged the dual roles of their supervisors as teachers and clinicians, leaving little doubt about the hierarchy of these roles:


학습자는 교육보다 진료에 우선을 두는 문화적 특권이 양질의 피드백의 availability에 영향을 미친다고 생각했다.

Learners recognised the impact of this cultural privi- leging of patient care over teaching on the availabil- ity of quality feedback:


세 문화권 중에서 유일하게 의학적 학습자는 때로는 그들의 존재가 교사들에게 부담이된다는 것을 느꼈습니다.

Uniquely among the three cultures, medical learn- ers sometimes even felt that their presence was a burden on their teachers:


교사 연수에서는 감독자가 보다 중심적 인 역할을 맡았는데, 여기에는 수업 할당 제어, 일관되게 학생의 성과 관찰 및 광범위한 피드백 제공이 포함되었습니다. 학생들은 자신이 받은 피드백을 이해하고 통합했음을 증명할 수있는 기회가 있다는 것을 중요하게 여겼다.

In teacher training, the supervisor assumed a more central role that included controlling lesson assign- ments, consistently observing the student’s perfor- mance and offering extensive feedback. Students, in turn, valued opportunities to demonstrate that they had understood and incorporated the feedback they received:


의학적 학습자처럼, 학생 선생님은 learned by doing했지만, 의학과는 달리, '하는 것'은 감독자의 지속적인 안내와 피드백에 의해 지원되었습니다.

Like medical learners, student teachers learned by doing, but, unlike in medicine, the ‘doing’ was sup- ported by continuous guidance and feedback from supervisors.

더 선명하고 대조적으로, 음악 학생의 학습은 교사 중심이었습니다. 숙련 된 교사의 지시없이 성취도를 향상시키는 것은 상상도 할 수없는 것이었다. 선생님의 의견 이외에 학습에 대한 유용한 영향을 파악하려는 한 학생의 노력은 전형적이었습니다.

In even sharper contrast, music students’ learning was very much teacher-focused; to improve in per- formance without instruction from a skilled teacher was reportedly unimaginable. One student’s struggle to identify useful influences on learning other than input from a teacher was typical:



'항상 선생님에게 의지 할 수 있습니다. 선생님께 무엇을해야할지 항상 물어볼 수 있습니다. 그들은 당신을 안내 할 수 있습니다. '(음악 학생, FG6)

‘You can always rely on your teacher, you can always ask your teacher what to do; they can guide you.’ (Music student, FG6)



(음악에서) 교사의 역할은 없어서는 안되는 것이었고, 피드백은 필수적이었다.

As the teacher’s role was so indispensable, feedback was considered vital:


학습자의 역할

Role of the learner


의료 학습자는 피드백을 (능동적으로) 찾는 데 대한 책임감을 인정했으며,

Medical learners accepted some responsibility for seeking feedback,



"의견이 충분하지 않다고 말할 수는 있지만, 나는 많은 어려움이 학습자에게 있다고 생각합니다 ... 나는 항상 그것이 내게 수작업으로 제공 될 것이라고 기대할 수는 없습니다."(Resident, FG2)

‘We can say that there’s not enough feedback, but I think a lot of times the onus is on the lear- ner… I can’t just always expect it to be hand-fed to me.’ (Resident, FG2)



이와는 대조적으로 학습자가 피드백을 이끌어낼 책임을 져야한다는 생각은 다른 전문 문화에서는 낯선 것이었다. 교사 훈련에서 학습자가 요구할 필요없이 일상적으로 피드백을 제공했기 때문에 피드백을 찾는 행동은 피드백이 수신되는지 확인하기보다는 설명을 얻거나 특정 질문에 응답하기위한 것이 었습니다. 음악에서 피드백 추구 행동은 완전히 불필요한 것처럼 보였습니다.

By contrast, the idea that learners should take responsibility for inviting feedback was more foreign to the other professional cultures. In teacher train- ing, feedback was provided routinely without the learner needing to ask for it, so feedback-seeking behaviour, when it occurred, was intended to obtain clarification or to respond to specific questions, rather than to ensure that feedback would be received at all. In music, feedback-seeking behaviour appeared to be entirely unnecessary.



'아니. (피드백은) 당신이 원하든 원치 않든간에 온다. '(음악 학생, FG9)

‘No. It comes whether you want it or not.’ (Music student, FG9)



신뢰도

Credibility


모든 문화에서 학습자들이 피드백을 얼마나 신뢰하는지가 그것이 의미있는 영향을 줄 수 있는 가능성에 강력한 영향을 미쳤다. 그러나 '신뢰성'이 모든 학습자에게 동일한 의미를 지니지는 않았다. 오히려 신뢰성에 대한 개념은 피드백이 제공된 전문 문화의 지울 수 없는 특성을 지니고 있었다. 예를 들어, 문화마다 피드백의 신뢰도에 영향을 주는 것이 달랐다. 의학 학습자들은 가르치는 능력이 중요하다는 것을 인정하면서도, 피드백의 신뢰성을 결정 짓는 강력한 요소로 감독자의 임상 기술을 꼽았다.

Learners across cultures identified the perceived credibility of the feedback they received as a power- ful influence on its likelihood of meaningfully impacting them. Credibility, however, did not have the same meaning for all learners; rather, notions of credibility bore the indelible stamp of the pro- fessional culture in which feedback was provided. For example, the culture shaped how teachers acquired credibility as sources of feedback. Medical learners, although they acknowledged teaching abil- ity as a desirable attribute, identified the clinical skills of their supervisors as a stronger determinant of the credibility of their feedback.



'교수가 환자와 상호 작용하는 것을 보았을 때 나는 그들의 행동이나 다른 사람들과 의사 소통하는 방식에 의구심을 표한다. 나는 그들의 피드백을 viable하다고 생각하지 않는다.'(Resident, FG2)

‘If I’ve seen them interacting with patients… and I’m questioning their performance or the way they communicate with other people, I don’t take their feedback as viably.’ (Resident, FG2)


'... 우리는 놀랍게도 실제로 교실에 오지 않은 교수진이 있습니다. 신뢰성은 그럴 때 떨어진다 '(교육 교수, 인터뷰 2)

‘…we have some faculty, surprisingly, who have never actually been in a classroom. I would say that’s where the credibility falls down.’ (Educa- tion professor, interview 2)


의학과 교수법과는 대조적으로, 음악 학생들은 훌륭한 연주 기술을 가진 학생들보다 교육용 기술이 잘 발달 된 교사를 선호했습니다. 음악 교사의 경우, 도움이되었지만 불충분 한 수행 능력; 교사는 또한 학생의 성과를 향상시키기위한 지침을 제공 할 수 있어야했습니다.

By sharp contrast with medicine and teaching, music students preferred teachers with well-developed instructional skills over those with virtuoso perfor- mance skills. For music teachers, the ability to per- formwas helpful but insufficient; a teacher also needed to be able to provide guidance to advance the student’s performance.


음악은 우리가 공부 한 세 가지 문화 중 유일하게 교사의 신뢰도가 부분적으로 교사의 학생의 수행능력에 의해 결정되었습니다. 열매를 맺는 학생들을 많이 보았던 선생님 (음악 학생, FG8)에 대한 음악 학생의 존경심은 강사 또는 멘토로서의 능력에 중점을두고 있으며, 자기자신의 능력에서 벗어났습니다. 한 사람.

Music was the only one of the three cultures we studied in which a teacher’s credibility was deter- mined, in part, by the performance of that teacher’s students. Music students’ respect for ‘the teachers you see who have had a lot of fruitful students’ (music student, FG8) signalled a shift in emphasis towards a teacher’s abilities as an instructor or men- tor, and away from his or her abilities as a per- former.


문화는 또한 신뢰가 위협받을 수있는 상황을 만들었습니다. 의학 학습자는 종종 관찰되지 않은 결과에 대한 피드백을 받았다.

Culture also created circumstances under which credibility might be threatened. Medical learners frequently received feedback on unobserved perfor- mances:


반대로 음악 학생은 자신의 공연을 보거나 들은 적이 없는 교사가 제공 한 피드백을 상상하지 못했으며, 그러한 것을 묻는 것 자체를 놀랍게 여겼다. 마찬가지로, 교육 문화에서, 직접 관찰없이 피드백을 창출 할 수 있다는 생각은 매우 낯선 것이었다.

Music students, by contrast, could not provide any examples of feedback being given by a teacher who had not seen or heard their performance; their sur- prise at even being asked this question suggested that this approach to feedback was alien to their cul- ture. Similarly, in the education culture, the very idea that feedback could be created without this level of observation was foreign:


의학에서는 관찰의 빈도가 적음에도 불구하고, 학습자는 여전히 관찰과 피드백의 신뢰성 사이의 강한 연관성을 확인했다.

Despite the infrequency of observation in medicine, learners still identified a strong link between obser- vation and the perceived credibility of subsequent feedback:


'나는 어떤 피드백이 정확하다고 생각한다면, 피드백에 매우 개방적이다. 특히 나를 관찰 한 사람이 지적할 경우에 그렇다.'(Resident, FG1)

‘I’m pretty open to [feedback], especially if I know it’s very accurate… and it’s being pointed out by a person who has observed me.’ (Resident, FG1)



구성성

Constructiveness


세 문화권 모두에서 학습자는 건설적인 것으로 인식되는 피드백을 선호했으나, '건설적'을 어떻게 정의하는가는 문화마다 다양했다. 음악에서 건설적인 것으로 여겨지는 피드백은 비판적이거나 교정적인쪽으로 기울어졌으며 학생들은 칭찬은 도움이되지 않는 것으로 평가 절하되었습니다.

Across all three cultures, learners preferred feed- back that they perceived as constructive, but again there were cultural variations in how constructive- ness was defined. Feedback that was considered to be constructive in music tended towards the critical or corrective, and students devalued praise as unhelpful:


음악 학생들은 교정적 피드백을 (부정적 피드백이 아닌) '긍정적 인 비판'(음악 학생, FG6)으로 개념화하였다. 대조적으로, 교사 훈련에서 건설적인 피드백은 긍정적인 측면에 확고하게 뿌리를 내리고, 좋은 성과를 강화하고 비평은 개선을 위한 제안으로 프레임하는 것이었다.

Music students appeared to readily appreciate the benefits of corrective feedback, conceptualising it as ‘positive criticism’ (music student, FG6) rather than as negative feedback. By contrast, feedback consid- ered to be constructive in teacher training was firmly rooted in positivity, reinforcing good perfor- mance and framing criticism as suggestions for improvement.



'그녀는 언제나 ' 가지 장점과 한 가지 고칠 점'을 말한다. 당신은 항상 두 가지 긍정적 인 말을하고 다음 단계를 수행합니다. 그래서 항상 건설적입니다. '(학생 교사, FG10)

‘She’d always do the “two stars and a wish”. You always say two positive things and then you do the next step. So, it’s always very constructive.’ (Student teacher, FG10)


의학에서 칭찬과 비판은 환경에 따라 건설적인 것으로 간주 될 수 있습니다. 긍정적 피드백은 특히 훈련 초기에 자신감을 확립하거나 구축 할 때 건설적인 것으로 간주되었습니다. 교정 피드백은 주의가 필요한 약점을 강조 할 때 건설적으로 여겨졌다.

In medicine, both praise and criticism could be considered constructive, depending on the circum- stances. Positive feedback was viewed as construc- tive when it established or built confidence, especially early in training. Corrective feedback was viewed as constructive when it highlighted weak- nesses requiring attention



모든 문화에서 공통적인 핵심 구성는 피드백을 행동 계획과 통합하는 것이었습니다. 개선을 위한 전략을 간략히 제시 한 실행 계획은 교정적 피드백조차도 건설적인 것으로 인식하게 해주었다. 그러나 행동계획의 퀄리티는 문화마다 달랐다. 음악 학생들은 상세하고 도움이되는 행동 계획을 빈번하게 언급했고,

A key element of constructiveness, common across cultures, was the incorporation of an action plan with feedback. An action plan, outlining a strategy for improvement, could permit even corrective feedback to be perceived as constructive. Cultures varied, however, in the typical quality of their action plans and thus in the extent to which the constructiveness of feedback was supported. Music students, in particular, spoke frequently of detailed and helpful action plans,


대조적으로 의학 학습자는 명시 적 행동 계획과 관련된 피드백의 사례를 떠올리기 위해 애를 썼다. 최악의 경우 실천 계획은 존재하지 않았으며, 기껏해봐야 디테일이 부족한 계획에 불과했다.

Medical learners, by contrast, struggled to provide examples of feedback linked to an explicit action plan. Action plans were non-existent at worst and under-detailed at best:




'의학에서 의사 소통은 매우 모호 할 수 있습니다. "더 많이 읽어야합니다."또는 "지식 기반이 부족합니다."(의사와 음악가, 인터뷰 8)

‘In medicine it can be very vague, like: “You need to read more” or “Your knowledge base is lack- ing.”’ (Doctor and musician, interview 8)



'그것이 부정적인 이유나 내가 그것을 어떻게 긍정적으로 만들 수 있는지에 대한 피드백이없는 부정적인 진술 일 뿐이라면, 그것은 나에게 와닿지 않을 것이며, 나는 그것을 수행하지 않을 것입니다.'(Resident, FG1)

‘If it’s just a negative statement without any feed- back on why it’s negative or how I can make it positive, then it wouldn’t stick with me, and I wouldn’t carry it on.’ (Resident, FG1)



고찰

DISCUSSION


피드백은 단순히 개인 간의 대화가 아닙니다. 그것은 복잡한 정보 교환이며, 사회 문화적 상황이 의미를 형성합니다. 지금까지 교육 문화가 피드백에 미치는 영향은 의학 교육 연구자들에게는별로 관심을받지 못했다. 이것은 아마도 학습에 대한 개인 수준의 이론에 특권을 부여하는 오랜 경향과 일치 할 것이다 .15,16 의학에서의 임상 교육은 주로 경험적이며 따라서 임상 학습의 성격을 탐구하는 연구의 대부분은 개인이 자신이 경험 한 것을 어떻게 해석하고 이해하는지에 초점을 맞추고있다. 반면에 사회 문화적 학습 이론은, 학습이 특정 상황에 놓여있는 것으로 학습을 본다. 학습에 대한 개인 이론 및 사회 문화적 이론은 서로 대립 할 필요가 없다 .20) 오히려, 개인의 학습과 경험에 대한 개인의 해석과 학습이 이루어지는 문화, 상황 및 환경은 학습 결과 형성에 기여한다.

Feedback is not merely a conversation between indi- viduals; it is a complex information exchange, the social and cultural contexts of which shape its meaning for learners. Until now, the effect of train- ing culture on feedback has received little attention by medical education researchers, which is perhaps consistent with the field’s longstanding tendency to privilege individual theories of learning.15,16 Although clinical education in medicine is primarily experiential and therefore heavily contextualised, most of the work exploring the nature of clinical learning has focused on how individuals interpret and make sense of the experiences they have.8,17 Socio-cultural learning theories, by contrast, view learning as situated within specific contexts and cul- tures.16,18,19 Individual and socio-cultural theories of learning need not be placed in opposition to one another.20 Rather, both the individual’s own inter- pretations of the events and experiences that com- prise his or her learning and the culture, context and environment in which the learning takes place contribute to shaping learning outcomes.21



의학교육에서 피드백은 개인의 피드백 전달 기술에 초점을 맞추는 경향이 있으나, 이는 피드백 개선에 문제가 된다. 사회 문화적 렌즈를 통해 볼 때 부적절한 개인을 타겟팅하는 접근법이 명확 해집니다. 피드백 전달은 확실히 강화 될 수 있지만, 교육자는 개별 학습자가 갖는 피드백에 대한 인식orientation에 영향을 줄 수있는 능력이 제한적입니다. 따라서 피드백의 영향력을 강화할 수있는 가장 좋은 기회는 이를 뒷받침 할 수 있는 문화적 발판을 마련하는 것입니다.

Our elaboration of feedback’s critical cultural dimension exposes as problematic many of our cur- rent approaches to improving feedback in medical education, which tend to focus on the feedback delivery skills of individuals.22,23 Viewed through a socio-cultural lens, the inadequacies of approaches targeting individuals become clear. Although feed- back delivery can certainly be strengthened, educa- tors have only a limited ability to influence the feedback orientation of individual learners. The richest opportunities to strengthen the impact of feedback may therefore involve creating the neces- sary cultural scaffolding to support it.



Shulman은 서명 교육에 관한 연구에서 다른 직업의 교수법에 대한 비교가 효과적 일 수 있으며, 기존에 고려되지 않았던 교육 향상에 대한 새로운 접근법을 제시 할 수 있다고 설명합니다. 교사 교육과 음악과 함께 의학의 학습 문화에 대한 우리의 비교 적 고려는 서명 교육이 다른 결과를지지하면서 다른 결과를지지하는 선택을 포함한다는 Shulman의 전제를 명백하게 보여줍니다.

In his work on signature peda- gogies, Shulman10 comments that comparisons of the pedagogies of different professions may be fruit- ful and may offer approaches to improving educa- tion that might not otherwise have been considered. Our comparative consideration of medi- cine’s learning culture alongside that of teacher education and music starkly demonstrates Shul- man’s premise that signature pedagogies involve choices that support certain outcomes while con- straining others.10


교사 교육 및 음악과 비교하여, 우리는 의학의 현재 훈련 문화가 사실상 피드백 문화가 아니라는 것을 발견했습니다. 이것은 새로운 발견이 아닙니다. 다른 연구자들은 다른 학습 문화와의 비교를 통해 개선 노력에 대한 명확한 시각을 제시한다 : 우리는 의학교육 문화에서 피드백의 신뢰성과 구성 성을 제한하는 특징을 교정 할 필요가있다. 교사 교육 및 음악 문화와 비교하면 의학의 피드백 문화에서 신뢰와 건설이 주요 취약성임을 알 수 있습니다. 또한, 이러한 취약점은 우리가 학습의 맥락을 어떻게 설정했느냐에 따라 존재합니다.

In comparison with teacher education and music, we found that medicine’s current training culture is not, in fact, a feedback culture. This is not a new finding; others have decried medicine’s failings in the feedback arena.24,27 What our study offers, through its comparison with other learning cultures, is a clear sightline for improvement efforts: we need to remedy the features of our culture that limit the credibility and constructiveness of feedback. By com- parison with the cultures of teacher education and music, we can see that credibility and constructive- ness, rather than representing strengths of medi- cine’s feedback culture, are its key vulnerabilities. Moreover, these vulnerabilities exist because of how we have set up the context of learning.


음악에있어 장기적인 교사-학습자 관계가 피드백의 중요성을 강조하면서도, 피드백 신뢰도와 구성성에 대한 인식을 용이하게 한다는 사실은 왜 우리가 그러한 관계(장기적 교사-학습자 관계)가 발전 할 수있는 상황을 만들어 내지 못하는지를 묻게됩니다. 선생님 교육 선생님이 학생들의 수업 시간에 앉아서 학습자가 가르치는 것을 지켜 볼 수 있도록 보호 해주기 때문에 행동 지침과 함께 상세한 학습 피드백을 제공한다는 것으로부터, 왜 우리의 임상 교육 문맥은 학습자의 수행을 관찰하고 논평하는 데 아주 적은 시간만을 기울였는지를 묻게 한다.

Knowing that the longstanding teacher–learner relationship in music facilitates a perception of feed- back credibility and constructiveness – even when feedback is pointedly critical – should lead us to ask why we aren’t creating contexts in which such rela- tionships can develop. Knowing that teacher educa- tion preceptors provide daily detailed feedback coupled with action plans to their learners – because their time is protected to enable them to sit at the back of the class and watch the learner teach – should lead us to ask why our clinical teaching con- text allots very little time and attention to observing and commenting on the learner’s performance.


우리는 시간 제약, 교수법에 대한 부적절한 보상, 개발력이 부족한 교원지도 기술과 같은 운영상의 어려움의 중요성을 기각하지 않습니다. 이러한 매우 실제적인 문제는 학습자의 직접적인 관찰이나 학습자의 피드백을 수반하는 상세한 행동 계획의 작성을 실제로 제한 할 수 있습니다. 그러나 이러한 운영 문제만으로는 피드백이 환자에게 어떻게 다루어 지는지에 대한 의미있는 변화를 이끌어 내기위한 노력이 불충분하다는 설명이 부족합니다.

We do not dismiss the importance of operational challenges such as time constraints, inadequate compensation for teaching, and underdeveloped faculty instructional skills; these very real issues may indeed limit the direct observation of learners or the creation of detailed action plans to accompany learners’ feedback. We believe, however, that these operational issues alone provide insufficient expla- nations for the inertia that plagues efforts to create meaningful change around how feedback is han- dled in medicine.


왜 의학의 문화가 피드백의 신뢰성과 건설성에 대한 취약성을 지속시키는지를 완전히 이해하려면, 직업의 근본적인 가치에 대한 신중한 검토가 필요합니다. 의학은 의사의 자율성과 독립성을 평가한다 .15,28 감독자가 학습자 성과를 일상적으로 관찰하는 것이, 교사와 학습자가 자율성을 존중하고 기대하는 문화에 편안하게 맞을 지 고려해 보아야 한다. 예컨대 학습자가 환자 진료에서 자율성을 갖고자 할 때, 교육받기를 거부하면서 caregiver와 manager로서의 역할을 위태롭게 만들 수 있다.

A fuller understanding of why medicine’s profes- sional culture allows vulnerabilities around the cred- ibility and constructiveness of feedback to persist requires a thoughtful examination of some of the fundamental values of the profession. Medicine val- ues doctor autonomy and independence.15,28 We might consider whether the routine observation of learner performance by supervisors will fit comfort- ably in a culture in which teachers and learners value and expect autonomy. One study, for exam- ple, identified learners’ desires for autonomy in patient care as a barrier to improving bedside teach- ing, noting that learners may avoid teaching situa- tions that they perceive might jeopardise their roles as caregivers and managers.29


점점 더 의학은 또한 자기주도적학습을 주요 가치로 삼고있다 .30 광범위하게 사용되는 역량 틀은 의학적 학습자가 지식 격차를 스스로 식별하고이를 치료할 계획을 세울 수있는 기술을 개발할 것이라는 기대를 강조한다 .31,32 우리는 교사가 의미있는 행동계획을 세우는데 만성적으로 실패하는 원인이 자기주도적 문화의 부산물이 아닐지 고민해봐야 한다. 

Increasingly, medicine also espouses self-directed learning as a professional value.30 Widely used competency frameworks high- light the expectation that medical learners will develop the skills to self-identify knowledge gaps and create plans to remedy them.31,32 We must con- sider whether the chronic failure of teachers to pro- duce the meaningful action plans that would make their feedback constructive might be a byproduct of this culture of self-directedness, which may view such detailed instruction as unnecessary, counter- productive or coddling.


마지막으로, 의학의 전문 문화는 탁월한 교육 기술을 가진 의사에게 주어지는 보상이 적다. 학습자가 교수 능력에 상관없이 피드백을 위해 가장 훌륭한 수행자를 바라는 한, 의료 교사는 가르침이 암시하는 역할 모델로 봉사 할 수 있습니다. 그러나 가장 강력한 역할 모델은 임상 적 역량뿐만 아니라 그들이하는 일을 어떻게, 왜하는지 설명 할 수있게 해주는 교육자로서의 기술을 갖추어 그 방향으로 학습자를 인도하는 것입니다 .33,34

Finally, medicine’s profes- sional culture may not reward doctors who develop exceptional instructional skills; as long as learners look to the best performers for feedback, regardless of their teaching ability, medical teachers may be con- tent to serve as role models whose teaching is impli- cit. The strongest role models, however, possess not only clinical competence, but also the teaching skills that enable them to explain how and why they do what they do and to guide learners towards the example they set.33,34


포커스 그룹과 인터뷰 데이터는 본질적으로 의미있는 피드백을 구성하는 요소에 대한 참가자의 시각 만 제공한다는 점에서 제한적입니다. 이러한 인식은 피드백이 실제로 효과적인지 여부를 반드시 나타내는 것은 아니다.

Focus group and interview data are inherently limited in that they provide only the participants’ perspectives about what constitutes meaningful feedback; these percep- tions do not necessarily indicate whether feedback is actually effective.


결론

CONCLUSIONS


피드백은 학습에 상당한 가치가 있지만 교사와 학습자 간의 직접적인 거래를 나타내지는 않습니다. 오히려 피드백은 복잡한 개인 및 대인 관계 및 문화적 및 맥락적 요소의 영향을받습니다. 학습을 향상시키기 위해 피드백 사용을 최적화하려면 개인이 프로세스를 인식하는 방법뿐만 아니라 해당 프로세스가 사용되는 문화에 따라 가치를 평가하는 방법에 대한 인식이 필요합니다.

Feedback has considerable value for learning, but it does not represent a straightforward transaction between teacher and learner. Rather, feedback is affected by complex individual and interpersonal dynamics and by cultural and contextual factors. Optimising the use of feedback to enhance learning requires an appreciation not only of how the pro- cess is perceived by individuals, but also of how it is valued by the culture in which its use is situated.








 2013 Jun;47(6):585-94. doi: 10.1111/medu.12150.

Beyond individualismprofessional culture and its influence on feedback.

Author information

1
Department of Clinical Neurological Sciences, Schulich School of Medicine and Dentistry, University of Western Ontario, London, Ontario, Canada. chris.watling@schulich.uwo.ca

Abstract

CONTEXT:

Although feedback is widely considered essential to learning, its actual influence on learners is variable. Research on responsivity to feedback has tended to focus on individual rather than social or cultural influences on learning. In this study, we explored how feedback is handled within different professional cultures, and how the characteristics and values of a profession shape learners' responses to feedback.

METHODS:

Using a constructivist grounded theory approach, we conducted 12 focus groups and nine individual interviews (with a total of 50 participants) across three cultures of professional training in, respectively, music, teacher training and medicine. Constant comparative analysis for recurring themes was conducted iteratively.

RESULTS:

Each of the three professional cultures created a distinct context for learning that influenced how feedback was handled. Despite these contextual differences, credibility and constructiveness emerged as critical constants, identified by learners across cultures as essential for feedback to be perceived as meaningful. However, the definitions of credibility and constructiveness were distinct to each professional culture and the cultures varied considerably in how effectively they supported the occurrence of feedback with these critical characteristics.

CONCLUSIONS:

Professions define credibility and constructiveness in culturally specific ways and create contexts for learning that may either facilitate or constrain the provision of meaningful feedback. Comparison with other professional cultures may offer strategies for creating a productive feedback culture within medical education.

PMID:
 
23662876
 
DOI:
 
10.1111/medu.12150


교수 학생간 비공식 멘토링 (Acad Med, 2005)

Informal Mentoring Between Faculty and Medical Students

Gail L. Rose, PhD, Margaret R. Rukstalis, MD, and Marc A. Schuckit, MD





많은 학교가 전문 교육 과정을 커버하고 전문적인 개발 도구로서의 멘토링의 중요성을 강조하지만 명확한 가이드 라인이나 지침은 거의 제공되지 않습니다. 1-3 멘토링은 자아를 줄 수 있는 능력, 즉  Erikson이 성숙한 사회적 발달 모델에서 "generativity"로 표현한 능력을 반영합니다 - .4 멘토는 "keeping the meaning"하는 것이며, 하나의 발달과제입니다. "보호자"는 전통을 이어갑니다. 지혜와 정의를 가지고 미래 세대에게 과거의 전통을 전달해줍니다. 자신의 직업 개발에서 일대일지도를받은 사람들은 자신의 개인 스타일을 형성하면서 이러한 경험을 낳을 수 있습니다. 그러나 새로운 교수진의 약 1/3만이 특정 멘토를 가지고 있다고보고합니다.

Although many schools cover pro- fessionalismin their curricula and high- light the importance of mentoring as a professional development tool, clear guidelines or directives are rarely provid- ed.1–3 Mentoring reflects the capacity to give the self away, an ability also charac- terized by Erikson as “generativity” in his model of adult social development.4 Mentors serve as “keepers of the mean- ing,” a developmental task based upon empirical data from Harvard’s Study of Adult Development.5 “Keepers” pass on the traditions of the past to future gener- ations with wisdom and justice without taking sides. Those who have received one-on-one guidance in their own career development may bring to bear these experiences as they shape their own per- sonal styles; however, only about one third of new faculty report having specific mentors.6



멘토링 사례

A Case Example of Mentoring


멘토링 관계

The Mentoring Relationship


전통적 멘토링은 경험이 부족한 초보자와 일반적으로 나이가 많은 경험 많은 베테랑 인 두 사람의 발달을 향상시키는 관계입니다 .7 Drs. R과 P가 보여 주듯이, 멘토들은 그들의 시간 투자를 가치만큼 가치있게 여기기 때문에 그 배치는 상호 이익이된다.


Traditional mentoring is a development- enhancing relationship between two per- sons: a less experienced novice and a more experienced veteran who is typically older.7 As the example of Drs. R and P illustrates, the arrangement is mutually beneficial, in that mentors value their investment of time as much as do prote´- ge´s.6


많은 멘토들은 전문 지식을 다른 사람에게 제공함으로써 얻는 강화된 자존심과 같은 감정적인 혜택친밀하고 지속적인 동료애를 개발할 수있는 기회를 장점으로 인정합니다. 그러한 관계는 오래 지속되고 다세대 성향을 가지는 경향이 있습니다. 결국, 프로 테지도는 젊은 동료, 연수생, 또는 학생에게 멘토로서 동시에 봉사 할 수 있기 때문입니다.


Emotional benefits, such as the enhanced self-esteem that comes from providing expert knowledge to another, and the opportunity to develop close, enduring colleagueship, are also acknowledged by many mentors. Such relationships tend to be long-lasting and multigenerational, as prote´ge´s eventually may serve concur- rently as mentors to younger colleagues, trainees, or students.8




용어 설명

Clarification of Terms



"멘토링"은 문헌에서 매우 광범위하게 정의 된 용어입니다. 소그룹 지도자,지도 교사, 교훈자, 감독자 또는 역할 모델과 같은 역할을 설명하는 데 다양하게 사용됩니다. 이 모든 것은 실용적인 지식을 습득 할 수있는 중요한 역할을합니다. 또한 충분한 접촉과 "케미"가 있다면 역할 모델role model이나 감독자supervisor가 학생에게 멘토가 될 수 있습니다 .9이 논문에서 우리는 자연스럽게 형성된 멘토링에 대한 토론에 초점을 맞춥니다. 특정 교과과정이나 직업적 목표를 넘어서는 개인적 및 전문적 개발을 장려하기 위해 junior와 senior간의 일대일, 상호, 헌신적, nonsexual 관계. 특히 우리는 교수진과 의대생 간의 멘토 관계에 중점을 둡니다.

“Mentoring” is a termdefined very broadly in the literature. It is at various times used to describe roles such as small-group leader, academic advisor, preceptor, supervisor, or role model. All of these are important roles that can facil- itate the acquisition of practical knowl- edge. Furthermore, a role model or su- pervisor may become with time a mentor to a student if there is sufficient exposure and “chemistry.”9 In this article, how- ever, we focus our discussion on mentor- ing defined narrowly as a naturally formed, one-to-one, mutual, committed, nonsexual relationship between a junior and senior person designed to promote personal and professional development beyond any particular curricular or insti- tutional goals. In particular, we focus on the mentor relationship between faculty and medical students.


공식 프로그램에서 멘토는 일반적으로 훈련을 받았으며, 매칭 전에는 학생에게 알려지지 않은 경우가 많습니다. 그러한 프로그램은 많은 혜택을 제공하며, 학생들이 의과대학 초기에 멘토를 찾을 수있는 기회를 제공합니다 .10 그러나 정식 프로그램을 통해 조직 된 관계는 업무 지향, 신뢰, 상호 헌신, 호환성compatibility, 강도, 폭, 기간, 구조 등의 영역에서의 멘토링과 질적으로 다릅니다 , 6,11-14 의대생을위한 혁신적인 멘토링 프로그램이 여러 곳에서 설명되었습니다 .11,15-17

In formal programs, mentors are typically trained, and are most often unknown to the student prior to the match. Such programs offer a number of benefits and provide opportu- nities for students to find mentors earlier in medical school.10 However, relation- ships organized via formal programs are qualitatively different fromspontaneous mentoring in such areas as task orienta- tion, trust, mutual commitment, compat- ibility, intensity, breadth, duration, and structure.6,11–14 A number of innovative mentoring programs for medical students have been described elsewhere.11,15–17


의대 학생들은 멘토십의 가치를 인정합니다.: 조사에 따르면 90 ~ 95 %의 학생들이 멘토링을 중요하거나 매우 중요하게 생각하거나 그러한 관계를 개발하는 데 관심이 있습니다 .10,18 학생들은 preceptors이나 attendings와 가장 자주 멘토십을 개발하며, resident들과 하기도 합니다.20 멘토링에 대한 열망에도 불구하고 약 3 분의 1의 학생 만이 멘토가 있다고보고하고있다 .18) 


전임상기간 동안에는 교수와의 개인적인 접촉이 제한되어 있으며, 임상실습 기간에는 짧은 기간동안 여러 과를 돌아야 하는 특성 때문에 학생들은 장기간에 걸친 강렬한intense 관계를 형성하지 못하는 어려움을 겪게 된다. 또한 멘토링을 해달라고 부탁하는 불편함과 비슷한 관심사를 가진 사람을 만나지 못하는 것도 학생들의 어려움에 포함됩니다 .18 이러한 이유로, 학생들이 교수와 비공식적으로 상호 작용할 수있는 기회를 적극적으로 찾고, 멘토링을 해줄 수 있을지를 직접적으로 물어볼 용기를 갖는 것이 매우 중요합니다 .21,21 마찬가지로 교수진은 학생의 요청을 받아들이는 것이 중요합니다. 또한 멘토링을 하는 것이 적합할 것 같고 헌신할 수 있을 때는 멘토링을 해 줄 필요가 있다.

Medical students value mentorship: sur- veys indicate that about 90%to 95%of students rate mentoring as important or very important, or are interested in devel- oping such a relationship.10,18 Students develop mentorships most frequently with preceptors or attendings,19 and with residents.20 In spite of the wide desire for mentoring, only about one-third of stu- dents report having a mentor.6,18 Limited personal contacts with professors during the preclinical years, and the short time frame and diversity of experiences during clerkships make establishment of long- termintense connections between stu- dents and faculty difficult.10,11 Other bar- riers perceived by students include discomfort asking for mentoring, and failure to meet someone with similar in- terests.18 For these reasons, it is critically important for students to proactively seek out opportunities to interact informally with faculty, and to find the courage to directly ask potential candidates for men- toring.12,21 Likewise, it is important for faculty to be receptive to requests by stu- dents, and to provide mentoring when it seems like a good fit and it is feasible to make the commitment.





멘토 : Integrity가 있는 선생님

Mentor: A Teacher with Integrity



멘토링은 가르치는 것과 똑같은 기술을 많이 사용합니다. 본질적으로 그것은 더 깊은 수준으로 가르치는 것과 같. 멘토는 일대일 방식으로 기술을 시연하고, 학생이 수행하거나 작성한 작업물을 비판하고 편집하며, 학생들의 진행 상황을 검토 및 추적하고 멘티가 장기 목표를 향해 나아갈 수 있도록 도전challenge합니다 .22,23 교사와 마찬가지로, 좋은 멘토는 분명하고 효과적으로 소통합니다. 학생들은 다양한 학습 스타일을 인식하고 학생들에게 가장 잘 도달 할 수 있도록 교육 방법을 조정합니다. 일부 학생들은 지시적이고 구체적이고 작업 지향적인 도움이 필요하지만, 어떤 학생들은 자신의 아이디어를 분명하게 말하거나 삶의 목적을 분명히하는 데 도움이 필요할 수 있습니다. 가르침을 넘어, 멘토들은 멘티의 개인적, 직업적 발전에 투자합니다. 멘티의 잠재력을 인식하고 멘티의 커리어를 promote 할 수있는 기회를 가질 수 있습니다.

Mentoring involves many of the same skills teaching does; in essence it is teach- ing taken to a deeper level. On a one-to- one basis, mentors demonstrate tech- niques, critique and edit students’ performed or written work, review and track students’ progress, and challenge mentees to take steps toward their long- termgoals.22,23 As teachers do, good mentors communicate clearly and effec- tively. They recognize different learning styles and adjust their instructional ap- proach to best reach a student. While some students need directive, concrete, task-oriented assistance, others may need help articulating their ideas or clarifying a life purpose. Beyond teaching, mentors invest in their prote´ge´s’ personal and professional development. They can rec- ognize potential and take opportunities to promote their mentees’ careers.


멘토가 무엇을 하는지도 중요하며, 멘토가 누구인지도 중요합니다 .10 멘토는 역할 모델로서 종종 행동, 태도 및 전망을 통해 지혜를 직접 전달합니다. 의사와 환자, 동료에 대한 존중의 정도, 신뢰성, 윤리는 모든 관찰자에게 분명합니다. 공감적으로 경청하는 모습, 개인적인 가치에 따라 생활하며 직업에 대한 열정과 긍정적인 태도를 전달하는 개인은 연수생에게 똑같이하도록 고무시킬 것입니다. 훌륭한 멘토는 학생과 개인적 및 정서적 수준으로 교류할 수있는 능력을 갖추고 있습니다. 그들은 좌절, 낙담 또는 불안감을 느끼고 있을 때 조언이나 확신reassurance을 구할 수있는 멘티의 감정 표현을 용인 할 수 있습니다. 유능한 멘토는 현재의 상황과 관련이있을 때 자신의 개인적인 경험과 어려웠던 경험을 나눌 의향이 있으며 교육과정 외 교육 주제를 논의 할 수 있습니다.

What the mentor does is important, and also who the mentor is matters a great deal.10 As role models, mentors often impart wisdomindirectly through their behaviors, attitudes, and perspectives. A physician’s degree of respect for patients and colleagues, trustworthiness, and eth- ics are evident to any observer. An indi- vidual who listens empathetically, lives in accordance with personal values, and conveys enthusiasmand positive attitude about the profession will inspire trainees to do the same. Good mentors have the ability to engage with a student on a per- sonal and emotional level. They are able to tolerate expressions of emotion in their mentees who may seek advice or reassurance when they are feeling frustra- tion, discouragement, or anxiety. Effec- tive mentors are open to discussing extra- educational topics and are willing to share their own personal experiences and struggles when they are relevant to the current situation.6



경험은 더 나은 멘토를 만든다

Experience Makes a Better Mentor



다양한 커리어 단계의 교수들은 멘토링에 대한 다양한 측면을 제시하며, 커리어 단계는 Dyad dynamics에 영향을 미칠 수 있습니다 .24 최근까지 학생의 입장이었기 때문에 주니어 교수들은 학생들의 시각과 요구 사항을보다 면밀히 파악할 수 있습니다.

Professors at different career stages offer different aspects of mentoring, and one’s career stage may have implications for the dyad’s dynamics.24 Since they were most recently in a trainee role, junior faculty may identify more closely with students’ perspectives and needs.


이제 막 훈련과정을 마친 주니어 교수진은 종종 최신 과학 지식을 잘 알고 있습니다. 그러나 경험이 상대적으로 부족하여 구제 능력에 확신을 갖지 못할 수도 있습니다. 또한 직업적 정체성이 끊임없이 변하기 때문에 경력 초기에 교수진이 일과 삶의 균형에 대한 개인적인 struggle투쟁에서 벗어나이 중요한 직업 분야와 관련하여 다른 사람들에게 지침을 제공하기가 어려울 수 있습니다.

Junior faculty, who are fresh out of train- ing, are often well aware of the most up- to-date scientific knowledge; however, they may not feel confident in their men- toring abilities due to their relative lack of experience. Furthermore, because their professional identities are constantly in flux, it can be difficult for faculty early in their careers to step back sufficiently from personal struggles with work and life balance to offer guidance to others regarding this important area of professionalism.


중견 교수들은 더 많이 (삶이) 확립되어 있고, 경험이 풍부하며, 점점 더 안정된 개인적 및 직업적 정체성을 갖는 경향이 있습니다. 이상적으로는 경력 경로와 능력에 대한 확신을 갖게되었습니다.

Midcareer academicians are more estab- lished, more experienced, and tend to have increasingly stable personal and professional identities. Ideally, they have focused career paths and growing confi- dence in their abilities.


시니어 교수는 차세대 사람들과 경험을 공유 할 수있는 풍부한 경험과 지식을 가지고 있습니다 .24 그들은 전문적인 명성을 쌓았으며 많은 경우 조직이나 직업 내에서 상당한 영향력과 영향력을 행사합니다. 그들은 발전위원회, 부서장, 또는 여러 분야의 통증 관리 클리닉을 지휘 할 수 있습니다. 시니어 교수 멘토는 종종 ​​기관의 기억을 지니고 있으며 자신의 분야에 대한 광범위한 역사적 견해를 가지고 있습니다. 학생들은 새로운 관점과 열정으로 에너지를 소중히하고 파생시킬 수 있습니다.

Late-career academicians have a wealth of experience and knowledge to share with those in the next generation.24 They have established professional reputations and in many cases wield considerable power and influence within an organization or profession. They may serve on advance- ment committees, chair a department, or direct a multidisciplinary pain manage- ment clinic. Late-career mentors often carry an institutional memory and have a broad historical view of their field. They may value and derive energy fromthe fresh perspective and enthusiasmoffered by a student prote´ge´.







멘토링에는 유연성이 필요합니다.

Mentoring Requires Flexibility


멘티는 다양한 목표가 있을 수 있다. 

  • 포괄적(예 : "성공적인 의사가 됨"), 

  • 구체적 (예 : "해부학 과정 통과"), 

  • 의학적으로 관련이있는 (예 : "신체 검사를 배우십시오"), 

  • 작업(학습) 관련 습관 (예 : "시간 관리 개선") 

  • 개인 (예 : "의대 외의 생활"),

Mentees have a variety of goals, which may be broad (e.g., “become a successful doctor”), specific (e.g., “pass my anatomy course”), medically related (e.g., “learn physical examskills”), work-habit related (e.g., “improve time management”), or personal (e.g., “have a life outside of medical school”),


학생의 발달 단계

Stages of a student’s development


학생의 단계에 따라 서로 다른 멘토링 전략을 권장합니다. 학생의 요구는 학생이 beginner에서 advanced로 진행함에 따라서 개발할 기술, 종단적인 관점, 멘토의 역할, 교육 방향의 원천에 따라 변화합니다 .24,25 초기에 멘토는 구체적인, 스킬 기반의 교육을 시행하여, 구체적인 task와 테크닉을 완수하는 것을 도와줌으로써 가장 효과적인 멘토링을 할 수 있다. 새로운 의대생들은 긍정적이고 건설적인 피드백에 대해 더 많은 것을 알게됩니다.

Different mentoring strategies are recom- mended for students at different stages. Students’ needs change in terms of skills to be developed, longitudinal perspective, role of mentor, and source of educational direction as students progress frombe- ginner to advanced levels of training.24,25 Early on, mentors can be most effective by providing specific, skills-based in- struction aimed at facilitating the acquisi- tion and mastery of techniques and con- crete tasks. New medical students benefit fromdetailed positive and constructive feedback.


처음에는 멘토가 지시자directive가 되어야하고, 단기간의 초점을 유지하고 구체적인 작업과 지연을 할당해야합니다. 반면 Advanced 학생의 멘토는 컨설턴트consultant 또는 sounding board 역할을 하는 경우가 많습니다. 그들은 추상적인 아이디어와 이론에 대한 개념적인 교류와 토론을 통해 연수생을 인도하고 지원합니다. 초점은 장기 계획 및 경력 개발로 바뀝니다.

In the beginning, mentors may need to be directive, to maintain a short-termfocus, and to assign concrete tasks and dead- lines. Mentors of advanced students, on the other hand, more often play the role of a consultant or sounding board. They guide and sup- port the trainee through conceptual ex- change and discussion of abstract ideas and theories. The focus turns to long- termplanning and career development.


간단히 말하면, 효과적인 멘토는 학생들의 발달 단계에 따라 교육적 접근 방식을 달리 함으로써, 멘티가 독립적이며 자신감있는 전문가가 되도록 돕는 점진적 "이유weaning 과정"을 수립합니다.

In short, effective mentors tailor their educational approach to the student’s stage of development, and insti- tute a gradual “weaning process” to help mentees become independent and confi- dent professionals.24




성별 및 인종

Gender and race


유연한 멘토링은 또한 학생의 사회 인구학적 정체성에 대한 인식을 수반합니다. 대부분의 경우, 학생 중 여성과 소수 민족의 비율이, 교사의 그것보다 높기 때문에ㅡ 교사는 다양한 성별과 인종을 지닌 학생들을 지도하도록 요청 받게됩니다. 학생의 성별과 인종은 의대 학교의 경험과 멘토의 선호도에 영향을 줄 수 있습니다. 그러므로 멘토는 오해misunderstandings를 막기 위해 차이점을 인정하고 멘티의 고유한 관점을 수용하도록 교육을 받습니다.

Flexible mentoring also entails awareness of a student’s sociodemographic identity. Teachers will be called upon to mentor students of different genders and races because, in most instances, the propor- tion of women and minorities among students is higher than it is among fac- ulty. A student’s gender and race can af- fect medical school experiences and men- toring preferences. Therefore, to prevent misunderstandings, mentors can ac- knowledge differences and accept educa- tion fromprote´ge´s about their unique perspectives.26


여성은 훈련과 직장에서 harassment를 경험했다고 보고 할 가능성이 더 높으며 27) 훈련 상황에서 권력을 행사하지 않도록 권장됩니다 .28 여성은 남성과 비교할 때 역할 모델을 할 수 있는 멘토를 선호하고, 신뢰감을 나타내며, 균형 잡힌 삶의 방식으로 사는 능력을 보여주는 멘토를 중요시합니다. 15,29-31 또한 여성은 여성 스승을 선호하는 경향이 큽니다.

Women are more likely to report experi- encing harassment in their training and careers,27 and are less often encouraged to assert power in training situations.28 Compared with men, women tend to prefer mentors who act as role models, exhibit trustworthiness, and demonstrate an ability to live a balanced life- style.15,29–31 Women are also more likely than men are to prefer a female mentor.


예를 들어, 여성들은 더 많은 심리적 인 지원을받는 경향이 있지만 스폰서 쉽과 비공식적 인 네트워킹은 덜받는 경향이있 다 .32-34

For example, women tend to receive more psychosocial support frommentors, but less sponsor- ship and informal networking.32–34



예를 들어, 한 의과대학 학생 중 44 %는 여성 멘토가 있다고 했으나 교수 중에 여성은 24 % 뿐이다.18 따라서 학생들이 점점 더 다양해짐에 따라 여성과 소수자 교수가 더 자주 멘토로 요청받을 가능성이 높습니다. 

For example, 44%of medical students at one institu- tion indicated they had a female mentor, yet women constituted only 24%of the faculty.18 Thus it is likely that women and minority faculty will be called upon more often to be mentors by students who are increasingly more diverse.






할 것과 하지 말아야 할 것

Mentoring Do’s and Don’ts



효과적인 멘토링 관계는 헌신과 정기적인 유지 관리가 필요합니다. 이것은 공동의 책임입니다. 목록 1은 "해야 할 것과하지 말아야 할 것"을 보여줍니다.

Effective mentoring relationships require commitment and regular maintenance; this is a shared responsibility. List 1 shows some “do’s and don’ts.”


또한 정기적인 상호 작용은 관계 설정에 중요합니다 .6 만남의 빈도와 기간에 대한 기대는 mutual 해야하며, 의사 결정의 목표와 관계의 초점에 따라 달라질 것입니다.

Furthermore, regular interactions are critical to the establish- ment of relationships.6 Expectations for the frequency and length of meetings should be mutual, and will vary depend- ing on the prote´ge´’s goals and the focus of the relationship.


또한 멘토는 멘티에 초점을 맞추고 시간 경과에 따른 진행 상황을 추적해야합니다 .37 멘토에서는 멘토가 조언을 제공하는 대신 주로 질문을 함으로써 멘티의 존중과 확신을 전달할 수 있습니다. 멘티가 자신의 개념conceptions을 구두로 말할 수 있을 때 독립성이 촉진됩니다 .24,38

Also important, mentors need to main- tain a focus on the mentee and track his or her progress over time.37 In meetings, mentors can convey respect and confi- dence in the mentee by primarily asking questions instead of giving advice; inde- pendence is fostered when the mentee can verbalize his or her own concep- tions.24,38



효과적인 멘토는 피드백을 제공하고 강점을 발견해줍니다.22 관계 유지를 위해서는 dyad 내에서 잘 진행되고 있는 것과 개선 할 것은 무엇인지에 대한 정기적인 재평가가 있어야 한다.

Effective mentors give feedback and identify strengths.22 Maintaining a relationship also involves periodic reassessment of what is going well within the dyad, and areas in which to improve.


좋은 멘토는 멘티에 초점을 두지 않고 자신의 의제를 홍보하거나, 멘티를 "공짜 노동력"으로 보거나, 멘티의 업적을 빼앗거나, 멘티를 멘토의 경력 경로, 철학 및 의견흉내내는mimic "클론"으로 만들려는 등 자신의 전문적 필요에만 독점적으로 초점을 둡니다. 즉, 효과적인 멘토는 멘티의 경력과 self-sufficiency을 촉진시키기 위해 노력합니다.

Good mentors avoid exclusive focus on their own professional needs, such as by promoting their own agenda instead of the mentee’s,39 viewing the mentee as “free labor,” taking credit for mentee’s accomplishments, or seeking to create a “clone” who mimics the mentor’s career path, philosophy, and opinions. In short, effective mentors work to promote the mentee’s career and self-sufficiency.


관계 유지는 상호 책임입니다. 양 당사자는 직접적이고 정직하게 통신해야합니다. 멘토는 멘티가 시간약속을 지키고, 기한을 맞추고, 약속commitment을 지키면서 기여할 것을 기대할 수 있습니다.

Maintaining a relationship is a mutual responsibility. Both parties need to com- municate directly and honestly. Mentors can expect mentees to contribute by be- ing on time, meeting deadlines, and fol- lowing through on commitments.21



회의에서 멘티는 건설적 비판을 받아 들일 수 있고, 실수를 인정하고, 제안에 응답 할 수 있어야 합니다. 멘티는 멘토가 "그 곳"에 있었고 장애물을 피할 수 있도록 도와주었다는 것을 인정해야 한다.40 멘티는 존경을 전달해야하며 subordinate로서의 역할을 받아 들여야한다 .41 전통적 멘토링 관계는 위계적이며, 멘토링의 장점은 많은 부분 이 위계에서 파생된 것이기도 하다. 관계가 진화함에 따라 멘토는 멘티가 점점 더 많은 책임과 도전을 받아들이도록 기대하고 격려 할 수 있습니다. 이것은 전문적 정체성professional identity을 개발하는 길의 일부입니다. 마지막으로, 멘토링 관계는 주기적으로 재평가되어야 합니다. 목표와 목적이 진화 할 것이며, 이러한 변화는 만남의 준비와 초점에 영향을 미칠 수 있습니다.

In meetings, mentees should be able to accept con- structive criticism, admit mistakes, and respond to suggestions; they should ac- knowledge that the mentor has “been there” and can help themavoid obsta- cles.40 Prote´ge´s should convey respect and ac- cept a subordinate role.41 Traditional mentoring relationships are hierarchical and, to a large extent, the benefits derived from them are a function of this hierar- chy. As the relationship evolves, mentors can expect and encourage their mentees to accept increasing responsibility and challenge. This is part of the path to de- veloping professional identity. Finally, mentoring relationships should be reas- sessed periodically. Goals and objectives will evolve, and these changes may have implications for meeting arrangements and focus.


멘토는 어떤 문제가 생길 수 있을지에 대해서 미리 주의를 기울여야 한다. 학생이 멘토에 지나치게 의존하게되면 관계가 비생산적으로 변할 수 있습니다. 42 예를 들어, 학생이 독립적 인 결정을 할 수 없거나 만들지 않고 대신 멘토에게 각 결정을 내리는 경우가 있다. 학생들은 멘토에게만 정보와 기회 의존해서는 안된다. 다양한 자원을 식별하고, 조사하고, 사용하는 것은 멘티가 육성해야 할 전문 기술입니다.

Mentors should be alert to problems that can arise; relationships can become coun- terproductive if the student becomes overly dependent on the mentor.42 For example, if a student cannot or does not make independent decisions but instead looks to the mentor to make each deci- sion, the student will not develop inde- pendence. Students should not rely on the mentor exclusively for information and opportunities. Identifying, investi- gating, and using a variety of resources is a professional skill to be fostered.


멘티의 수동성과 묵인은 또한 해로울 수 있습니다. 학생이 멘토의 관점을 일방적으로 받아들이고 "보트를 흔들지 않는"것 만이 능사라고 생각하면, 학생은 자신의 지적 관점에 대한 자신감을 높일 수 없습니다. 마지막으로 subordinate은 때로는 멘토을 과도하게 이상화 할 수 있습니다 .14 멘토는 크게 존경받으며 오류가없는 것처럼 보일 수 있습니다. 멘토는 제자에게 멘토도 인간이며 모든 대답을 반드시 지니고 있지 않다고 상기시켜야 할 수도 있습니다. 과도한 이상화는 실망으로 이어질 수 있습니다. 지나치게 멘토에게 의존적인 멘티에게는 스스로가 학습에 궁극적 인 책임이 있음을 상기시켜줄 수 있습니다.

A mentee’s passivity and acquiescence can also be detrimental. If a student believes it is better to accept the mentor’s perspective and not “rock the boat,” the student may not de- velop confidence in his or her own intel- lectual perspective. Finally, subordinates can sometimes overidealize their men- tors.14 Mentors are greatly admired and respected, and may seem infallible. It may be necessary to remind a mentee that mentors are indeed human and don’t necessarily have all the answers. Overidealization can lead to disappointment. Mentees who are overly dependent may be reminded that each student is ultimately responsible for his or her own learning.




Summary


좋은 멘토링 관계는 자기 지식과 자신의 스타일과 한계의 확인을 포함합니다. 개인은 멘토링을위한 역량이 강하고 효과 성은 모든 이상적인 성격의 구현을 필요로하지 않습니다. 멘토는 수련자들에게 그들이 할 수있는 것과 제공 할 수없는 것에 대해 명확해야합니다. 역할의 일부는 다른 사람에게 도움이 될만한 사람을 추천 할시기를 아는 것입니다. 중요한 멘토는 관계의 전반적인 목표를 유지하고 시간이 지남에 따라 멘티의 개인적 및 전문적 성장에 중점을 둔다.


Good mentoring relationships involve self-knowledge and identification of one’s style and limitations. Individuals have relative strengths in their capacity for mentoring and effectiveness does not require embodiment of every ideal at- tribute. Mentors should be clear with trainees about what they can and cannot offer; part of the role is knowing when to refer someone to another resource that might be more helpful. Impor- tantly, a good mentor maintains sight of the overall goal of the relationship and holds the focus for the mentee’s personal and professional growth over time.


17 Wilson H, Egan T, Friend R. Teaching pro- fessional development in undergraduate medical education. Med Educ. 2003;37:482–3.


Professional development in undergraduate medical curricula--the key to the door of a new culture?





 2005 Apr;80(4):344-8.

Informal mentoring between faculty and medical students.

Author information

  • 1Department of Psychiatry, University of Vermont, Burlington, VT, USA. Gail.Rose@vtmednet.org

Abstract

Mentoring skills are valuable assets for academic medicine faculty, who help shape the professionalism of the next generation of physicians. Mentors are role models who also act as guides for students' personal and professional development over time. Mentors can be instrumental in conveying explicit academic knowledge required to master curriculum content. Importantly, they can enhance implicit knowledge about the "hidden curriculum" of professionalism, ethics, values and the art of medicine not learned from texts. In many cases, mentors also provide emotional support and encouragement. The relationship benefits mentors as well, through greater productivity, career satisfaction, and personal gratification. Maximizing the satisfaction and productivity of such relationships entails self-awareness, focus, mutual respect, and explicit communication about the relationship. In this article, the authors describe the development of optimal mentoring relationships, emphasizing the importance of experience and flexibility in working with beginning to advanced students of different learning styles, genders, and races. Concrete advice for mentor "do's and don'ts"is offered, with case examples illustrating key concepts.

PMID:
 
15793017
[PubMed - indexed for MEDLINE]


형성적 피드백(Formative Feedback) (Review of Educational Research, 2008)

Focus on Formative Feedback (Review of Educational Research, 2008)

Valerie J. Shute

Florida State University





It is not the horse that draws the cart, but the oats. —Russian proverb


 

 

교육적 맥락에서 사용 된 피드백은 일반적으로 지식과 기술 습득을 향상시키는 데 결정적인 역할을한다 (예, Azevedo & Bernard 1995, Bangert-Drowns, Kulik, Kulik, & Morgan, 1991, Corbett & Anderson, 1989; Epstein et al. , 2002, Moreno, 2004, Pridemore & Klein, 1995). 성취에 미치는 영향 외에도 피드백은 학습 동기 부여에 중요한 요소로 묘사됩니다 (예 : Lepper & Chabay, 1985; Narciss & Huth, 2004). 그러나 학습을 위해서는 피드백에 관한 이야기가 너무 장밋빛 또는 단순하지 않습니다.

Feedback used in educational contexts is generally regarded as crucial to improv- ing knowledge and skill acquisition (e.g., Azevedo & Bernard, 1995; Bangert- Drowns, Kulik, Kulik, & Morgan, 1991; Corbett & Anderson, 1989; Epstein et al., 2002; Moreno, 2004; Pridemore & Klein, 1995). In addition to its influence on achievement, feedback is also depicted as a significant factor in motivating learn- ing (e.g., Lepper & Chabay, 1985; Narciss & Huth, 2004). However, for learning, the story on feedback is not quite so rosy or simple.


Cohen (1985)의 의견에 따르면 피드백은 "교육 설계에서 교육적으로 강력하지만, 가장 덜 이해되어있는 특성 중 하나입니다"(33 쪽). 수백 가지의 연구 보고서가 피드백의 주제에 관해 발표되었습니다.
According to Cohen (1985) feedback “is one of the more instructionally pow- erful and least understood features in instructional design” (p. 33). the hundreds of research studies published on the topic of feed- back


이러한 피드백 연구의 큰 범위 내에서 많은 결과가 상충하며 일관된 결과 패턴이 존재하지 않습니다.

Within this large body of feedback research, there are many conflict- ing findings and no consistent pattern of results.


형성적 피드백의 정의

Definition of Formative Feedback


형성 피드백은이 리뷰에서 학습 향상을 목적으로 자신의 생각이나 행동을 수정하려는 의도로 학습자에게 전달 된 정보로 정의됩니다.

Formative feedback is defined in this review as information communicated to the learner that is intended to modify his or her thinking or behavior for the pur- pose of improving learning.


이 분야에서 수행 된 대부분의 연구의 전제는 올바른 피드백이 학습 과정과 결과를 올바르게 개선 할 수 있다는 것입니다. 마지막 3 단어 - "올바르게 전달 된 경우"-이 검토의 요점을 구성합니다.

The premise underlying most of the research conducted in this area is that good feedback can significantly improve learning processes and outcomes, if delivered correctly. Those last three words—“if delivered correctly”—constitute the crux of this review.


Goals and Focus


이 글의 두 가지 목적은 (a) 특징, 기능, 상호 작용 및 학습에 대한 링크를 더 잘 이해하기 위해 피드백에 대한 광범위한 문헌 검토 결과를 제시하고 (b) 문헌 검토에서 발견 한 결과를 적용하는 것이다 형성 피드백과 관련된 일련의 가이드 라인을 작성합니다.

The dual aims of this article are to (a) present findings from an extensive liter- ature review of feedback to gain a better understanding of the features, functions, interactions, and links to learning and (b) apply the findings from the literature review to create a set of guidelines relating to formative feedback.


이 리뷰는 일반적인 요약 피드백과는 달리 작업 수준 피드백(task-level feedback)에 중점을 둡니다. 과제 수준 피드백은 일반적으로 학생들에게 문제에 대한 특정 대응에 대한보다 구체적이고 시기 적절한 (때로는 실시간) 정보를 요약 피드백과 비교하여 제공하며 학생의 현재 이해력 및 능력 수준을 추가로 고려할 수 있습니다. 예를 들어, Struggling student는 숙련 된 학생과 비교하여 형성피드백에서 더 큰 지지와 구조를 요구할 수 있습니다

This review focuses on task-level feedback as opposed to general summary feedback. Task-level feedback typically provides more specific and timely (often real-time) information to the student about a particular response to a problem or task compared to summary feedback and may additionally take into account the student’s current understanding and ability level. For instance, a struggling student may require greater support and structure from a formative feedback message com- pared to a proficient student.


방법

Method


절차

Procedure


Seminal articles in the feedback literature were identified (i.e., from sites that provide indices of importance such as CiteSeer), and then collected. The bibliog- raphy compiled from this initial set of research studies spawned a new collection- review cycle, garnering even more articles, and continuing iteratively throughout the review process.


포함 기준

Inclusion Criteria


The focus of the search was to access full-text documents using various search terms or keywords such as feedback, formative feedback, formative assessment, instruction, learning, computer-assisted/based, tutor, learning, and performance.


문헌 고찰

Literature Review


이 주제에 관한 많은 연구에도 불구하고, 학습에 피드백을 관련시키는 특정 메커니즘은 여전히 ​​거의 없고, (매우 소수의) 일반적인 결론이 있더라도 대부분 애매하다. 피드백 데이터에 대한 메타 분석을 수행하는 어려운 과제를 해결 한 연구원은 피드백 결과를 설명하기 위해 "일관적이지 못한", "모순 된", "매우 가변적 인" 이라고 묘사한다 (Azevedo & Bernard, 1995, Kluger & DeNisi , 1996). 10 년 후 그 기술 어는 여전히 적용됩니다.

Despite the plethora of research on the topic, the specific mechanisms relating feedback to learning are still mostly murky, with very few (if any) general conclusions. Researchers who have tackled the tough task of performing meta-analyses on the feedback data use descriptors such as “inconsistent,” “contradictory,” and “highly variable” to describe the body of feedback findings (Azevedo & Bernard, 1995; Kluger & DeNisi, 1996). Ten years later those descriptors still apply.


피드백은 학습과 수행의 중요한 촉진자로 널리 인용되었지만, 몇몇 연구에서 피드백이 학습에 영향을 미치지 않거나 쇠약하게 만드는 영향을 미친다 고보고했다
Feedback has been widely cited as an important facilitator of learning and per- formance but quite a few studies have reported that feedback has either no effect or debilitating effects on learning


예를 들어, 비판적critical이거나 통제하는controlling 것으로 간주되는 피드백 (Baron, 1993)은 종종 성과를 향상시키기위한 노력을 좌절시킨다 (Fedor, Davis, Maslyn, & Mathieson, 2001). 학습을 방해하는 피드백의 또 다른 특징은 다음과 같다 :

  • 동료들에 대한 학생의 입장을 나타내는 점수 또는 전반적인 점수를 제공하고

  • 그러한 규범적 피드백을 낮은수준의 특이성 (즉, 모호함)과 결합 시킴 (Butler, 1987; Kluger & DeNisi, 1998; McColskey & Leary, 1985; Wiliam, 2007, Williams, 1997).

 

For instance, feedback that is con- strued as critical or controlling (Baron, 1993) often thwarts efforts to improve performance (Fedor, Davis, Maslyn, & Mathieson, 2001). Other features of feed- back that tend to impede learning include:

  • providing grades or overall scores indi- cating the student’s standing relative to peers, and

  • coupling such normative feedback with low levels of specificity (i.e., vagueness) (Butler, 1987; Kluger & DeNisi, 1998; McColskey & Leary, 1985; Wiliam, 2007; Williams, 1997).


이 검토의 정의에 따르면 학습에 부정적인 영향을 미치는 피드백은 형성 적이지 않습니다.
In line with the definition in this review, feedback that has negative effects on learning is not formative.


피드백 목적

Feedback Purposes


다양한 형식의 피드백 외에도 다양한 기능이 있습니다. Black and Wiliam (1998)에 따르면 피드백의 두 가지 주요 기능이있다 : 지시directive 및 촉진facilitative.

  • Directive 피드백학생들에게 수정해야 할 내용을 알려주는 것입니다. 이러한 피드백은 촉진적 피드백과 비교하여보다 구체적으로 나타나는 경향specific이 있으며, 이는 학생들이 자신의 목표를 바꾸고 개념화하는 데 도움이되는 의견과 제안을 제공합니다.

In addition to vari- ous formats of feedback, there are different functions. According to Black and Wiliam (1998), there are two main functions of feedback: directive and facilita- tive. Directive feedback is that which tells the student what needs to be fixed or revised. Such feedback tends to be more specific compared to facilitative feedback, which provides comments and suggestions to help guide students in their own revi- sion and conceptualization.

 


 

인지 메커니즘과 형성적 피드백

Cognitive Mechanisms and Formative Feedback


형성적인 피드백을 학습자가 사용할 수 있는 몇 가지 인지 메커니즘이 있습니다.

There are several cognitive mechanisms by which formative feedback may be used by a learner.


 

첫째, 현재 performance 수준과 원하는 수준의 performance 또는 목표 사이의 차이를 나타낼 수 있습니다. 이러한 격차를 해소하면보다 높은 수준의 노력을 위한  동기부여가 될 수있다 (Locke & Latham, 1990; Song & Keller, 2001). 즉, 형성적 피드백은 학생이 과제 수행 능력 (또는 저조한 정도)에 대한 불확실성을 줄일 수있다 (Ashford, 1986; Ashford, Blatt, & VandeWalle, 2003). 불확실성은 그것을 줄이거 나 감당하기위한 전략에 동기를 부여하는 혐오적인 상태이다 (Bordia, Hobman, Jones, Gallois, and Callan, 2004). 불확실성은 종종 불쾌감을 느끼고 업무 성과에서주의를 산만하게하기 때문에 (Kanfer & Ackerman, 1989) 불확실성을 줄이면 더 높은 동기와 더 효율적인 업무 전략으로 이어질 수 있습니다.

First, it can signal a gap between a current level of performance and some desired level of performance or goal. Resolving this gap can motivate higher levels of effort (Locke & Latham, 1990; Song & Keller, 2001). That is, for- mative feedback can reduce uncertainty about how well (or poorly) the student is performing on a task (Ashford, 1986; Ashford, Blatt, & VandeWalle, 2003). Uncertainty is an aversive state that motivates strategies aimed at reducing or man- aging it (Bordia, Hobman, Jones, Gallois, & Callan, 2004). Because uncertainty is often unpleasant and may distract attention away from task performance (Kanfer & Ackerman, 1989), reducing uncertainty may lead to higher motivation and more efficient task strategies.


둘째, 형성 피드백은 학습자, 특히 초보자 또는 고생하는 학생들의 인지적 부하를 효과적으로 줄일 수 있습니다 (예 : Paas, Renkl, & Sweller, 2003; Sweller, Van Merriënboer, & Paas, 1998). 이러한 학생들은 고성능 요구로 인해 학습 중에 압도적으로 압도 될 수 있으므로 인지부하를 줄이기 위해 고안된 supportive 피드백의 이점을 누릴 수 있습니다.

Second, formative feedback can effectively reduce the cognitive load of a learner, especially novice or struggling students (e.g., Paas, Renkl, & Sweller, 2003; Sweller, Van Merriënboer, & Paas, 1998). These students can become cog- nitively overwhelmed during learning due to high performance demands and thus may benefit from supportive feedback designed to decrease the cognitive load.


마지막으로, 피드백은 부적절한 task strategies, 절차상의 오류 또는 오해를 교정하는 데 유용한 정보를 제공 할 수 있습니다 (예 : Ilgen et al., 1979; Mason & Bruning, 2001; Mory, 2004; Narciss & Huth, 2004). 수정 기능 효과는보다 구체적인 피드백에 대해 특히 강력 해 보인다 (Baron, 1988; Goldstein, Emanuel, & Howell, 1968).

Finally, feedback can provide information that may be useful for correcting inappropriate task strategies, procedural errors, or misconceptions (e.g., Ilgen et al., 1979; Mason & Bruning, 2001; Mory, 2004; Narciss & Huth, 2004). The corrective function effects appear to be especially powerful for feedback that is more specific (Baron, 1988; Goldstein, Emanuel, & Howell, 1968),


피드백 구체성

Feedback Specificity


피드백 특이성(Feedback specificity)은 피드백 메시지에 제시된 정보의 레벨로 정의된다 (Goodman, Wood, & Hendrickx, 2004). 즉, 특정 (또는 정교한) 피드백은 특정 응답이나 행동에 대한 정보를 정확도를 넘어 제공하며 촉진적facilitative이기보다는 지시적directive입니다.

Feedback specificity is defined as the level of information presented in feedback messages (Goodman, Wood, & Hendrickx, 2004). In other words, specific (or elaborated) feedback provides information about particular responses or behaviors beyond their accuracy and tends to be more directive than facilitative.


몇몇 연구자들은 학생들의 작업이 올바른지 아닌지를 나타내는 것보다는 답변을 개선하는 방법에 대한 세부 정보를 제공 할 때 피드백이 훨씬 더 효과적이라고보고했습니다 (예 : Bangert-Drowns et al., 1991; Pridemore & Klein, 1995) . 특이성이 결여 된 피드백은 학생들로 하여금 쓸데없는 것, 좌절하는 것, 또는 둘 다로 보게 할 수도 있습니다 (Williams, 1997). 또한 피드백에 어떻게 반응 할 것인가에 대한 불확실성을 야기 할 수 있고 (Fedor, 1991) 의도 된 메시지를 이해하기 위해 학습자가 더 많은 정보 처리 활동을 요구할 수있다 (Bangert-Drowns et al., 1991). 불확실성과인지 부하는 학습 수준을 낮추고 (Kluger & DeNisi, 1996; Sweller et al., 1998), 피드백에 대해 반응respond하려는 동기를 감소시킬 수도있다 (Ashford, 1986, Corno & Snow, 1986).

Several researchers have reported that feedback is significantly more effective when it provides details of how to improve the answer rather than just indicating whether the student’s work is correct or not (e.g., Bangert-Drowns et al., 1991; Pridemore & Klein, 1995). Feedback lacking in specificity may cause students to view it as useless, frustrating, or both (Williams, 1997). It can also lead to uncer- tainty about how to respond to the feedback (Fedor, 1991) and may require greater information-processing activity on the part of the learner to understand the intended message (Bangert-Drowns et al., 1991). Uncertainty and cognitive load can lead to lower levels of learning (Kluger & DeNisi, 1996; Sweller et al., 1998) or even reduced motivation to respond to the feedback (Ashford, 1986; Corno & Snow, 1986).


요약하면, 개념적이고conceptual 절차적인procedural 학습 작업에 대해, 명확하고 명확한 피드백을 제공하는 것이 합리적이고 일반적인 지침입니다. 그러나 이는 학습자의 특성 (예 : 능력 수준, 동기 부여) 및 학습 결과 (예 : 보존 및 전송 작업)와 같은 다른 변수에 따라 달라질 수 있습니다. 또한, 형성 피드백 자체의 특이성 차원은 문헌에서 기술 된 바와 같이 그다지 "특이 적이 지 않다".

In summary, providing feedback that is specific and clear, for conceptual and procedural learning tasks, is a reasonable, general guideline. However, this may depend on other variables, such as learner characteristics (e.g., ability level, moti- vation) and different learning outcomes (e.g., retention vs. transfer tasks). In addi- tion, the specificity dimension of formative feedback itself is not very “specific” as described in the literature.


형성적 피드백의 특징

Features of Formative Feedback


피드백에 대한 훌륭한 역사적 검토에서 Kulhavy and Stock (1989)는 효과적인 피드백이 학습자에게 두 가지 유형의 정보 (검증 및 정교화)를 제공한다고보고했습니다.

  • 검증: 대답이 정확한지,

  • 정교화: 메시지의 정보적 측면

...에 대한 간단한 판단으로 정의되며, 정확한 답을 얻기 위해 학습자를 안내하는 관련 단서를 제공합니다. 연구원은 효과적인 피드백은 검증과 정교화의 요소를 모두 포함해야 한다는 견해로 수렴하는 것처럼 보인다 (예 : Bangert-Drowns et al., 1991; Mason & Bruning, 2001).

In an excellent historical review on feedback, Kulhavy and Stock (1989) reported that effective feedback provides the learner with two types of information: verifi- cation and elaboration. Verification is defined as the simple judgment of whether an answer is correct, and elaboration is the informational aspect of the message, pro- viding relevant cues to guide the learner toward a correct answer. Researchers appear to be converging toward the view that effective feedback should include ele- ments of both verification and elaboration (e.g., Bangert-Drowns et al., 1991; Mason & Bruning, 2001).


확인 Verification


가장 일반적인 방법은 간단히 "정확함"또는 "부정확함"을 말합니다.보다 유익한 옵션이 있습니다
The most common way involves simply stating “correct” or “incor-rect.” More informative options exist


명시적 검증 중에서 응답 정확도를 나타 내기 위해 응답을 강조 표시하거나 마킹하면 (예 : 체크 표시와 함께) 정보를 전달할 수 있습니다. 암시 적 검증은 예를 들어 학생의 응답이 예기치 않은 결과 (예 : 시뮬레이션 내)를 초래할 때 발생할 수 있습니다.
Among explicit verifications, highlighting or otherwise marking a response to indicate its correctness (e.g., with a checkmark) can convey the infor- mation. Implicit verification can occur when, for instance, a student’s response yields expected or unexpected results (e.g., within a simulation).


정교화 Elaboration


피드백 정교화는 검증보다 훨씬 다양합니다. 예를 들어 (a) 주제를 다룰 때, (b) 응답을 할 때, (c) 특정 오류를 토론 할 때, (d) 예제를 제공 할 때, (e) 온화한 안내를 할 수 있습니다.
Feedback elaboration has even more variations than verification. For instance, elaboration can (a) address the topic, (b) address the response, (c) discuss the par- ticular error(s), (d) provide worked examples, or (e) give gentle guidance.


정교한 피드백(Elaborated feedback)은 일반적으로 정답을 다루고, 선택된 응답이 왜 틀린 지 설명 할 수 있으며, 정답이 무엇인지 나타낼 수 있습니다. 한 가지 유형의 정교화, 즉 반응-관련 피드백이 단순한 검증이나 "정확한 때까지의 대답"과 같은 다른 유형의 피드백보다 학생들의 성취, 특히 학습 효율성을 향상시키는 것처럼 보입니다 (예 : Corbett & Anderson , 2001, Gilman, 1969, Mory, 2004, Shute, Hansen, & Almond, 2007).
Elaborated feedback usually addresses the correct answer, may explain why the selected response is wrong, and may indicate what the correct answer should be. There seems to be growing consensus that one type of elaboration, response- specific feedback, appears to enhance student achievement, particularly learning efficiency, more than other types of feedback, such as simple verification or “answer until correct” (e.g., Corbett & Anderson, 2001; Gilman, 1969; Mory, 2004; Shute, Hansen, & Almond, 2007).


피드백의 복잡성과 길이

Feedback Complexity and Length


특정 피드백 하에서보다 구체적인 피드백이 일반적으로 덜 구체적인 피드백보다 좋을지라도 고려해야 할 관련 차원은 정보의 길이 또는 복잡성입니다. 예를 들어, 피드백이 너무 길거나 너무 복잡하면 많은 학습자가 주의를 기울이지 않아 사용하지 않게됩니다. 긴 피드백은 또한 메시지를 확산 시키거나 희석시킬 수 있습니다.

Although more specific feedback may be generally better than less specific feedback (at least under certain conditions), a related dimension to consider is length or complexity of the information. For example, if feedback is too long or too complicated, many learners will simply not pay attention to it, rendering it use- less. Lengthy feedback can also diffuse or dilute the message.


많은 연구 논문들이 피드백 복잡성에 대해 다루었지만 복잡성의 차원에서 주요 변수를 배열하려고 시도한 것은 소수다 (Dempsey, Driscoll, & Swindell, 1993; Mason & Bruning, 2001; Narciss & Huth, 2004). 각 목록의 정보를 하나의 컴파일로 집계했습니다 (표 1 참조).

Many research articles have addressed feedback complexity, but only a few have attempted to array the major variables along a dimension of complexity (albeit, see Dempsey, Driscoll, & Swindell, 1993; Mason & Bruning, 2001; Narciss & Huth, 2004). I have aggregated information from their respective lists into a single compilation (see Table 1),


형성적 피드백이 교정 기능을 제공하는 것이라면 가장 단순한 형태로도 (a) 학생의 대답이 옳은지 또는 틀린지를 확인하고 (b) 정확한 응답 (지시 또는 촉진)에 대한 정보를 학습자에게 제공해야합니다. .

If formative feedback is to serve a corrective function, even in its simplest form it should (a) verify whether the student’s answer is right or wrong and (b) provide information to the learner about the correct response (either directive or facilita- tive).


피드백 복잡성은 영향이 없다

No Effect of Feedback Complexity


정보의 양(=피드백 복잡성)은 피드백 효과와 유의한 관계가 없다.

Schimmel (1983) found that the amount of information (i.e., feedback complexity) was not significantly related to feedback effects.




TABLE 1 Feedback types arrayed loosely by complexity





피드백 복잡성의 부정적 영향

Negative Effects of Feedback Complexity


피드백 복잡성은 다양했습니다. 가장 낮은 수준은 단순히 정답이었으며, 가장 복잡한 피드백에는 검증, 정답 및 해답이 잘못 된 텍스트 통로의 관련 부분에 대한 포인터가 잘못된 이유에 대한 설명이 포함되었습니다. .

Feedback complexity was systematically varied. The lowest level was simply correct answer feedback, and the most complex feedback included a com- bination of verification, correct answer, and an explanation about why the incorrect answer was wrong with a pointer to the relevant part of the text passage where the answer resided.


요약하면, 피드백 복잡성에 대한 inconclusive한 발견은 형식적 피드백과 학습 간의 관계에 관련된 다른 중개 요인이있을 수 있음을 시사한다. 예를 들어, 피드백 복잡성보다 피드백의 더 중요한 측면은 학습 목표에 관한 정보 제공과 이를 달성하는 방법과 같은 컨텐츠의 특성과 품질 일 수 있습니다.

In summary, the inconclusive findings on feedback complexity suggest that there may be other mediating factors involved in the relationship between forma- tive feedback and learning. For instance, instead of feedback complexity, a more salient facet of feedback may be the nature and quality of the content, such as pro- viding information about learning goals and how to attain them.


목표-지향적 피드백과 동기부여

Goal-Directed Feedback and Motivation


목표 지향 피드백은 개별 응답 (즉, 개별 작업에 대한 응답)에 대한 피드백을 제공하기보다는 학습자가 원하는 목표 (또는 목표 집합)에 대한 진행 상황에 대한 정보를 제공합니다. 연구 결과에 따르면 학습자가 동기를 부여 받고 참여하게하려면 학습자의 목표와 이러한 목표를 달성 할 수 있다는 자신의 기대치가 밀접하게 일치해야 합니다 (Fisher & Ford, 1998, Ford, Smith, Weissbein, Gully, & Salas, 1998). 목표를 달성 할 수 없을 정도로 높게 설정하면 학습자는 실패를 경험하고 낙담하게됩니다. 목표가 너무 낮아 달성이 확실하지 않을 경우 성공은 더 많은 노력을 촉진 할 수있는 힘을 잃습니다 (Birney, Burdick, & Teevan, 1969).

Goal-directed feedback provides learners with information about their progress toward a desired goal (or set of goals) rather than providing feedback on discrete responses (i.e., responses to individual tasks). Research has shown that for a learner to remain motivated and engaged depends on a close match between a learner’s goals and his or her expectations that these goals can be met (Fisher & Ford, 1998; Ford, Smith, Weissbein, Gully, & Salas, 1998). If goals are set so high that they are unattainable, the learner will likely experience failure and become discouraged. When goals are set so low that their attainment is certain, success loses its power to promote further effort (Birney, Burdick, & Teevan, 1969).


Malone (1981)에 따르면 학습자가 목표를 달성하는 데 필요한 몇 가지 특징이 있습니다. 예를 들어, 목표는 개인적으로 의미 있고 쉽게 생성되어야하며 학습자는 목표 달성 여부에 관한 성과 피드백을 받아야합니다. Hoska (1993)는 목표를 두 가지 유형, 즉

  • 획득 (즉, 바람직한 것을 얻기 위해 학습자를 돕는 것)과

  • 회피 (즉, 학습자가 바람직하지 않은 것을 피하는 것을 돕기)로 분류했다.

According to Malone (1981), there are certain features that goals must have to make them challenging for the learner. For example, goals must be personally meaningful and easily generated, and the learner must receive performance feed- back about whether the goals are being attained. Hoska (1993) classified goals as being of two types: acquisition (i.e., to help the learner acquire something desirable) and avoidance (i.e., to help the learner avoid something undesirable).


동기 부여는 학습자의 성취에 중요한 중재 요소로 나타 났으며 (Covington & Omelich, 1984), 피드백은 목표 중심의 노력에 응답하여 전달 될 때 강력한 동기가 될 수 있습니다.

Motivation has been shown to be an important mediating factor in learners’ per- formance (Covington & Omelich, 1984), and feedback can be a powerful motiva- tor when delivered in response to goal-driven efforts.


목표-지향성은 사람들이 다른 종류의 목표를 향해 노력하도록 동기 부여되는 방식을 설명합니다. 개인은 과제에 대해 학습learning 또는 성과performance 지향적 태도를 취하는 것으로 생각합니다 (예 : Dweck, 1986).

  • learning orientation 은 새로운 기술을 개발하고 지능은 가단성이 있다는 신념을 바탕으로 새로운 상황을 마스터함으로써 자신의 능력을 향상시키려는 욕망을 특징으로합니다.

  • performance orientation은 자신의 능력을 다른 사람에게 보여주고, 다른 사람에게 좋은 평가를 받으며, 지능은 타고난 것이라고 믿는 바램을 반영합니다 (Farr, Hofmann, & Ringenbach, 1993).

Goal orientation describes the manner in which people are motivated to work toward different kinds of goals. The idea is that individuals hold either a learning or a performance orientation toward tasks (e.g., Dweck, 1986). A learning orientation is characterized by a desire to increase one’s competence by developing new skills and mastering new situations with the belief that intelligence is malleable. In contrast, performance orientation reflects a desire to demonstrate one’s competence to others and to be positively evaluated by others, with the belief that intelligence is innate (Farr, Hofmann, & Ringenbach, 1993).


연구에 따르면 두 가지 유형의 목표 지향이 개인이 과제 난이도와 실패에 어떻게 반응하는지에 차별적으로 영향을 미치는 것으로 나타났습니다 (Dweck & Leggett, 1988).

  • 학습 지향learning orientation은 실패 직면의 지속성,보다 복잡한 학습 전략의 사용, 도전적인 자료 및 과제의 추구로 특징 지워집니다.

  • 성과 지향performance orientation은 업무 (특히 실패의 직면)에서 벗어나고, 어려운 업무에 대한 관심이 적으며, 성공할 가능성이 적은 도전 과제 및 과제를 찾는 경향이 특징입니다.

이러한 연구 결과에 따르면 연구 결과는 일반적으로 학습 지향은 긍정적 인 결과와 관련이 있으며 성과 지향은 모호하거나 부정적 결과와 관련이 있음을 보여 주었다 (Button, Mathieu, & Zajac, 1996, Fisher & Ford, 1998, VandeWalle , Brown, Cron, & Slocum, 1999).

Research has shown that the two types of goal orientation differentially influ- ence how individuals respond to task difficulty and failure (Dweck & Leggett, 1988). That is, learning orientation is characterized by persistence in the face of failure, the use of more complex learning strategies, and the pursuit of challenging material and tasks. Performance orientation is characterized by a tendency to with- draw from tasks (especially in the face of failure), less interest in difficult tasks, and the tendency to seek less challenging material and tasks on which success is likely. Consistent with these labels, research has generally shown that learning ori- entation is associated with more positive outcomes and performance orientation is related to either equivocal or negative outcomes (e.g., Button, Mathieu, & Zajac, 1996; Fisher & Ford, 1998; VandeWalle, Brown, Cron, & Slocum, 1999).


학습자의 목표-지향성에 영향을 미치는 한 가지 방법 (예 : 수행 강조에 초점을 학습에 중점을 두는 것)은 형성적 피드백을 통해 이루어집니다.

One way to influence a learner’s goal orientation (e.g., to shift from a focus on performing to an emphasis on learning) is via formative feedback.


 

스캐폴딩으로서 형성적 피드백

Formative Feedback as Scaffolding


훈련 바퀴와 마찬가지로 스캐폴딩은 학습자가보다 advanced 활동을 할 수있게 해주 며, 그러한 도움없이 할 수있는 것보다 더 진보 된 사고와 문제 해결에 참여할 수있게 해줍니다.

Like training wheels, scaffolding enables learners to do more advanced activi- ties and to engage in more advanced thinking and problem solving than they could without such help.


기존에는 촉진적 피드백 (지침 및 단서 제공)이 지침 피드백 (교정 정보 제공)보다 학습을 향상시킬 수 있다고 제안하지만, 반드시 그런 것은 아닙니다. 사실, 일부 연구는 Directive 피드백이 실제로 주제 나 내용 영역을 학습하는 학습자 (특히 Knoblauch & Brannon, 1981; Moreno, 2004)의 경우보다 실질적으로 도움이 될 수 있음을 보여주었습니다. 스캐 폴딩은 학습 과정에서 학습자가 명시 적으로 지원하는 것과 관련되어 있기 때문에 교육적 측면에서 모델, 단서, 프롬프트, 힌트, 부분 해법 및 직접 교육 (Hartman, 2002)이 포함될 수 있습니다. 스캐폴딩은 학생들이 인지적 발판을 얻음에 따라 점차적으로 제거됩니다. 따라서 Directive 피드백은 학습 초기 단계에서 가장 유용 할 수 있습니다. 촉진 적 피드백은 나중에 도움이 될 수 있으며, 관련된 질문은 "언제입니까?"이다.

 

Vygotsky (1987)에 따르면, 학습자가, 외부 스캐폴딩은 기존의 지식 시스템이 새로운 학습을 위한 스캐폴딩의 일부가 되는, 보다 정교한 인지 시스템을 개발할 때 제거 될 수 있습니다. 피드백 타이밍 문제는 이제 더 자세히 논의됩니다.

Conventional wisdom suggests that facilitative feedback (providing guidance and cues, as illustrated in the research cited previously) would enhance learning more than directive feedback (providing corrective information), yet this is not nec- essarily the case. In fact, some research has shown that directive feedback may actu- ally be more helpful than facilitative—particularly for learners who are just learning a topic or content area (e.g., Knoblauch & Brannon, 1981; Moreno, 2004). Because scaffolding relates to the explicit support of learners during the learning process, in an educational setting, scaffolded feedback may include models, cues, prompts, hints, partial solutions, and direct instruction (Hartman, 2002). Scaffolding is grad- ually removed as students gain their cognitive footing; thus, directive feedback may be most helpful during the early stages of learning. Facilitative feedback may be more helpful later, and the question is: When? According to Vygotsky (1987), exter- nal scaffolds can be removed when the learner develops more sophisticated cogni- tive systems, where the system of knowledge itself becomes part of the scaffold for new learning. The issue of feedback timing is now discussed in more detail.


시기

Timing


It was my teacher’s genius, her quick sympathy, her loving tact which made the first years of my education so beautiful. It was because she seized the right moment to impart knowledge that made it so pleasant and acceptable to me. —Helen Keller


 

앞서 언급 한 피드백 변수 (예 : 복잡성 및 특이성)와 마찬가지로 피드백시기와 학습 결과 및 효율성에 미치는 영향에 관한 문헌에서 상충되는 결과가 있습니다.
Similar to the previously mentioned feedback variables (e.g., complexity and specificity), there are also conflicting results in the literature relating to the timing of feedback and the effects on learning outcome and efficiency.


일부 연구자들은 오류가 메모리에 인코딩되는 것을 방지하기위한 수단으로 즉각적인 피드백을 주장했지만, 지연된 피드백은 사전 간섭을 줄여 초기 오류를 잊어 버리고 올바른 정보를 간섭없이 인코딩 할 수 있다고 주장했습니다
Some researchers have argued for immediate feedback as a means to prevent errors being encoded into memory, whereas others have argued that delayed feedback reduces proactive interference, thus allowing the initial error to be forgotten and the cor- rect information to be encoded with no interference


 

지연된 피드백

Support for Delayed Feedback


즉각적 피드백

Support for Immediate Feedback


피드백 타이밍과 연결

Conjoining Feedback Timing Findings


문헌에서보고 된 이러한 상호 작용 중 하나는 피드백 타이밍과 작업난이도와의 관련성이 있다.

    • 작업이 어려우면 즉각적인 피드백이 도움이 되지만

    • 작업이 쉬운 경우 지연된 피드백이 바람직 할 수 있습니다 (Clariana, 1999).

이것은 이전에 Scaffolding으로 Formative Feedback 하위 섹션에서 제시 한 아이디어와 유사합니다.

One such interaction reported in the literature concerns feedback timing and task difficulty. That is, if the task is difficult, then immediate feedback is beneficial, but if the task is easy, then delayed feedback may be preferable (Clariana, 1999). This is similar to the ideas presented earlier in the Formative Feedback as Scaffolding subsection.


피드백 타이밍 요약

Summary of Feedback Timing Results


피드백 타이밍과 학습 및 성과 간의 관계를 조사한 연구는 일관되지 않는 결과를 나타냅니다. 하나의 흥미로운 관찰은 많은 현장 연구가 즉각적인 피드백 (Kulik & Kulik, 1988 참조)의 가치를 입증하는 반면, 많은 실험실 연구는 지연된 피드백의 긍정적 인 효과를 나타내는 반면 Schmidt & Bjork (1992; Schmidt et al.)

 

불일치를 해결할 수있는 한 가지 방법은 즉각적인 피드백으로 긍정적 인 학습 결과와 부정적인 학습 효과가 모두 활성화 될 수 있다는 점입니다. 예를 들어 즉각적인 피드백의 긍정적 인 효과는 의사 결정을 촉진하거나 실행 동기와 원인에 대한 결과의 명확한 연관성을 제공하는 것으로 볼 수 있습니다. 즉각적인 피드백의 부정적인 영향은 전송하는 동안 사용할 수 없는 정보에 대한 의존을 촉진하고 덜 신중하거나 주의 깊은 행동을 조장 할 수 있습니다. 이 가정이 사실이라면 즉각적인 피드백의 긍정적이고 부정적인 영향은 서로 상쇄 될 수 있습니다. 양자 택일로, 긍정적 인 영향이나 부정적인 영향이 실험적 맥락에 따라 전면에 나타날 수있다.

 

학습에 대한 지연된 피드백 효과에 대해서도 비슷한 주장이 제기 될 수있다. 예를 들어, 긍정적면에서 피드백 지연은 학습자가 능동적인 인지 및 메타인지 과정에 참여하도록 유도하여 자율감을 고취시킨다. 그러나 부정적인 측면에서 struggling하고 덜 동기 부여 된 학습자에 대한 피드백을 지연시키는 것은 지식과 기술 습득에 실망스럽고 해로운 것으로 판명 될 수 있습니다.

 

Research investigating the relationship of feedback timing to learning and per- formance reveals inconsistent findings. One interesting observation is that many field studies demonstrate the value of immediate feedback (see Kulik & Kulik, 1988), whereas many laboratory studies show positive effects of delayed feedback (see Schmidt & Bjork, 1992; Schmidt et al. 1989). One way to resolve the inconsis- tency is by considering that immediate feedback may activate both positive and neg- ative learning effects. For instance, the positive effects of immediate feedback can be seen as facilitating the decision or motivation to practice and providing the explicit association of outcomes to causes. The negative effects of immediate feed- back may facilitate reliance on information that is not available during transfer and promote less careful or mindful behavior. If this supposition is true, the positive and negative effects of immediate feedback could cancel each other out. Alternatively, either the positive or negative effects may come to the fore, depending on the exper- imental context. A similar argument could be made for delayed feedback effects on learning. For example, on the positive side, delayed feedback may encourage learn- ers’ engagement in active cognitive and metacognitive processing, thus engender- ing a sense of autonomy (and perhaps improved self-efficacy). But on the negative side, delaying feedback for struggling and less motivated learners may prove to be frustrating and detrimental to their knowledge and skill acquisition.


피드백과 다른 변인과의 관계

Feedback and Other Variables


학습자 수준

Learner Level


이 리뷰의 Timing Subsection에서 언급했듯이, 성취도가 낮은 학생들은 즉각적인 피드백으로 이익을 얻는 반면, 성취도가 높은 학생은 지연된 피드백을 선호하거나 이익을 얻을 수 있다고 제안했습니다. (Gaynor, 1981; Roper, 1977). 게다가 다양한 유형의 피드백을 테스트 할 때, Clariana (1990)는 능력이 부족한 학생들이 try-again feedback보다 correct response feedback을받는 것으로 이익을 얻는다 고 주장했습니다. Hanna (1976)는 또한 다양한 피드백 조건과 관련하여 학생의 수행 능력을 검증했다. 검증, 정교화, 피드백 없음.

    • Verification피드백 조건은 우수한 학생의 경우 가장 높은 점수를,

    • Elaborated 피드백은 낮은 학생의 경우 가장 높은 점수를 받았습니다.

    • 중상위 학생들을위한 검증과 정교한 피드백 간에는 유의미한 차이가 없었지만 이러한 피드백 유형 모두는 피드백이없는 것보다 우위에있었습니다.


 

As alluded to in the Timing subsection of this review, some research has sug- gested that low-achieving students may benefit from immediate feedback, whereas high-achieving students may prefer or benefit from delayed feedback (Gaynor, 1981; Roper, 1977). Furthermore, when testing different types of feedback, Clariana (1990) has argued that low-ability students benefit from receipt of correct response feedback more than from try again feedback. Hanna (1976) also examined student performance in relation to different feedback conditions: verification, elaboration, and no feedback. The verification feedback condition produced the highest scores for high-ability students and elaborated feedback produced the highest scores for low-ability students. There were no significant differences between verification and elaborated feedback for middle-ability students, but both of these types of feedback were superior to no feedback.


정답에 대한 확신

Response Certitude


Kulhavy and Stock (1989)는 정보 처리의 관점에서 피드백과 응답 증명 문제를 조사했다. 즉 학생들은 다양한 과제에 대한 각 답변에 따라 자신감 평가 ( "응답 성"등급)를 제공하게했습니다. 그들은 학생들이 스스로 답이 정확하다고 확신 할 때 피드백을 분석하는 데 더 적은 시간을 할애하고, 학생들이 스스로 답이 정확하지 않다고 생각할 때 피드백을 검토하는 데 더 많은 시간을 할애 할 것이라고 가설을 세웠습니다. 이것의 암시는 간단합니다. 즉, 자신의 대답이 틀렸다고 확신하는 학생들에게보다 Elaborated 피드백을 제공하고 정답의 정확성이 높은 사람들에게 더 Constrained 피드백을 제공합니다.


 

Kulhavy and Stock (1989) examined feedback and response certitude issues from an information-processing perspective. That is, they had students provide confidence judgments (“response certitude” ratings) following each response to various tasks. They hypothesized that when students are certain their answer is cor- rect, they will spend little time analyzing feedback, and when students are certain their answer is incorrect, they will spend more time reviewing feedback. The impli- cations of this are straightforward; that is, provide more elaborated feedback for students who are more certain that their answer is wrong and deliver more con- strained feedback for those with high certitude of correct answers.


목표 지향성

Goal Orientation


Davis, Carson, Ammeter, and Treadway (2005)는 learning orientation 이 낮은 사람들한테는 피드백 특이성 (저, 중등도, 고레벨)이 유의한 영향을 미친다는 것을 발견했다. 이 연구 결과는 피드백 활동의 전반적인 긍정적 효과를 뒷받침하며, 학습자가 high-performance 또는 low-learning 지향성을 가질 때 more specific 피드백을 사용할 것을 제안합니다.

Davis, Carson, Ammeter, and Treadway (2005) found that feedback specificity (low, moderate, and high levels) had a significant influence on performance for individuals who were low on learning orientation (i.e., high feedback specificity was better for learners with low learning orientation). They also reported a significant influence of feedback specificity on performance forpersons high in performance orientation (i.e., this group also benefited from more specific feedback). The findings support the general positive effects of feedback on performance and suggest the use of more specific feedback for learners with either high-performance or low-learning goal orientations. 


규범적(서로 비교하는) 피드백

Normative Feedback


McColskey와 Leary (1985)는 실패가 self-referenced 된 용어로 표현 될 때, 즉 다른 측정에 의해 평가 된 학습자의 알려진 수준의 능력과 관련하여 실패의 해로운 영향이 줄어들 수도 있다는 가설을 조사했다. 그들은 norm-referenced feedback에 비해 self-referenced feedback은 미래 성과에 대한 기대치롤 높여주고 노력에 대한 기여도가 증가한다는 것을 발견했다. (예를 들어, "나는 정말로 열심히했기 때문에 성공했다.") 능력에 대한 기여 (예 : '나 때문에 성공했습니다.')는 영향을받지 않았습니다. 주된 의미는 성취도가 낮은 학생들은 규범적 피드백이 아니라 자기 참조 된 피드백을 받음으로써, 자신의 progress에 주의를 집중할 수 있다는 것이다.

McColskey and Leary (1985) examined the hypothesis that the harmful effects of failure might be lessened when failure is expressed in self-referenced terms—that is, relative to the learner’s known level of ability as assessed by other measures. They found that, compared to norm-referenced feed-back, self-referenced feedback resulted in higher expectancies regarding future performance and increased attributions to effort (e.g., “I succeeded becauseI worked really hard”). Attributions to ability (e.g., “I succeeded because I’msmart”) were not affected. The main implication is that low-achieving students should not receive normative feedback but should instead receive self-referenced feedback—focusing their attention on their own progress. 


형성적 피드백의 프레임워크

Toward a Framework of Formative Feedback


To understand the world, one must not be worrying about one’s self. —Albert Einstein


 

Kluger and DeNisi (1996)


Kluger와 DeNisi (1996)는 피드백 개입 (FI)이 다양한 시각에서 성과에 미치는 영향을 조사하고 1900 년대 초 Thorndike의 고전적 연구에 이르기까지 수십 년간의 연구를 조사했다. 그들의 예비 피드백 개입 이론 (FIT)은 FI 효과를 조사하는 광범위한 접근법을 제공하며,

Kluger and DeNisi (1996) examined and reported on the effects of feedback interventions (FIs) on performance from multiple perspectives and spanning decades of research—back to Thorndike’s classic research in the early 1900s. Their preliminary feedback intervention theory (FIT) offers a broad approach to investigating FI effects,


FIT의 기본 전제는 FI가 세 가지 수준의 제어 (a) 작업 학습, (b) 작업 동기 부여, (c) 메타 태스킹 프로세스 (그림 1 참조) 사이에서 학습자의 관심의 위치를 ​​변경한다는 것입니다.

The basic premise underlying FIT is that FIs change the locus of a learner’s attention among three levels of control: (a) task learning, (b) task motivation, and (c) metatask processes (see Figure 1).

 


 

FI가 유도한 관심 영역의 위계가 낮을수록 FI에 따른 performance에 이득이 향상된다는 장점이 있습니다. 즉, 학습자가 과제의 측면 (즉, 그림 1의 아래 부분)에 초점을 두는 형성 피드백은 자기 자신에게주의를 환기시키는 FI (즉, 그림 1의 상단 상자)에 비해 학습 및 성취를 촉진합니다. , 이는 학습을 방해 할 수 있습니다.

The lower in the hier- archy the FI-induced locus of attention is, the stronger is the benefit of an FI for performance. In other words, formative feedback that focuses the learner on aspects of the task (i.e., the lower part of Figure 1) promotes learning and achieve- ment compared to FIs that draw attention to the self (i.e., the upper box in Figure 1), which can impede learning.


 

FIT는 다섯 가지 기본 논점으로 구성된다 :

(a) 행동은 목표 나 표준에 대한 피드백의 비교에 의해 규제된다.

(b) 목표 또는 표준은 계층 적으로 조직된다.

(c)주의는 제한적이므로 피드백 표준 갭 주의력을 행동 규칙에 적극적으로 참여시키는

(d)주의는 일반적으로 계층 구조의 중간 수준으로 향한다. 그리고

(e) FI는 관심의 장소를 바꾸어 행동에 영향을 미친다.

FIT consists of five basic arguments:

  • (a) behavior is regulated by comparisons of feedback to goals or standards,

  • (b) goals or standards are organized hierarchi- cally,

  • (c) attention is limited and therefore only feedback–standard gaps (i.e., dis- crepancies between actual and desired performance) that receive attention actively participate in behavior regulation,

  • (d) attention is normally directed to a moderate level of the hierarchy; and

  • (e) FIs change the locus of attention and therefore affect behavior.


Figure 2 summarizes the main findings.


 

이러한 결과의 한 가지 중요한 발견은 자기주의의 모델 (Baumeister, Hutton, & Cairns, 1990), 노력의 속성 ( Butler, 1987) 및 통제 이론 (Waldersee & Luthans, 1994).

One important finding from these results concerns the attenuating effect of praise on learning and performance, although this has been described elsewhere in the literature in terms of a model of self-attention (Baumeister, Hutton, & Cairns, 1990), attributions of effort (Butler, 1987), and control theory (Waldersee & Luthans, 1994).

 

 

 

 


 

Bangert-Drowns et al. (1991)


행동을 유도하기 위해서는, 학습자는 행동에 의해 초래 된 신체적 변화를 모니터링 할 수 있어야합니다. 즉, 학습자는 새로운 정보에 적응시키고 performance에 대한 자신의 기대치와 일치시킴으로써인지 조작 및 이에 따른 활동을 변경합니다.

The basic idea is that to direct behavior, a learner needs to be able to monitor physical changes brought about by the behavior. That is, learners change cognitive operations and thus activity by adapting it to new information and matching it with their own expectations about performance.


 

Bangert-Drowns et al. (1991)의 대부분의 변수는 텍스트 기반 피드백을 분석하여 5 단계 모델로 구성했습니다. 이 모델은 학습자가 피드백주기를 통해 움직이고 mindfulness의 구조를 강조하면서 학습자의 상태를 설명합니다 (Salomon & Globerson, 1987). Mindfulness는 "학습자가 상황 별 단서와 관련된 과제에 관련된 근본적인 의미를 탐구하는 반사적 과정"입니다 (Dempsey 외, 1993, 38 페이지).

Most of the variables Bangert-Drowns et al. (1991) analyzed comprised text- based feedback, which they organized into a five-stage model. This model describes the state of learners as they move through a feedback cycle and emphasizes the construct of mindfulness (Salomon & Globerson, 1987). Mindfulness is “a reflec- tive process in which the learner explores situational cues and underlying mean- ings relevant to the task involved” (Dempsey et al., 1993, p. 38).


5 단계는 그림 3에 묘사되어 있으며, 특히 성찰의 중요성과 연관된 다른 학습주기 (예 : Gibbs, 1988, Kolb, 1984)와 유사합니다.

The five stages are depicted in Figure 3 and are similar to other learning cycles (e.g., Gibbs, 1988; Kolb, 1984), particularly in relation to the importance of reflection.


1. 학습자의 초기 또는 현재 상태. 이것은 관심의 정도, 목표 지향성, 자기 효능감 정도 및 이전의 관련 지식에 의해 특징 지어진다.

1. The initial or current state of the learner. This is characterized by the degree of interest, goal orientation, degree of self-efficacy, and prior relevant knowledge.


2. 검색 및 검색 전략. 이러한 인식 메커니즘은 질문에 의해 활성화됩니다. 정교화의 맥락에서 저장된 정보는 정보에 대한 액세스를 제공하는 경로가 더 많기 때문에 메모리에서 찾기가 더 쉽습니다.

2. Search and retrieval strategies. These cognitive mechanisms are activated by a question. Information stored in the context of elaborations would be easier to locate in memory because of more pathways providing access to the information.


3. 학습자가 질문에 응답합니다. 또한, 학습자는 (자신의) 대답에 대해 어느 정도 확신을 갖고 있기 때문에, 피드백에 대해 기대하는 바가 있습니다.

3. The learner makes a response to the question. In addition, the learner feels some degree of certainty about the response and thus has some expectation about what the feedback will indicate.


 

4. 학습자는 피드백의 정보를 고려하여 응답을 평가합니다. 평가의 성격은 피드백에 대한 학습자의 기대에 달려 있습니다. 예를 들어,

  • 학습자가 자신의 응답을 확신하는데다가 피드백이 (그 응답이) 정확하다는 것을 확인해주면 검색 경로retrieval pathway가 강화되거나 변경되지 않을 수 있습니다.

  • 학습자가 응답을 확신하고 피드백이 정확하지 않다는 것을 나타내면 학습자는 부조화를 이해하려고 노력할 수 있습니다.

  • 피드백 확인 또는 disconfirmation에 대한 응답에 대한 불확실성은 학습자가 수업 내용을 획득하는 데 관심이 없다면 심층 성찰을 시뮬레이트 할 가능성이 적습니다.

 

4. The learner evaluates the response in light of information from the feedback. The nature of the evaluation depends on the learner’s expectations about feedback. For instance, if the learner was sure of his or her response and the feedback confirmed its correctness, the retrieval pathway may be strength- ened or unaltered. If the learner was sure of the response and feedback indi- cated its incorrectness, the learner may seek to understand the incongruity. Uncertainty about a response with feedback confirmation or disconfirmation is less likely to simulate deep reflection unless the learner was interested in acquiring the instructional content.


5. 응답 평가 결과 관련 지식, 자기 효능감, 관심사 및 목표를 조정합니다. 이러한 조정 된 상태는 이후의 경험과 함께 다음 "현재"상태를 결정합니다.

5. Adjustments are made to relevant knowledge, self-efficacy, interests, and goals as a result of the response evaluation. These adjusted states, with sub- sequent experiences, determine the next “current” state.


 

Bangert-Drowns et al. (1991)의 메타 분석과 후속 5 사이클 모델의 주된 결론은 피드백이 주의 깊게 받아 들여지면 피드백이 학습을 촉진 할 수 있다는 것이다. 반대로 피드백은 학습자가 자신의 기억 검색을 시작하기 전에 응답이 가능 해지거나 피드백 메시지가 학생들의 인지적 요구 (예 : 너무 쉽거나, 너무 복잡하거나, 너무 모호한 경우)와 일치하지 않는 경우와 같이 어리석은 행동mindlessness을 조장 할 경우 학습을 방해 할 수 있습니다.

The main conclusion from Bangert-Drowns et al.’s (1991) meta-analysis and subsequent five-cycle model is that feedback can promote learning if it is received mindfully. Conversely, feedback can inhibit learning if it encourages mindlessness, as when the answers are made available before learners begin their memory search, or if the feedback message does not match students’ cognitive needs (e.g., too easy, too complex, too vague).

 

 

 


 

Narciss and Huth (2004)


일반적으로 Narciss and Huth (2004)는 효과적인 형성 피드백을 설계하고 개발하는 것은 복잡한 학습 과제에 효과적인 피드백을 제공하기 위해 학습자의 특성뿐만 아니라 학습 내용(instructional context)을 고려할 필요가 있다고 주장했다. 형성 피드백의 설계를위한 개념적 틀은 그림 4에 묘사되어있다.

In general, Narciss and Huth (2004) asserted that designing and developing effective formative feedback needs to take into consideration instructional context as well as characteristics of the learner to provide effective feedback for complex learning tasks. The conceptual framework for the design of formative feedback is depicted in Figure 4 (modified from the original).


1. 지시. 학습 요소 또는 문맥은

(a) 학습 목표 (예 : 학습 목표 또는 일부 교과 과정과 관련된 표준),

(b) 학습 과제 (예 : 지식 항목,인지 행동, 메타인지 기술), 그리고

(c) 오류 및 장애 (예 : 일반적인 오류, 잘못된 전략, 오류의 출처).

1. Instruction. The instructional factor or context consists of three main elements: (a) the instructional objectives (e.g., learning goals or standards relating to some curriculum), (b) the learning tasks (e.g., knowledge items, cognitive operations, metacognitive skills), and (c) errors and obstacles (e.g., typical errors, incorrect strategies, sources of errors).


2. 학습자. 피드백 설계와 관련된 학습자에 관한 정보는

(a) 학습 목표 및 목표;

(b) 사전 지식, 기술 및 능력 (예 : 콘텐츠 지식과 같은 도메인 종속적, 메타인지 기술과 같은 도메인 독립적)

(c) 학업 동기 (예 : 학업 성취도, 학업 자기 효능감, 그리고 metamotivational 기술).

2. Learner. Information concerning the learner that is relevant to feedback design includes (a) learning objectives and goals; (b) prior knowledge, skills, and abilities (e.g., domain dependent, such as content knowledge, and domain independent, such as metacognitive skills); and (c) academic moti- vation (e.g., one’s need for academic achievement, academic self-efficacy, and metamotivational skills).


3. 피드백. 피드백 요소는 3 가지 주요 요소로 구성된다 :

(a) 피드백의 내용 (즉, 검증과 같은 평가 측면과 힌트, 큐, 유추, 설명 및 실제 예제와 같은 정보 측면) ),

(b) 피드백의 기능 (즉,인지, 메타인지 및 동기 부여),

(c) 피드백 요소의 제시 (예 : 타이밍, 일정 및 적응성 고려 사항).

3. Feedback. The feedback factor consists of three main elements: (a) the con- tent of the feedback (i.e., evaluative aspects, such as verification, and infor- mative aspects, such as hints, cues, analogies, explanations, and worked-out examples), (b) the function of the feedback (i.e., cognitive, metacognitive, and motivational), and (c) the presentation of the feedback components (i.e., timing, schedule, and perhaps adaptivity considerations).

 

 


 

Mason and Bruning (2001)


Mason and Bruning’s theoretical framework, depicted in Figure 5,


프레임 워크에서 가져온 일반적인 권장 사항은 간단한 (낮은 수준) 또는 복잡한 (높은 수준) 작업의 맥락에서

  • 성취도가 낮은 학생에 대한 즉각적인 피드백이 지연된 피드백보다 우월하다는 점입니다.

  • 지연된 피드백은 성취도가 높은 학생, 특히 복잡한 업무에서 권장된다.

 

The general rec- ommendation they have drawn from the framework is that immediate feedback for students with low achievement levels in the context of either simple (lower level) or complex (higher level) tasks is superior to delayed feedback, whereas delayed feedback is suggested for students with high achievement levels, especially for complex tasks.

 

 


 

Summary and Discussion


Recommendations and Guidelines for Formative Feedback


Tables 2, 3, 4, and 5 present suggestions or prescriptions based on the current review of the formative feedback literature.


사실, 비고츠키 (Vygotsky, 1987)는 지성이 동기 부여와 정서적 (또는 정서적 인) 측면에서 분리 됨으로써 심리학의 연구가 손상되었다고 지적했다. 이것은 정서적 혼란이 정신 활동 (예, 불안, 화가)을 저해 할 수 있다고 주장한 연구자들 (Goleman, 1995; Mayer & Salovey, 1993, 1997, Picard et al., 2004) , 우울한 학생들은 배우지 않습니다). 따라서 미래 연구의 한 가지 흥미로운 영역은 피드백과 결과 성과에서 정서적 인 요소들 간의 관계를 체계적으로 조사하는 것입니다.

In fact, Vygotsky (1987) noted that the study of psychology had been damaged by the separation of the intellectual from the motivational and emotional (or affective) aspects of thinking. This seems to be supported by a growing number of researchers (e.g., Goleman, 1995; Mayer & Salovey, 1993, 1997; Picard et al., 2004) who have argued that emotional upsets can interfere with mental activities (e.g., anxious, angry, or depressed students do not learn). Thus, one intriguing area of future research is to systematically examine the relationship(s) between affective components in feedback and outcome performance.


TABLE 2 Formative feedback guidelines to enhance learning (things to do)

 

  • 학습자가 아닌 과제에 집중하십시오.

  • 학습 향상을 위해 정교한 피드백을 제공하십시오.

  • 정교한 피드백을 다루기 쉬운 단위로 제시하십시오.

  • 피드백 메시지로 구체적이고 명확하게 작성하십시오.

  • 피드백을 가능한 한 간단하지만 (학습자의 요구와 교육 제약에 따라) 단순하게하지 마십시오.

  • 성과와 목표 사이의 불확실성을 줄입니다.

  • 서면 또는 컴퓨터를 통해 공정하고 객관적인 피드백을 제공하십시오. 피드백을 통해 "학습"목표 오리엔테이션을 추진하십시오.

  • 학습자가 솔루션을 시도한 후에 피드백을 제공하십시오.

Focus feedback on the task, not the learner.

Provide elaborated feedback to enhance learning.

Present elaborated feedback in manageable units.

Be specific and clear with feedback message.

Keep feedback as simple as possible but no simpler (based on learner needs and instructional constraints).

Reduce uncertainty between performance and goals.

Give unbiased, objective feedback, written or via computer. Promote a “learning” goal orientation via feedback.

Provide feedback after learners have attempted a solution.

 


 

TABLE 3 Formative feedback guidelines to enhance learning (things to avoid)


  • 규범적인 비교를하지 마십시오. 전반적인 성적 제공에 대해서는 신중해야합니다.

  • 학습자를 낙담 시키거나 학습자의 자존심을 위협하는 피드백을 제공하지 마십시오.

  • '칭찬'은 아끼십시오

  • 구두로 피드백을 제공하지 않도록하십시오.

  • 학습자가 적극적으로 참여하는 경우 피드백을 통해 학습자를 방해하지 마십시오.

  • 항상 정답으로 끝나는 점진적인 힌트를 사용하지 마십시오.

  • 피드백 프리젠 테이션 모드를 텍스트로 제한하지 마십시오.

  • 광범위한 오류 분석 및 진단의 사용을 최소화하십시오.

Do not give normative comparisons. Be cautious about providing overall grades.

Do not present feedback that discourages the learner or threatens the learner’s self- esteem.

Use “praise” sparingly, if at all.

Try to avoid delivering feedback orally.

Do not interrupt learner with feedback if the learner is actively engaged.

Avoid using progressive hints that always terminate with the correct answer.

Do not limit the mode of feedback presentation to text.

Minimize use of extensive error analyses and diagnosis.

 

 

 


 

TABLE 4 Formative feedback guidelines in relation to timing issues


  • 원하는 결과와 일치하도록 피드백 타이밍을 설계하십시오.

  • 어려운 작업의 경우 즉각적인 피드백을 사용하십시오.

  • 상대적으로 간단한 작업의 경우 지연된 피드백을 사용하십시오.

  • 절차 적 또는 개념적 지식을 유지하려면 즉각적인 피드백을 사용하십시오.

  • 학습 이전을 촉진하려면 지연된 피드백 사용을 고려하십시오.

Design timing of feedback to align with desired outcome.

For difficult tasks, use immediate feedback.

For relatively simple tasks, use delayed feedback.

For retention of procedural or conceptual knowledge, use immediate feedback.

To promote transfer of learning, consider using delayed feedback.

 

 


 

TABLE 5 Formative feedback guidelines in relation to learner characteristics


  • 학습성이 높은 학습자에게는 지연된 피드백 사용을 고려하십시오.

  • 저조한 학습자에게는 즉각적인 피드백을 사용하십시오.

  • 학습 성이 낮은 학습자의 경우 지시 (또는 정정) 피드백을 사용하십시오.

  • 학습성이 높은 학습자에게는 촉진 적 피드백을 사용하십시오. 저학년 학습자의 경우 스캐 폴딩을 사용하십시오.

  • 학습성이 높은 학습자에게는 검증 피드백으로 충분할 수 있습니다.

  • 저학년 학습자의 경우 올바른 응답과 정교한 피드백을 사용하십시오.

  • 낮은 학습 지향 (또는 높은 성취 지향)을 가진 학습자에게 구체적인 피드백을 제공하십시오.

 

For high- achieving learners, consider using delayed feedback.

For low-achieving learners, use immediate feedback.

For low-achieving learners, use directive (or corrective) feedback.

For high- achieving learners, use facilitative feedback. For low-achieving learners, use scaffolding.

For high- achieving learners, verification feedback may be sufficient.

For low-achieving learners, use correct response and some kind of elaboration feedback.

For learners with low learning orientation (or high performance orientation), give specific feedback.


 




The general question is: What level of feedback complexity yields the most bang for the buck?


 

 

 

 

 







    This article reviews the corpus of research on feedback, with a focus on formative feedback—defined as information communicated to the learner that is intended to modify his or her thinking or behavior to improve learning. According to researchers, formative feedback should be nonevaluative, supportive, timely, and specific. Formative feedback is usually presented as information to a learner in response to some action on the learner’s part. It comes in a variety of types (e.g., verification of response accuracy, explanation of the correct answer, hints, worked examples) and can be administered at various times during the learning process (e.g., immediately following an answer, after some time has elapsed). Finally, several variables have been shown to interact with formative feedback’s success at promoting learning (e.g., individual characteristics of the learner and aspects of the task). All of these issues are discussed. This review concludes with guidelines for generating formative feedback.


    임상교육에서의 피드백(Do's and Don'ts) (Perspect Med Educ. 2015)

    Guidelines: the do’s, don’ts and don’t knows of feedback for clinical education

    Janet Lefroy · Chris Watling · Pim W. Teunissen ·

    Paul Brand




    Do’s—educational activity for which there is evidence of efficacy

    Don’ts—educational activity for which there is evidence of no efficacy or of harms (negative effects)

    Don’t Knows—educational activity for which there is no evidence of efficacy


    Table 1 Summary of guidelines. For the individual clinical supervisor giving feedback









    Introduction


    요약 표

    In the summary (Table 1) we indicate the strength of this evidence and therefore of our recommenda- tion using the criteria outlined in Table 2.



    어떤 피드백이 유용할지를 알기는 쉽지 않다. (주는 것과 받는 것 사이의) 'feedback gap'이 있다. 이것이 supervisor에게 의미하는 것은 trainee의 요구와 수용도를 진단하지 않고 피드백을 주는 것은 노력낭비의 가능성이 있다. formative feedback의 효과는 trainee가 얼마나 스스로 향상되고자 하는지와 그 자신감의 강도에 달려있다.

    It is not easy to know what feedback will be useful to a trainee. There is a recognized feedback gap (between feed- back given and what is received by the trainee [1]). What this means for supervisors is that delivering feedback with-out  first  diagnosing  our  trainee’s  need  and  receptiveness risks wasting effort. The impact of formative feedback will depend on the strength of the trainee’s desire to improve and their confidence in their ability to do so [2].





    Foundation paper


    Methods and ‘way of working’


    결과

    Results



     

    Background evidence to guidelines for the individual clinical supervisor giving feedback


    프로세스에 관한 Do's

    Do’s for the process of feedback


    1. 피드백이 단순히 한 사람이 다른사람에게 도움을 주기 위한 정보를 주는 것이라 생각하면 안된다. 피드백은 문화/가치/기대/개인적 역사/관계/권위에 의해 영향을 받는 사회적 상호작용이다. 피드백을 conversation이 아니라 commodity라고 보면 안된다.

    Guideline 1. Do realize that feedback is not just one per- son providing information to another to help them improve. Feedback is part of a social interaction influenced by cul- ture, values, expectations, personal histories, relationships, and power. Do treat feedback as a conversation rather than as a commodity.


    부정적인 피드백을 어떻게 Reconciling and assimilating 할 것인지는 사회적 맥락에 따라 달라진다. 피드백의 효과와 역할에 대한 사회적/문화적 가치를 강조한 문헌도 있다. 피드백을 단지 'trainee의 수행을 향상시키기 위한 목적으로 trainee의 관측된 수행능력을 표준적 기준에 대해서 비교한 정보를 제공하는 것'으로 바라볼 때, 피드백을 통해 드러나는 문화/가치/기대/역사/관계/권위를 무시하는 것이다.

    Reconciling and assimilating negative feedback with views held by the individual was found to be influ- enced by social context. The differences between professions described in that study highlight the influence of social and cultural values on the role and impact of feedback [4]. Viewing feedback only as ‘specific information about the comparison between a trainee’s observed performance and a standard, given with the intent to improve the trainee’s per- formance’ [5] ignores the complex ways in which culture, values, expectations, personal histories, relationships, and power manifest themselves through feedback [6].




    2. 피드백이 효과가 있으려면 Trainee는 반드시 피드백을 믿을 만credible하다고 믿어야 한다. Credible한 피드백은 직접적 관찰을 통해서 well-informed되어있어야 하며, 피드백이 신뢰할만한trustworthy 곳에서 와야 한다. Supervisor로서 신뢰할만한 롤모델이 되어야 한다.

    Guideline 2. Do recognize that trainees must perceive feed- back as credible in order for it to be influential. Credible feedback is well-informed, typically by direct observation of the task or event, and it comes from a trustworthy source. Make sure that you as supervisor set a good example as a credible role model.


    학습자는 credible한 사람의 피드백은 가치있게 여기지만, credibility가 낮다고 생각되는 피드백은 무시하는 성향이 있다. 피드백의 credibility는

    • 출처의 credibility,

    • 그 피드백이 informed and created하는 프로세스,

    • 피드백의 내용과 특성 그 자체

    등에 의해 영향을 받는다.

    A number of qualitative studies have shown that learners value feedback that they deem to be credible, but may dis- miss feedback that they perceive to lack credibility [7–10]. The credibility of feedback is influenced by the credibility of the source, by the process by which the feedback was informed and created, and by the content and characteristics of the feedback itself [9].



    3. Trainee의 역량수준과 과제의 복잡도에 따라 피드백의 타이밍을 정하라

    Guideline 3. Decide the timing of feedback depending on the competence level of the trainee and on the complexity of the task.


    적절한 시간에 제공해야 피드백의 수용도와 활용이 높아진다. 그러나 '적절한 시간'의 의미는 모호하다. 예컨대 술기의 시뮬레이션 교육에서는 terminal feedback이 concurrent feedback보다 낫다.

    Studies of learners’ perceptions of effective feedback have highlighted the importance of timeliness to learners’ acceptance and use of feedback [7, 11], the concept of optimal timeliness appears to be a nuanced one. For exam-ple, for simulation training of procedural skills, terminal feedback (at the end of the task performance) may be supe-rior to concurrent feedback (during the task performance) for enhancing learning [12]. 


    타이밍에 따라 서로 다른 레벨의 피드백이 필요하다

    Hattie and Timperley [13] provide evidence that differentlevels of feedback deserve different timing.




    4. Trainee가 피드백을 찾아나서게끔, 그리고 그것을 자신의 퍼포먼스 향상에 사용하게끔 장려하라.

    Guideline 4. Do encourage trainees to look for feedback and use it to enhance their performance.


    자존감에 영향을 받을까봐 피드백을 두려워한다. trainer에게 좋은 인상을 남기고 싶을 수도 있고, 어떻게 향상되어야 할지에 대한 피드백 정보를 원할 수도 있다. 피드백-탐색에는 복잡하고 방대한 무의식적 심리가 있다.

    Our trainees may approach feedback with trepidation about the harm it might do to their self-esteem [15]; they may desire to make a good impression on their trainer among others; they may also desire the information which feedback gives them about how to improve [16]. These are the com- plex and largely unconscious psychological influences on feedback-seeking [17].



    Teunissen 는 목표지향성이 뚜렷할수록 피드백 탐색의 빈도가 더 높아짐을 보여주었다.

    Teunissen et al. showed that this relationship between goal orientation and increased fre- quency of feedback seeking also holds in a population of postgraduate medical trainees [20].


    내용에 관한 피드백의 Do's

    Do’s for the content of feedback


    5. 개별 trainee에 맞춤형 피드백을 하라. Trainee는 다음의 것에 도움을 받을 것이다.

    Guideline 5. Do tailor bespoke feedback to the individual trainee. The trainee might benefit from:

    • 잘 된 부분을 강화 ‘reinforcement of key points done well’;

    • 빼놓거나 개선되어야 할 부분을 지적 ‘identification of key points which might have been done better or omissions’;

    • 퀄리티 향상을 위한 전략 찾기 ‘working out strategies for improving the quality of their work’;

    • 자기-인식 향상 ‘an increased self-awareness’


     

    피드백은 '만족스럽지 못한 요소'에 대해서 제공될 때, 구체적인 학습 목표와 연결될 때 가장 효과적이다. 따라서 피드백의 내용은 supervisor와 trainee간 진단적, 지지적 대화에서 arise해야 한다.

    Feedback is most effective if directed at unsatisfactory ele- ments of performance and linked to specific learning aims [23]. The content of feedback should therefore arise from a diagnostic and supportive dialogue between supervisor and trainee [24].




    6. 어떻게 과제가 수행되었고, 어떻게 수행되었어야하는지에 대한 구체적 피드백을 주라

    Guideline 6. Do give specific feedback, focused on how the task was done and how that type of task should/might be done.


    퍼포먼스와 관련이 없는 일반적 정보 (좋은 퍼포먼스는 무엇이고 나쁜 퍼포먼스는 무엇인지) 등은 구체적인 코멘트보다 효과가 떨어진다. 구체성이 부족한 피드백은 medical training에서 오래 전부터 문제로 지적되어왔다. 그러나 피드백의 구체성을 향상시키는 것이 더 효과적인 학습을 가져왔다는 근거를 찾는 것도 모호하다. Goodman 등은 피드백의 구체성을 향상시키는 것이 초기 퍼포먼스에는 도움이 되었지만, exploration을 discourage하여 independent performance에 필요한 더 심화된 학습은 훼손하는 것으로 나타났다.

    General infor- mation unrelated to the performance, comments about a good or poor performance or compliments are less effec- tive than specific comments [27, 28]. Lack of specificity has repeatedly been identified as an all-too-common weak- ness of the feedback that is typically exchanged in medi- cal training [29]. When, however, one looks for evidence that increasing feedback specificity leads to more effective learning, the waters become murkier. Goodman et al. [30], for example, showed that increasing the specificity of feed- back benefits initial performance, but discourages explora- tion, potentially undermining the deeper learning required for independent performance.


    Kluger and DeNisi의 피드백 이론은 task에서 멀어지고 individual에 대한 피드백은 덜 효과적이라고 지적한다. 줄이면, 자존감에 위협이 되는 피드백은 도움이 되지 않는다.

    Kluger and DeNisi’s feedback intervention theory, also posits that feedback becomes less effective as attention shifts away from the task and toward the individual; in short, feedback that is threatening to self-esteem is unlikely to be effective [28].


    요약하자면, 피드백은 task level에서 이뤄질 때 가장 효과적이고, 자기조절이나 프로세스에 대한 피드백은 'deep processing and mastery of tasks'에 도움이 될 수 있다. 그러나 'person level'에 맞춘 피드백은 하지 마라.

    To sum up feedback is most effective when directed at the task level and may assist in ‘deep processing and mastery of tasks’ when it is about processing of tasks or self-regulation. A ‘Don’t’ is providing feedback that focuses on the person level.


     

    7. 피드백이 '최소한의 역량수준 달성'을 위한 것인지, '최적의 수행능력과 비교한' possible variation에 대한 성찰인지를 명확히 하라

    Guideline 7. Do make sure to indicate whether feedback is about necessary improvement for minimally acceptable per- formance or whether it is a reflection on possible vari ations to build upon adequate performance.


    자기조절이론에서는 두 개의 기본적 자기조절시스템이 공존하면서 상충한다고 말한다. 두 개의 시스템은 promotion- 그리고 prevention- 접근법이다. supervisor는 자신의 trainee가 promotion focus인지 prevention focus인지 파악해야 한다.

    Self-regulation theories suggest that within each of our trainees are two basic self-regulation systems which co- exist but may conflict [33]. These two systems—the pro- motion (doing things because you want to) and prevention (doing things because you have to in order to avoid harm) approaches—may both be active in response to feedback [34]. It is important that the supervisor recognizes that his/ her trainee is predominantly in promotion or prevention focus with respect to the focus of feedback, because

    • positive feedback is more effective in motivating performance improvement for learners in promotion focus, while

    • negative feedback is more useful in motivating performance improvement for learners in prevention focus [28].


    Prevention system은 performance goal을 가지는 사람들에게 왕성하다. 이들은 '난 이미 충분한 실력을 갖췄음'을 증명하고 싶어하며, 비판을 받고싶지 않아한다. Trainee가 (performance goal을 가지고 있을 때보다) learning goal 을 가지고 있을 때 피드백이 더 잘 작동한다. 따라서 피드백 그 자체가 trainee를 performance goal을 가지게 압력을 주어서는 안된다.

    The prevention system is active in individuals with performance goals—aiming to prove that one is already adequately competent and avoiding criticism. Feedback works best for learning when the trainee has learning goals rather than performance goals [17, 35] so it is important that the feedback itself should not push the trainee towards performance goals.



    성적은 피드백이 promotion and prevention response 모두를 자극하는 방식이다. 이러한 심리상태에서 성적을 받는 것은 향상전략을 탐구하는 행동을 강화해주는 것으로 나타났으며, 특히 성적부여의 기준이 확실할 때 그러하다. 성적의 나쁜 효과도 나타나곤 하는데, 고등교육과 의학교육에서 formative feedback에서는 grade를 optional하게 만들 것을 제안한다.

    Grades are a clear and non-nuanced form of feedback which can trigger both promotion and prevention responses in trainees [28]. Receiv- ing grades in this frame of mind was found to enhance the seeking of strategies for improvement, especially if criteria for allocation of grades are understood [26, 36]. Harmful effects of grades have also been noted in some participants in school, higher education and medical education [13, 26, 34], suggesting that making grades optional in formative feedback may be wise,


     

    8. Actionable한 피드백을 줘서 Trainee가 개선을 위한 전략을 수립할 수 있게 하라. Trainee의 수행능력에 대해 토론한 후, 현재 역량을 넘어설 수 있게 하는 '가이드'나 'scaffolding'을 제공하라

    Guideline 8. Do ensure that feedback is actionable, enabling the trainee to construct strategies for improve- ment. After discussing the trainee’s performance of a task, provide some guidance or ‘scaffolding’ to enable them to step beyond their current competence.


    학습자의 피드백 경험에 대한 연구는 피드백은 actionable할 때 그 가치가 높아짐을 강조한다. Actionable한 피드백은 학습자 발달의 로드맵을 보여주며, 강점을 강화하고 약점을 보완하기 위한 명확한 suggestion을 준다.

    Research into learners’ experiences of feedback has highlighted the value placed on feedback that is actionable [38]. Actionable feedback contains a roadmap for learner development; it provides explicit suggestions for building on strengths or addressing weaknesses in performance.


     

    ‘scaffolding’ 의 이론적 개념은 Wood의 '학습의 구성주의자 모델'에서 잘 개발되었다. 이들은 Vygotsky의 ‘zone of proximal development’ 에 근간을 두고 있다.

    The theoretical concept of ‘scaffolding’ by tutors has been well developed by Wood et al. in their constructivist model of learning [39]. They based this on Vygotsky’s many studies in children of how the learner is helped to develop into their ‘zone of proximal development’ (beyond their current ability) by social interaction with tutors or peers [40].


    성인학습의 사회적 상호작용에서도 scaffolding 개념이 도움이 된다.

    In the social interactions of adult learning the scaffolding concept can also be helpful [27, 41–43].


    Wood가 말한 scaffolding은..

    The tasks of scaffolding as described by Wood et al. are:

    ● Orient the learner to the task

    ● Simplify into steps

    ● Motivate to maintain effort to achieve the goal

    ● Highlight critical features of the task

    ● Control frustration and the risk of failure

    ● Provide a model of the required actions


    역량수준이 낮은 trainee에게 scaffolding은 directive feedback을 주거나 specific instruction을 주는 것이 될 수 있다. 높은 수준의 trainee에게는 less directive 할 수 있다. 즉 Suggestion, hints, tips 등이 그것이다.

    For trainees with a low level of competence, scaffolding involves giving directive feedback or specific instructions; for trainees with a high level of competence scaffolding can be less directive i.e. suggestions, hints and tips for (further) improvement (facilitative feedback) [27]. 


    9. 개선전략을 토의할 때 Trainee motivation에 관심을 가지라

    Guideline 9. Do attend to trainee motivation when discussing strategies for improvement.


    성인학습자에게 동기부여는 주로 '내부에서 형성'된다. 그리고 이는 피드백에 의해서 영향을 받는다. 충분히 동기부여가 되고 충분한 자기효능감을 위해서 supervisor은 피드백을 주면서 trainee의 반응을 보아야 한다.

    In adult learners, motivation is more likely to be internally generated [44] but it is no less important to learning, and is influenced by feedback [28, 34, 45]. In aiming for sufficient motivation to learn to do the task and sufficient self-efficacy that their effort is likely to succeed, clinical supervisors should check trainee response to their feedback as they go along.



    10. 피드백을 주는 구체적인 전략에 무관하게, trainee를 '성찰적 대화'에 참여하게 해서 '자기평가'와 'Supervisor의 관측&설명'을 같이 생각해볼 수 있게 하라

    Guideline 10. Regardless of the specific approach to feedback that is used, do engage the trainee in a reflective conversation that marries their self-assessment with your observations and elaborations.


     

    trainee를 토론에 참여시킴으로서, supervisor는 reflection-in-action and reflection-on-action을 활용하여 자신의 퍼포먼스에 대해 인식하게끔 할 수 있다. 그 이후의 코칭은 trainee의 자기평가에 대한 confirming 또는 challenging이 될 수 있다.

    By involving trainees in a discussion, supervisors can raise their awareness of their performance relative to their goals of quality performance through reflection-in-action and reflection-on-action [22, 32]. Coaching then includes confirming or challenging the trainee’s self-assessment,

     

    피드백의 효과에 영향을 주는 것은 많은데, 어떻게 피드백을 전달할 것인가는 task, recipient, relationship에 따라 달라질 수 있다. 피드백은 ‘A supported sequential process rather than a series of unrelated events’가 되어야 한다.

    There are many factors which influence the effect of feedback, and the choice of how to deliver the feedback will depend on the task, the recipient and the feedback relationship [23]. Feedback should be ‘A supported sequential process rather than a series of unrelated events’ [32].


    여러 피드백의 방법이 있으나, 뭐 하나가 가장 효과적인 것은 없다.

    Several approaches to feedback have been described in the literature (sandwich, Pendleton, reflective feedback conversation, agenda-led outcome-based analysis, feedfor- ward), but no single approach has been established to be the most effective.


     

    샌드위치

    The feedback sandwich (in which the supervisor describes

      • 잘 된것 what went well,

      • 개선할 것 what can be improved,

      • 잘 된 것 then re-emphasizes what went well) [46]

     

    칭찬의 심리적 효과를 활용하여 비판이 더 잘 수용되게 한다. 이러한 접근법은 피드백 관계의 미묘한 초반 관계에서 효과적이지만, 일단 관계가 확고하면 불필요할 수 있다.

    harnesses the psychological effect of praise to enable the reception of criticism. This approach is thought helpful especially in the delicate start of a feedback relationship, but unnecessary once the relationship is robust. Evidence of its effectiveness is lacking. 



    펜들톤

    Pendleton [47] outlined a method for giving feedback aiming to engage the learner in self-reflection and to bal- ance positive and critical feedback.


    ● Check the learner wants and is ready for feedback. 

    ● Let the learner give comments/background to the mate- rial that is being assessed. 

    ● The learner states what was done well

    ● The observer(s) state(s) what was done well

    ● The learner states what could be improved.

    ● The observer(s) state(s) how it could be improved

    ● An action plan for improvement is made.



    이 규칙은 투박하다거나 정형화되었다고 비판받으나, 피드백을 주고받는 것에 도움이 된다.

    The rules have been criticized as clunky and formulaic, but the framework can be helpful for learning to give and receive feedback.


     

    성찰적 피드백 대화

    The reflective feedback conversation unfolds like this: 


    ● The teacher asks the learner to share concerns about performance. 

    ● The learner describes concerns and what they would have liked to have done better

    ● The teacher provides views and offers support, then asks the learner what might improve the situation

    ● The learner responds, then the teacher elaborates on that response, correcting if needed, and checking understanding.



    대화는 '프로세스'이며 'event'가 아니다. Revisiting과 F/U이 필요하다.

    Importantly, the conversation should be viewed as a process rather than an event; revisit- ing and follow-up are often required.


     

    Agenda-led outcome-based analysis

    Agenda-led outcome-based analysis

    • starts with the trainee’s agenda,

    • looks at the outcomes they were aiming for,

    • encourages self-assessment and problem-solving,

    • pro- vides balanced feedback and suggests alternatives

     

    학습자 중심적 방법

    This method is described in Kurtz, Silverman and Draper’s Cal- gary Cambridge method for teaching communication skills [49] and is a learner-centred way of identifying the most helpful focus for a feedback discussion. 



    feedforward interview

    피드백은 아니다. 'best performance'에 초점을 두어 preferred standard 와 the actual state 의 차이를 회피하기 위한 전략.

    By contrast, the feedforward interview [34] is not actu- ally a technique for feedback. It aims to avoid creating a dis- crepancy between a preferred standard and the actual state of affairs (seen as a key element of feedback, but also recog- nized as problematic for trainees who have low self-esteem) by focusing learners on their best performances.

      • The trainee recalls peak moments in his/her performance and

      • is asked to reflect on what conditions in themselves and their surround- ings made that possible,

      • then considers strategies to ensure sustainable peak performance.

     

    Trainee에게 peak experience를 정기적으로 feedforward하여 'promotion'접근법에 필요한 ground를 마련하라고 제안함.

    Kluger and van Dijk recom- mend periodical feedforward interviews with trainees about their peak experiences, partly in order to prepare the ground for necessary feedback to be received with a ‘promotion’ approach.




    하지 말 것

    Don’t assume


    11. 한 방법이 늘 작동할거라고는 생각하지 말라. Player와 Context가 변화하면 가장 유용한 접근법도 변화한다.

    Guideline 11. Don’t assume that a single approach to feedback will be effective with all trainees or in all circum- stances. As the players and the contexts change, so too does the most useful approach to feedback.

     

     

    ● You know what a trainee wants to learn 

    ● You know why a trainee is struggling 

    ● You know if or why a trainee wants feedback 

    ● You know what information a trainee takes out of a situ- ation or feedback conversation


     

    임상과제나 교육과제에 따라 사람마다 매우 다르다. 피드백에 대한 반응도 학습자마다 다르고, 수행능력이 비슷해도 다를 수 있다. Dijksterhuis 는 개인간 피드백의 수용도와 반응도 차이를 보여주었고 Kluger and van Dijk는 regulatory focus theory가 이러한 차이의 원인을 설명해줄지도 모른다고 했다. Watling등의 임상학습환경에서의 연구

    Individuals vary in their orientation toward clinical and edu- cational tasks. Responses to feedback also differ between learners, even regarding similar performance on similar tasks. Dijksterhuis showed individual variability in the acceptance and responsiveness to feedback [50]. Kluger and van Dijk [34] proposed that regulatory focus theory might explain some of the observed variability in feedback responses, and Watling et al.’s naturalistic exploration of the usefulness of this theory showed it offered some insights into feedback responses in clinical learning situations [25].


    피드백의 impact에 대한 차이는 개인간 차이를 넘어선다. 피드백에 대한 반응은 학습문화와 학습문화가 피드백에 대해서 만들어내는 규범 또는 기대에 따라서도 다르다. 그리고 Context는 피드백의 관계적 맥락을 포함하는데, 이 역시 큰 영향을 준다. Telio는 최근 교사-학습자간 'educational alliance'가 피드백에 미치는 맥락적 영향을 강조했다. 이러한 차이가 있기 때문에 교사는 학습자에 대한 이해를 바탕으로 다양한 접근법을 사용해야 한다. 피드백 exchange는 교사와 학습자의 목표가 서로 합치되었을 때 가장 효과적일 것이다.

    Variability in the impact of feedback extends beyond the individual. Responses to feedback are also shaped by learning culture, and the norms and expectations it creates for feedback [38], And context, including the relational ele-ment of feedback, is increasingly recognized as influential;Telio [51] has recently highlighted the contextual influence of the ‘educational alliance’ that develops between teacher and learner on the feedback that is exchanged. In the face of such variability, teachers must develop versatile approaches to feedback that are grounded in an understanding of the learner. The feedback exchange is perhaps at its most effec-tive when teachers’ and learners’ goals are aligned [38]. 



    12. F/U 없이 피드백을 제공하지 마라. 향상된 퍼포먼스를 보여줄 기회가 따라오지 않는 피드백은 효과를 미치지 않는다.

    Guideline 12. Don’t provide feedback without follow-up. Trainees are unlikely to be influenced by feedback that is not followed by an opportunity for them to demonstrate improv- ing performance.







    13. (어디서 듣고 온) poorly informed 정보로 피드백을 주지 마라. 그렇게 하는 것은 피드백에 대한 가치만 떨어뜨릴 뿐이다.

    Guideline 13. Don’t provide feedback that is poorly informed (or is based on hearsay); doing so diminishes the value that trainees assign to feedback in general.


    Trainee들은 피드백을 원칙적으로는 중요하다고 생각하며, 피드백을 주는 것을 임상교사가 갖추어야 할 바람직한 행동이라고 생각한다. 그러나 현실에서 medical training동안 받는 피드백의 퀄리티는 낮다고 보고되고 있으며, 직접관찰이 제한되는 것 등의 이유로 피드백은 poorly informed된다. 그 결과, trainee는 외부 피드백의 가치를 일반적으로 평가절하하게 되고, 자기평가에 의존한다.

    Surveys have demonstrated that trainees value feedback in principle, and value the provision of feedback as a desired quality of clinical teachers [53]. In reality, however, the quality of the feedback received in medical training is often reported as low, and poorly informed due to factors includ- ing limited direct observation of performance. As a conse- quence, trainees may begin to devalue external feedback in general, relying instead on self-assessment [10, 11].



    14. 피드백이 정서에 미치는 부정적 영향을 과소평가하지 말라. 정서적 스트레스는 피드백 수용의 장애가 된다.

    Guideline 14. Don’t underestimate the emotional impact of feedback that is perceived as negative. Emotional distress may be a barrier to acceptance and use of feedback.

     


     

    Feedback intervention theory 는 피드백이 trainee의 자존감에 위협을 가하면 효과성이 떨어진다고 강조한다. Sargeant는 '자기평가와 반대되는 부정적 피드백을 받는 사람은 스트레스를 받고, (그러한 피드백을) 수용하고 그에 따라 행동할 수 있는 능력을 제한시키는 지속적 감정을 유발한다. Eva는 피드백의 해석과 수용이 감정의 복잡한 interplay에 영향을 받는다고 하였는데, 여기에는 자신감/공포감 등이 있고, 학습자에게 피드백을 전할 때 self-concept를 유지할 수 있게 해주는 것이 중요함을 강조하였다.

    Feedback intervention theory [28] posits that feedback which threatens self-esteem is much less likely to be effec- tive. Sargeant found that those who had received negative feedback that conflicted with their self-assessment experienced distressing and long-lasting emotions that limited their ability to accept and act upon the feedback [31]. Eva showed that the interpretation and acceptance of feedback was influenced by a complex inter- play of emotions, including confidence and fear, and high- lighted the importance of allowing the learner to maintain their self-concept when delivering feedback [2].



    15. 성적을 매기는 기준을 설명하지 않은 채로, 그리고 개선 전략을 주지 않은 채로 성적만 매기지 마라

    Guideline 15. Don’t give grades without explaining the criteria for allocation of grades and providing strategies for improvement.


    너무 자주 comment box에는 아무것도 쓰여있지 않은 상태로 성적이 부여된다. 성적은 학생의 동기를 저하시키거나 노력을 덜 하게 만들기 때문에 formally assessing하지 않고 성적을 주는 것은 피해야 한다.

    all too often grades are given with the comment boxes left blank [56]. Because of the potential for grades to demotivate or to reduce effort, it has been suggested that it might be wisest to avoid giving grades except when formally assessing the learner (in infre- quent ‘high stakes’ assessments) [58].



    잘 모르는 것.

    Don’t knows


    16. 피드백의 credibility를 결정하는 것은 무엇인가?

    Guideline 16. What determines the credibility of feedback?


    17. 얼마나 많은 내용을 피드백해야 하는가?

    Guideline 17. How much is the right amount of content when giving feedback?


    18. '개방적, 안전한 상호작용'은 무엇이 결정하는가?

    Guideline 18. What determines the ‘open and safe interac- tion’ in the feedback conversation?


    19. Trainee의 반응에 영향을 주는 것은?

    Guideline 19. What influences the trainee’s response? (constructive or destructive outcomes)


    Workplace learning theorists는 어떻게 개인적 요인과 사회문화적 요인이 상호작용하는지 이해해야 한다고 강조한다. Billett는 affordances and agency를 강조하였다.

    Workplace learning theorists (e.g. Eraut [61], Billett [62]) have highlighted the need to understand how individual and the sociocultural influences on learning interact. Billett emphasizes the notions of affordances and agency;

    • 학습환경은 다양한 Affordance(학습의 기회)를 제공한다
      a learning environment offers a range of affor- dances, or opportunities to learn, but

    • 개별 학습자는 Agency(주체성)을 발휘하여 이 Affordance에 engage해야한다.
      an individual learner must exercise agency to engage with those affordances.

     

    피드백은 affordance (좋은 피드백이 이뤄지는가?)와 agency(학습자들이 피드백에 engage하는가?) 둘 모두와 관련된다.

    Feedback challenges may lie with either affordance (is good feedback made available to learners?) or agency (do learn- ers choose to engage with feedback?), or both;


    20. 노골적인 동료와의 비교가 도움이 되는가? 노골적인 'standard'와의 비교는 도움이 되는가?

    Guideline 20. Is overt comparison with peers when made by the supervisor helpful to the trainee? Indeed, is overt comparison with required performance standards helpful?


    동료와 비교하는 것은 피드백을 덜 효과적으로 만드는데, 왜냐하면 task에서 벗어나 self에 초점을 맞추게 하기 때문이다. Self-referenced 와 other-referenced 피드백은 직간접적으로 self에 초점을 맞추게 만든다. 다른 사람과의 비교에서 부정적인 평가를 받으면 자존감에 위협을 느낄 수 있고 performance goal orientation을 유도하며, 학습을 방해한다. 그러나 이러한 우려에도 불구하고 comparison의 가치를 강조하기도 한다.

    What about comparison with peers? There is evidence that feedback becomes less effective as its focus moves away from the task and toward the self [13, 28]. Both self- referenced and other-referenced feedback (in)directly focus the attention to the self. Unfavourable comparisons with others may threaten self-esteem and promote a performance goal orientation, potentially hindering learning [17]. But despite these concerns, some research has suggested value in comparisons:



    21. Written summary가 학습에 도움이 되는가?

    Guideline 21. Does a written summary of the feedback dis- cussion enhance learning?


    의대생들은 비공식적 verbal feedback을 WBA에서의 written feedback보다 좋아한다.

    Medical students have been found to value informal ver-bal feedback more than formal workplace-based assessment(WBA) with written feedback [66, 67]. 


    written summary 의 가치는....

    The value of the written summary is therefore secondary but could include:


    Aiding reflection on the feedback at a later date 

    Aiding discussion between tutor and trainee at a later date 

    Enhancing tutor effort at the time of generating the feedback




    학습문화를 위한 가이드라인

    Guidelines for the learning culture: what elements of learning culture support the exchange of meaningful feed- back, and what elements constrain it?


    Do's

    Do’s


    22. 피드백을 학습프로세스에 포함시키기 위한 시스템 접근법을 갖추라

    Guideline 22. Do have a systems approach, building feed- back into the learning processes.


     

    각 기관은 장기적 교사-학습자 관계가 번창할 기회를 줄 수 있다.

    Institutions can create opportunities for longitudinal teacher-learner relationships to flourish, such as extended placements [68–70].



    Trainee 에 대한 기대는 reflection-on-feedback 일 것이며, 개선전략 도입을 강제할 시스템이 있어야 한다. 신입 Trainee는 특정 academic community의 규칙을 안내받아야 한다.

    Expectations of the trainee might be reflection-on-feedback with some system of reinforcing implementation of strategies for improvement [32]. New trainees will require induction into the rules of the particular academic community.



    피드백의 문화를 확립하려면, trainee에 대한 피드백 뿐만 아니라 supervisor에 대한 정기적 피드백 시스템도 있어야 한다.

    In order to ensure a climate of feedback, an institution should provide a system of regular feedback not only for trainees but also for supervisors [32, 72].


    FD course 제공 외에도 supervisor와 그들의 사회적 네트워크에 대한 교육적 support를 제공함으로써 피드백을 사용하고 받아들이도록 촉진할 수 있다.

    In addition to providing faculty development courses, educational support can be offered to supervisors and the supervisors’ social networks can be used and supported to facilitate acceptance and use of feedback [73].



    23. 장기적인 신뢰할 수 있는 supervisor-trainee 관계 형성을 지원하라. 영향력이 큰 피드백은 신뢰관계의 맥락에서 번창한다.

    Guideline 23. Do support the development of longitudinal, trusting supervisor-trainee relationships in medical train- ing; influential feedback thrives in the context of trusting relationships.


    Trainee가 supervisor와 관계형성이 되어있을 때 피드백의 credibility를 신뢰할 수 있고, 교사의 목표와 자신의 목표의 합치를 이룰 수 있다. Bok 등이 말한 바와 같이 교사-학생의 durable한 관계는 즉각적으로 유도한다.

    When trainees can build a relationship with their supervi- sors, it allows them to trust the credibility of the feedback they receive and the alignment of the teacher’s goals with their own. As Bok et al. showed, durable teacher-learner relationships also prompt readily [69]. 


    Bates 등에 따르면 그러한 관계가 '비판적 피드백의 건설적 해석'을 가능하게 해준다. 학생은 challenging or corrective 피드백을 supportive하게 해석한다. 신뢰가 있고 지지적인 관계에서 피드백은 더 credible하다고 여겨질 가능성이 높다. 피드백 프로세스에서 관계의 centrality를 인지하여, educational alliance라는 개념이 제안되었고, 이는 교사-학습자 관계의 link를 이해하고 그 안에서 이뤄지는 피드백의 impact를 이해하기 위한 것이다.

    Bates et al. [67] found that such relationships afforded ‘constructive interpretation of critical feedback’ (p. 366); students were able to interpret even challenging or corrective feedback as supportive. Within a trusting and supportive relationship, feedback is also more likely to be viewed as credible [50, 78]. Recog- nizing the centrality of relationship in the feedback process, the concept of the ‘educational alliance’ has been proposed as a framework for understanding the links between the teacher-learner relationship and the impact of the feedback generated within it [51, 79].



    24. 비디오를 사용하라

    Guideline 24. Do use video review with feedback as a com- ponent of training.


    Supervisor의 피드백은 직접 관찰을 할 때와 비디오로 볼 때 다르지 않을 수 있으나, Trainee는 자신의 강점과 약점을 confirm할 수 있다.

    The super- visor’s feedback may not differ whether following direct observation or following video observation but the trainee will be able to confirm the strengths and weaknesses in their own performance.


    Supervisor은 같은 비디오 녹화를 보고도 상당히 다른 피드백을 준다. Systematic review에서 비디오녹화를 자기평가하면서 보았을 때는 효과적이지 않았지만, 전문가의 피드백과 연결되었을 때는 더 효과가 좋았다. 

    Supervisors differ considerably in the feedback they give after reviewing the same videotaped consultation [80]. In a systematic review Hammoud et al. con- cluded that video review with self-assessment alone was not found to be generally effective for medical students, but when linked with expert feedback it was superior to tra- ditional feedback alone [81].



    25. 피드백이 일상화/정기화/가치로운 실천공동체를 형성하라

    Guideline 25. Do promote communities of practice in clin- ical workplaces in which feedback is routine, regular and valued.



    피드백이 '일반적인 것'이라고 여겨지는 환경이 필요하다. 이는 trainee가 supervisor에게 피드백을 줄 수 있음을 의미한다. 비판적 피드백이 learning culture에 의해서 normalize 되었을 때 더 활발히 exchange 된다.

    environment in which providing feedback is considered ‘normal’. This would mean, for example, that trainees are encouraged to give feedback to their supervi- sors [82, 83]. It has also been found in the training of ath- letes and musicians that critical feedback is exchanged more readily when it is normalized by a learning culture [38, 84].


     

     

    26. 근무지 교육시스템에서 공식적 역할을 하는 사람이 그 역할을 인식하고, 학습자의 교육적 목표가 무엇이 되어야 하는지 이해하게 하라

    Guideline 26. Make sure that those who have a formal role in a workplace’s educational system are aware of that role and understand what learners’ educational objectives should be.


    Boor 는 좋은 관계 외에도 학습자는 교육시스템을 이해하고, 그 지식을 개별 학습자에게 적용할 수있는 supervisor를 좋아한다. VDV는 programmatic assessment에서 다음과 같이 말했으며, 이는 피드백에 대해서도 마찬가지다.

    Boor et al. found that, next to the importance of a good relationship, learners value clinical supervisors who are aware of the educational system and expectations and who can apply that knowledge to the individual learner [85]. Van der Vleuten’s comments on programmatic assessment are useful here:

     

    ‘If a programme of assessment is to provide meaningful outcomes, all the players should understand what they are doing, why they are doing it, and why they are doing it this way. ’ [86]

    If we substitute ‘feedback’ for ‘assessment’, the comment rings equally true.


    27. 팀에서 정기적으로 피드백을 주고, 피드백을 주는 방식에 대해서 성찰하고, 피드백 제공 역량을 유지/향상시키기 위한 refresher course를 듣게 하라

    Guideline 27. Make sure that the team give feedback regu- larly, reflect on the practice of giving feedback, and follow refresher courses to maintain and improve competency in providing feedback.


    평가 프로세스에 대한 교수들의 insight가 없는 것도 문제이다. Tutor training이 필요한데, tutor는 academic and social needs를 정확히 진단해야 한다. 학습자의 관점을 이해하고 강조해야 하며, 적절한 scaffolding tool을 사용할 스킬이 있어야 한다.

    Lack of faculty insight in the assessment process remains an issue [87]. The feedback landscape described by Evans [1] indicates the need for tutor training: the tutor must accu- rately diagnose academic and social needs; understand and empathize with the learner’s perspective, and have skills to employ appropriate scaffolding tools.



    Don’ts


    28. 피드백 효과성 향상을 위해 FD에만 의존하지 마라

    Guideline 28. Don’t rely exclusively on faculty develop- ment to improve the effectiveness of feedback.


    역사적으로 피드백에 관한 FD가 피드백의 퀄리티를 향상시키기 위한 primary approach였다. 그러나 '어떻게 피드백을 줘야하는가'에 집중한 나머지 '어떻게 학습자가 받아들일 것인가'를 놓쳐왔다. 또한 효과적인 피드백을 만드는 학습문화의 역할, 건설적 비판의 normalizing, 피드백을 학습에 가치있게 만들기 등등도 놓쳐왔다. FD는 필요하지만 충분하진 않다.

    Historically, faculty development in feedback delivery has been the primary approach to improving the quality and effectiveness of feedback [29, 73]. This focus on how feedback is given ignores the important element of how it is received by learners [88]. The crucial role of

    • learning culture in making effective feedback possible,

    • normalizing constructive criticism, and

    • establishing the value of feed- back for learning

    ...is also missed by an approach focused on individual teachers [38]. Faculty development is important but not sufficient;



    29. mini-CEX같은 임상스킬의 formal assessment가 관측과 피드백 없이 종결되게  하지 마라

    Guideline 29. Don’t allow formal assessments of clinical skills, such as the mini-CEX, to be completed without obser- vation and feedback.


     

    모르는 것

    Don’t knows


    Guideline 30. What are the vital components that ensure a constructive system of workplace learning that caters to trainees, workers, and the educational system? How can the institution nourish a climate which encourages the provi- sion and seeking of feedback?


    '피드백이 효과가 있었나' 가 아니라 '어떤 상황에서 효과가 있는가'이다.

    They conclude that thereal question is not whether feedback works, but under what circumstances is works best.


    팀워크 프로세스에 대한 피드백, 팀워크 프로세스가 작동하는 동안의 피드백이 팀이나 팀원의 퍼포먼스에 대한 피드백보다 효과가 있었다.

    Feedback about and during the process of teamwork was more reliably effective than feedback about performance given to the team or to individuals within the team.



    Guideline 31. Is it most effective to give feedback to indi- viduals alone or in a group setting?




    Guideline 32. Does the use of formative assessment out- comes for summative purposes (such as having supervisors provide formative feedback that at the end of a rotation is also used for a summative assessment) corrupt a well-inten- tioned educational system?



    형성평가를 최종평가의 data point로 사용하는데 어려움을 겪고 있으며, 이로 인해서 형성평가가 assessment-for-leraning의 역할을 하지 못하고 있다.

    A qualitative study with clinical undergraduate veterinary students and their supervisors highlighted that both struggled with formative assessments that are used as ‘data points’ for a final summative judgment. As a result, the formative assessments did not play the powerful assess- ment-for-learning role they are meant to have in lum based on programmatic assessment [69].


    의학교육은 총괄평가와 형성평가의 경계를 흐릿하게 할 뿐만 아니라 교사의 코치로서의 역할과 평가자로서의 역할 구분도 흐릿하게 한다. 교사가 이렇게 dual role을 하게 될 때 피드백의 퀄리티와 impact가 훼손될 수 있다.

    Medical education not only blurs the line, at times, between summative and formative assessment, but also blurs the line for its teachers between the roles of coach and assessor. Recent literature has begun to challenge this approach, suggesting that the quality and impact of feedback may be compro- mised when the teacher is assigned this dual role [38, 91, 92].


    피드백 프로세스는 개선전략을 만들어내지 못하면 불완전한 것이다. 가장 좋은 피드백 프로세스 루프는 어떤 결과가 있었는지에 대한 subsequent assessment이다.

    The feedback process is incomplete if it does not result in the generation of strategies for improvement— either recommendations, or self-generated as a result of feedback. And the best feedback process loops back into a subsequent assessment with feedback about whether this has resulted in improved clinical performance.

     


     


    Summary



    34. Kluger A, Van Dijk A. Feedback, the various tasks of the doctor, and the feedforward alternative. Med Educ. 2010;44:1166.


     



     



     2015 Dec;4(6):284-99. doi: 10.1007/s40037-015-0231-7.

    Guidelines: the do'sdon'ts and don't knows of feedback for clinical education.

    Author information

    • 1Keele University School of Medicine, Clinical Education Centre RSUH, ST4 6QG, Staffordshire, UK. j.e.lefroy@keele.ac.uk.
    • 2Schulich School of Medicine and Dentistry, Western University, Ontario, Canada.
    • 3Maastricht University and VU University Medical Center, Amsterdam, The Netherlands.
    • 4Isala Klinieken, Zwolle, The Netherlands.

    Abstract

    INTRODUCTION:

    The guidelines offered in this paper aim to amalgamate the literature on formative feedback into practical Do'sDon'tsand Don't Knows for individual clinical supervisors and for the institutions that support clinical learning.

    METHODS:

    The authors built consensus by an iterative process. Do's and Don'ts were proposed based on authors' individual teaching experience and awareness of the literature, and the amalgamated set of guidelines were then refined by all authors and the evidence was summarized for each guideline. Don't Knows were identified as being important questions to this international group of educators which if answered would change practice. The criteria for inclusion of evidence for these guidelines were not those of a systematic review, so indicators of strength of these recommendations were developed which combine the evidence with the authors' consensus.

    RESULTS:

    A set of 32 Do and Don't guidelines with the important Don't Knows was compiled along with a summary of the evidence for each. These are divided into guidelines for the individual clinical supervisor giving feedback to their trainee (recommendations about both the process and the content of feedback) and guidelines for the learning culture (what elements of learning culture support the exchange of meaningful feedback, and what elements constrain it?)

    CONCLUSION:

    Feedback is not easy to get right, but it is essential to learning in medicine, and there is a wealth of evidence supporting the Do's and warning against the Don'ts. Further research into the critical Don't Knows of feedback is required. A new definition is offered: Helpful feedback is a supportive conversation that clarifies the trainee's awareness of their developing competencies, enhances their self-efficacy for making progress, challenges them to set objectives for improvement, and facilitates their development of strategies to enable that improvement to occur.

    KEYWORDS:

    FeedbackFeedback culture; Feedback relationship; Formative assessment; Workplace based assessment

    PMID:
     
    26621488
     
    PMCID:
     
    PMC4673072
     
    DOI:
     
    10.1007/s40037-015-0231-7
    [PubMed] 
    Free PMC Article


    Ppsychometrics를 활용한 신뢰성있는 인지부하 측정: 객관적인 교육프로그램 최적화 모델(Med Educ, 2016)

    Reliable cognitive load measurement using psychometrics: towards a model of objective teaching programme optimisation

    K. H. Vincent Lau & Jeremy Moeller






    CLT의 구인과 측정에 대해서 Young and Sewell은 psychometrics의 역할을 주장함.

    In their 2015 review detailing the challenges of construct and mea- surement in cognitive load theory (CLT), Young and Sewell pro- posed the role of psychometrics in the more reliable measurement of cognitive load (CL).1


     

    Sewell 등은 "만약 어떻게 피훈련자가 working memory resources를 사용하는지 이해한다면, CL이 불균형을 이루거나 과도한 영역을 찾아낼 수 있을 것이고, 그 영역이 바로 학습에 제한을 가져오고 환자에 대한 harm의 위함이 증가하는 영역일 것이다"

    Sewell et al. summarise the practical applica- tion as: ‘...if we understand how trainees are using working mem- ory resources, we can identify areas in which CL is imbalanced or overwhelming – areas in which learning is limited and risk of harm to patients increases.’2


    간단히 말해서 CLT는 작업기억이 학습의 주요 제한요인이라고 하는 이론이며, 세 하위유형으로 구분한다.

    Briefly, CLT posits that working memory is the major limiting fac- tor in learning and divides CL into three subtypes:

    • intrinsic load (IL), which relates to the inherent diffi- culty of the learning task; 
      과제 자체의 내재한 난이도
    • extraneous load (EL), which relates to teaching techniques and the envi- ronment, and
      교육 테크닉과 환경과 관련
    • germane load (GL), which relates to the student’s use of deliberate strategies to enhance meaningful learning.
      유의미한 학습을 향상시키기 위한 학생의 전략

     

    CL 하위유형을 측정하는 목적은 교육프로그램을 adjust하여 distraction을 최조화시켜 EL을 낮추는 것이다.

    The purpose of measuring CL subtypes is to adjust a teaching programme through iterations by, for example, lowering EL by minimising distractions.



    Sewell 은 한 연구에서 자기보고식의 19문항짜리 CL 하위유형 측정 도구를 개발하였다.

    In this study, Sewell et al. developed a self-report, 19-itempsychometric tool that reliably measures CL sub- types in colonoscopy.2


    이 연구에서 GL은 CL의 하위유형이라는 근거를 제시했지만, 모든 전문가가 이를 인정하는건 아니다. 

    The study also provides evidence that GL is a distinct CL subtype, a view not shared by all experts in the field.3


    CLT가 지난 수십년간 잘 성립되었고 널리 퍼졌지만 CL을 측정하려는 노력은 중간 정도의 성공만 거두었을 뿐이다. 일반적으로 말해서, IL은 여러 점수체계간 상관관계가 높지만 EL과 GL은 좀 더 elusive하다.

    Whereas CLT has been well estab- lished over a few decades and widely applied, efforts to measure CL have seen only moderate suc- cess. In general, IL is most reliably quantified with reasonably high degrees of correlation between different scoring systems,9 whereas EL and GL are more elu- sive.4,9


    CL 측정도구를 더 발전시켜야 한다. 가장 최근의 연구는 2013년의 trial이다.

    In this context, there was a need for further development of CL measurement tools using rigorous qualitative and quantitative meth- ods. The most recent and notable study is a 2013 trial examining CL in a university statistics course,11


    CL을 낮추는 것은 post-test에서 더 나은 수행능력과 관련있었다.

    It also demonstrated that decreasing CL was associated with better performance in a post-test.11


    이 연구에서는 CL 측정의 실현가능성 뿐 아니라, 측정도구 개발의 섬세한 디테일까지 묘사하였다. 

    Importantly, this study2 not only demonstrates the feasibil- ity of measuring CL, but describes in elegant detail the development of the measuring tool, providing a guide for future similar efforts.



    CBTM와 같은 high-impact 교육세팅에 있어서 validated CL측정도구는 반복과 리모델링을 통하여 최적화될 수 있을 것이다. 그러한 프로그램을 개발하려면 다음이 필요하다.

    in specific high-impact education settings like dissemi- nated multi-institution pro- grammes such as in CBTMs(computer- based teaching modules (CBTMs)), a validated tool for CL measure- ment will allow for optimisation through iterations and remod- elling. The development of such a programme may involve:

    • (i) identifying a well-defined and manageable learning setting (with complexity on the order of colo- noscopy teaching);
    • (ii) developing and validating a self-report tool to measure CL for this setting;
    • (iii) seeking additional validation by selecting a representative subset of subjects to undergo non-self- report external assessment such as by secondary task performance, and
    • (iv) optimising CL subtypes through iterative changes.


    더 넓은 맥락에서 이 연구는 실시간학습의 측정에 psychometrics를 적용할 수 있다는 것을 validate하였다. 재생산가능한 결과를 보여줌으로써 저자들은 학습자들이 기존에 생각했던 것보다 더 나은 reporter가 될 수 있으며, 자기보고로 수집한 데이터가 쉽사리 평가절하discredited 되지 말아야 한다고 주장했다.

    In a broader sense, the study also validates psychometrics in its new application to the measurement of real-time learning. By demonstrat- ing reproducible results, the authors suggest that learners may be better reporters than previously thought, and data obtained from self-report should not be easily discredited in educa- tion research.2









    13 van Merrie¨nboer JJG, Sweller J. Cognitive load theory in health professional education: design principles and strategies. Med Educ 2010;44 (1):85–93.


    14 Lau KHV. Computer-based teaching module design: principles derived fromlearning theories. Med Educ 2014;48 (3):247–54.


    15 Qiao Y, Shen J, Liang X, Ding S, Chen F, Shao L, Zheng Q, Ran Z. Using cognitive theory to facilitate medical education. BMC Med Educ 2014;14:79.


    17 Jelovsek JE, Kow N, Diwadkar GB. Tools for the direct observation and assessment of psychomotor skills in medical trainees: a systematic review. Med Educ 2013;47 (7):650–73.







     2016 Jun;50(6):606-9. doi: 10.1111/medu.13046.

    Reliable cognitive load measurement using psychometricstowards a model of objective teachingprogramme optimisation.

    Author information

    • 1New Haven, Connecticut, USA.
    PMID:
     
    27170078
     
    DOI:
     
    10.1111/medu.13046
    [PubMed - in process]


    숙련된 운동능력에 대한 과도한 생각: 또는 왜 가르치는 사람은 할 수가 없는가 (Psychonomic Bulletin & Review, 2008)

    Overthinking skilled motor performance: Or why those who teach can’t do

    KRISTIN E. FLEGAL

    University of Michigan, Ann Arbor, Michigan

    AND

    MICHAEL C. ANDERSON

    University of St. Andrews, St. Andrews, Scotland







    세계정상급 선수이든, 주말에만 즐기는 사람이든, 골프실력을 향상시키는 것은 마음으로부터 시작한다. 과도하게 샷에 대해 생각하는 것에서 벗어나서, 무의식에 맡겼을 때 무슨 일이 생기는지를 알면 놀랄 것이다.

    Whether you’re a world-class player or a weekend en- thusiast, improving your golf game begins with your mind. You may be amazed to discover what happens when you free yourself from overthinking your shots and let your unconscious mind play the game. 


    Marlin Mackenzie (1990)


     

    운동능력을 숙달한 모든 사람은 절차적지식(Procedural Knowledge)을 묘사하는 것이 말로는 쉽지만 실제로 하기에는 쉽지 않다는 것을 알 것이다. 절차적 지식을 묘사하는 것이 어려운 것에는 그만한 이유가 있다. 무수한 인지심리학 연구와 인지 신경과학연구로부터 절차적지식과 서술적지식(declarative knowledge)가 서로 다른 표상(representation)에 의해서 support되고, 서로 별개의 신경시스템에 의해서 중재됨을 보여주고 있다. 그러나 만약 숙달된 운동선수에 대해서 보자면, 절차적지식과 서술적지식의 관계는 중립적인 것조차 되지 못한다. 즉, 자신의 운동능력에 대해서 의식적으로 성찰하는 것이 그것을 적절히 수행하는데 오히려 방해가 된다. 여기서, 우리는 이 negative한 관계를 살펴보고자 한다. 이미 잘 학습한 스킬의 요소요소에 집중하면서 동시에 그 스킬을 수행할 때 스킬 수행이 저하된다. 우리가 주장하는 것은, (스킬에 대해) 과도하게 생각하는 것의 부정적 효과는 생각-수행이 동시적으로 일어날 때online distraction 뿐만 아니라, 절차적 스킬을 말로 묘사하는 것이 장기적으로도 나중의 수행에 부정적 영향을 준다는 것을 보여주려하는 것이다.

    Anyone who has mastered a motor skill appreciates that describing the procedural knowledge underlying one’s performance is, literally, easier said than done. Describing procedural knowledge is difficult with good reason. Abundant evidence from cognitive psychology and cognitive neuroscience indicates that procedural and declarative knowledge are supported by different repre- sentations and mediated by distinct underlying neural systems (e.g., Anderson, 1982; Fitts & Posner, 1967; Ga- brieli, 1998; Keele & Summers, 1976). But if skilled ath- letes are to be believed, the relation between procedural and declarative knowledge may be considerably less than neutral: Reflecting consciously on what one knows about a skill often undermines its proper execution. Here, we examine this putative negative relationship. There is em- pirical evidence that attending to the components of a well-learned skill can impair concurrent performance (Beilock, Carr, MacMahon, & Starkes, 2002; Gray, 2004; Jackson, Ashford, & Norsworthy, 2006). Our claim is that the negative effects of overthinking are not limited to online distraction, but also reflect a longer term im- pact of verbalizing procedural skills on later execution.


     

    언어의 뒤덮음 효과Verbal Overshadowing Effect

    The Verbal Overshadowing Effect


    말로 표현하기 어려운 인식경험을 말로 표현하는 것이 기억의 유지retention을 손상시킨다는 연구가 있다. Schooler and Engstler-Schooler 는 은행강도 얼굴과 같은 말로 표현하기 어려운 자극을 묘사하는 경우에 나중에 (그것을 하지 않은 사람보다) 그 얼굴 인식을 더 못한다는 것을 보여줬다. 이러한 용어는 '언어의 뒤덮음 효과'라고 불리는데, '말로 표현하는 것'이 언어-기반 표상representation을 만듦으로서 말로 표현하기 어려운 perceptual memory를 덮어버린다는 것이다. 이 현상은 perceptual experience의 세부적 사항이 말로 표현될 수 있는 수준을 넘어설 때 발생한다. 말로 표현하는 것이 어렵지 않은 것(구두 진술) 또는 논리적 문제해결에 대해서는 verbalization이 방해가 되지 않고 오히려 가끔은 더 기억을 촉진한다. 방해 효과는 오직 개개인이 묘사불가능한 특성을 갖는 stimulus를 묘사하려고 시도할 때 발생한다. 언어의 뒤덮음 효과는 맛, 오디션, 지도지억, insight 문제해결 등에서도 나타나면서 시각적 기억에만 해당되는 것이 아님을 보여줬다.

    There is a precedent for the hypothesis that verbaliz- ing ineffable perceptual experiences impairs later reten- tion. Schooler and Engstler-Schooler (1990) observed that participants who described a difficult-to-verbalize stimulus (the face of a bank robber) from memory were much worse at later recognizing that face than were par- ticipants who did not put their memory into words. This effect was termed verbal overshadowing, on the basis of the idea that verbalization creates a language-based representation that overshadows difficult-to-verbalize aspects of the perceptual memory. The phenomenon oc- curs when the details of a perceptual experience exceed what can be conveyed in words. For easy-to-verbalize tasks, such as recalling a spoken statement (Schooler & Engstler-Schooler, 1990) or logical problem solv- ing (Schooler, Ohlsson, & Brooks, 1993), verbalization does not impede and, in some cases, facilitates mem-ory. Disruption occurs only when individuals attempt to describe memory for a stimulus with indescribable qualities. Verbal overshadowing has been observed in such domains as taste (Melcher & Schooler, 1996), au- dition (Houser, Fiore, & Schooler, 2003), map memory (Fiore & Schooler, 2002), and insight problem solving (Schooler et al., 1993), establishing that the effect is not limited to visual memories per se.


    언어로 묘사하는 행동이 언제나 perceptual experience의 기억을 손상시키는 것은 아니다. 예를 들면 Melcher and Schooler 는 이 현상이 그 영역에 대한 그 사람의 전문성(perceptual and verbal expertise)에 따라서 달라짐을 보였다. 와인 음주가에 대해서, 중등도 스킬을 가진 사람만이 맛을 묘사한 다음 와인을 더 못 맞췄으며, 이것이 보여주는 것이 perceptual expertise보다 verbal expertise보다 뛰어날 때만 발생함을 뜻한다. 후속 연구에서, 참여자들은 버섯에 대한 perceptual 훈련과 conceptual 훈련을 받았는데, perceptual training을 받은 사람에서만 verbal description이 인식능력의 저하를 보였다..

    Verbal description does not always impair memories for perceptual experiences, however. For instance, Melcher and Schooler (1996) found that verbal overshadowing de- pends on one’s relative perceptual and verbal expertise in a domain. In a study of wine drinkers, only those at an inter- mediate skill level recognized a previously sampled wine less accurately after describing it, suggesting that impair- ment occurs only when perceptual expertise outstrips ver- bal expertise. In a later study (Melcher & Schooler, 2004), participants received either perceptual or conceptual train- ing on recognizing types of mushrooms, after which they described a target mushroom from memory. Importantly, verbal description impaired later recognition of the target mushroom only for participants who had received percep- tual training.


    언어의 뒤덮음 효과가 처음 보고되었을 때, 언어로 표현하는 것이 기억을 손상시키는 기전은 사람으로 하여금 그 언어묘사과정에서 형성한 묘사에 기억을 의존하기 때문이라고 생각했다. 만약 어떤 사람의 외모에 대해서 그것을 묘사하면서 잘 못 기억하고 있다고 했을 때, 그러한 실수가 지속되어서 이후의 기억까지 왜곡시킬 수 있다. 이러한 재코딩recoding 간섭은 말로 묘사한 내용이 잘못된 기억의 가능성에 영향을 줄 수 있다는 점에서 부합한다. 예컨대, 확실하게 기억하는 내용에 대해서만 묘사하라고 하거나, 이름을 붙이기 어려운 encoding에 대해서 그것의 이름이 무엇인지 알려주면 이후의 기억 정확성이 향상된다.

    When the verbal overshadowing effect was first re- ported, it was thought that verbalization impaired mem- ory by leading participants to rely on memory for their generated descriptions during the test. If participants misremembered an aspect of a person’s appearance while describing that person, the error might persist and distort later memory. This recoding interference account is con- sistent with findings that the contents of a verbal descrip- tion can influence the likelihood of misremembering. For example, warning participants only to describe details that they can confidently recall (Meissner, Brigham, & Kelley, 2001) or providing participants with the names that they generated at encoding for hard-to-name forms (Brandi- monte & Collina, 2008) have been shown to enhance sub- sequent memory accuracy.


    그러나 몇몇 연구결과는 다른 기전이 언어의 뒤덮음 효과에 기여할 수 있음을 보여주었다. 

    However, several findings suggest that other mecha- nisms must contribute to verbal overshadowing effects.

    • 첫째, 묘사의 정확성과 언어의 뒤덮음 효과 사이에 관계가 불명확하다
      First, there is often little relationship between descrip- tion accuracy and verbal overshadowing (Fallshore & Schooler, l995; Schooler & Engstler-Schooler, 1990; however, see Meissner et al., 2001).

    • 둘째, (기억해야 하는 얼굴 말고 다른) 얼굴을 묘사하거나, 심지어 차를 묘사하는 과제를 수행한 이후에도 이후의 target face에 대한 인식(능력)이 손상되었다.
      Second, describing a different face (Dodson, Johnson, & Schooler, 1997) or even describing a car (Westerman & Larsen, 1997) can impair later recognition of a target face.

    이러한 결과는 언어의 뒤덮음에 대한 다른 관점을 제시한다. 구체적으로는, 인코딩 과정의 모드가 정상적인 외형처리모드configural processing mode 에서 의식적 특성-기반 처리 모드conscious, feature-based processing로 전환된다는 것이다. 이러한 전이-부적합 프로세스 이론transfer-inappropriate processing theory 에 따라서, 언어로 표현될 수 있는 것에 제한이 가해지고, 이것이 연구대상자들로 하여금 두드러지는 특징discrete feature의 묘사에 집중하게 하면서, 한편으로는 일상적으로 인식(능력)에 도움이 되는 자극요소stimulus component들 사이의 perceptual relationship을 배제시키는 것이다.

    These findings led to an alternate view that verbal overshadowing re- flects a shift in the type of processing used to recognize a perceptual stimulus. Specifically, the effect may reflect a change from the normal configural processing mode engaged during encoding to conscious, feature-based processing during the test. By this transfer-inappropriate processing theory (Schooler, 2002; Schooler, Fiore, & Brandimonte, 1997), limits on what can be expressed with language lead participants to focus their descrip- tions on discrete features (e.g., eye color, size of the nose) while excluding perceptual relationships between stimulus components that normally support recognition.




    본 연구

    The Present Experiment



    운동능력의 기저에 깔린 절차적지식은 언어로 표현될 수 있는 것을 한참 넘어서기 때문에, 숙련된 수행능력을 묘사하는 것은 perceptual experience를 묘사하는 것과 유사한 수준의 어려운 과제일 수 있다. 만약 그렇다면, 어떤 스킬을 묘사하는 것이 그것의 retention을 손상시킬 것이다. 이러한 의식적 성찰과 숙련된 행동 사이의 부정적 관계를 보여주는 근거들이 있다. 스킬 습득 연구에서 중요한 개념idea중 하나는, 전문성개발은 말로 표현하기 쉬운 서술적기억표상declarative memory representations 을 말로 표현하기 어려운 절차적지식으로 바꿔가는 과정이라는 것이다. 이러한 전환의 결과 중 하나는 전문가가 어떤 스킬을 수행하면서 초보적인 단계에 집중하면, 오히려 그 스킬의 수행에 어려움을 겪을 것이라는 점이다. 이러한 관점과 일치하게, Beilock, Carr 등은 경험이 많은 골프선수에게 스윙의 특정 요소에 집중하게 만들면 퍼팅 수행능력이 손상됨을 보여주었다. 반대로 초심자들은 어떤 스킬 수행의 요소element에 집중하는 것이 도움이 되었다.

    Because procedural knowledge under- lying a motor skill typically far exceeds what can be expressed verbally, describing memory for skilled per- formance might pose difficulties similar to describing a perceptual experience. If so, perhaps describing a skill will impair its retention. There is evidence consis- tent with this negative relationship between conscious reflection and skilled action. An important idea in skill acquisition research (Anderson, 1982; Fitts & Posner, 1967) is that development of expertise involves a shift from declarative memory representations that are easy to articulate to procedural knowledge that is difficult to put into words. One consequence of such a change is that performance might suffer when experts attend to the elementary steps of their proceduralized skill dur- ing execution (e.g., Baumeister, 1984; Beilock & Carr, 2001; Lewis & Linder, 1997). Consistent with this view, Beilock, Carr, et al. (2002) found that requiring experi- enced golfers to attend to a specific component of their swing impaired their performance in a putting task. Nov- ices, by contrast, benefit from focusing attention on ele- ments of skill execution (Beilock, Carr, et al., 2002) or the internal production of movement (Perkins-Ceccato, Passmore, & Lee, 2003; but see Wulf & Su, 2007).



    이러한 사례들은 모두 스킬을 수행하는 도중에 그 자체에 집중하는 것의 영향을 연구한 것이다. 그러나, 단순히 어떤 스킬을 수행한 이후에 그 스킬에 대해서 언어적으로 생각해보는 것조차도 이후 스킬 수행을 방해할 수 있는가는 불분명하다. 만약 어떤 스킬을 수행하는 방식을 묘사하는 것이 이후 수행에 방해가 된다면, 언어의 뒤덮음 현상이 절차적지식에까지 확장되는 것이다.

    These examples all involve attention to a skill during its ex- ecution, however; it is unclear whether simply thinking verbally about a skill offline disrupts later skilled per- formance. If describing the manner in which a skill is executed impairs later performance of that skill, it would extend the verbal overshadowing phenomenon to the do- main of procedural memory.



    방법

    METHOD


    Participants


    Materials


     

    설계와 절차

    Design and Procedure


    Forty participants each were randomly assigned to the verbaliza- tion and to the no-verbalization conditions. Within each condition, 20 participants were assigned to each of the lower and higher skill conditions on the basis of their self-reported nine-hole golf score.


    In the learning phase, the putting task was explained, and all par- ticipants were allowed as many trials as necessary (within an unre- vealed limit of 20 min) to reach the criterion of three consecutive on-target putts.


    After learning, verbalizers spent 5 min writing a detailed descrip- tion of how they performed the task. These participants were advised to think back to everything that they focused on while putting and were instructed to “record every detail that you can remember, re- gardless of how insignificant it may strike you.”

     

    Control participants (nonverbalizers) performed a verbal distractor task for the same duration, providing valence ratings for words with no association to golfing.

     

    In the final test, all participants returned to the putting task, and were again allowed as many trials as needed to reach three consecutive on-target putts. The number of trials to reachieve the criterion was measured.





    RESULTS


    Learning Performance

     



    Test Performance


     


     

    Characteristics of Verbalization Content

     

     

     


     



    고찰

    DISCUSSION


    이번 연구에서, 단순히 숙련된 운동능력을 묘사하는 것 만으로도 이후 수행에 부정적 영향을 주었다. 숙달된 골퍼가 5분을 투자했을 뿐인데, 관련없는 과제를 수행한 대조군보다 2배의 퍼팅을 하게 되었다. 반대로, 초보 골퍼는 verbalization에 영향을 받지 않았고, 오히려 약간 이득이 있었다. 이러한 고스킬 골퍼와 저스킬 골퍼 사이의 차이는 verbalization한 양이나 유형에 영향을 받는 것은 아닌 것으로 보인다. 따라서, performer가 높은 수준의 proceduralized knowledge가 있지 않는 한 verbalization 그 자체가 스킬의 수행을 손상시키는 것은 아니다. 이러한 결과는 perceptual experience에서의 episodic memory연구 결과와도 마찬가지이다.

    In the present experiment, we demonstrated that merely describing one’s skilled motor performance could impair the execution of that skill later on. When higher skill golf- ers spent 5 min describing their recent putting experience, they took twice as many putts to reachieve the putting criterion on a later test than did control participants who spent 5 min performing an unrelated verbal activity. In contrast, lower skill golfers were not measurably affected by verbalization and, if anything, slightly benefited rela- tive to lower skill control participants. This difference be- tween higher and lower skill golfers appears unrelated to the amount or type of verbalization content. Thus, verbal description by itself does not impair skill execution, un- less the performer possesses a higher degree of procedur- alized knowledge. These results accord well with verbal overshadowing findings concerning episodic memory for perceptual experiences.


    비록 이전 연구가 운동능력을 지나치게 생각하는 것overthinking이 부정적인 효과를 가진다고 지적하긴 했지만, 이번 연구에서는 그러한 효과가 offline에서도 발생함을 보여준 첫 번째 연구이다. 스킬을 수행하는 동안 자신의 스킬에 대해서 생각하는 것이 이중-과제 간섭dual task interference를 일으키는 것은 직관적으로 생각했을 때 말이 되나, 스킬을 수행한 이후에 그것을 생각하는 것도 안 좋은 영향을 준다는 것은 놀라운 사실이다. 실제로, 고스킬 골퍼들은 수행능력이 떨어졌다. 이러한 offline impairment는 이중-과제 간섭에 대한 기존의 해석에 새로운 것을 제안한다. 이전에는 shared process 혹은 capacity의 경쟁 때문에 발생한다고 생각했다. 비록 이번 연구결과가 그러한 요인이 없다는 것을 보여준 것은 아니지만, 그것보다는 보다 지속적인 어떤 요인이 있는 것으로 보인다. 의식적 성찰conscious reflection은 기저의 표상에 지속적인 변화를 가할 수도 있다. 실제로, 그러한 지속적 효과는 episodic memory에 대한 언어의 뒤덮음 효과를 보여준 많은 경우에 유사하게 일어났다.

    Although prior work has documented the negative ef- fects of overthinking on motor performance, the present study is the first to demonstrate that such effects occur after thinking about performance offline. Whereas it may seem intuitive that consciously reflecting on one’s skill during execution would cause dual-task interference, it is surprising that simply describing one’s skill after the fact can be so disruptive. Indeed, our higher skill golfers were reduced to the performance level of our lower skill golfers after verbalizing for only 5 min. The observation of such offline impairment suggests new interpretations of previ- ous work on dual-task interference in skilled performance, which previously would have been attributed to impaired execution resulting from competition for shared processes or capacity. Although the present findings do not negate such factors, they indicate that something more enduring may also occur: Conscious reflection may induce persist- ing changes in access to the underlying representations. Indeed, the prediction that such persisting effects might occur followed by analogy from the many instances of verbal overshadowing in episodic memory.


    무엇이 이 지속적 손상을 일으키는 것일까?

    What produces this enduring impairment?

     

    • 한 가지 메커니즘은 처리모드 간 경쟁이다. 절차적지식과 서술적지식을 처리하는 신경시스템은 경쟁적으로 상호작용한다. 서술적지식에 열심히 몰두vigorously engaging하는 것이 일시적으로 절차적학습 시스템을 방해할 수 있다. 이 관점에서 얼굴을 묘사하는 것이 묘사하지 않은 얼굴의 retention까지 손상시킨다는 연구결과와 같이, 언어로 묘사한 스킬 뿐 아니라 묘사하지 않은 스킬까지 손상시킬 수도 있음을 추측하게 한다. 이러한 결과는 또한 "숨막히는 압박choking under pressure"와 같이 스트레스 상황에서 숙련된 운동선수가 스킬에 초점을 둔 집중이 늘어날 수 있는 근거가 된다.
      One pos- sible mechanism is global competition between modes of processing analogous to that proposed by the transfer- inappropriate processing hypothesis of verbal overshad- owing (Schooler, 2002). In fact, research on learning sys- tems in animals and humans suggests such a hypothesis, indicating that the neural systems mediating procedural and declarative learning competitively interact (Poldrack et al., 2001; Poldrack & Packard, 2003). For example, le- sions to medial temporal lobe structures in animals im- prove procedural learning. Similarly, functional neuroim- aging studies have shown that humans disengage medial temporal lobe activity after practice on a skill-learning task. If Poldrack and colleagues’ hypothesis is correct, vigorously engaging declarative memory (as in a 5-min period of intensive verbal retrieval and description) should temporarily disrupt procedural learning systems. If so, the present effects may constitute the first behavioral evi- dence for the competitive learning systems hypothesis in humans. One intriguing prediction of this view is that ver- bal description should impair not only the described skill, but also other nondescribed skills acquired in the same session, much as verbally describing a face impairs reten- tion of nondescribed faces (Dodson et al., 1997). Compe- tition between explicit and implicit learning systems has also been investigated in categorization tasks that require integration of information according to rules that are dif- ficult to verbalize (Maddox & Ashby, 2004). Such differ- ential effects of rule-based and procedural learning have been suggested to contribute to “choking under pressure” (Markman, Maddox, & Worthy, 2006), which offers an in- teresting account of evidence that stressful situations can increase skill-focused attention in experienced athletes (Baumeister, 1984; Beilock & Carr, 2001; Gray, 2004).

    • 스킬에 초점을 둔 집중skill-focused attention의 파괴적 효과가 남아있을 수 있다. 절차화된 스킬 중 언어로 표현한 요소에 대한 집중attending이 남아있음으로써, 기저에 깔린 표상을 각각의 요소로 decompile하는 변화를 주었을 수 있다. 스킬의 요소 중 말로 묘사한 것에 집중하는 편향이 지속되면서 이후에 그것을 수행할 때 online interference를 일어킬 수 있다. 그러나 단순히 어떤 스킬의 요소에 집중하는 것 자체로는 충분하지 않으며, 그 행동에 대한 기억을 말로 표현하는 것verbalizing이 결정적이다.
      A second explanation for our results is a residual dis- ruptive effect of skill-focused attention prior to task ex- ecution. Verbalization required participants to focus atten- tion on the elements of their skilled performance, similar to instructions to continuously monitor one component of a golf putting or soccer dribbling task in the study by Beilock, Carr, et al. (2002). In our study, as in that one, attention to the component actions (as must occur during the act of verbalization) hurt skilled participants’ perfor-mance and modestly improved the performance of nov-ices. Perhaps attending to the (verbalizable) components of a proceduralized skill induces enduring changes to the underlying representation by decompiling it into its con-stituent parts, as proposed by Masters (1992). Alterna-tively, verbalization may not affect the procedural repre-sentation, but may instead induce a lingering attentional bias toward the described components of the skill. Simply paying more attention to what one has just described may cause online interference during the final test session, yielding results like those observed by Beilock, Carr, et al. (2002). We must emphasize, however, that attention to the components of the skill by itself may not be enough; verbalizing memory for the action may be critical for im-pairing later performance. In the verbal overshadowing domain, related findings have shown, for example, that mental imagery of a perceptual stimulus is not sufficient to produce verbal overshadowing, implicating verbal pro-cessing as integral to this kind of memory error (Fiore & Schooler, 2002; Schooler & Engstler-Schooler, 1990). 



    교수자 자신이 스스로 가르치는 내용에 능숙해지는 만큼, 그 스킬에 대해서 성찰하고 그 스킬의 기본을 말로 설명하는 것이 수행능력에 안 좋은 영향을 줄 수 있다. 가르치는 사람은, 막상 그 만큼 잘 하지 못한다.

    Whatever the mechanistic basis, the present finding in- dicates that simply verbally expressing one’s recent motor action may sow the seeds of poor execution during later performance. This observation may have repercussions for athletes, who depend on effective mental techniques to prepare for their events (e.g., implicit learning in a gym- nastics routine can be disrupted by verbalization, as shown by Brandimonte, Coluccia, & Baldanza, 2008). Equally, sports coaches and other physical activity instructors may wish to reconsider their opinions on strategies for impart- ing knowledge about motor control. Whereas verbalization assists in the early stages of acquiring a skill, it may im- pede progress once an intermediate skill level is attained. To the extent that instructors themselves are skilled in what they teach, the recurring need to reflect upon and articulate the basis of their skill may pose costs to their performance. Indeed, unless a concentrated effort is made to maintain one’s procedural expertise, the verbalization necessary for teaching may hasten a decline in skill, suggesting a new view of an old adage: Those who teach, cannot do.




     


     


     





     2008 Oct;15(5):927-32. doi: 10.3758/PBR.15.5.927.

    Overthinking skilled motor performance: or why those who teach can't do.

    Author information

    • 1Department of Psychology, University of Michigan, Ann Arbor, Michigan 48109, USA. kflegal@umich.edu

    Abstract

    Skilled athletes often maintain that overthinking disrupts performance of their motor skills. Here, we examined whether these experiences have a basis in verbal overshadowing, a phenomenon in which describing memories for ineffable perceptual experiences disrupts later retention. After learning a unique golf-putting task, golfers of low and intermediate skill either described their actions in detail or performed an irrelevant verbal task. They then performed the putting task again. Strikingly, describing their putting experience significantly impaired higher skill golfers' ability to reachieve the putting criterion, compared with higher skill golfers who performed the irrelevant verbal activity. Verbalization had no such effect, however, for lower skill golfers. These findings establish that the effects of overthinking extend beyond dual-task interference and may sometimes reflect impacts on long-term memory. We propose that these effects are mediated by competition between procedural and declarative memory, as suggested by recent work in cognitive neuroscience.

    PMID:
     
    18926983
     
    [PubMed - indexed for MEDLINE]


    의학교육자 되기: 동기, 사회화, 항해 (BMC Med Educ, 2014)

    Becoming a medical educator: motivation, socialisation and navigation

    Emma Bartle* and Jill Thistlethwaite





    Background


    의학교육인력이 부족함에 대한 우려

    There is increasing concern about a medical education workforce shortage [1].


    의사들은 언제나 교육에 헌신해왔다. 실제로 이것은 의-전문직업성의 요소로서 인정받고 있다. 이러한 책무성은 영국의 fundation curriculum과 호주의 curriculum framework에도 나타나있으며, 'teaching'을 모든 junior의사의 핵심 역량으로 보고 있다.

    Doctors have always had a commitment to teaching; indeed this is a well recognised component of medical professionalism. This responsibility is further professio- nalised in both the foundation curriculum in the UK (‘demonstrates the knowledge, skills, attitudes and beha- viours to undertake a teaching role’ [2]) and the Australian curriculum framework for junior doctors (‘plans, develops and conducts teaching sessions for peers and juniors; uses varied approaches to teaching small and large groups; incorporates teaching into clinical work; evaluates and re- sponds to feedback on own teaching’ [3]), which include teaching as a core competence for all junior doctors, re- gardless of their career choices.



    대학의학은 교육/연구/진료라는 세 가지 상호관련된 기둥 위에 세워져 있다.

    Academic medicine is founded on the three pillars of clinical service, research and teaching, and the interrela- tionships between them [4].


    연구와 교육과 관련된 academic career를 선택하는데의 장애요인으로 다음이 연구된 바 있다.

    The deterrents to pursuing an academic career both in relation to research and teaching have been summarised, for example by Walport in the UK, [5] as

    • 진입 경로 불분명 a lack of clear entry routes,

    • 구조화된structured 진로 structured career pathways,

    • (직장으로) 선택가능한 지역의 유연성 문제 flexibility in terms of the geography of available places,

    • 진료와 학문 업무(와 생활)의 균형 balance of work between service and academia (and life), and

    • 수련을 마칠 때까지의 적저한 구조와 재정지원 가능성 the availability of properly structured and funded posts on completion of training.


    추가적으로, 임상교육자의 성공은 교육이 아니라 연구생산성과 진료을 기준으로 측정된다는 문제가 있다. Harmon이 지적한 바와 같이, '연구'는 전통적으로 대학에 기반을 둔 의료전문직만이 할 수 있는 독특한 기여unique contribution으로 인식되어 왔다. 또한 '의학교육자'의 정의에 대한 명확한 합의가 부족하다.

    Additionally, as with academic careers in other sectors, the success of clinical educators is measured in terms of research pro- ductivity and clinical service rather than teaching. As Harmon notes, it is research that is traditionally perceived to be the unique contribution of university-based aca- demia to the medical profession and wider community [6]. We also note a lack of consensus as to the definition of a ‘medical educator’.



    대학의학의 학문구조와 연구를 중시하는 문화가 연구보다는 교육을 선호하는 임상의사들의 inauthenticity와 marginalisation 감정을 유발한다는 것을 보여주는 연구

    The study indicated that the discipline structure of academic medicine and the research-focused culture of academic and institutional expectations could engender feelings of inauthenticity and marginalisation for those clinicians who favour teaching over research.



    연구의 Context

    The context of this study 


    호주 퀸스랜드. 지난 10년간 호주정부는 의과대학을 늘림으로써 의과대학생 수를 크게 늘렸고, 의대 졸업생 수도 늘어났다.

    The setting for this study was Queensland, Australia. In the last decade the Australian government has signifi- cantly increased the number of medical students through the expansion of existing medical schools and funding of new schools. This has led to a marked rise in the number of medical graduates [8]



    2007년, 퀸스랜드 보건부는 임상교육 및 훈련 영역의 역량 강화 필요성을 느껴서 2008년 Medical Education Registrar (MER) 프로그램을 만들었다. MWAC가 네 명의 MER에게 재정을 지원했다.

    In 2007, Queensland Health (the funder and supplier of the state’s health service, with a network of 17 hospital and health service districts across Queensland) identified its own need to build capacity in the area of clinical education and training [10] and in 2008 developed the Medical Education Registrar (MER) scholarship pro- gram for this purpose (Table 1). Medical Workforce Advice and Coordination (MWAC) provided funding for four fulltime hospital-based MER positions per year across Queensland from 2008–2012.




    12개월 프로그램

    The MER positions were designed as a 12-month period of developmental experience for junior doctors, within the context of the registrar (resident) career continuum.



     

    Table 1 The objectives of the MER position [11]

     



    이론적 프레임워크

    Theoretical framework



    사회-인지-경력이론(socio-cognitive career theory , SCCT)를 따라서 데이터 분석

    As discussed further below we adopted socio-cognitive career theory (SCCT) as a framework for the data analysis as we read through the transcripts.


    Bandura의 사회-인지 이론으로부터 나온 SCCT개인의 진로흥미/진로선택/수행능력의 상호작용과 어떻게 개인요인(기대성과, 자기효능감, 목표)이 장애요인에 대한 맥락적/경험적 지지와 관계되는지 이해하도록 도와줌. SCCT에서는 뛰어난 수행능력을 갖추기 위해서는 개개인이 component skill과 강력한 자기효능감이 필요하다고 가정함.

    Derived primarily from Bandura’s general social cognitive theory [13], SCCT provides a useful conceptual framework for understanding the interplay between an individual’s career interests, choice and performance, as well as understanding how personal factors, such as out- come expectations, self-efficacy beliefs and personal goals, can interrelate with contextual and experiential supports or barriers [12]. SCCT assumes that individuals require both component skills and a strong sense of self-efficacy to achieve competent performance [14].


    자기효능감은 특정 역할을 수행하는 능력에 대산 스스로의 판단, 구체적인 상황에서 성공에 대한 판단 등이며 타인/관찰학습/행동/맥락요인에 영향을 받는다.

    Self-efficacy is used to describe people’s self-judgements of their capability to perform a role and succeed in specific situations, and can be influenced by other people, observa- tional learning, behaviour and contextual factors [13,15].



    자기효능감에 대한 신념은 활동의 선택/노력과 끈기/사고 패턴/감정적 대응에 영향을 미친다. 자기효능감은 학업과 진로 관련 선택 및 그 수행능력의 예측인자.

    In particular, self-efficacy beliefs are thought to impact on an individual’s activity choice, effort and persistence, thought patterns, and emotional reactions [12]. Self-effi-cacy has been found to be predictive of academic andcareer-related choice and performance [16-18].


    기대성과outcome expectation은 개개인이 주어진 맥락에서 구체적인 행동이 어떤 결과를 가져올 것인가에 대한 추측이다. 개인목표는 한 사람이 자신의 행동을 조직화하고 행동을 guide하기 위해서 설정한 개인적/전문직적/생활적 목표이다.

    Outcome expectations refers to an individual’s estimate that a specific behaviour within a given context will lead tocertain outcomes [12,13]. Personal goals describe the per-sonal, professional and lifestyle goals set by individuals to organise their behaviour and guide their actions


    진로 장애요인이란 희망 진로에 불협화음을 내는 개인적/맥락적 요인 등이며, 기대성과에 부정적 영향을 준다.

    notion of career barriers, personal and contextual factors which result in dissonance among career aspirations, progress and achievement. Overall, car-eer barriers engender negative outcome expectations in those contemplating a particular career pathway [19]. 


     

    임상연구 진로 개발에는 여러 어려움이 있다.

    [20]. They found that the development of a clinical re- search pathway has many potential challenges, including

    • low self-efficacy beliefs,

    • over commitment,

    • negative out- come expectations,

    • ill-defined personal goals, and

    • the conflicting demands and expectations of the multiple environments an individual may inhabit.



    방법

    Methods


     

    자료 수집 

    Data collection

    1회 혹은 2회 

    Interviews with participants were conducted at either one or both of two time-points:


    인터뷰어 사이의 일관성 유지 

    We developed an interview question guide, ensuring that a level of consistency in the broad topics covered was maintained across the three interviewers (the two authors and a research assistant).


    자료 분석

    Data analysis


    프레임워크분석: 이미 정의된 연구질문에 기반하여 귀납적으로 접근함 

    The interviews were transcribed verbatim and firstly analysed by the two authors and the research assistant using framework analysis [23], a deductive approach based on our defined research questions and the medical education workforce issues prompting the study.


    주제 사이의 관련성, SCCT와의 비교, 결과의 해석 등을 논의하여 presented text를 최종 결정. RATS 가이드라인 따름.

    The association between themes, compari- son with SCCT and interpretation of the findings were discussed by authors both to finalise the text presented. Our study adheres to the RATS guidelines for reporting qualitative studies.




    결과

    Results


     

     

    진로 선택의 동기: 더 나은 교육을 위한 열망wanting

    Motivation for career choice: wanting to provide better education


     

    자신의 부정적인 경험이 (자신을 가르친 사람보다) 더 나은 방식으로 가르칠 수 있겠다는 관심을 갖게 해줌

    Their negative experi- ences in particular motivated their interest in being able to teach better than their own teachers.

     

    senior의사로서 교육이 기본 역할이라는 것을 알게 되면서, 교육스킬은 nurture가 필요한 것이며, natural하게 얻어지는 것이 아니라는 것을 알게 됨.

    while identifying that teaching is a fundamental role of the senior clinician, they highlighted that it is a skill that needs nurturing ra- ther than something that would come naturally to a doctor:



    Table 2 Themes arising from the data

     



    개인의 목표, 기대, 자기주도성의 필요성

    Personal goals, expectations and the need for self-direction


     

    MER들은 커리어 목표가 있고, 더 나은 교육자가 되려는 동기부여 요인이 있었지만, 그 외에 이 프로그램에서 무엇을 기대했는지 고려하지 않았음.

    While the MERs had career goals and the aim of deve- loping as better educators as motivating factors to undertake the MER position, many had not considered what they otherwise hoped to achieve during the post itself.


    이러한 목표설정의 부재, 잘 정의되지 않은 목표는 이 프로그램을 시작할 때 방향설정orientation이 부족한 것, 그리고 이 포지션에 있으면서 무엇이 가능하고 허용되는지를 결정하는 것에 대한 자기주도성이 필요했던과 관련됨.

    This lack of goal setting, or ill-defined goals, was partly related to the lack of orien- tation at the start of the post and partly due to the need for self-direction in determining just what was possible and permissible during the position itself:


    그러나 자기주도성(이 요구되었던 것)을 후향적으로 성찰해볼 때 장점도 있음. 

    However in retrospect this self-direction was seen as an advantage:


    동료와 선배 의사들이 이 역할이 무엇인지 잘 이해하지 못함.

    The lack of understanding of the role by their peers and more senior clinicians could be frustrating:


    '일부 병원에서는 의학교육을 행정으로 보기도 하며, 어떤 전공의들은 명백히 자신의 역할이 아닌 행정업무를 하고 있었다.'

    ‘Some hospitals also interpreted medical education as medical admin and so we found that some registrars were doing administration which is not part of it obviously.’ (12b) 



    임상 로테이션 중의 과도한 업무와 MER에서 요구되었던 자기주도성은 transition 기간을 더 어렵게 만들었는데, 시간이 너무 많아서, 혹은 그 포지션에서 맡은 역할들에게 요구되는 행동의 균형을 맞추기 위해서가 그 이유였다.

    The contrast between the heavy workload of a clinical rotation and the self-direction required as a MER made for a difficult period of transition, either because of the luxury of time or the balancing act required for the number of roles within the position:



     

    롤모델의 영향

    The influence of role models


    롤모델은 여러가지 위장된 형태로 존재한다.

    Role models took many guises: 


    '그들은 심지어 약 절반의 시간 동안 가르치고 있다는 것도 모른다. 그들에게는 이것이 너무 자연스럽고, 그들은 소수만 이해하는esoteric 것을 가르치는 것이 아니다. 내가 멘토/롤모델로서 생각하는 이 사람들은 중요한 것을 가르쳐 주었다.'

    ‘The fact that they don't even know they're teaching half the time, it comes so naturally to them and they're not teaching esoteric stuff. That's my point of view is that these people that I view as mentors or role models have taught me the important stuff.’ (4a)


    롤모델을 찾은 다음, 초심자들은 이들 롤 모델의 특성과 행동에 비추어 성찰을 하고 발전한다.

    Having identified their role models, novices need to reflect on those role models’ attributes and behaviour in order to improve:



    정체성 찾기

    Defining one’s identity


    MER은 교육자로서의 신뢰성과 환자진료에 대한 헌신 모두에 대해서 임상적 정체성을 유지하는 것이 중요함을 강조했다.

    The MERs emphasised the importance of continuing with their clinical identity in terms of credibility as an educa- tor and their commitment to patient care:


    일부 응답자는 스스로를 다양한 정체성이 있다고 했으며, 그러나 다른 사람들은 의료계medical community에서 자신의 포지션에 대한 보다 고정된 생각을 갖는다고 했다.

    Some respondents’ role saw themselves as having mul- tiple identities whereas others had much firmer ideas of their position in the medical community.



    연구(자)는 보다 덜 중요한 정체성이었다. 

    Research as a component of the role or their identity was much less important:



    지지: 관리자supervisor와 멘토 

    Supports: supervisors and mentors


    지지support와 전문직적 가이드professional guidance 가 MERS에서 가장 중요한 특정이었는데, 종종 이것은 두 명의 관리자가 존재하는 것 때문에 복잡해지기도 했다. 한 사람은 의학교육분야의 관리자이고, 다른 사람은 전공과의 관리자이다.

    In terms of support and professional guidance the super- visor was the most important figure for the MERS, though this was somewhat complicated by having two supervisors during the term: the one responsible for the medical education part of the role (and their experience varied) and the other the discipline supervisor:



    교육에 대한 지식이 충분하지 않은 관리자를 두는 것은 힘든 일이었으며, 프로젝트에 대한 기대에 관련된 문제였다.

    Not having a supervisor with sufficient knowledge of education was difficult and this was particularly an issue in relation to the expectation of doing the project:


    '내가 가장 힘들었던 것은 mentorship 문제였다'

     ‘Probably part of the reason that it was so frustrating…was the mentorship.



    교육을 계속 하는데의 장애요인: 연구

    Potential barriers to pursuing education: the need for research


    의학교육연구에 대한 열정은 많지 않았다. 연구 능력과 연구결과의 발표delivery는 academic career에서 기본적 요건이었고, 따라서 연구에 흥미가 없는 것은 academia에 남는 장애요인이었지만, 임상교육자로서의 역할에 대해서는 그렇지 않았다.

    There was not much enthusiasm for pursuing medical education research. Research capability and delivery were seen as fundamental requirements for an academic career, and therefore the lack of interest in research was a barrier to academia but not necessarily to the clinician educator role.


    의과대학에서 연구논문을 읽고 해석하는 과목이 있었지만 연구에 참여할 기회는 별로 없었다.

    Even though there had been courses on reading and interpreting research papers during medical school, there had been little exposure or opportunity to get involved in research prior to the MER role:


    evaluation을 수행할 자신감 혹은 능력의 부족

    There was also lack of capability or confidence to undertake evaluation:



    고찰

    Discussion


    Despite their initial beliefs that they could perform better than their own teachers, as participants navigated the role there was gradual recognition that teaching is a skill that needs to be nurtured and developed and does not necessarily come naturally.


    시니어 의학교육자로부터 교육 스킬을 배울 공식적 기회의 필요성. 임상에서 조금 멀어져 시간이 필요하나 이것이 부정적 영향을 줄 수도. 동료로부터 인정을 받아야 함. 주변에서 MER역할에 대해 명확한 이해가 부족했고, 이것이 주니어 의사가 교육자로서 professional socialisation 되는데 부정적 영향 가능성 있음.

    Recent studies on the developmental needs of junior doctors entering academic medicine have identified the provision of role models and creation of research opportunities as key requirements [21,22,25,26], yet there has been little discussion on the need to develop teaching skills. Our findings testify to the need to provide formal opportunities for junior doctors to learn these from senior medical educators, to facilitate their development as medical educators. This could require time away from clinical work, something that could negatively impact a MERs motivation to participate in this type of activity. It would also require recognition by colleagues of the credibility of an education career pathway; our findings illustrate there was no clear understanding of the MER roles by others in the system and they can be seen as a soft option. The failure of colleagues to recognise the MER positions as credible, negatively impacted on the junior doctors’ professional socialisation as educators.



    정체성 측면에서 '전문의로서 일하면서 교육에 흥미가 있고 스킬을 갖춘 의사'로 인정받고자 했고, 임상의에 대한 정체성이 첫 번째임, 교육자로서의 정체성은 두 번째였다. 동료로부터 인정을 받지 못하기도 했음. 개인적 요인과 맥락적 요인의 misalignment가 있을 때 부정적 결과가 나올 수도 있음.

    The sense of identity emerged as a strong theme from the data. The MERs were not motivated by the chance to develop an academic career but wished to be seen as doctors with an interest and skill in education while working as specialists. These junior doctors primarily described themselves as clinicians; the identity of educator was seen as secondary to their main role. A consistent concern about the primacy of clinical work was expressed, and some respondents felt a strong pull back to full time clinical practice. When discussing identity, the role of an educator in a clinical environment, though often described as complementary, was almost always implicitly viewed as secondary to that of clinician. The lack of acknowledgement by peers of the validity of the role contributed to a dissonance between career aspirations and achievement. Misalignment between personal and contextual factors such as these have been reported to engender negative outcome expectations for those considering a specific career pathway [19,27].



    병원에 따라 각자의 필요성에 맞춰 MER 포지션의 목적이 달라졌다. 이것이 일부 참여자에게는 힘든 일이었으나, 일부 자기-동기부여가 잘 되고 자신의 시간을 어떻게 사용할지 아는 참여자에게는 유용하기도 했음. 오리엔테이션이 도움이 될 것.

    The purpose of the MER position in different hospitals varied between sites based on hospitals’ needs, while also being capable of being tailored quite closely to the motivations of each MER. This different experience of structure and autonomy than in clinical roles was challenging for some participants as they navigated the role. However the lack of a formal position description was useful for those who were self-motivated and able to decide how they wanted to pursue their time. Certainly orientation would have been helpful and particularly a chance to talk to others in the role of those who have had the role previously.



    연구가 가장 큰 장애요인이었고, 연구에 대해서는 별 열정이 없었음. 실제 연구경험도 적었고, 어떻게 교육적 인터벤션을 평가할지에 대한 아이디어가 부족햇음. 그들은 자신의 병원에서 임상교육을 개선시키고 싶었으며, 교육에 관심이 있는 전문의가 되고 싶어했음. 또한 병원에 지원할 때hospital posts 다른 사람보다 걸출한 모습을 보여주고 싶었음. 이러한 결과는 "임상교육자의 등장을 더 촉진시키기 위해서는, 현재 연구를 중심으로 임상의사들을 의학교육분야로 사회화socialize시키는 방식이 변화할 필요가 있따"라는 Kumar 등의 권고와 부합함.

    The need for research was the biggest barrier to participants when considering whether to continue on a medical education career pathway upon completion of the post. There was not much enthusiasm for pursuing medical education research. Unlike the findings of O’Sullivan et al. whose respondents spoke of early exposure to research opportunities, our participants had little practical research experience other than an introduction to critical appraisal at medical school. This meant they had limited ideas of how to evaluate educational interventions or indeed gauge the success of their own projects. Teaching, hands-on and development, was the main objective and there was no particular interest in research or academia. They want to enhance clinical teaching in the hospitals and become specialists with an interest in education. This interest may also be of use for them to stand out from others applying for hospital posts. These findings support the recommendations of Kumar et al.[7], who in a recent study of clinician educators at the University of Sydney concluded that to facilitate the rise of the clinician educator, the current focus on socialising clinicians into medical education in terms of research will need to change substantially.




     


     





     2014 May 31;14:110. doi: 10.1186/1472-6920-14-110.

    Becoming a medical educatormotivationsocialisation and navigation.

    Author information

    • 1Centre for Medical Education Research and Scholarship, School of Medicine, The University of Queensland, 288 Herston Road, Herston, QLD 4006, Australia. e.bartle@uq.edu.au.

    Abstract

    BACKGROUND:

    Despite an increasing concern about a future shortage of medical educators, little published research exists on career choices inmedical education nor the impact of specific training posts in medical education (e.g. academic registrar/resident positions). Medical educators at all levels, from both medical and non-medical backgrounds, are crucial for the training of medical students, junior doctors and in continuing professional development. We explored the motivations and experiences of junior doctors considering an education career and undertaking a medical education registrar (MER) post.

    METHODS:

    Data were collected through semi-structured interviews with junior doctors and clinicians across Queensland Health. Framework analysis was used to identify themes in the data, based on our defined research questions and the medical education workforce issues prompting the study. We applied socio-cognitive career theory to guide our analysis and to explore the experience of junior doctors in medical education registrar posts as they enter, navigate and fulfil the role.

    RESULTS:

    We identified six key themes in the data: motivation for career choice and wanting to provide better education; personal goals, expectations and the need for self-direction; the influence of role models; defining one's identity; support networks and the need for research as a potential barrier to pursuing a career in/with education. We also identified the similarities and differences between the MERs' experiences to develop a composite of an MER's journey through career choice, experience in role and outcomes.

    CONCLUSIONS:

    There is growing interest from junior doctors in pursuing education pathways in a clinical environment. They want to enhance clinical teaching in the hospitals and become specialists with an interest in education, and have no particular interest in research or academia. This has implications for the recruitment and training of the next generation of clinical educators.

    PMID:
     
    24885740
     
    [PubMed - indexed for MEDLINE] 
    PMCID:
     
    PMC4047547
     
    Free PMC Article


    의과대학 2학년 학생의 동기부여신념, 정서, 성취(Med Educ, 2010)

    Second-year medical students’ motivational beliefs, emotions, and achievement

    Anthony R Artino,1 Jeffery S La Rochelle2 & Steven J Durning2







    INTRODUCTION


    안타깝게도, 개개인을 성공에 이르게 하는 요인은 잘 이해된 바 없다. 더 나아가 의학교육 문헌들은 인지적 요인에 우선적으로 초점을 맞추고 있다(과거 학업성취, 표준화 시험 점수). 이들은 small to moderate 정도의 분산만을 설명해줄 뿐이다.

    Unfortunately, the factors that lead to individual success are not well understood. Moreover, the medical education liter- ature tends to focus primarily on cognitive factors (e.g. prior academic achievement and standardised test scores), which typically explain only small to moderate amounts of variance in academic out- comes.2,3


    최근, 정서적 요인의 중요성(동기부여와 감정)이 관심을 받고 있다. 이들 교육연구자들은 인지적 요인 단독의 효과를 넘어서 어떻게 감정이 성취성과에 영향을 주는지를 연구하고 있다. 본질적으로, 이들 연구자들은 인간의 마음은 단순히 정보를 처리하고 인지라는 것은 정보를 조작하는 것이며, 학습은 정보를 습득하고 저장하는 것이라는 식으로 학습을 컴퓨터에 비교하는 지나치게 단순화된 비유에 반대하는 것이다. 대신, 교육심리학자들은 인간의 사고는 훨씬 더 알쏭달쏭하며, 신축성있고, 동기부여와 감정에 영향을 받는다고 본다. 이러한 복잡성을 감안하면, 현재의 학습과 수행에 관한 모델은 정서적 요인을 고려해야 한다.

    Recently, the importance of affective factors (e.g. motivation and emotion) has received greater emphasis among educators in fields outside medi- cine.4,5 In particular, these educational researchers have begun to explore how affect might ultimately influence achievement outcomes, above and beyond the effects of cognitive factors alone.6,7 In essence, these educators have argued against the over-sim- plified computer metaphor of learning, whereby the human mind is simply a processor of informa- tion, cognition primarily involves the manipulation of that information, and learning is merely the acquisition and storage of information. Instead, contemporary educational psychologists have proposed that human thinking is much more ‘fuzzy’ and flexible, and is subject to motivations and emotions that may serve multiple purposes at any given time.8 Given this complexity, current models of learning and performance often include consideration for affective factors.





    이론적 프레임워크

    Theoretical framework


    사회인지이론에서는 인간의 기능functioning이 다음의 세 가지 요인이 역동적이고 상호작용는 것이라고 본다.

    Social cognitive theory assumes that human functioning results from the triadic, dynamic and reciprocal interaction of

    • 개인적 요인(신념, 기대, 태도, 이전 지식)
      personal factors (e.g. beliefs, expectations, attitudes and prior knowledge),

    • 행동(개별적 행동, 선택, 말)
      behav- iours (e.g. individual actions, choices and verbal statements), and

    • 사회적, 물리적 환경(자원, 행동의 결과, 타인, 물리적 환경)
      the social and physical environment (e.g. resources, consequences of actions, other people and physical settings).9

     

    따라서 그림 1의 모델은 학생이 자신의 능력과 학습활동의 가치에 대해서 갖는 동기부여적 신념에 영향을 주는 학습환경의 맥락적 특성들(과제 특성, 교육 자원, 사회-문화 요인)을 나타낸 것이다. 즉, 이들 신념은 감정에 영향을 미치는데 즐거움, 불안 등이 그것이다. 이 감정들은 다양한 학업성과와 연결되어 있다.

    Thus, the model in Fig. 1 proposes that contextual features of the learn-ing environment (e.g. task characteristics, instruc- tional resources and other broader socio-cultural factors) affect students’ motivational beliefs about their capabilities and the value of learning activities. In turn, these beliefs influence discrete achievement emotions, such as enjoyment and anxiety,6 which then link to various academic outcomes, such as student achievement and satisfaction.


    이 연구에서는 의과대학생의 학업적 성취를 예측하기 위하여 동기부여적 신념motivational belief와 성취관련 감정achievement emotion을 사용하였다. 동기부여적 신념에 대해서는 두 가지 구인을 고려하였다. 하나는 과제의 가치task value이고, 다른 하나는 학업적 자기효능감이다.

    In this study, motivational beliefs and achievement emotions were used to predict medical students’ academic achievement in an introductory clinical reasoning course. In terms of motivational beliefs, two constructs were considered.

    • Task value: 어떤 과목이 얼마나 재밌고, 중요하고, 유용한지에 대한 학생의 판단. 비-의학 분야에서는 TV가 높을수록 다양한 주요 성과(미래 학습활동에 대한 선택, 학업 성취)가 높다.
      The first is task value, which can be defined as students’ judgements of how interesting, important and useful a course is to them.10 Research in non-medical contexts has typi- cally demonstrated that task value beliefs positively predict many important outcomes, such as choice of future learning activities10 and academic achievement.11

    • 학업적 자기효능감Academic self-efficacy: 특정 학업 과제를 성공적으로 수행할 수 있다고 판단하는 정도. SE가 높을수록 다양한 학업성과에 영향을 준다.
      The second motivational belief examined was academic self-efficacy, which can be defined as stu- dents’ judgements of their capabilities to successfully perform specific academic tasks.9 Generally, research has shown that self-efficacy beliefs positively influence many academic outcomes, including, for example, choice of activities,12 level of effort13 and academic achievement.14


    성취감정Achievement Emotion은 개인적 요인의 두 번째 부분이다. Pekrun은 AE의 통제-가치 이론을 제안하였다. 통제-가치 이론Control-value theory은 AE가 성취-관련 활동과 연관된 개별 감정들이라고 정의했으며, 여기에는 즐거움(새로운 것을 배울 때의), 불안(고부담 시험에 대한), 지루함(길고 재미없는 강의) 등이 있다. Pekrun에 따르면, AE는 개인의 Motivational belief(SE, TV)에 의해 결정된다.

    Achievement emotions represent the second set of personal factors in the conceptual model. Pekrun6 has proposed a control-value theory of achievement emotions. Control-value theory defines achievement emotions as discrete emotions that are associated with achievement-related activities such as, for example, the enjoyment that often comes from learning some- thing new, the anxiety associated with taking a high- stakes examination or the boredom that may occur during a long, uninteresting lecture. According to Pekrun,6 achievement emotions are determined, in part, by individuals’ motivational beliefs, such as self- efficacy and task value beliefs. Limited educational research in secondary schools and post-secondary universities has indicated that achievement emotions predict students’ use of learning strategies, choice of future courses and academic achievement.15,16


    개념 모델에서 묘사된 관계는 상호적이다.

    Finally, consistent with social cognitive theory, the relationships depicted in the conceptual model are assumed to be reciprocal.


     


     

    방법

    METHODS


    참여자

    Participants


    F Edward He´bert School of Medicine의 2학년 학생. 미국의 유일한 federal medical school.

    The participants were second-year medical students enrolled in the F Edward He´bert School of Medicine, Uniformed Services University of the Health Sciences (USU). The USU is the only US federal medical school and matriculates approximately 170 medical students each year.



    교육 맥락

    Instructional context


    2학년 시작시 임상추론입문ICR 과목

    The instructional context was a second-year course called Introduction to Clinical Reasoning (ICR). This course was chosen for the present study because it represents students’ first exposure to clinical decision making


    진행: 강의와 소그룹 세션

    Generally speaking, each ICR session

    • begins with an overview lecture on the topic,

    • which is followed by mandatory small-group sessions on the topic.

     

    강의의 목표: 용어/병태생리/실제 접근

    In the overview lecture (30–50 minutes), the general goals are:

    • to teach relevant terminology;

    • to review and reinforce pertinent pathophysiology, and

    • to illustrate a practical approach to the topic.

     

    소그룹 세션의 목표(1)

    In the small-group sessions, the general goals are two- fold:

    • to illustrate major diagnostic entities encom- passed within the topic, and

    • to teach typical ‘patterns’ of presentation for these diagnostic entities and key decision points to help students arrive at the diagnosis.

    소그룹 세션의 목표(2)

    This second general goal of the small-group session includes teaching students to

    • identify key findings,

    • recognise problems and construct problem lists,

    • build clinical vocabulary,

    • identify syndromes,

    • compare and contrast similar diagnoses seen with a given topic, and

    • formulate a differential diagnosis that the student can defend using the presenting data.


    절차

    Procedures


    척도

    Measurements


    설문

    Surveys


    1/3 종료 후 설문

    End-of-trimester 1 survey


    MB를 다음을 가지고 측정

    Students’ motivational beliefs were measured using two subscales adapted from Artino and McCoach:18


      • 1 a 6-item task value subscale assessed students’ judgements of how interesting, important and useful the clinical reasoning course was to them, and 

      • 2 a 5-item self-efficacy subscale assessed students’ confidence in their ability to learn the material presented in the course.


    약간의 워딩 변화

    Several minor wording changes were made to the motivational beliefs subscales;



    2/3 종료 후 설문

    End-of-trimester 2 survey


    AE를 다음을 가지고 측정

    Students’ achievement emotions were measured using a shortened version of the class-related emo- tions section of the Achievement Emotions Ques- tionnaire (AEQ):19


      • 1 a 4-item enjoyment subscale assessed students’ course-related enjoyment; 

      • 2 a 6-item anxiety subscale assessed students’ course-related anxiety, and 

      • 3 a 5-item boredom subscale assessed students’ 3 course-related boredom.


    약간의 워딩 변화

    Once again, changes were made to the original subscales to reflect the specific medical education context studied here.




    학업성취

    Achievement outcomes


    Course examination grade


    National board shelf examination score




    분석

    Analysis


    Prior to analysis, the data were screened for accuracy and missing values, and each survey item was checked for normality. Following data screening, three sets of analyses were conducted.

    • First, confir- matory factor analysis (CFA) techniques were used to validate the hypothesised survey structure and identify survey modifications that would result in a refined, more parsimonious measurement model.

    • Factors identified in the CFA were then subjected to reliability analysis, and descriptive statistics and Pearson correlations were calculated.

    • Finally, a causal model was estimated using structural equa- tion modelling (SEM). Built upon the multivariate techniques of factor and path analysis, SEM is a flexible and powerful statistical tool that allows researchers to test a priori hypotheses regarding the inter-relationships between both observed and latent variables (for a detailed explanation of SEM and its applicability in medical education research, see Violato and Hecker22).

    In the present study, the aim of the SEM was to test the hypothesised linear relations between the latent beliefs and emotions variables and students’ academic achievement.

    All CFA and SEM analyses were conducted using AMOS 7.023 and the remainder of the analyses were conducted using SPSS 16.0 (SPSS, Inc., Chicago, IL, USA).


     



    결과

    RESULTS


    대부분의 종단연구에서 자료의 결측과 (연구대상자의) attrition은 흔한 문제이다.

    In most longitudinal studies, missing data and attri- tion are frequent problems; this study was no excep- tion. Among the 174 students enrolled in the ICR course, 136 agreed to complete both surveys (giving a 78% response rate). The sample included 86 men (63%) and 50 women; their mean age was 24.9 years (SD = 1.5).


     

    CFA

    Confirmatory factor analysis


    두 설문의 convergent와 discriminant validity를 검사하기 위하여 시행. MLE가 사용되었고, chi-square가 model fit을 평가하기 위해서 사용됨. 일반적으로 chi-square에서 유의하지 않은 결과가 good model fit을 의미한다. 그러나 chi-square 검사는 샘플 크기와 상관관계의 크기에 영향을 받기 때문에, 연구자들은 chi-square에만 의존하지 않는다. 여기에 몇 가지 추가적인 fit indices를 사용하는데, 여기에는 자유도 비율degrees of freedom ratio, CFI, RMSEA등이 있다.

    A CFA was conducted to examine the convergent and discriminant validity of the two surveys. Maximum likelihood estimation was used to esti- mate the parameters and a chi-square test was conducted to assess model fit. Generally, a non- significant chi-square result indicates a good model fit.24 However, because the chi-square test is affected by the sample size and the size of the correlations in the model, researchers do not normally rely on the chi-square test as the sole measure of model fit. Therefore, several additional fit indices were considered together with the chi-square test. These indices included the chi-square : degrees of freedom ratio (also referred to as the normed chi-square statistic), the com- parative fit index (CFI), and the root-mean-square error of approximation (RMSEA).




    26개의 문항을 다섯 개의 latent variable에 load함.

    The 26 survey items used in this study were hypoth- esised to load onto five distinct latent variables: task value, self-efficacy, enjoyment, anxiety and boredom. Based on the model fit guidelines outlined by Hu and Bentler,25 the resulting goodness-of-fit indices indicated that the model fit the data only marginally well. In particular, the chi-square result was statisti- cally significant (v2 [289, n = 136] = 565.89, p < 0.001), and although the normed chi-square statistic (1.96) was < 3.0, the CFI (0.80) was < 0.90 and the RMSEA (0.08) was > 0.06 (the latter two statistics indicated a marginal model fit).



    model fit을 개선하기 위해서 최종 solution을 위한 trimming시행. SE척도에서 하나 배제, Anxiety에허 두 개 배제.

    Next, in an attempt to improve model fit, standar- dised residuals and modification indices were exam- ined and five items were identified as having large standardised residuals and⁄ or large modification indices. Because one of the objectives of the CFA was to further refine the measurement model, these five items were trimmed from the final solution (see recommendations in Brown).26 The trimmed items included one item from the self-efficacy scale (‘I’m confident I can learn in the context of the small-group sessions’), two items from the anxiety scale (‘I feel uneasy during the small-group discussion sessions’ and ‘I feel nervous during the small-group discussion sessions’) and two items from the boredom scale (‘I feel this course is fairly dull’ and ‘I’m generally uninterested in the course material’).


    두 번째 CFA가 시행되었고, 결과가 향상됨.

    Following the trimming procedure, a second CFA was conducted; all fit indices improved as a result of these modifications. The chi-square result remained statistically significant (v2 [179, n = 136] = 259.92, p < 0.001); however, the normed chi-square result (1.45) went down to < 3.0, the CFI (0.92) went up to > 0.90 and the RMSEA (0.05) went down to < 0.06, all indicating that the revised model was an adequate fit to the data. The survey items retained in the final solution are provided in Table 1, along with their means and SDs.




    Descriptive statistics and Pearson correlations



     


     

    Evaluating the structural equation model




    DISCUSSION


    의학 외 다른 교육연구에서는 정서요인을 중요하게 인정하고 있음.

    Recently, educational researchers in fields outside medicine have acknowledged the critical role per- sonal affective factors, like motivation and emotion, play in learning and performance.4–8,11


    이번 연구의 결과는 가설로서 제기한 일부 관계를 지지해줌. 특히 TV는 Enjoyment, Boredom과 연관이 있었음. 이것은 통제-가치 이론의 연구결과와도 부합하는 것으로, 그 과목이 흥미롭고 유용하다고 생각하는 학생일수록 그것을 공부하는게 즐겁고, 덜 지루하다. 유사하게 학생의 SE는 Anxiety와 부적 상관관계에 있었다. 즉, 더 그 과목을 잘 할수 있다고 생각하는 학생일수록 불안을 덜 느낀다. 이 역시 통제-가치 이론과 부합하며, Bandura가 원래 개념화한 자기효능감, 그리고 그것이 학습의 정서적 요소에 미치는 영향과도 부합한다. 더 나아가서 이들 연구결과는 의학교육자들이 학생의 TV와 SE를 가지고 학생의 성취 감정향상을 볼 수 있음을 시사한다.

    Findings from this study provide some support for the hypothesised relationships. Specifically, task value beliefs were positive predictors of students’ course- related enjoyment and negative predictors of their reported boredom. Thus, the direction and magni- tude of these effects, which are consistent with previous empirical work using control-value theory,6,15,16 suggest that students who believed the course was interesting, important and useful were also more likely to enjoy it and less likely to become bored. Similarly, students’ academic self-efficacy was a negative predictor of anxiety, indicating that those who were confident they could learn the course material were also less likely to experience course- related anxiety. The direction and size of this effect is also consistent with control-value theory6,15,16 and Bandura’s9 original conceptualisation of self-efficacy and its influence on affective components of learn- ing. Further, these results suggest that medical educators may observe improvements in students’ achievement emotions by first addressing students’ task value beliefs and self-efficacy perceptions (for specific instructional recommendations, see Schunk et al.11).


    성취감정에 있어서, Enjoyment는 NBME점수와, Anxiety/Boredom은 과목 성적과 관련이 있었음. 즉, '즐거움enjoyment'는 의과대학에서 진행되는 이후 성취도에 영향을 주며, Anxiety/Boredome은 보다 즉각적, 과목-관련 학업성취에 영향을 준다.

    In terms of achievement emotions, course-related enjoyment was positively related to students’ NBME shelf examination scores, whereas both anxiety and boredom were negatively related to students’ course examination grades. These results suggest that enjoyment, a positive emotion, may have important direct effects on subsequent achievement outcomes in medical school. By contrast, anxiety and boredom, both negative emotions, may have direct effects on more immediate, course-related achievement outcomes.


     

    이 정도의 medium effect size는 limited empirical evidence와 부합.

    Finally, the overall effects for the model were R2 = 0.20 and 0.14 for the course examination grade and national board shelf examination score, respec- tively. These medium effect sizes are consistent with the limited empirical evidence linking achievement emotions to scholastic achievement.6,15,16




    의학교육에 대한 함의

    Implications for medical education


    의대생들은 high-functioning하고 successful하며, 본질적으로 강력한 동기부여신념이 있고, 부정적 감정을 잘 조절할 수있는 대응기전을 갖추었다는 은연중의 가정이 있다. 비록 본 연구결과는 강력한 동기부여신념을 지지해줄지는 모르나, 부정적인 성취감정의 영향에 면역이 되어있지는 않다는 것을 보여준다.

    There is an implicit assumption that medical students are predominantly high-functioning and successful, and possess inherently strong motiva- tional beliefs and advanced coping mechanisms with which they can assuage negative achievement emotions. Although the results presented here do confirm the presence of strong motivational beliefs, the findings suggest that medical students are not immune to the effects of negative achievement emotions.


    교육자들은 과목의 구조/내용/교육법/점수체계 등을 고려하고, 이것들이 MB와 AE에 어떻게 영향을 주는지 고려해야 함.

    In addition, these findings suggest that educators should consider course structure, content, teaching method and grading scheme – and how these factors might impact both motivational beliefs and achievement emotions over time – as these personal factors could potentially affect other important performance outcomes.


    6 Pekrun R. The control-value theory of achievement emotions: assumptions, corollaries, and implications for educational research and practice. Educ Psychol Rev 2006;18:315–41.


    22 Violato C, Hecker KG. How to use structural equation modelling in medical education research: a brief guide. Teach Learn Med 2007;19:362–71.



     


     


     


     




     2010 Dec;44(12):1203-12. doi: 10.1111/j.1365-2923.2010.03712.x.

    Second-year medical students' motivational beliefsemotions, and achievement.

    Author information

    • 1Department of Preventive Medicine and Biometrics, Uniformed Services University of the Health Sciences, 4301 Jones Bridge Road, Bethesda, MD 20814-4712, USA. anthony.artino@usuhs.mil

    Abstract

    CONTEXT:

    A challenge for medical educators is to better understand the personal factors that lead to individual success in medical school and beyond. Recently, educational researchers in fields outside medicine have acknowledged the importance of motivation and emotion in students' learning and performance. These affective factors have received less emphasis in the medical education literature.

    OBJECTIVES:

    This longitudinal study examined the relations between medical students' motivational beliefs (task value and self-efficacy),achievement emotions (enjoyment, anxiety and boredom) and academic achievement.

    METHODS:

    Second-year medical students (n=136) completed motivational beliefs and achievement emotions surveys following their first and second trimesters, respectively. Academic achievement was operationalised as students' average course examination grades and national board shelf examination scores.

    RESULTS:

    The results largely confirmed the hypothesised relations between beliefsemotions and achievement. Structural equation modelling revealed that task value beliefs were positively associated with course-related enjoyment (standardised regression coefficient [β] = 0.59) and were negatively related to boredom (β= -0.25), whereas self-efficacy beliefs were negatively associated with course-related anxiety only (β = -0.47). Furthermore, student enjoyment was positively associated with national board shelf examination score (β = 0.31), whereas anxiety and boredom were both negatively related to course examination grade (β= -0.36 and -0.27, respectively). The overall structural model accounted for considerable variance in each of the achievement outcomes: R(2) = 0.20 and 0.14 for the course examination grade and national board shelf examination score, respectively.

    CONCLUSIONS:

    This study suggests that medical students' motivational beliefs and achievement emotions are important contributors to their academic achievement. These results have implications for medical educators striving to understand the personal factors that influence learning and performance in medical training.

    © Blackwell Publishing Ltd 2010.

    PMID:
     
    21091760
     
    [PubMed - indexed for MEDLINE]


    재교육의 어려움: 이론적 방법론적 통찰 (Med Educ, 2013)

    The remediation challenge: theoretical and methodological insights from a systematic review

    Jennifer Cleland,1 Heather Leggett,2 John Sandars,2 Manuel J Costa,3 Rakesh Patel,2,4 & Mandy Moffat,1








    도입

    INTRODUCTION



    의학 학위를 받기 위한 시험에서(의과대학 시험에서)의 성취도가 면허시험에서의 수행능력과 졸업 후 임상역량과 관계됨을 보여준 바 있다. 의과대학때 잘 못했던 학생들은 의사가 되어서도 그러하다.

    Research has identified that measures of attainment in medical degree examina- tions can predict subsequent performance on licens- ing examinations and clinical competence after medical school.1,2 In addition, weak medical students go on to become weak doctors,3,4


    struggling students가이드나 서포트를 계속 잘 받지 못할 수 있고 의사들은 종종 underperformance에 fail 주는 것을 꺼려한다. 따라서 학습에 문제가 있는 학생들은 계속 그상태가 해결되지 못한 상태로 남게 되며, 반복적인 실패와 underperformance를 겪는다. 적절한 시점에 나쁜 performance에 대해서 개입하는 것은 해로운 학습 및 행동 패턴을 가진 학생들을 임상에서 문제를 일으키기 전에 즉각적으로 손볼 수 있게 해준다.

    struggling students may con- tinue with little guidance or support7 and supervising clinicians are often reluctant to fail underperfor- mance.8–10 Thus, students’ learning problems remain unaddressed, leading to repeated failure and under- performance.5,6,11 Timely intervention for poor per- formance has the potential to enable the individual to deal with adverse learning and behaviour patterns promptly before these cause problems in clinical practice. 


    대부분의 재교육 프로세스는 세 단계를 밟는다. 문제 학생의 진단 혹은 발견 / 재교육 인터벤션 / 재시험. 하지만 이 재교육 프로세스는 교수들에게 상당한 시간을 투자할 것을 요구하며, 더 문제는 교수들은 재교육 인터벤션의 효과에 대해서 확신이 없다는 점이다.

    Most remediation processes consist of three steps: identification or diagnosis; a remediation intervention, and retesting.12–14 However, these remediation processes place substantial time de- mands on faculty staff.7,13 Furthermore, faculty members report uncertainty about the efficacy of remediation interventions.13


    이러한 결과는 지필고사 혹은 임상시험에서 구체적인 재교육 인터벤션이 있기 전과 후의 수행능력을 평가하여 그러한 인터벤션이 효과가 있었다고 결론내리는 다수의 연구들과 배치되는 결과이다. 그러나 "single studies는 그로부터 발견한 지식을 개념/인구집단/세팅/시점에 대해서 일반화하기에 제한적이며, 광범위한 퍼즐의 일부만 보여줄 뿐이다'

    These findings are at odds with those of a number of studies that have evaluated performance on written or clinical examinations before and after a specific remediation intervention7,14–38 and concluded that the intervention was effective. However, single studies ‘are limited in the generalisability of the knowledge they produce about concepts, populations, settings and times’ and ‘frequently illuminate only one part of a larger explanatory puzzle’.39



    방법

    METHODS


    연구 선택

    Study eligibility and selection


     

    자료 추출

    Data extraction


    우리는 TREND 체크리스트를 활용함. 그 이유는 이것이 활용한 이론의 보고, 인터벤션의 묘사, 여러 조건의 비교, 연구 설계 등을 강조하기 때문이다.

    We used the TREND (transparent reporting of evaluations with non-randomised designs) checklist48 to guide data extraction. This was selected as it emphasises the reporting of theories used and descriptions of intervention and comparison con- ditions, and research design, in evaluation studies that use non-randomised designs, and so was felt to be appropriate for review given the nature of the research studies on the topic.

     

    우리는 우리의 주관성을 인정하고, 협동적 탐구cooperative enquiry 원칙을 적용함(관찰결과에 대한 토론, 비판적 성찰과 확장)

    We explicitly acknowledged our subjectivity and used the principles of cooperative enquiry (i.e. discussing findings, and critically reflecting and expanding on them50) on an ongoing basis to address this within the group.


     

    자료 통합과 분석

    Data synthesis and analysis


    qualitative synthesis of the data 를 했음

    A qualitative synthesis of the data was selected as this method was deemed appropriate in the context of our research questions (Evidence for Policy and Practice Information and Co-ordinating Centre [EPPI-Centre; http://eppi.ioe. ac.uk/cms]49).




    결과

    RESULTS



    대부분의 연구는 연구대상자가 적었다.6-377, 중간값 23. 표본크기를 계산한 연구는 단 하나.

    Most studies included very small sample numbers, ranging from six to 377, with a median of 23. Only one study stated a sample size calculation.55


    대부분의 연구는 (인터벤션) 이후에 수행된 구체적인 시험에서의 결과에 초점을 두었다(USMLE 등). 실제로 많은 연구들은 공공연하게 (재시와 같은) 구체적인 시험을 위한 시험의 기술과 내용에 초점을 맞추고 있었다.

    Most (n = 22, 71%) studies focused solely on performance on a specific, subsequent examination (e.g. resits, re-taking a standard examination such as the US Medical Licensing Examination, or the next standard examination in a programme).14–22,24,26–30,32–35,37,51,52 Indeed, many studies overtly focused on examination technique and content boosting for a specific examination (which the participants were required to re-sit or re-take to progress their education or training).


    8개 연구는 재교육에 있어서 보다 holistic한 관점을 유지하였는데, personal support를 제공하였거나, 지식과 스킬의 교수-학습에 대한 보다 포괄적인 접근법을 취했다.

    The remaining eight studies (26%) took a more holistic perspective to remediation and either in- cluded the provision of personal support or took a broader approach to learning and teaching skills and knowledge.7,23,25,31,36,38,53,54


    복잡성complexity에 대해서도 평가했다. 다수의 상호작용적 요소를 활용한 경우 complex 한 인터벤션으로 정의했다. 대부분의 연구는 구체적으로 무엇을 했는지, 얼마나 길게, 왜 했는지 자세하게 기술하지 않았다. 대부분의 인터벤션은 다양한 활동을 했음을 보고했으나(e.g. tutorials, directed reading, skills practice, feedback, examination prac- tice, case presentations), 그 접근법을 적절한 이론에 따라 정당화한 연구는 거의 없었다. 예를 들면 대부분의 연구는 단순히 프로그램 요소가 멘토링이나 튜터리얼을 포함했다고만 쓰고, 그 내용이나 형태, rationale 등을 쓰지 않았다.

    Studies were assessed for complexity.46 An interven- tion was defined as complex if it utilised several interacting components. Dimensions of complexity can include, for example, the number of and inter- actions between components, number and variability of outcomes and the degree of flexibility or tailoring of the intervention permitted. The majority of the studies reviewed did not report in detail what they did, why and for how long. Most interventions reported a variety of activities (e.g. tutorials, directed reading, skills practice, feedback, examination prac- tice, case presentations), but few studies clearly justified their approaches on the basis of appropriate theory (with the notable exceptions of 14,18,19,30,53). Many studies simply stated, for example, that the programme elements included mentoring or tutorials without describing the content or format of, or rationale for, these tutorials or mentoring ses- sions.17,34,35

     

     



    고찰

    DISCUSSION


    대부분의 연구는 근거의 퀄리티를 보면 퀄리티가 낮았다. 이는 주로 통제되지 않은 사전-사후 연구로, 소수의 학생을 대상으로 한 것, 장기 성과 척도가 없는 것 등이 원인으로, 인터벤션의 효과를 호손효과 혹은 background effect와 구분하기 어렵게 한다.

    This review established that the addressing of underperformance in medical students or doctors in training is an active area of primary research, but the majority of studies identified would be classed as being of low quality according to the criteria for grading quality of evidence.39 The evidence comes predominantly from uncontrolled before-and-after studies with small samples and few process or long- term outcome measures, which may not convincingly distinguish intervention effectiveness from back- ground effects or the Hawthorne effect.56,57



    복잡성에 대한 이슈는 명확하다. 연구의 설계나 방법을 보면 디테일이 부족했다. 구체적으로 어떤 요소가 차이를 만들어냈는지를 밝히는데 도움이 되지 않았다.

    The issue of complexity is clear. The designs and methods of the studies reviewed, and the lack of detail reported on the precise nature of many of the interventions, do not allow us to identify which components of the process actually made a differ- ence.


     

    일반적으로 인터벤션은 '같은 것을 한번 더 하는 것'인 경향이 많았다. 원래의 교수법이 배우는데 적절한 도움이 되지 않았으면, (비록 시니어 교수에 의해 소그룹으로 진행되었다 해도) 같은 것을 한 번 더 하는게 두 번째에 도움이 될 것이라고 볼 이유가 희박하다.

    Generally, interventions tended to represent ‘more of the same’, such as additional or intensive knowledge or skills teaching. If the original teaching did not help students to learn appropriately, there seems little reason to assume that ‘more of the same’ will do so a second time around, even if this is delivered in small groups by senior faculty staff.


     

    학생들을 서포트해서 다음 단계로 넘어가게 만드는 것의 윤리성은 그들의 poor한 수행능력을 지속하게 만든다는 점에서 좋게 보아도 의문점이 많을 뿐이다. 또한 제한된 교수의 자원이 향상이 없는 progression을 지원하는데 사용되어야 하는지에 대해서도 논란이 있으며, 교수들이 senior 학생일수록 fail시키기 어려워한다는 여러 근거를 종합할 때, weak student가 weak doctor가 될 가능성이 높기 때문이다.

    The ethics of sup- porting students to progress to the next stage of training only to continue to perform poorly (e.g. 6,15,38 ) are, at best, questionable. It is also debatable whether scarce faculty resources should be used to support progression without improvement, which may take weak students further towards registration as potentially weak doctors,3,4 when the evidence sug- gests that faculty members find it harder to fail senior students.8

     

    더 나아가, 우리는 어떤 종류의 추가적 서포트가 효과가 있었으며, 얼마나 추가적인 교육이 중요한지 모른다. '만족도'에 대한 측정은 우리의 이해에 별로 기여하는 바가 없다. 그러나 어떤 연구들은 무엇이 차이를 만들어냈는가를 밝히고자 했으며, 여기서 특정 서브그룹이 재교육에 가장 잘 반응한다는 것을 암시한다.  

    Moreover, we do not know what types of extra support work, or how much extra teaching is critical. The process measure of ‘satisfaction’, where em- ployed, does not add much to our understanding of barriers, facilitators or what precisely works. However, those studies that explicitly attempted to tease out what makes a difference14,38 are a welcome addition to the literature, hinting as they do that particular subgroups of students may respond best to remediation.

     

    이러한 결과는 놀랍지 않다. 첫째로, 오래 전부터 poor한 수행능력의 이유는 무수하다라는 것이 인식되어왔으며, 이는 poor performer가 균일한 집단이 아님을 보여준다. 둘째로 복잡한 인터벤션에서 나온 근거는 서로 다른 서브그룹간 서로 다른 효과가 있음을 보여주었다.

    These findings are unsurprising. Firstly, it has long been recognised that reasons for poor perfor- mance are myriad; that is, poor performers are not a homogeneous group. Secondly, evidence from com- plex interventions in clinical areas has recently highlighted that these have different effects in different subgroups.61

     

    가장 널리 활용되는 이론적 프레임워크는 대체로broadly 인지적이며, 자기조절, 메타인지, 성찰을 활용하고, 피드백을 주고 받는 것을 포함한다. 이는 매우 적절한데, weak 학습자와 strong 학습자는 조절 프로세스에서 질적 양적 차이가 있음을 교육 연구에서 보여주고 있다. 자기조절수준이 높은 학습자는 SRL 스킬이 낮은 학습자보다 학업적으로 더 성과가 높다.

    The most widely used theoretical framework was broadly cognitive, using self-regulation, meta- cognition and reflection, and the giving and receivingof feedback.14,38 This seems very appropriate: educa-tion research has indicated quantitative and qualita- tive differences in regulation processes and activities between weak and strong learners (e.g. 62). Highly self-regulated learners are academically more suc- cessful than those students with low levels of skill in self-regulated learning (SRL) or those who lack regulation in their learning (e.g. 63).

     

    의과대학생이 처음 의과대학에 들어왔을 때부터 자기조절을 잘 할것이라고 기대할 수 없다. 실제로, 탐색적 연구의 결과를 보면 successful한 의과대학생과 unsuccessful한 의과대학생이 SRL에 차이를 발견할 수 있다. 30여년의 교육 연구를 보면 SRL 테크닉에 대한 분명한 훈련이 효과가 있음을 보여줘왔다. '학습에 관한 학습' 과정을 의과대학 1학년 학생들에게 제공하는 것이 medical career의 초반 단계에 스스로의 SRL 접근법을 발견하게 해주고 변화를 할 수 있게 해줌으로서 미래의 underperformance의 가능성을 낮춰주는 효과적인 접근법이다.   

    We cannot assume that students can self-regulate when they enter medical school: indeed, an exploratory study suggests that differences in SRL in successful and unsuccessful medical student learners are identifi-  able.64 Thirty years of education research has identi- fied that explicit training in SRL techniques is effective65,66 in terms of improving learning out- comes for students. We suggest that this framework could make important contributions to traditional medical training assessment frameworks that have been used to identify and remediate strugglers (see also67–69). The provision of ‘learning to learn’ courses for Year 1 medical students may provide an effective approach to helping students at an early stage of their medical careers to identify their SRL approach and make changes that might reduce their chances of future underperformance.70

     

    대부분의 재교육에 대한 노력은 의과대학의 후반부에 있는 학생들이 대상이 된다. 그러나 조기 재교육 인터벤션은 많은 struggling student의 특징이기도 한 cycle of underperformance를 멈출 수 있는 잠재력을 지니고 있다. Struggling student는 낮은 자기효능감 신념과 학습에 부정적인 감정을 가지고 있어서 어려운 학습과제를 지속하고자 하는 동기부여에 영향을 준다. 이러한 학생들은 그들이 struggling student로 확인되는 이후 최대한 이른 시기에 성공을 경험할 필요가 있으며, 이를 통해서 스스로의 학습과 수행능력에 대한 통제 경험을 느껴볼 수 있게끔 해야 한다.

    Most remediation efforts are targeted at learners in the latter years of medical school. However, early remediation interventions have the potential to stop the cycle of underperformance that is characteristic of many struggling students. Struggling students have low self-efficacy beliefs and negative feelings about learning that directly influence their motivation to persist with difficult learning tasks.71 These students need to experience success as soon as they are identified as struggling so that they can feel a sense of control over their learning and performance.

     

    교육, 학습에 초점을 둔 변화와 학습의 실제 프로세스에 대한 평가를 포함하는 것이 효과가 있을 것이다. 그렇게 해야만이 underperformance에 대한 정확한 진단을 할 수 있고, 그것이 발생하는 이유를 찾아낼 수 있으며, '시험 잘보기 코칭'이 아닌 '학습을 위한 학습'에 초점을 둔 재교육을 위한 타당한 이론적 기반을 제공해줄 것이다.

    changing the focus of teaching, learning and assessment to include assessment of the actual processes of learning would progress work in this area. Doing so should enable the accurate diagnosis of underperformance and the early identification of the reasons for its occurrence, as well as providing a sound theoretical basis for remediation that focuses on ‘learning to learn’ rather than ‘examination coaching’.


     



    38 Winston KA, van der Vleuten CP, Scherpbier AJ. An investigation into the design and effectiveness of a mandatory cognitive skills programme for at-risk medical students. Med Teach 2010;32 (3):236–43.


    64 Cleary TJ, Sandars J. Assessing self-regulatory processes during clinical skill performance: a pilot study. Med Teach 2011;33 (7):368–74.


    67 Durning SJ, Cleary TJ, Sandars J, Hemmer P, Kokotailo P, Artino AR. Perspective: viewing ‘strugglers’ through a different lens: how a self-regulated learning per- spective can help medical educators with assessment and remediation. Acad Med 2011;86 (4):488–95.


    68 Sandars J, Cleary TJ. Self-regulation theory: applica- tions to medical education: AMEE Guide No. 58. Med Teach 2011;33 (11):875–86.


    69 White CB, Gruppen LD. Self-regulation learning in medical education. In: Swanwick T, ed. Understanding Medical Education. Chichester: Wiley-Blackwell 2010;271–82.


    70 Sandars J. Pause 2 Learn: developing self-regulated learning. Med Educ 2010;44 (11):1122–3.


    71 Artino AR Jr, Hemmer PA, Durning SJ. Using self-reg- ulated learning theory to understand the beliefs, emotions, and behaviours of struggling medical students. Acad Med 2011;86 (10 Suppl):35–8.






     2013 Mar;47(3):242-51. doi: 10.1111/medu.12052.

    The remediation challengetheoretical and methodological insights from a systematic review.

    Author information

    • 1Division of Medical and Dental Education, University of Aberdeen, Aberdeen, UK. jen.cleland@abdn.ac.uk

    Abstract

    OBJECTIVES:

    Remediation is usually offered to medical students and doctors in training who underperform on written or clinical examinations. However, there is uncertainty and conflicting evidence about the effectiveness of remediation. The aim of this systematic review was to synthesise the available evidence to clarify how and why remediation interventions may have worked in order to progress knowledge on this topic.

    METHODS:

    The MEDLINE, EMBASE, CINAHL (Cumulative Index to Nursing and Allied Health Literature), ERIC (Educational Resources Information Centre), Web of Science and Scopus databases were searched for papers published from 1984 to April 2012, using the search terms 'remedial teaching', 'education', 'medical', 'undergraduate'/or 'clinical clerkship'/or 'internship and residency', 'at risk' and 'struggling'. Only studies that included an intervention, then provided retest data, and reported at least one outcome measure of satisfaction, knowledge, skills or effects on patients were eligible for inclusion. Studies of practising doctors were excluded. Data were abstracted independently in duplicate for all items. Coding differences were resolved through discussion.

    RESULTS:

    Thirty-one of 2113 studies met the review criteria. Most studies were published after 2000 (n=24, of which 12 were published from 2009 onwards), targeted medical students (n=22) and were designed to improve performance on an immediately subsequent examination (n=22). Control or comparison groups, conceptual frameworks, adequate sample sizes and long-term follow-up measures were rare. In studies that included long-term follow-up, improvements were not sustained. Intervention designs tended to be highly complex, but their design or reporting did not enable the identification of the active components of the remedial process.

    CONCLUSIONS:

    Most remediation interventions in medical education focus on improving performance to pass a re-sit of an examination or assessment and provide no insight into what types of extra support work, or how much extra teaching is critical, in terms of developing learning. More recent studies are generally of better quality. Rigorous approaches to developing and evaluating remediation interventions are required.

    © Blackwell Publishing Ltd 2013.

    PMID:
     
    23398010
     
    [PubMed - indexed for MEDLINE]


    상상해보자: 의학교육의 새 패러다임 (Acad Med, 2013)

    Just Imagine: New Paradigms for Medical Education

    Neil B. Mehta, MBBS, MS, Alan L. Hull, MD, PhD, James B. Young, MD, and James K. Stoller, MD, MS






    위대한 돌파구는 필사적으로 필요로 했던 것이 어느 순간 갑자기 달성되면서 생겨난다

    Big breakthroughs happen when what is suddenly possible meets what is desperately necessary. 

    —Thomas L. Friedman, “Come the Revolution,” New York Times, May 15, 2002


    2010년 'Educating Physicians: A Call for Reform of Medical School and Residency는 두 번째 플렉스너 보고서라고도 불리며, 의학교육이 오늘날 마주한 중대한 도전을 지적한다.

    The 2010 publication Educating Physicians: A Call for Reform of Medical School and Residency,1 often referred to as the “second Flexner Report,” points out the substantial challenges facing medical education today.


    현재 의학교육 모델의 한계

    Shortcomings of the Current Model of Medical Education


    비효율성, 비유연성, 학습자-중심 결여

    A stark inventory of the shortcomings of the current model of medical education includes inefficiency, inflexibility, and lack of learner-centeredness.


    지금의 성적은 효과적인 의사가 되기 위해 필요한 진정한 스킬/행동/특성을 반영하지 못한다. 교수가 학생을 평가하는 것은 무척 variable하며, 종종 문제해결능력 또는 비판적 사고능력이 아니라 대인관계의 특성을 반영한다.

    Thus, these grades likely do not reflect true skills, behaviors, and attributes needed to be an effective physician. Faculty assessments of students are highly variable and often reflect interpersonal characteristics2 rather than problem- solving and critical thinking skills.


    또한, 현대 임상환경은 교수가 교육에 헌신하기 힘들게 한다. 많은 학생들이 임상실습동안 병력청취나 신체검진에 대해서 observed 되지도 못한 채 실습을 마친다.

    Also, realities of the modern academic clinical environment can challenge faculty’s commitment to recruitment and teaching. Strikingly, many students go through the required clinical rotations without once being observed taking a history or examining a patient.3


    GME도 문제이긴 마찬가지이다. 동료peer to peer 교육의 질은 GME에서 중요한 요소인데, 레지던트의 교육 스킬을 향상시키는데 관심을 두지 않고, 교육병원은 진료와 교육에 대한 의무mission의 균형을 맞추지 못하고 있다. 최종적으로 로테이션 동안에 임상경험은 '운에 맡기는' 상황이다. 이러한 갭은 전공의 근무시간 제한으로 인해서 더 악화된다.

    Graduate medical education (GME) is challenged as well. For example, the quality of peer-to-peer teaching, an important element of GME, is compromised by inattention to improving residents’ teaching skills and an unfavorable imbalance between the service and education missions of the teaching hospital.2 Finally, clinical exposure during rotations can be “hit-or- miss,” leaving gaps in trainees’ exposure. These gaps may be exacerbated by limitations on clinical exposure due to resident duty hours restrictions.


    의학교육은 사회로부터도 도전을 받고 있다. 의사 부족에 대비하여 더 많은 의사를 양성하라는 요구가 있지만 달성되지 못하고 있다. 또한 의과대학 졸업생의 부채가 $150,000를 넘어서면서 일차의료 전공을 하려는 학생이 줄고 있다.

    Medical education faces challenges at a societal level as well. There is a clear but unmet need to train more physicians to meet a massive projected physician shortage.4 Because the debt level of most students graduating from medical schools exceeds $150,000,5 choices to pursue primary care specialties may be undermined by the need to seek more remunerative specialties to pay off debt.6


    의학교육의 개혁

    Reforming Medical Education


    이상적인 상태에서는 모든 학생은 필수적인 입원환자 경험과 외래환자 경험을 쌓을 수 있어야 한다. 환자를 보는 것에 대한 감독을 받아야 하며, 환자를 본 것에 대한 형성적 피드백을 받고, 스스로의 지식, 기술을 쌓고 전문직의 사회화 과정에 활용해야 한다.

    In an ideal future state, all students would experience every essential inpatient and ambulatory clinical experience, would be observed during these encounters, and would receive formative feedback on such interactions to guide them in improving their knowledge, skills, and socialization to the profession.


    의학교육의 수준을 올리기 위해서 플렉스너는 최소한의 입학요건을 제안하였고, 모든 의과대학이 대학에 affiliation되어야 한다고 권고했다. 플렉스너는 일방적 강의를 비판하고 학생이 learn by doing 해야 한다고 했다. 또한 교육과정이 유연하여 공식적  학습formal learning을 임상경험과 연구와 통합시킬 수 있게 해야한다고 주장했다.

    To improve the standards of medical education, Flexner recommended that minimum admission standards should be established and that all medical schools should be affiliated with a university. Flexner criticized didactic teaching in lecture halls and wanted students to learn by doing. He believed that curricula should be flexible and should allow for integration of formal learning with clinical experiences and research.


    플렉스너는 이러한 제안이 '현재와 가까운 미래, 길어봐야 한 세대'를 위한 것이다 라고 인식했다.

    Flexner realized that his recommendations were for “the present and the near future—a generation at most.”8


    현재, 의학교육을 재구조화 하고 향상시키려는 노력이 다시 한 차례 이뤄지고 있다. ACGME는 시간-기반 수련모델을 역량-기반 모델로 바꾸고자 한다. 

    Today, efforts are under way to reframe and enhance medical education once again. The Accreditation Council for Graduate Medical Education (ACGME) Milestone Project is attempting to move from a time-based to a competence-based framework for progression through medical training.


    2007년 ten Cate와 Scheele는 EPA개념과 STAR를 주장하며, 현재의 역량 프레임워크와 실제 임상 진료행위의 갭을 연결시키고자 했다. 

    In 2007, ten Cate and Scheele10 proposed the concept of entrustable professional activities (EPAs) and statements of awarded responsibilities (STARs) to bridge the gap between the competency framework and practical clinical practice.


    AAMC와 NBME는 다른 기관과 협력하여 의학교육과 진료행위의 연속체에 걸쳐 학습을 추적하는 도구를 개발중이다. 

    The Association of American Medical Colleges (AAMC) and National Board of Medical Examiners are working with other accrediting agencies to develop a tool for tracking learning across the continuum of medical training and practice.11,12



    파괴적 혁신

    Disruptive Innovations


    이러한 것들 모두 종합하면, 현재의 과제는 '파괴적 혁신'이라 불리는 급진적인 새로운 패러다임을 받아들이는 것이다. 구체적으로 Bower와 Clayton은 '어떻게 새로운 급진적 패러다임이 현재 산업리더 industrial leader로부터 소외받아온 소비자들에게 더 단순하고 더 편리하고 더 맞춤화되고 더 저렴한 방식으로 이득이 되게 할 수 있는가'로 파과적 혁신을 주창했다.

    Taken together, current challenges invite radical new paradigms, which have been dubbed “disruptive innovations.” Specifically, Bower and Clayton have introduced the concept of disruptive innovations to describe how new radical paradigms can produce simpler, more convenient, more customizable, or cheaper ways of benefiting consumers who are currently being ignored by industry leaders.13


    대학의학은 국가적 문제 해결을 위한 의학교육 개혁의 급박한 필요성을 인정해야 한다. sound한 교육모델에 기반한 혁신은 더 광범위한 보건의료인력을 더 낮은 비용으로 양성할 수 있을 것이다. 이러한 맥락에서 Willliam Bennett은 "테크놀로지 대학의 메카라 할 수 있는 실리콘벨리는 대학과 교육자들이 할 수 없는, 아니 하지 않으려고 하는 방향으로의 고등교육 혁신을 진행중에 있다"라고 지적했다.

    Academic medicine must recognize the urgent need for medical education reform that will help solve the nation’s problems. Innovations must be rooted in sound pedagogic models that can help create a larger health care workforce at a lower cost. In this context, William Bennett,14 the former U.S. secretary of education, has pointed out that the “mecca of the technology universe (Silicon Valley) is in the process of revolutionizing higher education in a way that educators, colleges and universities cannot, or will not.”



    거꾸로 교실과 MOOC

    Flipped classrooms and massive open online courses


    의학교육 외 분야에서 다수의 강력한 파괴적 변화가 이미 일어나고 있다. Khan Academy 등

    In nonmedical education, a number of powerful disruptive changes already under way are changing the educational landscape. For example, the Khan Academy15 started in 2006


    Open access online courses는 이미 20년 전에 가능했으나 MOOC이란 개념은 2008년 “Connectivism and Connected Knowledge” 에 관한 강의가 전 세계 2300명의 지원자를 모집하며 유명해졌다. 이후 MOOC은 Sebastian Thurn 가 2012년 1월 스탠포드 대학의 테뉴어를 반납하고 Udacity를 시작한 것을 계기로 세계를 매료시켰다.

    Open access online courses have been available for at least two decades,18,19 but the concept of massive open online courses (MOOCs) was popularized by a group of learning researchers when a course on “Connectivism and Connected Knowledge” in 2008 attracted over 2,300 worldwide participants.20 However, MOOCs did not take the world by storm until Sebastian Thurn ceded his tenured position at Stanford University in January 2012 to start Udacity, a start-up offering MOOCs at low or no cost.21



    Khan Academy and Udacity로 인해서 전통적인 교육 역할을 잃을까 두려웠던 대학들은 이제 무료 온라인 코스를 제공하기 위해 협력하고 있다. Udacity가 런칭되고 얼마 지나지 않아 Coursera가 설립되었다. Coursera의 초기 파트너는 Stanford, the University of Pennsylvania, the University of Michigan, and Princeton University였으며, 이제는 190개국에서 150만명 이상이 Coursera의 198개 MOOC(33개 대학)을 수강중이다 이후 하버드와 MIT는 edX collaboration을 발표했다.

    Stimulated and possibly threatened by the fear of losing their traditional role in education by initiatives like the Khan Academy and Udacity, universities are now collaborating to offer free online courses. Shortly after Udacity launched, Coursera22 was founded by two Stanford faculty members with expertise in machine learning and artificial intelligence and their application to biomedical sciences. Coursera’s first university partners were Stanford, the University of Pennsylvania, the University of Michigan, and Princeton University. Currently, over 1.5 million students from 190 countries are enrolled through Coursera in 198 MOOCs from 33 universities.23 Soon after Coursera began, Harvard University and the Massachusetts Institute of Technology announced their edX collaboration,24 which will offer free content from the two universities to anyone in the world. Both Coursera and edX will offer certificates of mastery.


     

    이전 세대의 learning management systems 와 달리 새로운 시스템은 학생-중심적이며 타당한 교육적 이론에 기반을 두고 있다.

    Although previous generations of learning management systems faltered because they focused more on tracking and managing instruction and content, these new systems are student-centered and are based on sound pedagogic principles. They aim to

    • promote active, retrieval-based learning;
    • customized feedback based on analysis of vast amounts of data created by students’ performance;
    • real-time collaboration; and
    • peer learning while also creating an experience mimicking one-on-one tutoring.

    디지털 배지

    Digital badges


    디지털 배지는 또 다른 파괴적 혁신이다. 디지털 배지는 학습자에게 수여되는 전자 이미지로서, 지원서나 레쥬메에 포함될 수 있고 웹사이트나 블로그에 들어갈 수도 있다. 이 개념은 2010년 바로셀로나 컨퍼런스에서 처음 시작되었는데, 다양한 formal and informal 학습공간에서의 학습을 capture하는 것을 도와준다. 이후 곧 디지털배지는 MacArthur Foundation 로부터 2백만달러의 투자를 받게 된다.

    Digital badges are another disruptive innovation in the education world27 with implications for medical education. Digital badges are electronic images that follow learners through their lifetimes and can be included in applications and resumes or displayed on Web sites and blogs. The concept originated in 2010 at a conference in Barcelona, Spain, to help capture learning that occurs in multiple formal and informal learning spaces. Soon thereafter, digital badges received a substantial endorsement when the MacArthur Foundation funded a $2 million “Badges for Lifelong Learning Competition.”28


    베지에는 메타데이터가 들어있어서 수여자의 이름, 수여 기관, 수여기관의 정보, 수여자가 이 배지를 받기 위해서 해야 했던 것들, 이 배지를 받기 위하여 수여자가 충족시킨 기준의 근거 등이 포함된다. 따라서 디지털 배지는 스킬/성취/퀄리티를 더 섬세한granular 방식으로 보여준다. 디지털 배지는 실제 상황에서의 스킬을 마스터했다는 것을 보여주는 것으로, 고용주에게는 (일반적으로 학위에서는 잘 드러나지 않는) 전문성의 근거가 될 수 있다. 디지털배지를 수집하고 보여주는 것은 테크놀로지 세대에 있어서 동기부여 요인이 될 수 있다. 표준화된 온라인 플랫폼이 개발된 바 있다.

    Badges encode metadata containing information such as the badge recipient’s name, the institution (or individual) awarding the badge, information about the endorser (i.e., the organization that certifies or approves the badge or the badge provider), information about what the recipient had to do to get the badge, and evidence that the recipient met the criteria to earn the badge. Thus, digital badges can provide concrete evidence of skills, achievements, and qualities in a more granular manner than traditional grades and degrees. They reflect mastery of real-life skills and are valued by employers looking for evidence of expertise not often reflected by college degrees.28 Collecting and displaying electronic badges can be motivating for a generation that has grown up with technology. Standardized online platforms have been developed (e.g., Openbadges.org) for badge sponsors, badge issuers, and badge earners, allowing the issuing, collection, management, and sharing of badges across multiple Web sites and learning management systems.



     

    의학교육의 새로운 모델을 위한 비전

    Vision for a New Model for Medical Education



    우리는 협력적 온라인 학습환경을 위한 central environment를 만들 수 있다.

    We could develop a central online collaborative learning environment


    우리는 다학제간 협력을 구축할 수 있다. MOOC을 수강하는 무수한 학생은 가상의, 다학제적, 협력적 환경을 언제나 제공받을 수 있을 것이다.

    We could ensure multidisciplinary collaboration by building communities of learning. The vast numbers of students in these MOOCs would ensure that they would always have other students online at the same time helping to build a virtual, and most likely multidisciplinary, collaborative environment.


    MOOC은 교수들의 교육방식도 바꿔놓을 것이다. 예를 들어 교수들은 단순 강의를 제공하는 역할에서 벗어나, 면대면, 온라인 소그룹, 온라인 일대일 토론을 제공할 수도 있다. 이는 "거꾸로 교실"의 한 형태이며, 학생들은 기본적 학습 자료를 스스로 수업 전후에 공부하고, 아주 귀중한 (그리고 비싼) 교수들의 시간은 문제해결을 위한 협력적 학습에 사용될 수 있을 것이다.

    Massive online learning could also affect faculty practices. For example, faculty members freed from providing didactic sessions could be available for face- to-face, online small-group, or online one-on-one discussion. This would “flip the classroom”—that is, students could learn basic didactic material on their own before and after class, and valuable (and expensive) faculty time could be used for collaborative learning or problem- solving.29


    학생들은 배지 공급자를 선택할 수 있고, 의과대학과 궁극적으로는 인증기구가 설정한 파라미터에 따라서 스스로의 스케줄을 결정할 수 있다.

    Students could choose their badge providers and schedule their advancement through the curriculum guided by the parameters set by the medical school and ultimately by the accreditation bodies.


    임상실습 스케줄은 학생들이 house staff, allied health personnel, and faculty 들과 더 많은 시간을 보내게끔 바뀔 수 있으며, 이들이 로테이션동안의 학습목표에 대한 배지수여자가 된다. 의과대학의 역할은 철저한 교육훈련이 확실히 이뤄지게끔 하고, 배지수여자들에 대한 교수개발을 하고, 교육과정동안 학생의 모니터링과 자문 역할을 하는 것이다.

    The clerkship schedules would be modified to ensure that students spend more time with house staff, allied health personnel, and faculty who are certified badge providers for the learning objectives of the rotation. The role of the medical school would be to ensure rigorous training, certification, and continuing faculty development for the badge providers as well as close monitoring and advising of students throughout the curriculum. 


    디지털 배지는 EPA로 정의된 특정 기술을 마스터했음을 보여주눈데 사용될 수 있으며, 디지털 STAR이다.

    Digital badges could be used to record and display mastery of specific skills as defined in EPAs and thus would be the digital equivalents of the STARs.


    스킬의 유지를 위해서 배지는 유효기간이 설정될 수 있다. 또한 새로운 프로세스와 절차procedure가 진료의 새로운 기준이 될 수 있는 것처럼, 배지가 추가적으로 업데이트 될 수 있다.

    To support maintenance of skills, the specific badges could carry expiration dates. Also, as new processes and procedures become standard of care, the certification in women’s health would be updated with a need for additional badges.


    학생은 MOOC과 디지털배지의 데이터를 바탕으로 전자 포트폴리오를 유지한다. 이는 고용주, 대학, 동료, 환자, 면허기관과 공유된다. 교수자-중심에서 학생-중심으로, 선형적, 시간-기반 교육에서 숙달-기반mastery-based 진전progression으로 바뀐다.

    Students could maintain an electronic portfolio with data from MOOCs and digital badges they earn during their medical training. They would share this with employers, privileging hospitals, colleagues, patients, and state licensing boards. The focus would shift from teacher-centered to student-centered learning and from linear, temporal-based teaching to mastery-based progression.


    파킨슨병을 위한 온라인 학습 커뮤니티가 ParkinsonNet이라는 이름으로 네덜란드에 만들어진 바 있다. 아홉 단계 프로세스로서 지역 커뮤니티를 통해 저비용으로 파킨슨병 환자를 지원한다.

    A model for establishing an online learning community focused on Parkinson disease has been set up in the Netherlands.30 The program, called ParkinsonNet, is a nine-step process to help provide multidisciplinary care for patients with Parkinson disease in a cost-effective regional community network.


    the AAMC, the Khan Academy, and the Robert Wood Johnson Foundation 는 MCAT준비를 위한 무료 온라인 교육비디오를 만들고 있다.

    In a visionary move, the AAMC, the Khan Academy, and the Robert Wood Johnson Foundation are collaborating to create videos as a free online resource for students preparing for the Medical College Admission Test. This is an effort to help students from diverse and economically and educationally challenged backgrounds to enter the medical profession.31




    가능성을 상상하라

    Imagine the Possibilities


    두 번째 플렉스너 리포트는 네 가지 감탄할 만한 목표를 제시한다.

    The “second Flexner Report” identifies four laudable goals to improve medical education:

    (1) standardization of learning outcomes and individualization of the learning process,

    (2) integration of formal knowledge and clinical experience,

    (3) development of habits of inquiry and innovation, and

    (4) focus on professional identity formation.1





     





    8 Ludmerer KM. Commentary: Understanding the Flexner Report. Acad Med. 2010;85: 193–196.





    Just imaginenew paradigms for medical education.

    Author information

    • 1Dr. Mehta is associate professor of medicine and director of education technology, Cleveland Clinic Lerner College of Medicine of Case Western Reserve University, Cleveland, Ohio. Dr. Hull is professor of medicine and associate dean for curricular affairs, Cleveland Clinic Lerner College of Medicine of Case Western Reserve University, Cleveland, Ohio. Dr. Young is professor of medicine and executive dean, Cleveland Clinic Lerner College of Medicine of Case Western Reserve University, Cleveland, Ohio. Dr. Stoller is Jean Wall Bennett Professor of Medicine and Chairman,Education Institute, Cleveland Clinic, Cleveland, Ohio.

    Abstract

    For all its traditional successes, the current model of medical education in the United States and Canada is being challenged on issues of quality, throughput, and cost, a process that has exposed numerous shortcomings in its efforts to meet the needs of the nations' health care systems. A radical change in direction is required because the current path will not lead to a solution.The 2010 publication Educating Physicians: A Call for Reform of Medical School and Residency identifies several goals for improving the medical education system, and proposals have been made to reform medical education to meet these goals. Enacting these recommendations practically and efficiently, while training more health care providers at a lower cost, is challenging.To advance solutions, the authors review innovations that are disrupting higher education and describe a vision for using these to create a new model for competency-based, learner-centered medical education that can better meet the needs of the health care system while adhering to the spirit of the above proposals. These innovations include collaboration amongst medical schools to develop massive open online courses for didactic content; faculty working in small groups to leverage this online content in a "flipped-classroom" model; and digital badges for credentialing entrustable professional activities over the continuum of learning.

    PMID:
     
    23969368
     
    [PubMed - indexed for MEDLINE]


    의학교육에 대한 새로운 상상: 행동할 때 (Acad Med, 2013)

    Medical Education Reimagined: A Call to Action

    Charles G. Prober, MD, and Salman Khan

     

     

     

     

     


     

    우리의 신념은 의과대학생에게 평생에 걸쳐 지식을 쌓을 수 있는 프레임워크를 제공해야 한다는 것이다. 그리고 생의학 혹은 의료의 특정 영역에 적성과 열정이 있는 학생은 그 분야를 더 깊이 추구해야 한다.

    Our belief is that medical students should be provided a framework on which knowledge can be built over a lifetime of learning. And students who have aptitude and passion for developing a focus in a specific area of biomedicine or medical practice should pursue this area more deeply.

     

     

    교실 뒤집기

    Flipping the Classroom

     

    The One World School House: Education Reimagined에서 살만 칸은 교육의 새로운 모델을 제시했다.

    In The One World School House: Education Reimagined, one of us (S.K.) described a new model of education, informed in part by ongoing work with K–12 students.1

     

    '거꾸로교실'에서 이전에 교실에서 가르치던 내용은 집에서 학습하고, 숙제는 동료와 함께, 교사의 지도하에 협력적으로 교실에서 수행한다.

    There is a “flipping of the classroom”: Lessons previously taught in class are learned at home, and “homework” is performed in the classroom in collaboration with peersand guided by teachers.

     

     

    의학교육에 대한 새로운 상상

    Reimagining Medical Education

     

    초중등교육에 대한 새로운 모델이 의학교육에도 적용가능할 것이다.

    We believe that the model for reimagining K–12 education is equally relevant to medical education.

     

     

    그림 1은 새로운 모델의 핵심 요소 세 가지를 그리고 있다.

    Figure 1 depicts the three key components of our proposed model for medical education:

    • 핵심지식의 프레임워크 확립 building a framework of core knowledge;
    • 지식을 상호작용적인, 강렬한, 참여적 형태로 심기 embedding the knowledge in richly interactive, compelling, and engaging formats; and
    • 일부 영역에서는 심화 학습 추구 encouraging in-depth pursuit of knowledge in some, but not all, domains.
     

    핵심지식의 프레임워크 확립

    Building a framework of core knowledge

     

    우리가 제안하는 의학교육의 핵심 요소는 '필수 전임상 교육과정'이다. 이 교육과정은 이후 학습의 토대가 되며 진실(evergreen)인 것으로 알려진 것에 초점을 둔다. 

    The central element of our medical education proposal, depicted at the top of Figure 1, is the core preclinical curriculum. This curriculum should focus on medical knowledge that is foundational and known to be true (“evergreen”).

     

    이 과정의 목표는 이후 수업에서 구성요소(building block)으로 역할을 할 수 있는 제한된 양의 필수적인 자료 배우는 것이다. 놀랍게도 핵심 교육과정은 국가적 차원에서 결정되는 것이 아니다. 핵심 교육과정은 유기적인 것으로, 비록 학교 간 상당히 높은 비율로 유사하더라도, 여러 시대에 따라 각 의과대학마다 새롭게 만들어지고 자라나는 것이다ㅏ.

    Rather, a goal should be to identify a limited amount of critical material that serves as the building blocks for subsequent lessons. It is striking that such a core curriculum is not defined on a national basis. Core curricula tend to be organic, arising and growing over time at each medical school, even though a high proportion of core content will be similar between schools.

     

    의과대학 핵심 교육과정을 통일시키는 주요 동력은 USMLE 내용이다. 

    The one unifying driver of medical schools’ core curricula appears to be the content of the USMLE.

     

    의과대학 교육과정과 무관하게 학생들은 의과대학에서의 실라버스가 아닌 서드파티 교육자료를 가지고 이 시험을 준비한다.

    Students, irrespective of their own medical school’s curriculum, typically prepare for these examinations by using third-party review material rather than their course syllabi.

     

     

    학생들은 그들이 배운 교육과정이 표준화된 국가시험의 내용을 반영하지 못한다는 사실을 깨닫고 매우 좌절하거나 스트레스를 받는다. 스탠포드의 전임상과정의 학생을 대상으로 한 연구에서 73%의 학생이 교육과정의 내용과 USMLE step 1을 위해서 알아야 하는 내용 사이에 불합치가 스트레스의 주요 원인임을 지적했다.

    Students often express a high degree of frustration and stress when they recognize that their school’s curriculum does not mirror the content of standardized national examinations. In a recent survey of preclinical students at Stanford, 73% identified this perceived misalignment between curricular content and what they “needed to know for USMLE Step 1” as one of their major sources of stress (Porwal A, Newell G. Unpublished data. June 2012).

     

    이는 의과대학 교육과정이 '시험을 위한 준비'가 되어야 한다고 말하는 것은 아니다. 오히려, 의과대학 교육과정과 NBME 사이에 신중한 합치를 이루어야 할 필요를 말하는 것이다. 이 목적을 위하여 우리는 의과대학 협력체를 구성하여 교육과정의 핵심 내용에 대한 합의된 의견을 대표할 수 있는 자료를 만들 책임을 갖도록 제안한다. 

    This is not to suggest that medical school curricula should be designed to “teach to the test.” Rather, there needs to be a conscious alignment between those responsible for creating medical school curricula and the National Board of Medical Examiners. To that end, we propose the creation of a medical school collaborative, charged with the identification of material that would represent a consensus opinion on the core content of the curriculum.

     

    핵심 내용이 정해지면, 우리는 10분 정도의 짧은 비디오의 library를 만들 것을 권고한다. 학습자는 이것을 가지고 학교 교육과정이 조직된 것과 같은 순서로 학습 내용에 접근할 수 있다. 

    Following the identification of the core content, we further propose the creation of a library of short (~10 minute) videos that learners can use to access the content in an order consistent with the organization of their school’s curriculum.

     
     

    여러 학교가 같은 내용으로 비디오를 만드는 것이 도움이 될 것이다. 학생들은 자신의 학습 스타일과 가장 잘 맞는 것을 선택할 것이고 시간이 지나면 '최고의 비디오'가 자연스럽게 드러날 것이다. 

    We believe that it would be advantageous for multiple schools to produce videos on the same core content. Students could select the version of the presentations most consistent with their learning style. Over time, the “best” videos would emerge.

     

    학습자료는 시간이 지나면 업데이트되어야 하나, 이 과정은 필수 내용을 선정하는 과정에서 과학적 검증을 거친 것을 선택함으로써 최소화될 수 있다. 짧은 비디오 형태의 교육은 당대의 발견을 적절한 시기에 도입하는데 도움이 될 것이다.

    Material would need to be updated over time, although this need would be minimized through the selection of core content that has withstood the test of scientific validation. The short video format would facilitate the timely introduction of contemporary discoveries.

     
     

     

    지식을 상호작용적인, 강렬한, 참여적 형태로 심기

    Embedding knowledge through interactive formats

     

     

    두 번째 요소는 역동적 상호작용 세션이다.

    The second defining element of our medical education proposal is the creation of dynamic interactive sessions.

     

    이들 비디오는 SMILI의 일부일 뿐이다. SMILI 워킹그룹은 교수/학생/교육과학자/학습전문가/정보기술전문가 등으로 구성되어 있다. 우리의 핵심 목표는 수업을 보다 학생 중심의 상호작용적 형태로 진화시키고자 하는 교수를 돕는 것이다.

    These videos are, in fact, only a fraction of our overall Stanford Medicine Interactive Learning Initiative (SMILI).3 Our SMILI working group includes faculty, students, educational scientists, learning specialists, and information technology experts. Our central goal is to support faculty who want to evolve their classes into a more student-centric, interactive format.

     

    이러한 세션은 종종 서로 다른 전공의 교수들의 참여를 통해서 이득을 얻는다.

    These sessions often benefit from the participation of faculty with different types of expertise,

     

     

    비록 학생들은 교육과정 평가에서 수업을 개선할 다양한 지점을 찾아내지만, 141명의 응답자 중 82%는 기본적인 강의-기반 형태도바 이러한 형태의 모델을 더 선호한다. 가장 흔한 우려는 시간 관리이다. 학생들은 강의(또는 비디오)에 할당된 시간을 줄이지 않고 단순히 상호작용적 세션만 더하는 것을 가장 걱정한다.

    Although the students identified a number of opportunities for improvement in course evaluations, 82% of 141 respondents favored this model of instruction when compared with a primarily lecture- based format (Ransohoff K, Xie J. Unpublished data. December 2012). The most common concern expressed by our students was time management. The students expressed concern about simply adding interactive sessions without concurrently reducing the amount of time allocated for didactic instruction (by video).

     

     

    일부 영역에서는 심화 학습 추구

    Encouraging in-depth pursuit of specific knowledge

    학생들은 핵심 교육과정을 넘어선 'deep dive'를 하게 권장되어야 한다. 

    Students are encouraged to take “deep dives” beyond their core curriculum.

     

    스탠포드의 scholary concentration는 deep dive의 한 사례이지만 이것만 있는 것은 아니다.

    Examples of “deep dives” include, but are not limited to, what we currently offer students for their “scholarly concentration” pursuits at Stanford:

     

    핵심은 학습자의 적성과 열정을 지지하고 다가가는 것이다.

    The key is to tap into and support the individual learner’s aptitude and passion.

     

    최근, Bruce Alberts는 "표면적 학습의 실패"를 강조하였다.

    A recent editorial, authored by Bruce Alberts,4 the editor-in-chief of Science, underscored the “failure of skin-deep learning.” Alberts argues that we need to replace the current overview of subjects with a series of deep explorations. He cites research that demonstrates that “the most meaningful learning takes place when students are challenged to address an issue in depth.”4

     

     

    Effecting Change Through Multi-Institutional Collaboration

     

    협력적인, 다기관의 노력이 필요할 것이다.

    Perhaps the result would not be a one-world medical school house but a collaborative, multi-institutional effort to reimagine medical education.

     

     

     

     

     

     

     

     

     

     

     

     

     

     


     2013 Oct;88(10):1407-10. doi: 10.1097/ACM.0b013e3182a368bd.

    Medical education reimagined: a call to action.

    Author information

    • 1Dr. Prober is senior associate dean for medical education and professor of pediatrics, microbiology, and immunology, Stanford School of Medicine, Stanford, California. Mr. Khan is founder and executive director, Khan Academy, Mountain View, California.

    Abstract

    The authors propose a new model for medical education based on the "flipped classroom" design. In this model, students would access brief (~10 minute) online videos to learn new concepts on their own time. The content could be viewed by the students as many times as necessary to master the knowledge in preparation for classroom time facilitated by expert faculty leading dynamic, interactive sessions where students can apply their newly mastered knowledge.The authors argue that the modern digitally empowered learner, the unremitting expansion of biomedical knowledge, and the increasing specialization within the practice of medicine drive the need to reimagine medical education. The changes that they propose emphasize the need to define a core curriculum that can meet learners where they are in a digitally oriented world, enhance the relevance and retention of knowledge through rich interactive exercises, and facilitate in-depth learning fueled by individual students' aptitude and passion. The creation and adoption of this model would be meaningfully enhanced by cooperative efforts across medical schools.

    PMID:
     
    23969367
     
    [PubMed - indexed for MEDLINE]

     

     

    "내가 의대에 적합한걸까?" 1학년 학생들의 확신결여 현상에 대한 이해(Med Teach, 2015)

    ‘‘Am I cut out for this?’’ Understanding the experience of doubt among first-year medical students

    RHIANON LIU1, JOSEPH CARRESE1,2,3, JORIE COLBERT-GETZ4, GAIL GELLER1 & ROBERT SHOCHET1 

    1Johns Hopkins University School of Medicine, USA, 2Johns Hopkins Bayview Medical Center, USA, 3Johns Hopkins Berman Institute of Bioethics, USA, 4University of Utah School of Medicine, USA




    의과대학생들이 느끼는 정신적 고통의 수준은 높은 편인데, 그로 인해 우울, 탈진, 의욕상실 등을 경험한다. 의과대학생의 distress는 유급부터 자살충동까지 여러 부정적 결과를 초래한다.

    Medical students experience high rates of distress, often taking the form of depression, burnout, and loss of empathy over the course of medical training (Compton et al. 2008; Hojat et al. 2009; Dyrbye et al. 2011a). Medical student distress is further associated with negative personal consequences, ranging from thoughts of dropping out to suicidal ideation (Dyrbye et al. 2010c).


    의과대학생들이 distress를 받는 한 가지 이유는 샘솟는 '의구심'이다. 의과대학생들이 노출된 여러 스트레스 요인들에는 사회적 지지가 결여된 환경에서의 생활, 재정 문제, 수면 부족, 과도학 학습시간, 인간이 겪는 고통과 죽음에 대한 대면 등이 있다.

    One source of medical student distress that has not been adequately studied is emergence of doubt. Medical students are exposed to a range of stressors that include living away from social supports, financial debt, lack of sleep, long hours of study, and encountering human suffering and death (Compton et al. 2008).


    연구 설계

    Study design


    mixed-methods study를 수행하였다. 

    We conducted a mixed-methods study involving a survey and focus groups examining the phenomenon of doubt among first-year medical students at the Johns Hopkins University School of Medicine (JHUSOM) in June, 2012. 


    We asked students to answer 13 questions embedded in an annual, online advising program survey: 

    • nine questions about doubt were developed based on literature review of medical student well-being, and 
    • four questions reflecting other measures of distress from a validated well-being index (Table 1) (Dyrbye et al. 2010b, 2011b). 


    For the focus groups, we created a semi- structured interview guide based on literature review and expert opinion. We tested the guide in a pilot focus group, then revised it prior to use. One study team member (R.L.) served as the focus group facilitator. Questions in the survey and interview guide addressed types of doubt, coping with doubt, and impact of doubt. The distress questions on the survey addressed burnout, depression, stress, and loss of empathy.



    학업생활을 하면서..
    1. 나는 의과대학이 나에게 옳은 선택이었는지에 대한 의심을 한 적이 있다.
    2. 나는 JHUSOM이 나에게 맞는 의과대학이었는지에 대한 의심을 한 적이 있다.
    3. 나는 내가 의과대학 학업환경에서 성공할 수 있을지에 대한 스스로의 능력에 대한 의심을 한 적이 있다.
    4. 나는 의과대학에서의 학업 외 다른 생활에 대해서 의심을 한 적이 있다.
    5. 의과대학생활에 대한 의심으로 인해서 내 스스로의 목적이 뭔지 의문을 가지게 되었다.
    6. 의과대학생활에 대한 의심으로 인해서 내가 누구인지 의문을 가지게 되었다.

    의과대학생활에 대해 의심이 생겼을 때...
    7. 건강한 형태로 대응하고자 노력한다.
    8. JHUSOM의 문화는 내가 그러한 의심을 표현하는 것을 주저하게 한다.
    9. 그런 경우에 어떤 도움을 받을 수 있는지 잘 모르겠다.

    스트레스 문항
    10. 의과대학이 정서적으로 냉담한 사람이 되도록 만든다는 걱정을 합니까?
    11. 의과대학에서 탈진을 경험합니까?
    12. 지난 몇 달 간, 기분이 저하되거나, 우울하거나, 희망이 없다는 기분을 느낀 적이 있습니까?
    13. 지난 몇 달 간, 모든 것이 감당하지 못할 정도로 쌓여있다는 기분을 느낀 적이 있습니까?




    자료 분석

    Data analysis

    처음 세 문항을 가지고 두 그룹으로 분류, 이후 logistic regression 시행

    For the survey items, we dichotomized students based on their responses to the first three items (Table 1, questions 1–3). Students who responded ‘‘agree’’ or ‘‘strongly agree’’ to at least two of these items were classified into the moderate/high doubt group, and the remaining students into the low/no doubt group. We then used logistic regression to compare the likelihood of these groups ‘‘agreeing’’ or ‘‘strongly agreeing’’ with statements about coping with doubt and impact of doubt (Table 1, questions 5–9), and to compare the likelihood of these groups experiencing the four types of distress (Table 1, questions 10–13).


    포커스그룹 자료 분석

    For the focus groups, one author (R.L.) transcribed the audio-recordings and then four members of the study team (R.L., R.S., J.C., G.G.) independently coded the transcripts. Each transcript was read by at least two readers, and coded using an editing style of analysis (Miller 1999). We iteratively reviewed our codes to identify major themes.


    질적 결과

    Quantitative Results


    1번 문항에 대해서는 46%가, 2번 문항에 대해서는 39%가, 3번 문항에 대해서는 51%가 스스로 의심을 표했다.

    Forty-six percent (51/112) of students doubted (agreed or strongly agreed) whether medical school was the right choice for them, 39% (44) doubted whether JHUSOM was the right choice, and 51% (57) doubted their ability to succeed in the academic environment of medical school.


    이 세가지에 기초하여 20%는 고의심, 29%는 중증도 의심, 22%는 저의심, 29%는 무의심 집단으로 구분.49%-51% 정도의 비율

    Based on response patterns for these three items, 20% (23) experienced high doubt, 29% (32) moderate doubt, 22% (25) low doubt, and 29% (32) no doubt. In sum, 49% (55) experienced moderate/ high doubt, while 51% (57) experienced low/no doubt.


    저의심/무의심 집단에 비해서, 중등도의심/고의심 집단은 우울, 무기력 등을 겪을 가능성이 두 배 이상 높음

    Compared to those withlow or no doubt, students with moderate or high doubt weretwice as likely to experience being down, depressed, orhopeless and to experience emotional hardening. 






    Qualitative Results


    Student responses werecategorized into three broad themes: types of doubt, ways ofcoping with doubt, and impact of doubt (Table 5)


    확신결여의 유형 Types of doubt


    내가 정말 의사가 되고 싶은가? Do I want to become a doctor?


    내가 의사가 될 만큼 유능한가? Am I capable of becoming a doctor?


    의심에 대처하는 자세 Coping with doubt


    의심의 영향 Impact of doubt




    고찰


    '의심'의 경험과 그것을 관리하는 것이 의과대학생의 웰-빙에 중요한 요인이다. 1학년 학생 중 doubt의 유병률이 매우 높다는 것이 놀라운 결과로서, 거의 절반이 중등도- 고- 의심 상태임을 확인할 수 있었고, 이는 주로 의과대학 진로에 대한 자신의 열망이나 능력에 대한 것이었다. 더 나아가서 고의심 집단은 스스로의 정체성이나 목적에 대해서도 의심하는 경향이 높았다.

    Our results suggest that the existence of doubt and its management are indeed important components of medical student well-being. A striking finding from this study is the high prevalence of doubt among first-year medical students. Nearly half of these students reported moderate to high levels of doubt, largely related to uncertainty about their desire or ability to pursue a career in medicine. Furthermore, students who experienced higher levels of doubt were more likely to question their sense of identity and purpose.


    질적, 양적 결과를 조합하면 의과대학생이 받는 스트레스를 '의심'의 한 현상으로 이해하는 것도 가능한데, 이 때 의심은 catalyst나 mediator, 혹은 독자적인 형태로 존재할 수 있다

    The combination of quantitative and qualitative results linking doubt to other forms of distress enriches our understanding of medical student distress by including the phenomenon of doubt, either as a catalyst or mediator of known forms of distress, or as a distinct form of distress.


    연구결과는 개인의 목적이나 자기자신이 누군가에 대한 의심이 만연해 있음을 지적하는데, 의과대학이 개인적 성장과 전문직정체성형성(PIF)에 결정적 시기임이 보여진 바 있다. Cohen 등은 PIF에 부정적 영향을 줄 수 있는 훈련과정으로서 높은 기대, 지식/술기 부족에 대한 내적 공포, 배제적이고 위계적 문화, 고통과 죽음을 직면하는 정서적 무게감 등을 꼽았다. 이번 포커스그룹에서 학생들은 이러한 이슈에 대해서 비슷한 이야기를 많이 했다. 의학교육계가 정체성형성에 초점을 둔다면, 학생이 의심과 관련해서 이를 인식하고 극복하고자 하는 노력에 관심을 기울이는 것이 유용할 수 있다.

    Study participants indicated that pervasive doubt led them to question both their personal purpose and their sense of who they were. Previous work has shown that medical school is a critical time for personal development and professional identity formation (PIF) (Cohen et al. 2009; Holden et al. 2012). Cohen et al. identified several aspects of the training process that may negatively influence PIF, including pressures like high expectations and internal fear of inadequate know- ledge or skills, the exclusive and hierarchical culture of medicine, and the emotional weight of facing suffering and death (Cohen et al. 2009). Our focus group participants raised many of these same issues, supporting the idea that doubt is an important factor in medical students’ PIF. As the medical education community increases its training focus on identity formation, adding students’ perceptions and struggles with doubt to the discourse may be useful (Jarvis-Selinger et al. 2012).


    포커스그룹에서는 의심의 긍정적 측면과 파괴적 결과가 모두 나타났다. 긍정적 측면은 의심을 어떻게 다루어야 하는지 의과대학 초기에 경험할 수 있어서 미래의 진료 상황에서도 비슷한 불확실성에 대응할 수 있다는 것이다.

    Focus group participants described both helpful and destructive consequences to their experiences with doubt. On the positive side, learning how to manage a sense of doubt early in medical school could prepare students to deal with future uncertainty in the context of patient care


    이번 연구에서 우려되는 점은 학생들이 의심을 품고 있을 때 탈진이나 우울에 빠질 수 있다는 것이다.

    A concerning finding of our work, however, is that students felt harboring doubtcould lead to burnout and depression. 


    그렇다면, 어떻게 학생들이 의심을 건설적으로 다루고, 성장할 수 있도록 도울 수 있을 것인가? 이다.

    A critical question, then, is how to help students manage their doubt constructively, enhancing their growth in the face of inevitable uncertainty, rather than letting it overwhelm and discourage them.


    Dunn 등이 연구한 의과대학생의 모델에서는 '대응력 원천(coping reservoir)'를 제안했다. 각 학생은 각각의 개인적 원천이 있는데 성격/기질/대응방식에 따라 달라진다. 이 원천은 건강하거나 불건강한 대응방법으로 인해 채워지거나 비워질 수 있다. 더 나아가서 스트레스와 같은 요인은 원천을 고갈되게 할 수 있고, 사회적 지지는 이것을 채워준다. 이 모델은 본 연구와 일치하는데, 학생들이 말한 '고갈 요인'은 많은 경우 의심의 형태로 존재했으며, '충전 요인'은 의심을 대처하는 효과적인 방법이었다. 의과대학생과 의사의 회복탄력성이 강조되고 있다. 의심을 의과대학에서 받는 스트레스의 중요한 요인으로 바라볼 때 학생들의 탈진을 예방하고 회복탄력성을 길러줄 수 있을 것이다.

    One model of medical student well-being by Dunn et al. (2008) introduces the idea of a coping reservoir. Each student has an individual reservoir, determined by personal traits, temperament, and coping style. The reservoir can then be filled or drained by healthy or unhealthy coping methods. Furthermore, other factors like stress can drain the reservoir, while social support can fill it. This model corresponds to the processes described by students in our study. Students described many of the ‘‘depleting factors’’ as types of doubt, and many of the ‘‘replenishing factors’’ as helpful ways of coping with doubt. There is increasing recognition of the importance of physician resilience and of training medical students to be resilient (Epstein & Krasner 2013; Nedrow et al. 2013; Zwack & Schweitzer 2013). Addressing doubt as an important component of medical student distress may help educators guide students towards resilience rather than burnout during a grueling training process and challenging career.





    Glossary 


    Coping reservoir: 

    A term used to describe the positive and negative strategies used to cope with stress and how they interact. Dunn LB, Iglewicz A, Moutier C. 2008. 

    A conceptual model of medical student well-being: Promoting resilience and preventing burnout. Acad Psychiatr 32(1):44–53. 


    Identity formation: 

    The development of the distinct personality of an individual regarded as a persisting entity in a particular stage of life; a person’s mental representation of who he or she is in which individual characteristics are possessed and by which a person is recognised or known. Erikson EH. 1950. Childhood and society. New York: W. W. Norton. Josselson R. 1987. Finding herself: Pathways to identity development in women. San Francisco: Jossey-Bass.





    Jarvis-Selinger S, Pratt DD, Regehr G. 2012. Competency is not enough: integrating identity formation into the medical education discourse. Acad Med 87(9):1185–1190.











     2015 Dec;37(12):1083-9. doi: 10.3109/0142159X.2014.970987. Epub 2014 Oct 16.

    "Am I cut out for this?" Understanding the experience of doubt among first-year medical students.

    Abstract

    PURPOSE:

    Existing research shows that medical students experience high levels of distress. The purpose of this study was to understand howmedical students experience doubt, and how doubt relates to distress.

    METHODS:

    A mixed-methods study was conducted among first-year students at the Johns Hopkins University School of Medicine in June 2012.Students answered survey questions and participated in focus groups about doubt and other forms of distress.

    RESULTS:

    Ninety-four percent (112) of students responded to the survey, with 49% reporting a moderate or high degree of doubt. Compared to those reporting no or low doubtstudents with moderate/high doubt were significantly more likely to question their purpose and identity, struggle to cope with doubt, and experience depression and emotional hardening. Twenty-eight percent of students (34/112) participated in focus groups to explore their doubt, and three themes emerged: types of doubt, ways of coping with doubt, and impact of doubt.

    CONCLUSIONS:

    Doubt is highly prevalent among first-year medical students, affects their identity and purpose, and has positive and negative consequences. Doubt among medical students merits awareness and further study, as it may be an important mediator of students' emerging identity and sense of well-being.

    PMID:
     
    25319402
     
    [PubMed - in process]


    비공식적 학습용 웹사이트 분석(IJSDL, 2014)

    ANALYZING THE HUMAN LEARNING AND DEVELOPMENT POTENTIAL OF WEBSITES AVAILABLE FOR INFORMAL LEARNING

    Minkyoung Kim, Eulho Jung, Abdullah Altuwaijri, Yurong Wang, and Curt Bonk






    Despite the existence of overlap, we defined each of the six categories distinctly:


    1. Language learning 


    resources use technology-aided language learning with an integration of sound, voice interaction, text, video, and animation. It empowers self-paced interactive learning environments that enable learners to achieve learning outcomes without being restricted to place or time. Often, such environments involve numerous opportunities for participation users and multiple methods for motivating their success. Online language learning often entails high levels of self-directed and reciprocal learning or supporting peer learning (see Ehsani & Knodt, 1998)



    2. Outdoor and adventure learning 


    is a hybrid online educational environment that provides students with opportunities to explore real-world issues through authentic learning experiences within collaborative online learning environments. Inquiry-based learning including teamwork, authentic data analysis, and project-based learning is encouraged (see Doering, 2006). 


    3. Social change/global 


    resources seek to educate and inform people about issues and needs relating to social change, including poverty, hunger, AIDS, civics, the environment, etc. Technology is often used to create innovative ways to spread social good and access to learning worldwide. It is also used to empower and inspire people for the right cause.


    4. Virtual education 


    refers to learning environments where teacher and student are separated by time or space, or both. Course delivery can be through course management applications as well as various multimedia and Web 2.0 tools. Virtual education may be managed by organizations and institutions that have been created through alliances and partnerships to facilitate teaching and learning. Some virtual education websites provide learner services such as advising, learning assessment, and program planning (see Farrell & the Commonwealth of Learning, 2001). Our categorization and ratings are limited to virtual education resources that are available to individual learners at no cost.


    5. Learning portals 


    are centralized learning centers or repositories that contain an aggregation of educational information on a topic, often current or continually updated. Learners explore according to their own interest, time, and space. Learning portals support user and context learning, and are less centered on administration of that content and the results of the learning. 


    6. Shared online video 


    includes any educational video (YouTube or other webstreamed videos) that can be watched or shared. Some such sites offer syndicated programming and professional documentaries or filmmaking, whereas others are supported by lay people. These sites often allow for interaction via comments and annotation. They often allow for downloading of content.




    Table 1. Definitions of Evaluation Criteria





    Table 3. Top 25 Rated Learning Websites



    Table 4. Top 25 Websites According to Informal Learning Criteria and Category











    ANALYZING THE HUMAN LEARNING AND DEVELOPMENT POTENTIAL OF WEBSITES AVAILABLE FOR INFORMAL LEARNING

    Minkyoung Kim, Eulho Jung, Abdullah Altuwaijri, Yurong Wang, and Curt Bonk


    The advancement of learning technology in recent decades has broadened the possibilities for online learning in both formal and informal settings. This research was designed to reveal the essential characteristics of successful online resources and technology tools that are important resources for self-directed learning. Over the span of a year, a team of researchers collected and analyzed 305 informal learning websites and virtual education websites available at no cost to individual learners. The websites were categorized into the following six subject domains: language learning, outdoor and adventure learning, social change and global learning, virtual education, learning portals, and shared online video. Content analysis was employed to evaluate the 305 websites using eight evaluation criteria: content richness, functionality of technology, extent of technology integration, novelty of technology, uniqueness of learning environment/learning, potential for learning, potential for lifechanging impact, and scalability of the audience. The six categories or types of informal learning were then compared by applying the eight criteria.

    "문제"학생: 이 문제는 누구의 문제인가? AMEE Guide No.76

    The ‘‘problem’’ learner: Whose problem is it? AMEE Guide No. 76

    YVONNE STEINERT

    McGill University, Canada






    Introduction

    이 가이드는 문제학생들을 대처하는 방법에 대한 프레임워크를 제시하고자 한다.

    Clinical teachers often work with students or residents whom they perceive as a “problem”. For some, it is a knowledge deficit that first alerts them to a problem; for others it is an attitudinal problem or distressing behaviour (Steinert & Levitt 1993). And in some cases, it is difficult to know if the learner is, indeed, presenting with a problem. The goal of this Guide is to outline a framework for working with “problem” learners, which includes strategies for identifying and defining learners’ problems, designing and implementing appropriate interventions, and assuring due process. The potential stress of medical school and residency training will also be addressed, as will a number of prevention strategies. Although some of the issues involved in teaching students and residents may differ (e.g. length of exposure to the learner; available methods of assessment), the principles for working with “problem” learners remain the same. Moreover, although many of the examples in the Guide come from working with students and residents in medical specialties, the approaches apply to learners in all of the health professions (e.g. Clark et al. 2008). Identifying learners’ problems early – and providing guidance from the outset – can be an important investment in the training and development of future health professionals. It is hoped that this Guide, based on experiences in working with students and residents (Steinert & Levitt 1993; Steinert 2008) will be of help to clinical teachers, program directors, and faculty developers.


    정의 Definitions

    다양한 용어가 사용된 바 있다. ABIM에서는 "문제 레지던트"를 "윗사람 - 대체로 프로그램 관리자나 수석전공의 - 으로부터 인터벤션이 필요할 정도로 중대한 문제를 일으킨 전공의"라고 정의하고 있다. 다른 연구자는 "정동적, 인지적, 구조적, 대인관계적 어려움으로 인해서 학업 수행능력이 크게 떨어지는 사람"이라고 정의한 바 있으며, 정서적 스트레스나 물질 남용으로 인한 이차적 손상으로 인한 문제를 지적하기도 한다. 본 가이드에서는 "지식, 태도, 술기의 중대한 문제로 인해서 훈련 프로그램의 기대치를 충족시키지 못하는 학생/레지던트"라고 정의하고자 함.

    A variety of terms have been used to describe the “problem” learner: the “resident in difficulty”; the “troublesome learner”; the “disruptive student”; and the “impaired physician” (Shapiro et al. 1987; Grams et al. 1992; Gordon 1993; Steinert et al. 2001; Yao & Wright 2001). The American Board of Internal Medicine (1999) has defined a “problem resident” as a “trainee who demonstrates a significant enough problem that requires intervention by someone of authority, usually the program director or chief resident”, whereas Vaughn et al. (1998) have provided the following definition: “a learner whose academic performance is significantly below performance potential because of a specific affective, cognitive, structural, or interpersonal difficulty”. The term has also been used to refer to impairment, secondary to emotional stress or substance abuse (Grams et al. 1992). This Guide will define a “problem” learner as a student or resident who does not meet the expectations of the training program because of a significant problem with knowledge, attitudes or skills (Steinert 2008).



    유병률 Prevalence

    Prevalence를 보고한 연구는 적지만 적게는 5.8%에서 9.1%까지 있다. 한 연구에 따르면 가장 흔한 문제는 '불충분한 의학지식', '부족한 임상판단', '비효율적 시간사용' 등이다. 

    Studies reporting the prevalence of “problem” learners are limited (Roback & Crowder 1989; Yao & Wright 2000; Reamy & Harman 2006). However, reported rates vary from 5.8% over a four-year period in a Psychiatry program (Yao & Wright 2000) to 9.1% over a 25-year period in a Family Medicine program (Reamy & Harman 2006). In one study (Yao & Wright 2000), the most frequent problems identified by teachers were: insufficient medical knowledge (48%); poor clinical judgment (44%); and inefficient use of time (44%). In another study (Reamy & Harman 2006), insufficient knowledge and attitudinal problems were identified as the most common challenges, followed by interpersonal conflict, psychiatric illness, family stress and substance abuse. Not surprisingly, “problem” residents rarely identify themselves (Yao & Wright 2000).


    중요하게 기억해야 할 사실 중 하나는, '문제학생'을 다룰 때 우리는 흔히 선생으로서 선입견을 갖기 쉽지만, 대부분의 학습자는 강한 학업수행능력과 높은 성공 동기를 가지고 있다는 사실이다. 또한 Brenner et al은 "대부분의 지원자는 졸업할 때까지 별다른 간섭이 필요 없는 성공적인 레지던트가 될 것이다. 이들은 그 길을 가는 동안 평균적인 정도의 어려움을 겪을 것이다"라고 했다. 그러나 문제학생의 존재는 프로그램 전체에 영향을 줄 수 있다. 왜냐하면 이들에 대한 모니터링, 상담, remediation등이 프로그램과 교수의 자원을 잡아먹기 때문이다. 일부 교육자들은 '문제학생'의 존재가 프로그램 전체의 integrity를 손상시키거나, 동료들의 경험에 악영향을 줄 것을 우려한다.

    It is also important to remember that, although working with “problem” students or residents can easily color our perceptions as teachers, the majority of learners demonstrate strong academic performance and high motivations to succeed (Hays et al. 2011). Moreover, as Brenner et al. (2010) have stated, “most applicants will become successful residents who progress without interruption towards graduation, facing only the usual stumbles of normal professional development along the way”. However, the presence of a “problem” learner can significantly affect an entire program (Brenner et al. 2010), as increased monitoring, counseling, or remediation may tax the resources of both the program and the faculty. Some educators also fear that the presence of a “problem” learner may damage the integrity of the training program or negatively influence the experience of peers (Yao & Wright 2001).


    선생으로서 우리는 어떤 학생이 이러한 '문제아'가 될 것인가를 알고 싶어 한다. 미리 안다면 회피할 수 있기 때문이다. 그러나 지금까지 여러 연구에서 의과대학/레지던트 지원자를 screen하거나 예측하는 용도로 신뢰할 수 있는 요인은 밝혀진 바 없다.

    As teachers, we often wonder if it is possible to predict who will become a “problem” learner, hoping that we can avoid some of the anguish that is related to this educational experience. To date, however, studies have not been able to isolate factors that we can reliably use to either screen applicants to medical school/residency or predict future problems (Dubovsky et al. 2005; Brenner et al. 2010).



    증상과 증후 “Signs and symptoms”

    학습자가 어려움에 있음을 보여주는 다양한 종류의 증상이 잇다. 

    A range of “signs” may suggest that a learner is in difficulty (Evans & Brown 2010; Evans et al. 2010). These signs include 

      • failing a written or practical test; 
      • poor (or late) attendance at regularly scheduled events; 
      • inadequate knowledge or clinical skills that are inconsistent with stage of training; 
      • unprofessional behaviors with patients or peers; 
      • poor interpersonal skills; 
      • a lack of insight; 
      • anxiety; 
      • depression or reluctance to become part of the team. 
      • A lack of professional behavior is also a common indicator (Bennett et al. 2005; Greenburg et al. 2007). 


    Hay등은 전형적인 문제로 학습기술의 부족, 조직 기술의 부족, 정신건강 악화, 미성숙함, Insight 부족, 개인적 위기 등을 언급한다.

    In an exploratory study, Hays et al. (2011) developed a framework of “typical” problems that included poor learning skills, poor organizational skills, poor mental health, immaturity, poor insight and major personal crises. Interestingly, a lack of insight has been identified as one of the most difficult problems to address.


    학습자들이 다양한 요인의 결과로서 어려움을 겪을 수 있다는 것을 알아야 한다.

    It is also important to note that learners can encounter difficulty as a result of many factors, including exhaustion and fear of failing, substance abuse, illness, family and personal issues or academic challenges (Bennett & O’Donovan 2001; Tyssen & Vaglum 2002; Evans & Brown 2010). Mental and physical illnesses, as well as learning disabilities, are relatively common in the general population; not surprisingly, they frequently occur among medical students and residents as well (Frank-Josephson & Scott 1997; Faigel 1998; Dyrbye et al. 2005; Midtgaard et al. 2008).



    문제학생을 다루기 위한 프레임워크 A framework for working with “problem” learners

    Although different approaches to working with problem learners exist in the literature (e.g. Shapiro et al. 1987; Gordon 1993; Vaughn et al. 1998; Kahn 2001; Mitchell et al. 2005), the following framework, which has been described previously (e.g. Steinert & Levitt 1993; Steinert 2008) and is outlined in Table 1, has been found to be helpful to clinical teachers and program directors.





    직관에서 문제 파악까지 

    From intuition to problem identification


    학생이나 전공의의 문제를 정의하는 것은 여러 단계를 거치게 되며, 무언가 이상하다는 직관이나 느낌에서 시작되는 경우가 많다. 

    Defining a student's or resident's problem usually involves several steps (Steinert & Levitt 1993), beginning with a hunch or intuition that something is amiss. This intuition may come from the direct observation of a learner with a patient or repeated interactions in both formal and informal settings. When teachers (or primary supervisors) first suspect a problem, they should ask themselves three initial questions in order to verify their suspicion: What is the problem? Whose problem is it? Is it a problem that must be changed? Answering these questions will help to determine whether the learner actually has a problem, what it might be, and whether something needs to be done. By going through this process, teachers will also be able to develop a working hypothesis that they can later confirm with the learner and other colleagues.


    문제가 무엇인가? What is the problem?

    우리의 경험상, 학습자의 문제는 대체로 지식, 태도, 술기 중 하나의 범주에 들어간다. 

    In our experience, learners’ problems usually lie in one of three areas: knowledge, attitudes or skills (Steinert 2008). 

        • Knowledge problems, sometimes called cognitive difficulties (Hicks et al. 2005), often include deficiencies in basic or clinical sciences. Attitude problems (often manifested as behaviors) usually include difficulties related to motivation, insight, doctor-patient relations or self-assessment. 
        • For many, attitude problems are easy to identify but challenging to resolve. 
        • Skill deficits can include problems with interpretation of information, interpersonal or technical skills or clinical judgment and organization of work. More importantly, there is often an overlap between skill deficits and attitudinal problems (Steinert & Levitt 1993). (...)







    누구의 문제인가? Whose problem is it?

    문제가 어디 있는지를 찾아내는 것은 문제 정의 단계에서 가장 어려운 것 중 하나다. 우리의 경험에 따르면, 교사들은 주로 학생들한테 문제가 있다고 생각한다. 그러나 문제는 교사나 시스템에 있을 수도 있다.

    Determining where the problem lies may be one of the most challenging aspects of problem definition. Based on our experience, it appears that teachers often assume that it is the learner who has the problem. However, difficulties may also lie with the teacher or the system.


    교사의 문제 Teachers’ issues

    교사는 다양한 역할을 하고 있으며, 스스로 원하는 수준만큼 역할을 다해내지 못한 이유로 학생이나 레지던트에게 '문제아' 딱지를 붙일 수도 있다. 모든 경우에 교사들은 확인된 문제를 위해서 스스로 어떤 기여를 하고 있는가를 분석해봐야 한다. 예컨대, 교사들은 단순히 개인적으로 스트레스를 받거나 교사 역할에 불만족스럽기 때문에 - 학생이 문제학생이 아님에도 - '문제아'라는 딱지를 붙이곤 한다.

    Teachers play many roles (Whitman & Schwenk 1997) and may label a student or resident as a “problem” because they cannot fulfill the role they wish to fill (Steinert & Levitt 1993). Teachers also enter educational situations with specific assumptions, expectations and experiences, all of which can lead to problems; so can the teachers’ own stresses or biases. At all times, teachers should try to carefully analyze to what extent they are contributing to the identified problem. For example, they may label a learner a “problem” because they are personally stressed or dissatisfied with their teaching role, not because the learner is “in trouble”.


    문제학생을 다루는 것은 교사들에게 다양한 반응을 불러일으킨다. 보통 다음과 같다.

    Working with “problem” learners also engenders a variety of reactions in teachers. Common responses reported by teachers include the following (Steinert 2008):


            • Denial (Maybe he's just having a bad day …)
            • Avoidance (I think I’ll schedule another clinic during my teaching session.)
            • Desire to rescue or protect (If I work hard enough, I will be able to help her …)
            • Anger/frustration (Oh no! Why do I always get the challenging residents?)
            • Helplessness/impotence (It's so hard! We’ll never be able to do it.)
            • Acceptance (Let's get on with it and design a good remediation!)


    당연하지만, 교사의 감정은 학습자의 감정을 보여주는 거울이기도 하다. 따라서 개인의 반응을 확인하는 것은 유용한 평가도구가 될 수 있다.

    Not surprisingly, teachers’ sentiments often mirror the learner's feelings. Identifying personal responses can, therefore, serve as a useful assessment tool.



    학습자의 문제 Learners’ issues

    학습자의 문제에는 다음과 같은 것들이 있을 수 있다.

    In addition to gaps in knowledge, attitudes or skills (as described above), learners’ problems can include: 

            • stress relating to training or career concerns; 
            • life stresses, such as immigration, moving to another location, marriage or divorce; 
            • medical or psychiatric illness; 
            • substance abuse; 
            • learning disabilities or interpersonal conflict. 


    25%의 인턴은 약간의 우울함을 느끼며, 12.%의 주니어 의사는 알코올 남용을 하고 있다. 동시에 학습자의 기대/가정/반응이 문제 파악에 영향을 줄 수 있다. 추가적으로 학생이나 레지던트에게 '문제아'라는 딱지를 뭍이는 것도 중요한 영향력을 발휘하는데, 가급적 교사들은 어떤 딱지도 붙이지 말아야 한다. 장점보다 단점이 더 많다.

    As an example, in one report, 25% of interns were mildly depressed and 12.5% of junior doctors were misusing alcohol (Lake & Ryan 2005). At the same time, learners’ expectations, assumptions, and reactions to the perceived problem (e.g. a sense of inadequacy or insecurity; anger or fear of losing control) may also contribute to problem identification. In addition, the process of labeling a student or resident as a “problem” can have a significant impact, and whenever possible, teachers should try to avoid all labels. They may cause more harm than good.


    시스템의 문제 Systems’ issues

    시스템의 문제는 보통 찾아내기가 쉽지 않을 수 있다. 

    Systems problems, which are often difficult to identify, can include unclear standards and responsibilities beyond perceived levels of competence, an overwhelming workload, inconsistency in teaching or supervision, or a lack of feedback or assessment (Steinert & Levitt 1993). Learners will often report that they do not receive feedback from their supervisors on a routine basis or that their summative assessment is a “surprise,” while teachers will say that they did not have enough time to observe performance. Clearly, this challenge lies with the educational system and not the learner. Other systems’ issues include reduced clinical exposure, fragmentation of clinical teams (Evans et al. 2010), conflicting demands or expectations, and difficult patient problems. In multiple ways, identifying systems’ constraints is critical in defining the problem and designing an appropriate intervention. At the same time, teachers must feel supported by the system and know that they have access to resources when dealing with challenging situations.


    반드시 바뀌어야 하는 문제인가? Is it a problem that must be changed?

    학습자와 학습자의 동료들과 대화를 나누기 전에,교사들은 과연 그 문제가 반드시 해결되어야 하는 문제인지, 그리고 더 중요하게는, 만약 해결되지 않는다면 무슨 일이 생길지 생각해봐야 한다. 많은 교사들인 학습자들이 기쁘고 협력적이기를 바라나 이러한 기대는 현실적이지 않으며, 교사들은 어떤 행동이 자신들의 목적이나 가정에 위배되기 때문에 '문제'라고 바라보지는 않았는지 스스로 물어야 한다. 동시에, 초기에 문제를 발견하는 것이 중요한데, Evans et al은 "어려움에 빠진 학습자가 발견되더라도, 이들은 보통 큰 문제가 생기기 전까지는 방치되곤 한다"라고 했다. 가능하다면 이러한 '큰 문제'를 피해나가야 한다.

    Before talking to the learner and other colleagues, a critical next step, teachers should ask themselves whether a particular problem must be changed, and more importantly, what would happen if it was not addressed (Steinert 2008). Although many teachers would like their learners to be happy, pleasant and cooperative (Steinert & Levitt 1993), this expectation is not realistic, and teachers must ask themselves whether they have labeled specific behaviors as problematic because they interfere with their own objectives or assumptions. It is not surprising for a teacher to realize that a suspected problem does not need to be addressed. At the same time, early identification is critical, for as Evans et al. (2010) have stated, “although learners in difficulty are often recognized, they frequently go unchallenged until a critical event occurs”. To the extent that is possible, we should try to avoid these critical events.



    문제 발견에서 문제 정의까지 

    From identification to problem definition


    Once teachers have identified the problem(s) and considered their own role in the process, careful data-gathering is needed to confirm the teachers’ working hypothesis. This step includes a detailed description of the problem (e.g. when did it start; what makes it worse), the learner's perception of the problem, the learner's strengths and weaknesses in knowledge base, attitudes and skills (if not already identified), the learner's relevant life history (e.g. current life stresses; substance abuse; coping strategies), the teacher's perceived strengths and weaknesses, and colleagues’ perceptions, feelings, expectations and assumptions (Steinert 2008).


    임상 교사들은 흔히 학생과 직접 이야기하는 것을 꺼린다. 일부는 그것은 자신들이 역할이 아니라고 느끼기 때문이며, 일부는 효과적으로 수행할 기술이 없기 때문에 오히려 벌집을 건드리는 격이 되는 것은 아닐지 걱정하기 때문이다. 일부 교사들은 이미 너무 일이 많아서 그런 것을 할 여력이 없다고 하고, 어떤 사람들은 보복적 법률 소송을 당할 것을 걱정한다. 이러한 것과 무관하게, 다음과 같은 질문이 필요하다.

    Importantly, clinical teachers are often reluctant to talk to the learner directly. Some believe that it is not their role to do so; others feel that they lack the skills to do so effectively or worry that they are opening a potential “can of worms” that will make things worse (Evans et al. 2010). Some teachers feel that they are already “overstretched” and cannot take the time to get involved, whereas others fear reprisal through legal action (Lake & Ryan 2005). Irrespective of these sentiments, however, a direct approach is needed as teachers work through the following questions:



    문제가 무엇인가?

    1. What is the problem?

    지식인지 태도인지 술기인지. 문제를 개선시키거나 악화시키는 요인 뿐 아니라 관찰가능한 행동이나 패턴을 찾아보아야 함. 학습자가 가진 문제의 "functional inquiry"를 위해서 임상기술을 활용할 수도 있음.

    Teachers need to ascertain a detailed description of the learner's problem(s) and must decide if it is primarily one of knowledge, attitude, or skill. They must also try to identify observable behaviors and patterns as well as factors that either alleviate – or exacerbate – the problem. In multiple ways, teachers should rely on their clinical skills in order to conduct a “functional inquiry” of the learner's problem(s).



    문제에 대한 학습자의 생각은 어떠한가?

    2. What is the learner's perception of the problem?

    교사가 문제가 있을 것이라는 의심을 가졌을 때, 그것이 실제로 문제인가를 확인하기 위해서는 학생이나 레지던트와 이야기해볼 필요가 있다. 어떤 경우에 많은 교사들은 이 단계를 회피하려고 하지만, 학습자가 자신의 어려움과 강점, 동기와 가정에 대해 어떤 생각을 가졌는지를 알아보는 것은 중요하며, 필수적 첫 단계이다. 더 중요하게는 학습자 중심의 인터뷰가 문제에 대한 학습자의 인식, 문제의 역사와 관련된 요인, 개인적 요인 등을 밝혀줄 수도 있다. 이러한 면담이 그 자체로 인터벤션이 될 수 있음을 기억하는 것이 중요한데, 왜냐하면 일부 학습자들은 그들이 겪고 있는 문제에 대해서 이야기할 기회를 갖는 것 자체를 좋아할 것이며, 자신에게 관심을 가지고 지지해주려는 교사에 대해 감사할 것이기 때문이다.

    Talking to the student or resident is the most important step in confirming the teacher's suspicion that there is, indeed, a problem. For some reason, many teachers try to avoid this step, but ascertaining the learner's perception of his/her difficulties and strengths, motivations and assumptions, as well as training and career objectives, are an essential first step. More specifically, a learner-centred interview may uncover the learner's perception of the problem (as well as its causes), the history of the problem and related factors (e.g. academic difficulties) and personal factors (Evans & Brown 2010). It is also important to remember that such an interview can be considered an intervention in itself, as some learners welcome the opportunity to talk about what is troubling them and appreciate the teacher's support and interest in helping them from the outset.



    자신의 강점과 약점에 대해서 어떻게 생각하는가?

    3. What are the learner's perceived strengths and weaknesses?

    학습자와의 대화는 학습자의 장점과 개선이 필요한 부분에 대한 철저한 평가가 필요하다. 그러나 안타깝게도 교사들은 부족한 부분을 찾아가는 식의 접근법에 의존한다. 대신 학습자의 강점과 개인적 자질에 대한 평가가 필요하다.

    The discussion with the learner should include a thorough assessment of his or her strengths and areas for improvement in knowledge, attitudes and skills. Unfortunately, teachers often rely upon a deficit-based approach to teaching and learning; instead, an appreciation of the learner's strengths and personal qualities is needed. This information may also be gleaned by observing the learner in multiple situations (and different electives or rotations) or talking to colleagues and other members of the health care team. As described above, learners may struggle for a number of reasons. It behooves us to explore these issues together with the student or resident – and to draw upon our clinical skills in the assessment process.


    학습자의 관련 과거 경험은 무엇이 있는가?

    4. What is the learner's relevant life history?

    비록 교사들은 '개인적'질문을 함으로서 도를 넘을 수 있다는 걱정을 하기도 하지만, 이러한 벙보가 진단을 내리고 적절한 개입 계획을 결정하는데 필요하다. Yao와 Wright는 학습자의 낮은 수행능력은 다음의 것과 관련되어 있을 수 있다고 했다.

    Teachers often ask themselves how much – and what kind of – information they should gather. In fairness to the learner and the teacher's ability to make an accurate diagnosis and treatment plan (Steinert & Levitt 1993), teachers should inquire about current life stresses, recurrent problems and support systems. It is also important to inquire whether the learner has experienced similar problems in the past or whether this is a new challenge for him/her. As an example, a student with a learning disability is often aware of this problem long before the teacher has made the diagnosis. Although teachers are often concerned that they may be crossing a boundary by asking “personal” questions, this information is needed to make a diagnosis and to determine an appropriate intervention plan. Yao and Wright (2001) have suggested that a learner's poor performance may be related to one of the following causes: 

        • behavioral issues, such as those related to professionalism; 
        • medical conditions, including psychiatric illness; 
        • difficulty coping with stress; 
        • substance abuse and cognitive issues, including learning disabilities. 


    Mitchell et al은 "레지던트의 수행능력을 수행능력에 영향을 미치는 배경요인에 대한 이해 없이 이해하려고 하는 것은 개별 환자와 그 환자가 처한 상황에 대한 이해 없이 특정 치료방침에만 매달리는 것과 같다"

    This classification may be helpful in guiding this line of questioning. As Mitchell et al. (2005) have stated, “attempting to understand resident performance without understanding factors that influence performance is analogous to examining patient adherence to medication regimens without understanding the individual patient and his or her environment”.



    교사와 시스템의 강점 및 약점은 무엇인가?

    5. What are the teacher's – and the system's – perceived strengths and weaknesses?

    Cleland et al은 의학교육자들이 학생들의 underpeformance를 보고하기 꺼려하는 것을 보여준 바 있다.

    As stated earlier, the problem may lie with the teacher and/or the system. It is therefore important to ascertain the teachers’ own strengths (and areas for improvement) in knowledge, attitudes and skills, as well as his/her current life stresses and challenges. In an interesting study, Cleland et al. (2008) explored the reluctance of medical educators to report underperformance in students. In multiple ways, their findings, which included teachers’ attitudes towards a specific student (as well as failing students in general), normative beliefs and motivations, skills and knowledge, and environmental constraints, are all relevant in this context. We must also be aware of the potential role that the system can play in contributing to a “problem” situation. As stated earlier, it is worthwhile to identify systems issues so that we can try to minimize their influence as a contributing factor to the learner's problem.



    그 학습자의 동료들은 어떻게 바라보고 있는가?

    6. How do colleagues perceive the learner?


    (...)


    자료를 효과적으로 모으기 위해서는 임상 교사들은 다양한 상황에서 학습자를 관찰해야 하며, 환자의 문제를 학생/레지던트의 문제와 함께 보아야 하고, 그들의 평가가 동료들의 평가와 일치하는지도 확인해보아야 한다. 공식적 시험 결과도 도움이 될 수 있으며, 다른 로테이션에서의 피드백도 도움이 될 수 있다. 그러나 학습자를 직접 관찰하는 것, 그리고 직접 이야기해 보는 것의 중요성을 간과해서는 안되며 Yao와 Wright가 보고한 바와 같이, 문제는 보통 '직접 관찰' 또는 '결정적 사건'을 통해서 드러나기 때문이다.

    To gather data effectively, clinical teachers need to observe learners in multiple situations, systematically review patients’ problems with students and residents, and work to ensure that their assessments are congruent with those of their colleagues (Steinert & Levitt 1993). Formal test results may also be helpful (Evans et al. 2010), and when appropriate, so is feedback from other rotations. However, the importance of direct observation and talking to the learner cannot be undermined. As noted by Yao and Wright (2001), problems are most often identified through direct observation (82%) and critical incidents (52%).



    From definition to intervention

    어떤 문제들은 긴급한 조치가 필요할 수 있고, 어떤 것은 시간이 더 필요할 수도 있다. 앞에서 기술한 바와 같이, 학습자를 모든 단계에 포함시키는 것이 중요하며, 계획이 무엇이든 인터벤션은 학습자의 well-being에 대한 진지한 관심을 가지고, 환자와 환자 가족의 안전을 고려하여 진행되어야 한다.

    Once a working diagnosis has been established, teachers must design an appropriate intervention. This step includes a consideration of the problem(s) to be addressed, the available intervention options, who should be involved in the intervention, the proposed timeline for both the intervention and the evaluation of outcomes, and the process for documentation. Some problems (e.g. psychiatric illness; substance abuse) will require urgent attention (Steinert 2008); others will require additional time for observation or monitoring. As stated previously, it is essential to involve the learner in every step. In addition, whatever the plan, the intervention should ideally be conducted with genuine concern for the well-being of the learner (Winter & Birnberg 2002) and the safety of patients and their families.


    어떤 문제를 해결하고자 하는가?

    1. What problem are you trying to address?

    대부분의 문제가 독립적으로 발생하는 것이 아니므로, 문제의 우선순위를 정하고 어떤 것을 먼저 해결할 것인가를 정하는 것이 중요하다. 교사간, 교사와 학습자간의 합의를 이루는 것이 중요한 첫 단계이다. 이단계에서 교사들은 학습자들로 하여금 문제를 인지하고 인정하도록 도와야 한다. 또한 학습자가 가능한 전략이나 해결책에 대한 조언을 구하게 해야 한다. 경험에 따르면 공동의 의사결정이 필수적이다. 미리 설계된 인터벤션은 학습자가 그 계획에 동의하지 않으면 대체로 실패하고 만다.

    Most problems are complex in nature and do not occur in isolation. It is therefore important to prioritize the perceived problems and to decide which one will be addressed first. Consensus between teachers, and between the teacher and the learner, is also a critical first step. During this phase, the teacher may need to help the learner recognize and acknowledge the issues affecting performance (Evans et al. 2010) and solicit feedback on possible strategies and solutions. Based on experience, shared decision-making is essential; in fact, the designed intervention will usually fail if the learner does not agree with the intended plan.


    확인된 문제를 어떻게 해결할 것인가?

    2. How will you address the identified problem?

    A number of interventions, outlined in Table 2, can be considered when working with “problem” learners. In some instances, the clinical teacher will be involved in all components; at other times, program directors or other senior administrators will be responsible (Steinert 2008). However, in all situations, we must be aware of what options are available to us and one person must be accountable. Frequently, time with monitoring, or further assessment, is sufficient. In other cases, we need to enhance teaching and learning opportunities, either by increasing time for observation or feedback, or by arranging one-on-one coaching with staff or peers. In some situations, workloads might need to be reduced to allow for independent study and reading (for knowledge problems) or increased practice and feedback (for skill-related deficits). Alternatively, a formal remedial program may be required, with clearly defined goals and objectives, learning strategies, and evaluation methods (Steinert 2008). Although suspension, probation or dismissal (from the program) are not desirable options, they must, at times, also be considered (Ikkos 2000).






    추가 시간

    Additional time

    As in medicine generally, time can be an effective healer (Steinert & Levitt 1993). Some learners can overcome their difficulties by moving out of a particularly challenging or stressful rotation, or by working with a different clinical teacher. Others gain confidence or skill as time progresses. Whenever possible, additional time should be accompanied by careful monitoring through observation.


    추가 평가와 모니터링

    Further assessment and monitoring

    In other situations, further assessment will be needed. This will include spending more time with the learner and carefully monitoring what they do. It will also involve observing the student or resident in different contexts, with different patients and families. Including colleagues and other members of the team in this assessment phase can be equally beneficial. It is often surprising how invaluable team coordinators’ comments can be with regard to a student's or resident's behaviors with patients and other health professionals.


    일대일 토론

    One-on-one discussions

    One-on-one discussion with the learner constitutes an important strategy that is often taken for granted. Although frequently not considered part of an intervention, meeting with the learner, to review specific issues or concerns, can be very worthwhile. Such a meeting can also be used to clarify expectations (which learners often feel are not explicit) and discuss pre-assigned readings, clinical problems or identified deficits (e.g. problem-solving).


    교수 학습 기회 향상

    Enhanced teaching and learning opportunities

    At times, increased observation and feedback can help to address identified problems. This is especially true for knowledge-based problems or skill-related deficits. More frequent case discussions and chart reviews can facilitate knowledge acquisition, as can mini-tutorials, review of patient management problems and discussion of pre-assigned readings. Increased opportunities to observe role models in action can encourage the acquisition of interpersonal skills, as can time in a simulation-based environment. The latter can also help to address deficiencies related to technical skills, interviewing skills and team work. A skill-based training course, tailored to individual needs, might also be recommended.


    근무량 감축

    A reduced clinical workload

    A reduced clinical workload, with protected time to focus on knowledge or skill acquisition, may at times be in order. If the learner is feeling overwhelmed by the clinical demands (in relation to their own expertise and competence), a lesser workload may decrease stress so that learning can occur.


    로테이션, 장소, 감독관 변경

    A change in rotation, venue or supervisor

    Changes at the system level should also be considered. Changing the learner's rotations (e.g. scheduling an easier rotation, working in a different setting or clinical environment) can be another alternative, as can changing the primary supervisor or adding other teachers (with different skill sets) to the roster. Working with “problem” learners is generally quite time-consuming for teachers, and sharing the workload may be beneficial to all concerned.


    동료나 멘토의 지지

    Peer or mentor support

    Medical school and residency training can be a stressful time for students and residents (Dyrbye et al. 2005) At times, a supportive peer or teacher can be very helpful. The role of peers in working with “problem” residents has been debated by clinical teachers and residents alike; however, the value of “near-peer” support cannot be underestimated as long as peers maintain confidentiality and respect.


    레미디얼 프로그램

    A remedial program, with defined goals, objectives and strategies

    The above components are frequently used in a more formal remedial program, which may include a variety of teaching methods (e.g. videotape reviews of clinical encounters, role plays of difficult doctor–patient interactions) or extra rotations in a specific discipline, with protected time for increased supervision, study and review (Steinert & Levitt 1993). Known to address specific problems with reasonable success, such programs require clearly defined goals and pre-determined outcomes. Moreover, in some settings, they have had considerable success with both students (Schwartz et al. 1998) and residents (Catton et al. 2002).


    상담, 치료

    Counseling or therapy

    Although most clinical teachers find this a difficult option to pursue, counseling or therapy may be indicated, especially if the learner is presenting with aggressive or depressive symptoms, substance abuse, or psychiatric problems. Learning disabilities can also not be ignored as an underlying factor for perceived problems and often require intervention (Coles 1990). This is also an area where outside consultants or expertise should be sought.


    휴가

    A leave of absence

    A survey of internal medicine programs from 1979 to 1984 found that 1% of the residents required a leave, and 56% of the programs granted leaves of absence because of “emotional impairments” (Smith et al. 2007). Although teachers are often reluctant to consider this option, it should be part of the repertoire of interventions, especially as leaves of absence are one of the suggested options for health-related problems including substance abuse (Long 2009).


    정학/퇴학

    Probation, suspension or dismissal

    명확한 정책이 있어야 하지만, 한편으로는 Ikkos가 언급한 바와 같이, 문제학생을 다루는 법적 제도적 장치는 국가이나 기관에 따라서 다르다.

    Academic dismissal(학업능력에 따른 퇴학)과 displinary dismissal(그 조직이나 기관의 정책에 위배되는 행동에 따른 퇴학)은 구분되어야 한다.

    무엇이 성공인지에 대한 장기적인 관점이 필요하다. 

    In order for this option to work, clear policies must be in place. It is also true that this intervention is dependent on local norms and values, and as Ikkos (2000) had said, the legal and administrative framework to deal with “problem” learners differs across countries and authorities. In addition, only a few reports describe termination policies in medical training programs (Irby et al. 1981; Tulgan et al. 2001). However, this option must be seriously considered, despite teachers’ reluctance to do so. Irby and Milam (1989) distinguish between academic dismissals, which result from academic or clinical performance issues, and disciplinary dismissals, which follow violations of institutional rules or policies. Irrespective of the nomenclature, however, we might need to dismiss learners from their programs when remediation efforts fail (Catton et al. 2002). As Winter and Birnberg (2002) have stated in the description of their work with impaired residents, we must have a long range view of success and “recognize that suspension or dismissal may only be a temporary setback … short-term failure, including relapse, may in fact lead to long-term success”. It is also important to remember that re-directing a student to another specialty – or career – may not be a failure in the long run.


    Dudek et al은 교사가 학생을 낙제시키기를 머뭇거리게 하는 네 가지 요인을 밝혔다.

    In an interesting study, Dudek et al. (2005) identified four factors to explain teachers’ reluctance to fail students and residents: 

          • a lack of documentation; 
          • a lack of knowledge about what to document; 
          • anticipation of an appeal; and 
          • a lack of remediation options. 

    These factors are equally important in this context and must be addressed by program directors, educational leaders and administrators. In fact, we must put systems into place to protect our teachers as well as our learners.



    흔한 인터벤션의 방법들은 다음과 같다.

    As described previously (Steinert 2008), experience has shown that common interventions include: 

        • increased observation and feedback (for gaps in knowledge or skills); 
        • increased time with a faculty advisor (for knowledge deficits, attitudinal problems, interpersonal conflict or family stress); 
        • weekly study sessions, core content review and videotaping of clinical encounters (for knowledge, attitudinal or skill problems); and 
        • psychiatric counseling (for attitudinal problems, interpersonal conflict, family stress or substance abuse). 



    어떤 성과가 기대되는지, 인터벤션의 실패는 어떨지가 초창기에 결정되어야 한다.

    Anticipated outcomes, and consequences of failed interventions, must also be determined early in the process, though it is heartening to note that close to 90% of “problem” learners succeeded after a structured intervention or remediation program (Winter & Birnberg 2002; Reamy & Harman 2006).



    인터벤션에 누가 관여할 것인가?

    3. Who will be involved in the intervention?

    비록 규정에 따라 정해져 있을 수도 있지만, 가능하다면 프로그램 관리자나 관련된 부학장이 인터벤션 계획에 관여해야 한다.

    At times, the primary supervisor (or clinical teacher) will be responsible for both designing and implementing the intervention. At other times, another member of the team or outside consultant will be involved. Although this decision is often dependent on institutional policy or local norms, whenever possible, the program director or associate dean (or someone in a similar position) should be consulted and involved in the intervention plan. So should the student or resident. Depending on the design and complexity of the intervention, and the specific educational context, it may also be helpful to have more than one person involved in the intervention plan, and ideally, this should be discussed with the learner. In all cases, it is important that the learner is comfortable with the teacher(s) involved in the intervention, all of whom should have the time and expertise to deal with the learner's difficulties. As highlighted above, peer support can also be invaluable.


    인터벤션의 time frame은 어떻게 되는가?

    4. What is the time frame for the intervention?

    교사들이 흔히 하는 실수는 명확한 목적이나 목표, 시간계획 없이 인터벤션에 뛰어드는 것이다. 

    Teachers often err by “jumping into” an intervention without clear goals, objectives or time frames. Clearly, both the teacher and the learner would benefit from knowing how long the intervention will last and what the expected outcomes will be. It is also important to recognize that time frames may be context-specific. For example, much of undergraduate training occurs in one-month blocks; postgraduate training often provides more time for intervention and problem resolution. Clearly, the dimension of time must be seriously considered.


    인터벤션을 어떻게 평가할 것인가?

    5. How will the intervention be evaluated?

    Whatever the intervention, learners often lament that they do not know what is expected of them. Accordingly, the criteria for success must be carefully laid out from the outset. For example, if the teacher and learner are working on improving technical skills, the expectations for success should be clearly enunciated at the outset and a system for evaluating progress should be determined. It is equally important to schedule regular, pre-arranged meetings between the learner and the supervisor to monitor ongoing progress, to determine whether the intervention plan has been able to achieve its specified goals (Steinert & Levitt 1993), and to make mid-course corrections. These meetings should also be scheduled before the intervention starts so that they are not viewed as a method of crisis intervention. Finally, it is essential to outline what consequences will be considered if no improvement is noted. At times, the problem may need to be re-defined; at other times, the remediation program will need to be extended or altered. And as stated earlier, probation or dismissal may need to be considered as a viable option. In this era of outcomes-based education, clear outcomes are needed at every step of the way.


    인터벤션의 기록을 어떻게 남길 것인가?

    6. How will the intervention be documented?

    필수적 요소임에도 이 단계는 보통 생략되거나 우연에 맡겨지곤 한다.

    Although thorough documentation is an essential component of all interventions, this step is often omitted or left to happenstance. For example, 

        • teachers must document the identified problem (with supporting data), 
        • the discussions with the learner and colleagues, 
        • the intervention plan, and 
        • the observed outcome of designated activities. 

    Some teachers find it helpful to write up the intervention plan as a “learning contract”, outlining how the problem will be dealt with, in a particular time period; others prefer to keep carefully documented process notes. Though often skeptical at first, learners frequently express appreciation at knowing what is expected of them and what outcomes are desired. Documentation is also essential in ensuring due process.



    정당한 절차를 어떻게 확보할 것인가?

    7. How will due process be assured?

    교사들은 반드시 정당한 절차에 따라 협력적으로 접근해야 하며, 공정함을 담보해야 하고, 비밍르 유지해야 하며, 충분한 정보를 제공하고 동의를 받아야(informed consent)한다. 공정함이란 학습자가 교육 프로그램의 목적을 알고, 승진의 규칙을 아는 것이다. 이는 또한 정기적으로 피드백이 주어지며, 교사의 평가는 직접 관찰한 객관적 자료에 기반한다는 것을 말한다. Documentation은 자연정의(natural justice)를 공고히 하는데 중요하며, 교사들은 평가, 인터벤션, 토론 등을 기록해야 한다. 동시에 이러한 정당한 절차는 bilateral한 과정이며, 동료들을 위하여 natural justice를 확실히 해야 함을 기억해야 한다. 많은 교사들이 문제학생을 다루는데 있어서 '외로움' '취약함'등을 어려움으로 꼽았다. 

    Teachers must work collaboratively to ensure due process (Rankin & Kelly 1986; Rose 1989) and to guarantee fairness, confidentiality, and informed consent. Fairness implies that the learner is aware of the program's educational objectives and rules of promotion. It also implies that feedback is given on a regular basis and that the teachers’ evaluations are based on first-hand exposure and objective data. Documentation is critical in assuring natural justice, and teachers must be encouraged to document their assessments, interventions, evaluations and discussions with the learner. At the same time, we must remember that due process is a bilateral process and we must work to ensure natural justice for our colleagues. Many a teacher has commented on the “loneliness” and “vulnerability” that they experience when working with “problem” learners (Steinert 2008).


    조직 차원의 정책을 개발하고 학습자의 문제를 다루기 위한 프로토콜을 만드는 것은 레지던트의 권한과 정당한 절차를 확실히 하기 위해서 중요하다. 비록 이러한 정책이나 프로토콜이 각 조직마다 다르다고 하더라도, "chain of command"를 반드시 명시하여 누가 어떤 부분에 책임이 있는지, 보고 구조는 어떻게 되는지, 평가와 개입의 time frame은 어떤지, 명확하고 세심한 기록의 필요성 등이 기술되어 있어야 한다.

    Developing an institutional policy and protocol for handling learners’ problems can also help to assure residents’ rights and due process. Although such a policy and protocol will differ for each organization (or institution), it should describe the preferred sequence of events, the “chain of command” and who is responsible for which part of the protocol, the reporting structure, the time frame for assessment and intervention, and the need for clear and careful documentation. For example, some schools have entrusted a Board of Examiners (Catton et al. 2002) to handle residents’ problems; others have designated program directors or postgraduate deans to be responsible. Irrespective of the chain of command, it is important that all faculty members are aware of local policies and protocols and that the institution maintain a uniform approach to learners requiring attention. 


    왜 robust system이 필요한가?

    Long (2009) has described a number of reasons why it is important to have robust systems in place to work with “problem” learners. This includes 

        • the need for uniformity, 
        • the development of expertise, and most importantly, 
        • the early identification of learners in difficulty.



    Prevention of problems

    의과대학과 수련기간은 많은 학습자들에게 스트레스가 심한 혼란의 시기이다. 스트레스의 원인에는 다음과 같은 것들이 있다.

    Medical school and residency training is “a time of stress and turmoil for many learners” (Dabrow et al. 2006). As stated earlier in this Guide, and as described in the literature, these stresses come from a number of sources, including 

      • communication problems in the workplace, 
      • feelings of not being respected, 
      • the constraints of collaborative work, 
      • the potential gap between the medical school and clinical care, 
      • work overload, 
      • responsibility towards patients, 
      • worries about career plans and a perceived lack of knowledge (Luthy et al. 2004). 

    Depending on their life experiences and coping strategies, students’ responses to stress may – or may not – be adaptive (Dyrbye et al. 2005). Although a full discussion of prevention strategies is beyond the scope of this article, a number of approaches are worth considering. 


    유용한 프레임워크

    For example, Langlois and Thach (2000) have provided a helpful framework by which to look at the prevention of difficult learning situations, modeled along the lines of primary, secondary, and tertiary prevention. 

      1. At the level of primary prevention (i.e. preventing the problem before it occurs), they suggest a well-developed orientation program that includes the sharing of course expectations, a discussion of mutual goals and objectives, and ongoing assessment. 
      2. With respect to secondary prevention (i.e. early detection), they concur with the suggestions made in this Guide and re-affirm the importance of paying attention to early clues, responding quickly, and providing ongoing feedback and monitoring. 
      3. Tertiary prevention (i.e. managing a problem to minimize impact) is of course more complex and includes a number of carefully crafted intervention strategies; it is also wise at this stage to not try to “rescue” the learner by ignoring the problem or accepting poor performance. 

    "다양한 잠재적 위험 상황은 기대를 설정하고, 피드백을 주고, 사려깊은, 지속적 평가를 제공함으로서 예방 가능하다"

    Interestingly, few prevention programs for teachers in distress have been described in the literature. However, each of these suggestions would be equally relevant to the teacher and the system. As Langlois and Thach (2000) have said, “many potentially difficult situations can be prevented by setting expectations, giving feedback, and providing thoughtful, ongoing evaluation”.



    수련과정의 스트레스 인정

    Acknowledge the stress of training

    As Hays et al. (2011) have said, “academically bright and ambitious medical students must cope with a combination of curriculum, assessment, career choice, [and] personal, family and social pressures”. As teachers, we must acknowledge the stress and strain of undergraduate and postgraduate training and offer support to deal with systemic issues (Howell & Schroeder 1984; Peterkin 1991). We must also provide an educational environment that allows for learner differences, timely feedback and ongoing assessment so that problems are identified early and evaluations are not a “surprise”. In addition, we should consider the role of faculty advisors or mentors, so that learners can receive support and guidance in an atmosphere of trust and respect. Peer support, which can help to guard against delay in problem identification, can also be a useful intervention (Steinert 2008).


    학습 기술과 평생학습 전략 증진

    Promote study skills and life-long learning strategies

    Although life-long learning is often identified as an important attribute of competent practitioners, the skills inherent to this process are not frequently taught. Perhaps, it is time to re-dress this gap and teach students and residents ways in which to maximize learning in the workplace, direct their own learning, seek input from others, and use evidence at the point of care (Teunissen & Dornan 2008).


    관련된 교육 이벤트 구성

    Organize relevant educational events

    Some programs have held annual retreats to combat stress in residency training (e.g., Klein et al. 2000). Others have developed wellness (or assistance) programs to deal with the stress inherent in medical training (Borenstein 1985; Zoller et al. 1985). Irrespective of the program design, these activities include a discussion of relevant stresses and ways of identifying high stress levels, strategies for coping with stress, and information about available resources. Some programs have also included psychiatric counseling as part of their wellness or assistance program (Dabrow et al. 2006). As an example, the program at the University of South Florida College of Medicine offers confidential evaluation, brief counseling, and referral services (as appropriate). Importantly, this program is not focused solely around crisis intervention; it also incorporates a number of components of a successful assistance program: total confidentiality; easy access; education regarding availability of services and overall integration with the educational program (Dabrow et al. 2006). Educational courses and seminars on professionalism may also be warranted (Marco 2002). Demonstrating a lack of unprofessional behaviour is often seen among “problem” learners. It is, therefore, important to both teach and assess these behaviors in an explicit manner (Cruess et al. 2009) and make expectations clear.


    교수 개발

    Develop your faculty

    교육의 목적과 구성

    As stated earlier, most teachers do not feel prepared to handle “problem” learners effectively and faculty development has a critical role to play in this context. In our setting, we frequently offer workshops on the “problem” student and resident to our faculty members. The goal of these workshops is to provide a systematic framework for teachers “to help them in their task by emphasizing early identification, accurate diagnosis, and appropriate interventions” (Steinert et al. 2001). Workshop topics include: defining the problem; data gathering: confirming the diagnosis; designing and implementing the intervention; and assuring residents’ rights. Participants work in small groups and are encouraged to focus on their own challenges and lessons pertinent to their own settings. Program evaluations have shown that this workshop can be an effective way to sensitize teachers to the challenges of working with “problem” learners, to increase their knowledge and skill, and to help them become more aware of systems issues that may impact learner progress. Muller et al. (2000) have also highlighted the benefits of a faculty development workshop in helping teachers to apply an “interactional model to working with learners in difficulty”. As they pointed out, such an activity can help faculty to explore critical issues, test out their assumptions, identify new ways of working with learners’ challenges and begin to work collaboratively.


    Some general principles

    In closing, some general principles will be emphasized. Although “success” is not always possible, most “problem” learners do succeed in finding their way to a fulfilling career.


    조기 발견이 중요하다. Early identification is critical

    As Evans et al. (2010) have stated, “early identification and early support, before the trainee or student runs into major difficulties, should be regarded as the gold standard for educational supervision.” Most educators have encountered learners with significant gaps in knowledge or professional behaviors that have not been addressed earlier in their training. We fail this group by not failing them, and at a minimum, we must provide them with feedback, remedial guidance, and a plan (LeBlanc & Beatty in press).


    학생이나 레지던트는 고되다. It is not easy to be a student or resident

    As teachers and program directors, we need to remember that it is not easy to be a student or resident. It is also true that some learners complete their trajectory without any problems, but the essence of training can be stressful for many. Awareness – and acknowledgement – of this fact can be very helpful for both the learner and the teacher.


    성과에 초점을 두자 An outcomes approach is warranted

    문제 해결에는 두 가지 프레임이 있다. 하나는 '문제'적 관점이고, 다른 하나는 '성과'적 관점이다. 

    Claridge and Lewis (2005) describe two frames for problem solving: a problem frame and an outcome frame. 

        • In the former, which focuses on the details of the problem and the deficiencies at hand, the over-riding motivation is to “escape”. 
        • The outcome frame, on the other hand, focuses on internal motivation to change, finding solutions and moving towards a positive outcome. Belief in the individual as resourceful and capable underlies this frame, as does the notion of exploration and change. Clearly, all of these factors are important in working with “problem” learners.










     2013 Apr;35(4):e1035-45. doi: 10.3109/0142159X.2013.774082. Epub 2013 Mar 15.

    The "problem" learner: whose problem is it? AMEE Guide No. 76.

    Author information

    • 1Centre for Medical Education, Faculty of Medicine, McGill Universit, Canada. yvonne.steinert@mcgill.ca

    Abstract

    Clinical teachers often work with students or residents whom they perceive as a "problem". For some, it is a knowledge deficit that first alerts them to a problem; for others it is an attitudinal problem or distressing behaviour . And in some cases, it is difficult to know if the learner is, indeed, presenting with a problem. The goal of this Guide is to outline a framework for working with "problem" learners. This includes strategies for identifying and defining learners' problems, designing and implementing appropriate interventions, and assuring due process. The potential stress of medical school and residency training will also be addressed, as will a number of prevention strategies. Identifying learners' problems early - and providing guidance from the outset - can be an important investment in the training and development of future health professionals. It is hoped that this Guide will be of help to clinical teachers, program directors and faculty developers.

    PMID:

     

    23496125

     

    [PubMed - indexed for MEDLINE]


    외국대학 출신 학생들의 국내 의학전문대학원 적응에 관한 질적 연구 (KJME, 2015)

    Returning students’ perspectives on adjusting to medical graduate school in Korea: an interview study

    박소연, 권오영, 윤태영

    So Youn Park, Oh Young Kwon and Tai Young Yoon

    경희대학교 의학전문대학원 의학교육 및 의인문학교실

    Department of Medical Education and Medical Humanities, Kyung Hee University School of Medicine, Seoul, Korea



    1996년 대통령자문 교육위원회가 처음 제안한 의학전문대학원 제도의 도입 취지는 법학이나 의학 분야의 전문인들에게 보다 높은 수준의 교양과 전문성을 제공함으로써 세계무대에서 활발하게 활동할 수 있도록 하자는 것이었다[1]. 당시 기존의 의학교육은 다양한 인재를 양성하는 기능이 취약하다고 비판 받았으며, 각 학교들은 대학의 다양화와 특성화 방법에 대해 고민하였다[2]. 이에 따라 의학전문대학원 체제에서는 이러한 요구에 부응하는 한 가지 방편으로 지원자들에게 다양한 입학기회를 제공하였으며 그 중 하나가 국외 소재 외국대학에서 학사학위를 취득한 학생들을 선발하는 특별전형이다. 한국의학교육 평가인증 규정에 따르면 학생 선발에 대해 특성화된 대학원 편제에 합당한 다양한 경력을 소유한 학생 선발 여부가 평가의 한 항목으로 포함되어 있으며[3], 이러한 특별전형을 통해 현재 해마다 일정 비율의 외국대학 출신 학생들이 의학전문대학원에 입학하고 있다. 일례로 2011년 기준 주요 13개 의학전문대학원 입학생의 외국대학 출신 합격자 비율은 평균 6.2%였다[4].




    1. 의학전문대학원 진학 동기

    연구 대상자로 면접에 참여한 학생들은 총 16명으로 각각 1학년 5명, 2학년 1명, 3학년 4명, 그리고 4학년 6명이었다. 이들의 특성을 입학 배경별로 살펴보면 다음과 같다. 먼저 성별로는 남학생 7명, 여학생 9명이며, 학사학위를 받은 국가는 캐나다 4명, 미국 12명이다. 이전 전공은 인문사회계열이 3명, 공학계열 1명, 자연계열 12명이다. 한국의 의학전문대학원 진학 동기로는 더 좋은 취업 기회를 얻기 위해서거나 (9명), 다양한 학문적 경험을 위해 입학하였으며(3명), 기존 학업 수행 국가에서의 진학이 어려워서 택한 경우(2명), 그리고 한국에 있는 가족과 지내고 싶어서 진학한 경우(2명)가 포함되어 있다. 학생들이 의학전문대학원에 대한 진학을 본인 스스로 결정하였다는 사실은 기본적으로 의학전문대학원 진학 당시 학습에 대한 내적 동기를 가지고 있었음을 시사해준다. 재학 중인 대학원을 선택한 배경에는 대학원의 순위나 평판도(7명), 교육 과정 및 교수진(1명)에 대한 고려 이외에도 지인이나 친척의 추천(3명)과 입학조건(5명)이 영향을 미쳤다. 본 연구에서는 면접 결과를 바탕으로 의학전문대학원생들의 학교에 대한 적응을 크게 학업적 측면과 문화적 측면으로 나누어 분석하였다.







    2. 의학전문대학원 적응 문제

    1) 학업적 측면

    먼저 학업을 성취하는 데 걸림돌이 되는 구조적 원인으로는 경직된 의대 학습 문화, 제한적인 교수-학생 관계, 부족한 학업지원 서비스 등이 지적되었다. 학습량이 방대한 의학의 특성상 빽빽하게 수업 스케줄이 정해져 있고 학생들은 강의실에서 주로 수동적으로 수업에 임하기가 쉽다. 의학전문대학원생들은 이러한 환경 속에서 위축감을 느끼거나 본인의 학습 능력에 대해 한계가 있다고 쉽게 규정하기도 하였다(case 2, case 6).

    한국사회 분위기인 것 같은데, 외국은 수업도중 잘 이해가 안되는 부분은 “교수님 다시 한 번 설명해 주세요.”라고 바로 그 자리에서 말하거든요. 그런데 여기에선 분위기가 경직되어 있어서 질문하는 것이 수업에 방해되는 것 같아 질문하기가 어려워요. 학업량도 많아서 의문을 가질 틈도 없이 교수님이 말씀하시면 ‘그런가보다.’ 하고 넘어가요(case 6).


    또 다른 요인으로는 제한적인 교수-학생 간 관계가 지적되었다. 의과대학에서의 교수-학생 관계는 타 학과와는 달리 졸업 후에도 인턴, 레지던트 과정을 통해 계속 유지될 수 있기 때문에 학생들은 교수들의 영향력에 대해 좀 더 심각하게 인식하고 있고 사제 간의 관계를 좀 더 어렵게 느낀다. 외국에서 학위를 받은 경험이 있는 의학전문대학원생들도 적극적으로 지도교수님들과 상담하는 경우는 전체 면담자의 32%에 지나지 않았다. 상담을 경험한 학생들의 주된 상의 내용은 개인적 고민에 대한 내용과 향후 진로에 관한 내용이 주가 되었으며 그 외에 시험 및 성적에 대해 상의한다고 응답하였다. 그러나 상당수의 학생들이 학업 과정의 어려움에 대해 특별한 조언이나 관심을 받지 못하고 스스로 해결하는 것으로 나타났다(case 15, 16).

    학교생활이 가끔 힘들 때도 있다는 것을 교수님들이 좀 알아줬으면……. 아무도 모르잖아요. 특히 1, 2학년 때는 하루 종일 수업을 받아들이면서 ‘아, 이런 거구나.’ 하고 있는 것이 너무 힘들었어요(case 15).

    1학년 때 도서관에서 밤새면서 이게 뭐 하는 걸까 고민한 적도 있고……. 버티면 지나간다는 느낌은 있는데 순간순간 힘들었어요(case 16).


    학과 강의를 어느 정도 이해하는가라는 질문에 대해서는 13명의 학생이 80% 이상 이해한다고 대답하였으며 나머지 3명은 60%~80% 이해한다고 응답하였다. 수업의 이해를 어렵게 하는 요인들로는 전공 특성상 내용이 어렵고 수업 진행속도가 빠르기 때문이라는 답변이 가장 많았으며 그 외에 본인의 기초학습이 부족하거나 예습, 복습을 하지 않아서라는 대답이 뒤를 이었다. 학과 과제를 수행하는 데 소요되는 시간은 일주일에 평균 5시간 이상에서 10시간 미만이 소요된다는 응답이 가장 많았다. 예습, 복습에 투자하는 시간은 개인별로 차이가 컸는데 가장 많은 6명의 학생들이 일주일에 평균 5시간 이상에서 10시간 미만이라고 응답하였으며, 그 뒤를 이어 4명이 20시간 이상 투자한다고 대답하였다. 학업 만족도에 대한 질문에서 19%가 불만족한다고 대답하였으며 그 내용은 Table 1과 같다. 학업과 관련된 부분에서 특히 불만족하는 부분은 장학금 혜택과 학업지원서비스, 그리고 학교 시설 및 교육 여건 조성에 관한 부분이었다. 그 외에도 진로지도서비스와 행정지원서비스에 대한 부분이 만족도가 낮은 것으로 확인되었다.




    2) 문화적 측면

    학생들은 의학전문대학원의 문화가 상대적으로 위계질서를 더 많이 강조하며 경직되어 있다고 인식하고 있었다. 또 민감한 사안들(성적 발언이나 특정 계층 비하 발언)에 대해 특별한 문제의식 없이 쉽게 언급하는 분위기도 문제로 지적되었다.

    문화 차이가 확실히 있는 것 같아요. 제 입장에선 이 정도 의견을 제시해도 정당한 거라고 생각하는데 선배들이 봤을 때는 ‘그건 아니지.’라고 하는 부분이 있어요(case 1).

    이건 제가 조금 예민하게 받아들이는 것일 수도 있는데, 동아리에서 선배들이 친해지려고 하는 것은 알겠는데 내가 이것을 외국에서 들었다면 아니라고 했을 것 같은 성적인 발언들을 만난 지 얼마 안 되어서 쉽게 하는 모습을 봤어요(case 1).



    이러한 문화에 대해 입학 전에 어떠한 사전 정보도 받지 못하고 있어 입학 후 문화 충격이 좀 더 심하다고 볼 수 있다. 특히 입학 직후인 1학년 때 문화충격이 가장 심한 것으로 나타났지만 상급 학년으로 올라가면서 점차 적응해 가는 것으로 인식하고 있었다. 흥미롭게도 학년이 올라갈수록 현재 문화가 특별히 바뀔 것을 기대하기 보다는 본인이 적응해야 하는 것으로 받아들이는 모습을 보였다.

    이런 문화에 대해 알고만 들어왔어도……. 이 정도이다라고 솔직하게만 말해 줬어도 괜찮을텐데 예상을 못한 것들이 갑자기 다가오니까……(1학년).

    학교생활에 있어서 위계질서가 다른 집단보다 강하다 보니까 그것에 대해 적응 못하는 학생들이 있을 수 있어요. 이건 여기 생리니까 변화시키기는 어려울 것 같고, 겪는 과정에서 잘 견딜 수 있도록 도와주어야 할 것 같아요(2학년).

    학업적인 것만 해결이 되면 다른 부분들은 크게 문제가 되는 것 같지는 않아요. 솔직히 우리 학년에서 문화에 적응을 못하는 것은 거의 못 봤어요. 3학년 정도 되면 병원 등의 환경에서 어떻게 행동해야 하는지 알거든요(3학년).



    의학전문대학원 생활을 하면서 면담 학생의 43%가 차별을 느낀 적이 있다고 응답하였으며, 주된 내용으로는 성별에 대한 차별이 가장 많았다. 학생들이 의학전문대학원에서 얻은 가장 큰 성과로 꼽은 것은 다양한 학문적 경험과 다양한 사람들과의 교류였으며, 다음으로 더 좋은 취업기회 확보를 언급하였다. 의학전문대학원 과정 중 생활에 대한 전체적 불만족 비율은 학업 불만족도에 비해 상대적으로 낮은 6%로 확인되었다. 앞서 언급한 문화적 측면 이외에 특히 불만족스러운 부분으로 지적된 것은 기숙사 시설의 부족이었다(Table 1).



    졸업 후 계획에 대해 아직 특별한 계획을 세우지 못한 4명을 제외하고 7명이 한국에서 수련 과정을 지속하겠다고 대답하였으며 나머지 5명은 외국에서 취업을 하고 싶다고 응답하였다. 이러한 경향은 한국 문화에 적응하는 데 어려움을 겪었던 경우 더욱 강하였다.

    외국에서 사는 것에 대한 필요성은 한국에서 힘든 것이 어느 정도 반영된 것일 수도 있고. 여기 학교생활이 그렇게 만만한 것은 아니거든요. 한국에서 계속 학교를 다닌 학생들과의 차이가 느껴져요. 적응하는 것도 그렇고, 시스템을 받아들이는 것도(case 12).

    한국 와서 느낀 것이 나이가 좀 있는 상황에서 입학한 친구들에게는 불이익이랄까? 갈 수 있는 곳도 좀 제한되어 있고 그런 것이 있는 것 같아요. 외국에서는 전혀 그런 것을 못 느끼다가 한국에서 그런 것을 느꼈어요. 그래서 외국의사자격 시험을 보고 나갈까 해요(case 16).





    Abstract

    Purpose:

    The recent trend of switching from medical graduate school to medical school in Korea raises questions about the adjustments that students must make in medical education. We examined the perceptions of medical graduate students with regard to their adaptation in medical education.

    Methods:

    Sixteen semistructured, in-depth interviews were administered to medical graduate students who received their first degrees in foreign countries. The interviews addressed their perceptions of their experience in medical graduate school and on how well they adjusted to medical education.

    Results:

    Students perceived their adaptation to medical graduate school in two dimensions: academic achievement and cultural adjustment. In academic achievement, a limited student-teacher relationship was recognized by students. Students tended to be passive in the classroom due to an uncomfortable atmosphere. They also reported witnessing culture shock in relation to the paucity of information on entrance into medical graduate school. Freshmen voiced many difficulties in adjusting to the unique culture in medical graduate school, in contrast to upper classmen. However, only 32% of students experienced helpful mentoring for their problems.

    Conclusion:

    Students’ perspectives should guide all decisions made about medical education in an altered educational system.Self-regulated learning and a good mentoring program can help prepare students for medical education and professional life.


    의학교육에서 소셜미디어 활용 (Academic Medicine, 2013)

    Social Media Use in Medical Education: A Systematic Review

    Christine C. Cheston, MD, Tabor E. Flickinger, MD, MPH, and Margaret S. Chisolm, MD




    PURPOSE:

    두 가지 연구질문 (1)소셜미디어를 활용한 intervention이 만족, 지식, 태도, 기술에 어떠한 향상을 가져왔는가? (2)남은 과제와 기회는 무엇인가?

    The authors conducted a systematic review of the published literature on social media use in medical education to answer two questions: (1) How have interventions using social media tools affected outcomes of satisfaction, knowledge, attitudes, and skills for physicians and physicians-in-training? and (2) What challenges and opportunities specific to social media have educators encountered in implementing these interventions?

    METHOD:

    The authors searched the MEDLINE, CINAHL, ERIC, Embase, PsycINFO, ProQuest, Cochrane Library, Web of Science, and Scopus databases (from the start of each through September 12, 2011) using keywords related to social media and medical education. Two authors independently reviewed the search results to select peer-reviewed, English-language articles discussing social media use in educational interventions at any level of physician training. They assessed study quality using the Medical Education Research Study Quality Instrument.

    RESULTS:

    14개의 연구가 포함되었다. 지식(시험 점수), 태도(공감), 기술(성찰적 글쓰기) 등의 향상이 있었다. 가장 많이 보고되는 기회는 학습자의 참여가 늘어난 것, 피드백이 증가한 것, 협력이 늘어난 것, 전문직업적 발전이 향상되는 것 등이었다. 가장 흔하게 보고된 과제는 기술적인 것, 학습자 참여도, 개인정보/보안의 문제 등이었다.

    Fourteen studies met inclusion criteria. Interventions using social media tools were associated with improved knowledge (e.g., exam scores), attitudes (e.g., empathy), and skills (e.g., reflective writing). The most commonly reported opportunities related to incorporating social mediatools were promoting learner engagement (71% of studies), feedback (57%), and collaboration and professional development (both 36%). The most commonly cited challenges were technical issues (43%), variable learner participation (43%), and privacy/security concerns (29%). Studies were generally of low to moderate quality; there was only one randomized controlled trial.

    CONCLUSIONS:

    Social media use in medical education is an emerging field of scholarship that merits further investigation. Educators face challenges in adapting new technologies, but they also have opportunities for innovation.





    웹기반 도구는 여러 장점이 있음.

    Web-based tools offer several advantages over in-person/print educational tools: they can overcome physical or temporal barriers, provide searchable content, and encourage interactivity.8


    Web 2.0을 다음과 같이 정의내린 바 있음.

    McGee and Begg15 define Web 2.0 as a “collection of Web-based technologies that share a user-focused approach to design and functionality, where users actively participate in content creation and editing through open collaboration between members of communities of practice.”


    그러나 소셜미디어의 활용은 환자의 개인정보나 온라인 프로페셔널리즘에 대한 우려를 낳고 있음

    However, use of social media by physicians and medical trainees has given rise to concerns about patient privacy and online professionalism.18–21



    Social media tools used

    Blogs were the most commonly employed social media tool (n = 10; 71%), followed by wikis (n = 3; 21%), Twitter (n = 2; 14%), and Facebook (n = 2; 14%).









     2013 Jun;88(6):893-901. doi: 10.1097/ACM.0b013e31828ffc23.

    Social media use in medical education: a systematic review.

    Author information

    • 1Harvard Medical School, Boston, Massachusetts, USA.

    Abstract

    PURPOSE:

    The authors conducted a systematic review of the published literature on social media use in medical education to answer two questions: (1) How have interventions using social media tools affected outcomes of satisfaction, knowledge, attitudes, and skills for physicians and physicians-in-training? and (2) What challenges and opportunities specific to social media have educators encountered in implementing these interventions?

    METHOD:

    The authors searched the MEDLINE, CINAHL, ERIC, Embase, PsycINFO, ProQuest, Cochrane Library, Web of Science, and Scopus databases (from the start of each through September 12, 2011) using keywords related to social media and medical education. Two authors independently reviewed the search results to select peer-reviewed, English-language articles discussing social media use in educational interventions at any level of physician training. They assessed study quality using the Medical Education Research Study Quality Instrument.

    RESULTS:

    Fourteen studies met inclusion criteria. Interventions using social media tools were associated with improved knowledge (e.g., exam scores), attitudes (e.g., empathy), and skills (e.g., reflective writing). The most commonly reported opportunities related to incorporating social mediatools were promoting learner engagement (71% of studies), feedback (57%), and collaboration and professional development (both 36%). The most commonly cited challenges were technical issues (43%), variable learner participation (43%), and privacy/security concerns (29%). Studies were generally of low to moderate quality; there was only one randomized controlled trial.

    CONCLUSIONS:

    Social media use in medical education is an emerging field of scholarship that merits further investigation. Educators face challenges in adapting new technologies, but they also have opportunities for innovation.

    PMID:
     
    23619071
     
    [PubMed - indexed for MEDLINE]


    의과대학생이 만날 미래, 어떻게 준비할 것인가?

    How Should Medical Education Be Planned for Medical Students’ Future?

    정지훈

    경희사이버대학교 모바일융합학과

    Jihoon Jeong

    Department of Mobile Convergence Technology, Kyung Hee Cyber University, Seoul, Korea






    서 론

    우리는 과거와 달리 너무나 빠르게 변화하는 환경에 살고 있다.의학분야도 예외는 아니다. 현재의 기술의 발전속도가 가져오는 새로운 미래의학 패러다임의 변화, 그리고 인터넷의 발달로 의료서비스에 대한 사회적 인식과 제도의 변화까지 점쳐지고 있는 가운데앞으로 10–20년 뒤의 의료환경은 현재와는 크게 달라져 있을 가능성이 높다. 이는 현재의 의과대학생들이 전공의 수련을 마치고 실제활동하는 중간에 현실화된다는 것을 의미하므로 현재의 의학교육의측면에서 당면한 문제라고 할 수 있다.


    이처럼 미래를 대비하기 위한 의학교육의 혁신 노력은 국내외에서 꾸준히 있어 왔다. 연세대학교에서는 미래 의료환경에 적합한의사를 양성하기 위한 교육방법의 혁신을 위해 2004년 광혜교육과정을 도입하면서 학문단위별 교과목에서 과목의 통합성을 증대시키고, 선택과목과 문제중심학습의 확대, 인문사회의학 과목 개설 등의혁신을 단행한 바 있고(Ryue et al., 2009), 이런 변화는 한국의많은 의과대학의 의학교육 변화를 가져왔다. 또한 양은배와 맹광호는 “미래 의학교육을 위한 5가지 제언”이라는 논문(Yang &Meng,2014)을 통해 의사들이 갖추어야 하는 역량에 기초한 교육을 실시하고, 의과대학교육, 졸업 후 수련교육의 연계성이 확보되어야 하며,의학교육의 질 개선 활동강화, 의사양성시스템의 재정비, ‘대학의학’의 개념정립이라는 5가지 원칙을 제시하였다. 가장 최근에는 서울대학교에서 임상의사 이외의 다양한 분야로 진출하는 데 도움을 줄수 있는 학생경력개발센터를 개소하고, 바뀌는 미래의 의료 패러다임 변화에 맞는 다양한 미래지향적인 인재개발에 나선다는 보도가나오기도 하였다(Lee, 2014). 해외에서도 미래의 의료환경과 관련한다양한 의학교육의 변화를 위한 노력이 경주되고 있는데, 특히 영국의학협회는 1993년 “미래의사(Tomorrow’s doctors)”라는 지침서를 발간하고, 2003년과 2009년에 개정판(General Medical Council,2009)을 만들면서 미래에 대한 대비를 해나가고 있다.


    이와 같이 국내외에서 다양하게 미래의 의료환경의 변화와 이에대비하기 위한 의학교육에 대한 논의가 있지만, 현재 진행되는 미래의료 패러다임 변화의 속도에 비추어 볼 때 보다 근본적인 고민과대비가 필요한 시점이다. 미래에 대한 대비를 할 때에는 추상적으로미래를 이야기하기보다는 구체적인 목표시기를 특정하는 것이 미래예측에 도움이 된다. 이에 본 논문에서는 현재의 의과대학 1학년들이교육과 수련을 마치고 실질적인 의료현장에 투입되기 시작하는2025년 정도를 시작으로 10년 정도의 시기인 2025–2035년 사이에예상되는 미래의료 패러다임 변화의 중요한 양상들을 보다 심도있게 들여다보고, 현재의 의학교육 변화의 방향성, 그리고 의과대학생들이 개인적으로 대비를 해야 하는 내용들을 제시하고자 한다.



    소비자 중심의 의료 패러다임과 Health 2.0


    미래 의료환경의 변화 중에서 가장 근 미래에 등장할 중요한 변화는 information technology (IT) 기술 발전으로 인해 수백 년을 지탱해온 의료의 기본 속성이라고 할 수 있는 ‘지식의 비대칭성’이 깨질수 있다는 점이다. 보건의료산업에 다른 어떤 산업보다 규제가 많은것은 ‘지식의 비대칭성’이 가져오는 시장실패 때문이고, 사회보험이나 민간의료보험, 국영 의료체계 등이 나오게 된 것도 보건의료산업의 이런 특징이 가장 근본적인 역할을 했음은 주지의 사실이다.이런 기본 가정이 최근 이른 바 ‘구글 환자(google patient)’로 대별되는 환자 집단의 등장과 소셜 미디어와 네트워크의 확장으로 깨지는사례가 많아지기 시작했다(Meisel, 2011).


    이런 변화의 소용돌이 속에 급부상한 개념이 Health 2.0이다. Health 2.0의 정의는 월스트릿저널(Wall Street Journal, 2006)에서언급한 소셜 네트워크 중심의 건강의료, Scott Shreeve가 백서로정리한 보다 자세한 정의(Shreeve, 2011) 등이 있지만, 간단히 이야기하면 의료의 소비자로서 수동적인 역할만 수행을 했던 환자들이능동적으로 바뀌며, 공유와 참여, 집단지성으로 대표되는 웹 2.0기술이 이러한 판도의 변화를 촉진하는 것을 의미한다. Health 2.0패러다임은 전통적인 공급자 중심의 의료서비스체계에 도전장을던지는 의미가 있기 때문에, 이를 제도화하는데 강한 저항이 있을수밖에 없다. 만약 이런 사회적 혼란을 적절하게 중재하지 못하면국가적인 건강 관련 비용의 증가와 커다란 사회적인 갈등을 초래할수 있는 불씨가 될 수 있다.


    앞으로 스마트폰과 스마트 시계 등의 첨단 IT 기기의 보급과이를 지원하는 앱세서리, 웨어러블 디바이스의 공급이 확대되면 환자들의 적극적인 건강행위는 더욱 활발해질 가능성이 높으며, Health 2.0 패러다임은 더욱 강화되면서 전통적인 의사와 환자 관계에도 큰 영향을 미치게 될 것이다. 현재 전 세계에서 다양한 형태의IT 융합형 의료서비스가 개발되고 실험 중에 있으며, 최근에는 애플과 구글, 삼성전자 등의 커다란 IT기업들도 본격적으로 뛰어들고있어서 그 변화의 폭도 더욱 커질 것으로 예상된다. 또한 이와 같은Health 2.0 패러다임에 맞는 의사와 환자 상호작용과 관련한 아카데믹한 연구들도 많이 진행되고 있는데, MIT의 뉴미디어 의학그룹에서 진행하는 CollaboRhythm의 경우 의사와 환자 상호작용의 새로운 패러다임을 위한 기술적인 프레임워크 프로젝트로 주머니 속의 주치의를 목표로 하고 있다. 이외에도 무선의 다양한 접속망과 협업의사 결정, 그리고 환자에게 효과적인 교육을 위한 첨단 인터페이스와 시각화, 치료에 대한 순응도를 높이기 위한 다양한 방법들에대한 많은 연구가 진행되고 있어 이런 연구결과들이 임상에 적용되기 시작하면 현재와는 다른 의사-환자 관계 모델들이 많이 등장하게될 것이다.



    주류로 자리잡는 디지털 헬스


    Health 2.0이 주로 의사와 환자 관계와 소셜 네트워크와 인터넷의보급에 따른 패러다임 변화를 이야기했다면, 디지털 헬스는 디지털기술을 바탕으로 하는 의료의 변화를 바라보는 다소 기술적인 시각이다. 거의 대부분의 사람들이 모바일 기술을 바탕으로 연결이 되고,저렴하면서도 디자인이나 기능성을 모두 갖춘 제품적인 완성도가높은 각종 스마트 헬스 제품들의 보급이 확대되면서 디지털 헬스산업이 의료서비스의 큰 변화를 끌어낼 것으로 예상된다.


    인터넷과 스마트 디바이스와 연결되는 수많은 가정용 의료기기들이 보급되면서 건강한 삶을 영위하기 위한 활동들과 간단한 질병의 진단 및 치료, 관리 등은 집에서도 이루어질 것으로 전망되고있는데, 이런 변화를 확대하기 위해 미국 Food and Drug Administration에서는 이런 기술들에 대한 지나친 규제보다는 스마트 규제(smart regulation)라는 이름으로 보다 쉽게 관련한 의료기기나 스마트 건강 앱 등을 시장에 내놓을 수 있는 환경을 조성하고있다(Hamburg, 2014).


    이런 변화와 함께 2014년 세계 최대의 가전박람회인 Consumer Electronics Show에서는 헬스스팟(HealthSpot)이라는 일종의 자동판매기 내지는 동네 편의점형 의원이 등장해 큰 눈길을 끌었다.헬스스팟은 2013년부터 미국의 미네소타 주에서 진료를 시작하였는데, 한 명의 간호사가 관리하는 무인 진료실에 들어서면 전 세계의다양한 의사들과 원격진료로 만날 수 있으며, 필요하면 원격진료기기를 이용해서 각종 검사를 직접 받을 수 있는 시스템을 패키지화한 것이다. 헬스스팟은 2014년 5월, 800만 달러의 투자를 유치하며미네소타 주를 중심으로 도입이 확대되고 있다(Comstock, 2014).이는 앞으로 유사한 형태의 원격진료시스템이 다양한 방식으로 보급될 수 있다는 것을 의미한다.


    이러한 거시적인 변화의 양상을 반영하듯 유명한 경제지인 포브스는 수많은 이슈들을 제치고 2013년을 ‘디지털 헬스(digital health)’의 해가 될 것이라고 발표한 바 있다(Nosta, 2013). 이런 변화는 벤처투자 양상에서도 나타난다. 디지털 헬스와 관련한 투자상황을 정기적으로 발표하는 RockHealth 리포트에 따르면, 2014년상반기 디지털 헬스와 관련한 투자 액수는 2013년 상반기에 비해176%나 급증하였다(Rock Health, 2014). 이는 의료기기 분야의13%, 바이오테크 분야의 28% 성장은 물론이고 디지털 헬스 관련하여 전년도에 기록한 39% 성장률도 월등히 뛰어넘는 수치다. 이런변화는 세계적인 기업들의 움직임에서도 느낄 수 있다. 의료기기분야에서 세계 최대의 기업인 GE는 스타트업 헬스(Startup Health)라는 조직과 손을 잡고 소비자 건강 관련 제품들에 대한 3년짜리엑셀러레이터 프로그램을 시작하면서 본격적인 스타트업 양성에들어갔으며, 애플은 영국 NHS, 캠브릿지 대학병원, Mayo Clinic,스탠포드 대학병원, UCLA 메디컬 센터 및 세계 최대의 EHR(electronic health record) 업체인 Epic 등과 손을 잡고 본격적인의료서비스와 애플에서 자체 개발한 HealthKit의 연계작업에 들어갔다. 또한 구글은 Google Fit이라는 운동과 관련한 기술을 내놓고디지털 헬스 시장진입을 타진하고 있으며, 동시에 노화와 관련한건강문제를 해결하기 위한 중장기적인 연구를 위해 Calico라는 회사를 설립하였다.



    저렴한 유전자검사 비용과 개인화된 의료의 시대


    미래의 의학과 의료서비스와 관련한 전망을 할 때 또 한 가지빼놓을 수 없는 것이 유전자검사를 기반으로 하는 개인화된 의료(personalized medicine)의 시대가 실체화될 가능성이 높아지고 있다는 점이다. 개인의 유전자 염기서열 전체를 해독하는 것에 과거에는 엄청난 비용과 시간이 소요되었지만, 비용과 시간이 줄어드는속도가 18개월에 용량은 2배, 가격은 반으로 떨어진다는 메모리반도체에서의 무어의 법칙보다 훨씬 가파르게 속도와 가격이 떨어지면서 이제는 각 개인의 유전자를 모두 해독하는데 100만 원 정도를달성하기 직전에 와있다(Hayden, 2014).


    또한 미국의 DNA 기술 스타트업인 냅시스(Nabsys)는 고체반도체 유전자 염기서열 분석기를 2013년 상용화한다고 발표하였는데,이는 진공관에서 트랜지스터로 진화하면서 기계공학의 시대에서전자공학의 시대로 바뀐 것에 비견되는 변화라고 할 수 있다. 앞으로유전자검사 기기는 더욱 작고 저렴해지게 될 것이다. 영국 옥스퍼드나노포어 테크놀로지스(Oxford Nanopore Technologies)에서도유사한 DNA 분석기술의 상용화를 발표하였다. 결국 거대 장비와서버를 통해서 DNA를 분석하는 것이 아닌 나노포어 기술 등을활용해서 소형장비로 DNA를 분석하는 쪽이 대세가 될 가능성이높다(Schneider &Dekker, 2012).


    개인화된 의료가 성공하기 위해서는 DNA를 해독하는 비용이급격히 떨어지고, DNA의 정보가 실제로 치료에 영향을 줄 수 있을정도의 가치가 있어야 하며, 이런 정보를 이용한 적절한 치료법이나약제를 제약회사에서 만들어내야 한다. 아직까지 개인화된 의료가활성화되지 않은 것은 비용과 효과적인 측면에서의 검증이 되지않았기 때문인데, 최소한 비용 부분은 최근의 급격한 기술의 발전으로 수년 내에 급격하게 떨어질 것으로 예상되며, 나머지 조건들도머지 않은 시기에 충족이 가능하다는 것이 일반적인 평가이므로개인화된 의료의 시대가 그리 멀다고는 할 수 없다. 그렇지만 유전자기반의 개인화된 의료를 맹목적으로 반기는 것도 문제라는 시각도있다. 2012년 4월 존스홉킨스 대학의 연구자들은 유전자들과 특정질병과의 연계성을 해석하는 것이 특별히 기존의 전통적인 방법을이용해서 접근하는 것에 비해 별로 나을 점이 없다는 의견을 제시하였다(Roberts et al., 2012).


    여러 가지 부작용이나 비용 등에 대한 우려에도 불구하고 현재와같은 속도로 경제성이 개선되고, 쉽게 유전자검사를 하면서 개인화된 의료가 가능해진다면 이에 따른 새로운 의료시스템에 대한 근본적인 변화가 나타날 가능성이 높다. 현재의 대량생산과 표준화된지침을 중심으로 하는 보험체계나 진료방식에도 상당한 변화가 불가피하게 될 것이며 최선의 진단과 치료를 원하는 인간의 존엄과 관련한 근본적이고도 보편적인 권리와 비용효과성에 입각한 경제성 원칙이 충돌할 가능성도 있다.


    보건의료시스템의 비관적 미래 시나리오와 낙관적 미래 시나리오


    Health 2.0 패러다임의 변화, 디지털 헬스 기술의 발전, 개인화된의료의 시대의 도래와 같은 커다란 미래 의료의 변화와 함께 간과할수 없는 것은 보건의료시스템의 전반적인 변화이다. 우리나라보다심각한 보건의료시스템의 전반적인 문제를 노출하고 있는 미국의경우에는 공공과 민간에서 다양한 미래 시나리오를 그려 보면서미래의 닥칠 커다란 시스템상의 문제에 대한 대비를 하고 있다.가장 최근에는 Institute for Alternative Futures에서 2014년“Health 2030: a scenario exploration”이라는 제목의 미래 시나리오 보고서를 내놓기도 하였다(Institute for Alternative Futures,2014).


    우리나라의 상황에도 보건의료시스템에 대하여 이와 같은 시나리오 기법으로 비관적 미래 시나리오와 낙관적 미래 시나리오를제시해 보고자 한다. 제대로 된 시나리오 기법을 적용하기 위해서는수개월의 시간과 여러 인력들이 투입되어 작업을 해야 하지만, 본논문의 목적은 미래를 대비하기 위한 의학교육 및 의대생들에게인사이트를 주기 위한 것이므로 저자의 직관을 중심으로 한 2가지 시나리오를 제시한다.


    우리나라는 2000년에 고령인구비율 7%의 고령화 사회에 진입하였으며, 2018년에는 고령인구비율 14%의 고령사회, 2026년에는고령인구비율 20%의 초고령사회에 도달할 것으로 예측되고 있다.고령화와 높아진 기대수명으로 인해 의료비용과 관련한 의료보험재정과 이를 부담하는 인구 사이의 갈등과 반목이 높아질 것으로예상되며, 심각할 경우 시스템이 전혀 동작하지 않는 파산의 가능성도 완전히 배제할 수는 없다.


    먼저 비관적 미래 시나리오는 다음과 같다. 생산가능 인구의 감소와 고령화로 의료보험 재정은 고갈되고, 단일보험체계에 의존하는우리나라의 보건의료시스템은 붕괴 직전상태에 이른다. 공공보건의료와 관련한 정책당국과 건강보험관리공단, 건강보험심사평가원 등에서는 고갈된 재정으로 인해 의료공급자들에게 적절한 수가를 지급하지 못하게 되며, 이는 정책당국에 대한 불신을 불러와 의료서비스공급자는 의료보험 의무지정에서 탈피하여 경제력이 있는 부유층을대상으로 하는 서비스를 하려는 방향으로 방향전환을 하고, 국민들은 부실한 의료서비스에 대한 반발로 의료보험료를 내려 하지 않는악순환의 고리에 들어가게 된다. 정치인들은 새로운 건강보건의료와 관련한 자금을 마련하겠다는 재정계획을 발표하지만, 현실적이기보다는 정치적인 구호에 그치게 되며, 예산은 삭감되고 실질적인진전은 이루어내지 못하게 된다. 공공의료시스템은 약화되고, 국민들은 부유층을 중심으로 시장 중심의 의료시스템을 적극 도입하여의료시스템을 재건하려고 노력한다.


    의료서비스 공급자들을 이런 새로운 시장 중심의 의료시스템에몰려들게 되는데, 시장 중심의 민간의료시스템은 의료서비스 공급의 양극화를 가져와서 부유층과 빈곤층의 의료서비스 격차를 크게벌리게 된다. 이런 의료의 양극화는 정치적 불안정성과 경제에도타격을 입혀 추가적인 경제불황을 일으키며, 세수부족은 또 다시의료보험 재정을 약화시키는 악순환이 지속된다.


    일반화된 소셜 미디어와 온라인 네트워크는 이런 악순환을 끊는역할을 하는 것이 아니라 불신과 집단 이기주의를 가속화시키는역할을 하게 되며, 정부정책에 대한 불신과 각 이해집단의 충돌을일으키는 도화선 역할을 한다. 정부정책에 대한 불신은 가장 필수적인 예방백신 등에 대한 접종거부로 이어지며, 갑자기 유행하는 독감등과 같은 유행성 전염병에 대한 백신접종이 제대로 이루어지지않으면서 전국 규모의 대규모 전염병이 유행하고, 많은 사람들이감염되어 쓰러지고, 이 중 많은 수는 사망에 이르게 된다. 이런 위기상황에 공공의료시스템을 지키기 위해 노력하는 의료서비스 제공자들은 부족한 재정과 정부 및 국민과의 소통의 원활하지 않은 문제,그리고 과로로 효과적인 대처를 하지 못한다. 이와 같은 커다란전염병의 재앙은 사태를 해결하게 만들기는커녕 공공의료시스템을지켰던 사람들과 국민들, 그리고 정치인들 사이의 반목과 책임 떠넘기기를 가속화하고 전면적인 민간의료시스템으로의 이양을 선언하게 만들고, 공공의료시스템은 붕괴한다.


    공공의료시스템을 믿지 못한 일부 시민들은 ‘시민과학(citizenscience)’ 서비스를 통해 자신들이 직접 건강을 지키기 위한 의료시스템 구축에 나서기 시작한다. 이들은 초기에 질병의 감시와 모니터링을 중심으로 하는 크라우드 소싱작업을 시작으로 점차 지역사회를기반으로 하는 협동조합의 체계를 갖추어 나가기 시작한다. 이들이만든 네트워크와 이들이 수집한 데이터들은 새로운 시민협동 의료시스템의 탄생을 유도하며, 과거에는 없었던 새로운 의료전달체계와시스템이 등장한다. 그러나 이런 움직임은 중산층 이상의 기술에익숙하고, 초연결사회를 잘 받아들이는 계층의 전유물로 빈곤층은이런 네트워크에 끼어들지 못하면서 의료의 사각지대에 놓이게 된다.


    그렇다면 낙관적 미래 시나리오는 어떻게 될까? 다양한 환경 관련기술의 발달로 깨끗한 물과 공기가 공급되어 환경과 관련한 다양한건강의 위해요소들이 제어된다. 먹는 물의 수질이나 공기의 질 등은인터넷 등으로 실시간으로 알 수 있다. 차량에는 체온, 심전도 등과같은 생체신호를 이용하여 건강을 관리할 수 있고, 상용화된 무인자동차는 교통사고의 발생률을 크게 낮춘다.


    백신 기술의 발달로 다양한 전염병을 예방할 뿐 아니라 알쯔하이머병이나 암, 각종 만성질환에도 치료적으로 이용되어 저렴하게 건강한 생활을 유지할 수 있게 된다. 개인 유전자분석의 일반화는맞춤형 치료제 사용을 유도하여 의료비를 절감시키고, 약의 부작용이 최소화되며, 건강수명이 연장되고, 예방적인 건강행위를 활성화시켜 전반적으로 의료비용이 감소하게 된다.


    늘어나는 고령인구는 삶의 질이 좋아지면서 실질적인 노동을할 수 있는 연령이 늘어나게 되며, 현재 많은 비용이 들어가는 질병의치료에도 새로운 신약과 면역치료제, 세포치료제 및 자동화 치료기기의 발전으로 건강한 고령화 시대를 열게 되어 고령화로 인한 일각의 의료보험 재정의 위기를 극복하게 만든다. 노화나 비만 등을자연현상이 아닌 질병으로 인식하며, 노화와 비만 등을 치료하는의학 기술이 발전하여 건강한 삶을 유지하는 수명기간이 크게 늘어나게 되어 고령화에 대한 인식과 용어 자체가 크게 변하게 된다.


    유전자재조합, 세포 기술 등의 바이오 기술 및 IT 기술의 발전은보다 싸고 안전한 치료 및 예방목적의 의약품 생산을 가능하게 하며,제품의 효능과 안전성 예측을 용이하게 만들어 적은 자본으로 수익성 있는 많은 건강의료 또는 제약회사들이 등장할 수 있게 된다.바이오 장기, 세포치료제 기술과 인체와 연결하여 동작할 수 있는저렴한 사이보그 의체 기술의 발전은 과거에는 희망이 없었던 다양한 질병을 앓고 있는 환자들의 희망이 된다.


    공공보건과 의료기관 및 가정에서 모든 국민들이 건강 데이터수집과 관리 서비스를 무상으로 받을 수 있으며, 다양한 비영리기관이나 지역사회 커뮤니티에서 더욱 건강한 삶을 위한 서비스를 발표하고 많은 사람들이 이를 이용한다. 지역사회에서 누구나 참여할수 있는 바이오해커 연구실이 활성화되어 어렸을 때부터 다양한바이오와 IT 기술을 융합한 다양한 과학 기술 개발을 할 수 있게되며, 이런 변화는 전문가들에게 의존하는 것이 아니라 국민들 스스로가 건강관리와 건강행위를 할 수 있는 Health 2.0 운동이 또 다른차원으로 발전하게 만든다.


    의사들은 더 이상 의료서비스 공급을 독점하는 역할을 하기보다는 국민들의 정신적, 사회적인 상담사 역할을 포함한 전반적인 생활을 같이 봐주는 전인적인 카운슬러의 역할을 하기 시작하며, 국민들이 수집한 다양한 데이터를 해석하고 설명해주는 등 새로운 역할을부여받게 된다. 물론 커다란 종합병원이나 전문화된 병원에 취직한소수의 의사들은 기존의 의사들과 비슷한 역할을 수행하게 된다.


    여기에 제시한 비관적 시나리오와 낙관적 시나리오는 양극단의시나리오를 예측한 것이므로 어느 한 방향으로 흐르기보다는 기술의발전과 사회의 합의 정도에 따라서 이들 시나리오의 중간 정도 수준에서 타협될 가능성이 가장 많다. 의료는 보수적인 특징을 가지기때문에 상당한 위협이 현실화하기 전에는 선제적인 변화가 나타나기는 어렵다. 그러므로 초기에는 비관적 시나리오에 가깝게 흐를 가능성이 많지만 실제로 위기가 닥친 이후에는 정책당국과 사회, 그리고참여자들의 입장도 바뀌기 때문에 중장기적으로는 낙관적 시나리오의 방향으로 상당 부분 변화할 것으로 예측하는 것이 가장 합리적이다. 어느 방향의 변화가 나타나더라도 현재와 같은 의학교육시스템에서 배출된 의사들이 대처하기는 쉽지 않을 것이다.


    미래에 대비하는 의학교육과 의과대학생들에 대한 조언


    지금까지 2025–2035년 미래의료의 이슈들로 소비자 중심의 의료패러다임과 Health 2.0, 주류로 자리잡는 디지털 헬스, 저렴한 유전자 검사비용과 개인화된 의료의 시대, 보건의료시스템의 커다란 변화를 제시하였다. 이런 이슈들을 바탕으로 자연스럽게 현재의 의학교육과 의과대학생들에게 미래를 대비하기 위한 시사점을 도출하고자 한다.


    먼저 소비자 중심의 의료 패러다임과 Health 2.0 개념의 부상은의사와 환자 관계를 새롭게 정립할 것이다. 이는 결국 소비자가의사들 이상으로 의료정보를 많이 얻게 될 것이라는 것을 의미하며,일부 잘못된 의료정보로 인한 부작용도 있겠지만 대세가 될 것이다.이런 상황에서 부작용을 우려하기보다는 보다 적극적인 의사-환자소통을 통해 신뢰관계를 회복하고, 경우에 따라서는 커뮤니티활동등을 적극적으로 해나가는 의사상이 요구된다. 이를 위해서는 의과학 위주의 현재의 의학교육체계에 환자 및 지역사회에 대한 이해를바탕으로 보다 인간적으로 접근하는 소양을 길러주는 교육과정을대폭 강화할 필요가 있다. 의과대학생들도 의과학 공부를 열심히하는 것 이상으로 소통의 방식과 인간의 심리, 사회의학, 그리고소셜 미디어 등의 다양한 소통의 도구 등에 대해서 관심을 가지고 적극적으로 사용하는 연습을 할 필요가 있다.


    디지털 헬스의 부상에 따른 의학교육과 의과대학생들에게 필요한 대비방법은 상당 부분 Health 2.0 패러다임에 대한 대처와 공통분모가 많다. 추가적으로 중요한 것은 디지털 및 IT 기술 전반에 대한관심이다. 앞으로 디지털 기술의 발전이 의료서비스의 혁신을 일으킬 가능성이 많은데, 현재 상황은 의사들보다는 IT 기술에 정통한엔지니어와 기업들이 변화를 주도하고 있다. 이제는 의사들도 IT기술에 대해서 더욱 많은 공부를 하고 필요로 하는 서비스나 의료기기 등을 제작할 필요가 있다. 이를 위해서는 의학교육에 의료정보학과 의공학 등과 같은 공학교육을 보강할 필요가 있으며, 새롭게등장할 기술기반 의료혁신기업이나 스타트업에도 과감하게 뛰어드는 의과대학교 졸업자들이 많이 나와야 한다.


    개인화된 의료의 시대와 보건의료시스템 전반의 커다란 변화에대비하기 위해서는 의학교육에 있어서 이런 변화가 가져올 사회적인부담이나 보건의료시스템 전반의 이해에 대한 교육이 강화되어야한다. 과거에는 자신의 전문분야만 공부하고 병원이나 의원에 찾아오는 환자에 대해서 임상진료를 하면 되었다. 그렇지만 이제는 과거의 안정된 보건의료시스템이 계속 변화할 가능성이 제기되고 있기때문에 전체적인 시스템 변화의 가능성을 최소한 인지하고 대비할수 있는 수준의 지식은 가지고 있어야 할 것이다.



    결 론


    의료의 발전에는 다양한 기술의 발전이 불가분의 관계로 많이엮여 있다. 이 논문을 통해 언급한 디지털 헬스 기술이나 개인화된의료와 관련한 기술 이외에도 첨단수술방이나 정확한 진단과 관련한부분까지도 기술의존적인 경향은 점점 심해질 것이다. 이는 결국의사 개인의 능력보다는 첨단 기술에 대한 의존도와 병원을 운영하는 주체의 투자 등에 의해 의료서비스의 질이 좌우될 수 있음을의미하며, 커다란 병원과 자본력이 뒷받침된 의료서비스 공급자들의 영향력이 점점 커질 것이라고 예상할 수 있다.


    그렇다면 동네에 있는 동네 의사들과 의원들이 필요 없어질까?그렇지는 않을 것이다. 건강관리 및 질병관리 서비스를 받아야 하는대상은 많아질 것이고, 이러한 요구를 충실하게 만족시킬 수 있는능력을 가진 동네 의원들과 건강 제공자들이 중요한 역할을 하게될 것이다.


    앞으로 도래할 미래의 의료환경은 그 어느 때보다 환자와 의사의신뢰가 중요해 질 것이다. 이러한 신뢰를 확보하기 위해서는 환자들도 기본적으로 의사를 불신하거나 비도덕적으로 보는 선입견을 깨야할 것이고 의사들도 환자들에 대해서 더더욱 인간적으로 접근할수 있어야 한다. 이미 소비자 중심의 의학, 그리고 Health 2.0및디지털 헬스의 시대는 시작된 것이나 마찬가지다. 의사들도 환자,그리고 일반인들의 건강생활에 대한 주도적인 역할을 인정하고 이들과 의료진들과의 적극적인 소통과 협업을 통해 보다 건강한 사회를만들어가는 노력을 기울여야 할 때다. 또한 IT 기술을 포함하여의공학 기술의 발전이 가져올 미래의학에 대해서 더욱 관심을 가지고, 공부도 하고, 이 분야에 뛰어드는 의과대학 졸업생들이 많이나와야 한다.


    그런 측면에서 과거 그 어느 때보다 강력한 의학교육의 혁신과 의과대학의 대비가 필요한 시점이다. 그리고 이런 변화는 의학교육을 조금 바꾼다고 대처할 수 있는 수준을 넘어선다. 가능한 심각하게미래의학과 미래의 의료시스템의 변화에 대해서 알리고, 이에 대한 대처를 의과대학생들이 스스로 할 수 있도록 하지 못한다면 미래의료시스템의 변화속도와 의학교육의 변화속도의 격차로 인해 그 중간시기에 제대로 대비하지 못한 의과대학 졸업생들은 커다란 어려움을 겪을 가능성이 높다.


    현재의 의학교육은 임상의학과 기초의학으로 나뉘어져 있기는 하지만, 대체로 의과학이라는 테두리에서 벗어나지 않고 있다. 과거에 학문단위별 교과목에서 과목의 통합성을 증대시키고, 선택과목과 문제중심학습을 확대하는 등의 개혁이 있어왔지만, 이 역시도결국 임상의사와 기초의과학자를 양성하는 수준을 벗어나지는 못하고 있다. 이런 의학교육으로는 2025년 이후에 예상되는 미래의료환경에 학생들을 제대로 적응시키기 어렵다. 의학교육에 현재보다훨씬 다학제적인 특성을 가미해야 하며, 특히 공학과 사회학, 디자인과 기업가 정신과 같이 변화에 쉽게 적응하고, 다양성을 확보할수 있는 교과목들을 대폭 확충하고, 임상의사와 기초의과학자라는틀에 박힌 틀을 벗어날 수 있는 가능성을 학생들에게 많이 보여주는그런 의학교육이 필요하다.






    This study focused on investigating the future medicine and health care industry paradigm shift and suggesting

    the right direction for the medical education system in order for students to be better prepared in the

    near future. Here, I will discuss four issues related to the future of medicine: Health 2.0, digital health,

    personalized medicine, and innovations of the public healthcare system. Every issue has lessons for medical

    education and teaching for students who major in health professions. However, it is obvious that the present

    is an important period of time as, currently, we are at an early stage in the future health care environment.

    Recently, there have been rapid transformations in various fields of medicine. Therefore, if we fail to lead

    medical education in the right direction, medical students will suffer from major problems in coping with

    these changes.


    Keywords: Forecasting, Trend, Medical informatics, Individualized medicine, Suggestion

    의과대학생은 어떠한 수업 피드백을 받고 싶어 하는가?

    What kind of feedback do medical students want?

    김종엽1,2, 나백주1, 윤정민1, 강재구1, 한승연1, 황원민1, 허예라

    Jong-Yeup Kim1,2, Baeg Ju Na1, Jungmin Yun1, Jaegu Kang1, Seungyeon Han1, Wonmin Hwang1 and Yera Hur1

    1Faculty Development Committee, Konyang University College of Medicine, and 2Konyang University Myunggok Medical Research Institute, Daejeon, Korea

    1건양대학교 의과대학 교수개발위원회, 2건양대학교 명곡의과학연구소





    고찰

    좋은 수업을 위해서는 철저한 준비와 효과적인 내용 전달방법, 짜임새 있는 구성, 교수와 학생의 상호작용 관계, 확실한 피드백이 잘 갖추어져야 한다[5]. 그렇지만, 교수들은 교육준비나 교육시행에 비해 마지막 과정인 교육평가와 피드백을가장 어려워한다[1]. 이를 개선하고자 건양대학교 의과대학에서는 워크숍 등을 통해 피드백에 대한 교수개발을 시행하고 있으며, 통합교육과정을 처음 개발하는 과정(process)부터각 교육과정별로 피드백 시간을 따로 배정해 평가 후 피드백자체를 의무화하였다. 또한, 학기 말에는 수업 참여 교수 전원과 학생 대표가 모여 학기 수업 전반에 관한 피드백을 나누고있다. 이번 연구 결과에서도 알 수 있는 것처럼 실제 피드백시행률이 매우 높았던 점과 피드백에 대한 학생들의 불만족수준이 비교적 낮은 것은 교수들의 노력에 학생 대다수가 긍정적인 평가를 했던 것으로 보인다.


    경험한 피드백이 무엇이었냐고 물었을 때는 집단을 대상으로 한 문제풀이 형태가 81.3%로 대다수를 차지했다. 이는 평가 직후 시행한 피드백의 시간적 정황 때문일 수도 있지만, 학생의 학업 성취 상태에 따른 개인별 피드백이라기보다는 다수 학생을 대상으로 한 교수자 중심의 일괄적 피드백이 행해졌음을 유추할 수 있는 대목이다. 이는 Archer [6]가 이야기했던 지필 시험에 대한 피드백을 교수자들이 꺼린다는 내용과는 상반되는 부분이었으나, 고득점 학생들에 대한 피드백이 간과되는 경향이 있다는 그의 지적과 맥락을 같이 하는 부분이었다. 이렇듯 피드백의 중요성에 대해서는 공감대가 형성되고 있으나, 의과대학 교수들의 피드백은 아직 부족한 수준이다.


    학생들은 이번 설문에서 현행 피드백의 문제점에 대해서도지적했으며, 가장 높은 채택률을 보인 응답은 피드백의 체계적인 측면이 부족했다는 점이었고, 그 다음으로는 적절한 시간에 이루어지지 않았다는 점을 꼽았다. 특히, 의학과 4학년학생들의 응답이 가장 부정적이었다. 4학년부터 임상의학 과목의 비중이 높아지는 점을 감안하면 기초의학 교수들보다임상의학 교수들의 피드백이 아직 덜 체계적이고, 학생들에게 실질적인 도움이 되지 못했던 것으로 파악된다.


    과제와 시험 후의 피드백 형태에 대한 질문에서 학생들이‘과제에 대한 교수님의 의견이 적힌 서면 피드백’과 ‘시험의 난이도, 성적분포 등에 대한 피드백’을 가장 많이 희망한 것은 학생들이 다수에 대한 개략적인 피드백보다 개개인의 성취에초점을 맞춘 개별적 피드백을 희망하고 있음을 의미하며, 이는 Ahn [7]이 제시한 효과적인 피드백 전략과 일맥상통하는부분이다. Bienstock et al. [8] 또한 좋은 피드백의 구성 1)학생의 자기평가, 2) 교수자의 평가, 3) 수행 목표 수립, 4) 요으로 제시하며, 이 중 교수자 평가에서는 관련성 있는 내용과 건설적인 내용을 통해 학습자의 강점은 키워지고 실수는교정할 수 있도록 도와야 한다고 했다. 이는 교수자가 학생의학업 성취에 대해 개별적인 평가를 바탕으로 피드백이 이루어져야 함을 강조한 것이다. 문제는 이처럼 체계적인 피드백을 위해서는 교수자의 업무량이 현재보다 상당히 증가한다는점이다. 이는 업무량이 현재도 많은 의과대학 교수에게 있어,개인의 열정만으로는 극복할 수 없는 부분으로 훌륭한 피드백 문화가 대학에 뿌리내리기 위해서는 대학차원 그리고 더나아가 정부 차원에서의 지원이 필요할 것으로 사료된다[9].이 연구는 일개 대학의 학생들을 대상으로 설문을 시행했다는 제한이 있다. 따라서 해당 연구의 결과를 모든 의과대학에 그대로 적용하기에는 어려움이 있다. 개별 대학에서 보다나은 피드백을 위해서는 강의평가 시 피드백에 대한 항목도필수적으로 포함하여, 교수의 피드백에 대한 학생들의 의견을 주기적으로 확인해야 할 것이다.






    Korean J Med Educ > Volume 26(3); 2014 > Article


    일개 의과대학 피어튜터링 프로그램 소개 및 효과분석

    A Peer Tutoring Program Introduction and Effects Analysis in Medical College

    이수현·전우택·양은배

    연세대학교 의과대학 의학교육학과

    Su Hyun Lee · Woo Taek Jeon · Eun Bae Yang

    Department of Medical Education, Yonsei University College of Medicine, Seoul, Korea






    서 론

    의과대학의 교육과정은 짧은 시간에 많은 내용의 지식과 술기를습득해야 할 뿐 아니라 상대평가와 유급제도로 인해 성적에 대한스트레스가 다른 전공 대학생들에 비해 상당히 큰 편이다. 실례로연세대학교 의과대학에서 본과 1, 2, 3학년들의 스트레스를 조사한결과 학업 및 성적이라는 응답이 가장 높은 것으로 확인됐다(이수현, 2012). 국외연구에서도 의과대학 학생들은 많은 양의 정보를 학습해야 하는 부담감, 시간압박, 사회적 활동기회의 상실 등으로 심각한 스트레스를 경험하는 것으로 나타났다(Enns et al., 2001). 이같은 학업스트레스는 유급이나 휴학으로 이어지게 되는데 한 연구에 따르면 의과대학, 의학전문대학원(의전원) 학생 중 10% 내외가학교생활부적응과 더불어 학습부진 그리고 유급 또는 휴학으로 이어지는 경험을 하고 있으며 이미 유급을 경험했던 학생들은 유급을 반복할 가능성이 높아 이들이 학업실패의 악순환을 반복하지않도록 도울 필요가 있음을 제안한 바 있다(한의령 외, 2012). 이같이 의과대학 학생들의 학업에 대한 고충이나 어려움은 국내외를막론하고 의과대학 교육에 있어서 해결해야 할 하나의 과제로 많은대학들은 학생들이 경험하고 있는 학습의 어려움을 해결해주기 위해 노력하고 있다.


    대부분의 대학에서는 학교생활적응이나 학업에 어려움을 경험하는 학생들을 돕기 위해 튜터링 프로그램을 운영하고 있다. 튜터에 의해 이루어지는 다양한 학습지도활동을 통칭하여 튜터링이라하고 있으나 튜터링의 의미는 각 나라마다 다르게 해석되고 있다.영국, 호주, 뉴질랜드, 이탈리아 등에서 튜터링은 주로 대학생의 세미나 지도 등에 배정된 대학원생이나 강사에 의해 운영되는 학습활동을 의미하며 미국의 경우 특정 교과목의 교습을 원하는 개인혹은 그룹의 요청에 의해 대학원생, 선배, 동료들이 교과목을 교습하는 상호자율 학습보조활동으로 받아들여지고 있다(선복근 외,2009).


    학생과 학생 간에 개별지도로 이루어지는 색다른 교육형태인 피어튜터링(peer tutoring)은(김지현, 2011) 동일하거나 유사한 학과의 동기 간 또는 선후배 간의 학습지도활동을 의미한다(Miciano,2006). 피어튜터링은 독립적이고 자기 주도적인 학습을 할 수 있도록 도와주는 중요한 교수법으로(Barrows, 1992) 특정 교과목에서우수한 실력을 가진 상급생들이 튜터가 되어 해당 과목을 이수하는데 도움을 받고자 하는 후배 학생들, 즉 튜티들을 대상으로 학업성취에 도움을 주는 형식으로 운영된다(황은영, 2008). 이 같은 피어튜터링의 장점은 다음과 같다. 튜터의 경우 비슷한 시기에 동일한경험을 했기 때문에 왜 튜티가 힘들어하는지 더 잘 이해할 수 있으며 튜티들의 입장에서는 교수보다 동료나 선배에게 어려움을 토로하는 것이 더 편하게 느낀다는 장점이 있다(Haist et al., 1998; Santee& Garavalia, 2006).


    선행연구에 따르면, 피어튜터링은 거의 모든 교과영역, 인종, 성별등에 일관되게 튜티의 학업성적 향상에 효과적이라고 한다(Robinsonet al., 2005). 구체적으로 피어튜터링의 효과를 보고한 선행연구들을 살펴보면 다음과 같다. 피어튜터링은 대학생활적응과 자율적인 학습기술의 향상에 도움이 되며(Beasley, 1997), 학습습관의 변화와 독립적인 사고기술을 발달시키며(Barrows, 1992), 자기주도적 학습태도, 자기개념, 학습효율감 등에서도 긍정적인 변화를가져온다(Barrows &Tamblyn, 1980; Robinson et al., 2005).


    국내연구에서도 피어튜터링의 효과가 긍정적임을 일관성 있게보고하고 있다. 서금택(2008)은 튜터링의 효과성 분석연구를 통해서 튜티들은 피어튜터링을 통해 대인관계능력과 융통적 사고가 향상되고 학습에 대한 동기부여가 되며 학과성적의 향상으로 자신감이 향상되었고 이론과 현장체험을 통한 통합적 사고가 길러졌다고보고하였다. 한편, 튜터들은 리더쉽 향상, 튜티들에게 설명하는 과정에서 자신의 이해력 향상, 학습을 준비함으로써 자기 주도적 성향이 향상, 타인을 지도하면서 자신을 돌아볼 수 있는 자아발견의계기를 마련했다고 보고하고 있다. 즉 피어튜터링은 도움을 받는튜티뿐 아니라 튜터들에게도 긍정적인 변화를 이끈다고 보고하였다. 또 다른 연구에서도 튜터링은 튜티의 학습능력 향상과 대인관계능력, 의사소통능력, 리더쉽 등 다양한 개인과 조직의 관계에 대한 기술을 익힐 수 있다고 한 바 있다(선복근 외, 2009). 염민호와 박선희(2008)는 튜터링을 포함하는 학습공동체가 대학생들의 심층학습을 유도하여 전공과목을 학습하는데 도움을 주고 있다고 하였으며, 김인수(2010) 역시 대학생의 튜터링은 학업성취도를 높이는데 효과가 있으며 특히 학업성취도가 평균 이하인 튜티들이 튜터링으로 인한 학업성취 상승의 혜택을 많이 받은 것으로 보인다고하였다. 이 같은 선행연구들은 튜터링 프로그램의 긍정적인 효과가학업성취와 같은 특정 영역에서만 나타나는 것이 아니라 정서적 영역을 포함한 여러 영역에 걸쳐 있으며 튜터와 튜티가 튜터링으로 인해 다양한 혜택을 얻을 수 있음을 시사한다(이은준과 김태형, 2011).


    그러나 앞서 소개된 연구결과들은 다른 나라의 사례이거나 국내의과대학이 아닌 다른 전공 분야에서 진행된 결과를 보고한 것이다. 문화가 다른 외국대학 사례나 교육과정 운영체계가 다른 일반대학의 피어튜터링 모형을 그대로 국내 의과대학에 적용하는 데는분명 한계가 있다. 가장 큰 이유는 커리큘럼에서 찾을 수 있다. 

    • 첫째, 의과대학 학생들이 한 학기에 이수해야 하는 학점이 일반대학전공에 비해 더 많다. 일반대학 전공의 경우, 학생의 상황에 따라 한학기에 19-22학점을 4년 동안 수강하지만 의과대학은 매 학년, 매학기 이수해야 할 학점이 다르며 일반대학에 비해 이수해야 할 학점이 더 많다. 예를 들어, 연세대학교 의과대학 2012년 2학기 수강학점은 본과 1학년 24.5학점, 본과 2학년 25학점으로 최소 3학점에서 6학점까지 더 이수해야 한다. 
    • 둘째, 의과대학은 전공강의가 블록강의로 진행된다. 일반대학 전공과목 강의는 일주일에 한 번 3시간씩 운영되는데 반해서 의과대학은 전공과목들이 블록강의로 진행되기 때문에 과목에 따라 최소 16-106시간 동안 연속적으로 진행된다. 짧은 시간에 집중적으로 강의가 진행되기 때문에 하루 강의 분량을 소화해내기도 쉽지 않은 환경이다. 
    • 셋째, 일반대학 전공과목의 경우 한 학기 동안 2번의 정규시험(중간, 기말고사)을 치르지만 의과대학은 블록으로 진행되는 전공과목 강의가 끝나면 바로시험을 보게 된다. 따라서 시험이 매우 빈번하게 있으며 시험 분량도 상당히 많기 때문에 차분히 이해하면서 공부를 하려면 시험범위를 1독하기도 쉽지 않다. 예를 들어 연세 의과대학 1학년의 경우매주 또는 2-3주에 한 번씩, 한 학기에 10번 정도의 시험을 치러야한다. 2학년의 경우 시험횟수는 줄지만 마찬가지로 블록강의로 진행되고 강의 분량이 많기 때문에 1학년 때와 큰 차이가 없다. 이같이 본과 1, 2학년들은 짧은 시간에 많은 양의 지식을 소화해야 하고많은 전문용어와 개념들을 암기해야 하며 유급을 피하려면 일정수준의 성적을 유지해야 하기 때문에 단순 암기중심의 공부는 불가피한 부분이 있다.


    이같은 의대 현실로 볼 때 일반대학에서 운영하는 피어튜터링프로그램처럼 튜터가 튜티에게 직접 학습내용을 가르치거나 예습,복습을 관리하는 형태로 프로그램을 운영하는 것은 현실적으로어렵다. 따라서 의과대학 현실에 맞는 피어튜터링 모형이 개발될 필요가 있다. 이에 본 연구는 연세 의과대학에서 운영하는 피어튜터링 프로그램을 소개하고 일반대학의 피어튜터링 프로그램과의 차이를 확인하고자 한다. 다음으로 의과대학 학생들이 피어튜터링을통해서 무엇을 경험하는지 즉, 튜티와 튜터들의 튜터링 목표나 기대는 무엇인지, 피어튜터링이 학습활동과 학교생활적응에 실질적인 도움이 되는지, 튜터와 튜티가 지각하는 튜터링의 성과는 무엇인지에 대해서 살펴보고자 한다. 현재까지 피어튜터링 프로그램에대한 소개나 성과를 분석한 연구들은 대부분 일반대학 프로그램에 대한 보고로 교육과정 운영이 매우 다른 의과대학 학생들도 유사한 경험과 결과를 보고하는지 살펴볼 필요가 있다. 마지막으로튜티의 성적에 영향을 미치는 튜터링 관련 변인조사를 통해 튜터링운영과 평가를 위한 기초정보를 제공하고자 한다. 이상의 내용을연구문제로 제시하면 다음과 같다. 1) 연구문제 1: 의과대학 피어튜터링 프로그램과 일반대학 피어튜터링 프로그램의 차이점은 무엇인가? 2) 연구문제 2: 의과대학 피어튜터링 프로그램에서 튜터와 튜티가 무엇을 경험하는가? 3) 연구문제 3: 튜티의 성적에 영향을 미치는 튜터링 관련 변인은 무엇인가?


    결론 및 논의


    의과대학 학생들은 스트레스와 심리적 문제로 고통 받고 있으며스트레스의 많은 부분은 과도한 학습량, 실패에 대한 두려움, 낙제와 같은 교육과정과 관련된다(Malik, 2000). 또한 의대강의는 대형강의 위주로 진행되며 짧은 시간 동안 많은 지식을 전달하는 교수방법을 고려해 볼 때 학습과 학교적응에 어려움을 경험하는 학생들에게는 개별화된 접근이 필요하다. 실제로 학습을 위한 가장 좋은 방법으로 개별화된 교수법이 효과적임이 알려져 있으며(Waugh, 1982)이런 의미에서 피어튜터링은 의과대학에서 학습의 어려움을 경험하는 학생들에게 도움이 되는 적절한 교수법 중 하나라 할 수 있다.미국과 영국 의과대학에서는 피어튜터링과 유사한 또래지원 학습 프로그램(peer-assisted learning, PAL)을 운영하고 있으며 이에대한 성과를 몇몇 연구를 통해 보고하고 있다. PAL은 의학교육에서 잘 이해되도록 가르치는 방법으로 자리매김해왔으며 학습자와피어튜터 모두에게 다양한 도움을 제공한다. PAL은 의학교육에서해부학 분야, 문제 중심 학습, 의사소통 기술 훈련, 임상시험 그리고소생술훈련에서 긍정적인 효과를 나타내는 것으로 확인됐다(Nikendeiet al., 2009). 그러나 아직 국내 의과대학에서는 피어튜터링에대한 연구가 활발히 진행되고 있지 않은 상태이다. 따라서 본 연구는 연세 의과대학 피어튜터링 모형을 소개하고 튜터링 활동을 통해서 튜터와 튜티들이 어떤 경험을 하는지, 피어튜터링의 효과가 무엇인지를 분석하고자 하였다.


    연구를 위해 2012년 1학기 피어튜터링 프로그램에 참여한 40명(튜터 20명/튜티 20명)의 학생들을 대상으로 학기 말 평가기간 동안 설문조사 및 인터뷰를 통해 수집한 자료를 분석하였으며 5점 척도로 구성된 튜터링 관련 설문 문항과 튜티들의 1학기 평점평균을가지고 상관분석을 실시하였다. 이장에서는 분석결과를 바탕으로몇 가지 논의를 하고자 한다.


    첫째, 튜터링 과정에서 튜터와 튜티는 학업성적 향상, 학교생활적응, 대인관계 및 선후배 유대관계형성이라는 동일한 목표를 가지고 튜터링에 임하는 것이 확인되었다. 또한 튜터는 튜터링의 성과를선후배 유대관계형성, 학업지원, 정서적 지지라고 반응했으며 튜티들은 학업적 도움, 선후배 유대관계형성, 심리적 안정감 획득이라고하였다. 반응의 순위는 차이가 있지만 학업, 관계, 정서적 측면에서성과가 있음을 확인하였다. 본 연구결과는 피어튜터링이 학업성취에 긍정적인 영향을 미치며(김인수, 2010; Robinson et al., 2005; Santee& Garavalia, 2006) 튜티들은 튜터링을 통해서 학습능력과 대인관계능력이 향상된다는 선복근 외(2009)의 연구결과와 일치한다.


    또한 튜터링을 통해서 튜티뿐 아니라 튜터들도 도움을 받은 것으로 확인되었는데 구체적으로 튜터들은 튜터로서의 역할수행을 통해 책임감, 자기이해를 포함한 리더쉽 발달을 경험했으며 선후배 유대감을 형성하는데 도움이 됐고 튜티의 학업을 도와주는 과정에서본인의 학습동기도 향상되었음을 보고했다. 이 같은 결과는 튜터링과정에서 튜터들은 리더쉽이 향상되며 자기 이해력 및 자기 주도적성형이 향상된다는 서금택(2008)의 연구와 일치하는 결과이다.


    본 연구에서 유의하게 살펴봐야 할 결과는 튜터들과 튜티들이 정서적 지지를 제공하고 받은 것을 튜터링 성과로 지각한다는 점이다. 일반적으로 튜터링의 목적은 주로 학업발달에 초점을 두며(Sobral,2002) 학습공동체형성, 학습동기증진, 커뮤니케이션 능력 배양의 기회제공(황은영, 2008)으로 알려져 있다. 이 같은 정의로 볼때 튜터링의 목적에 정서적 측면은 포함되어 있지 않지만 튜터링의성과로 정서적 측면이 나타난다는 것은 튜터링이 인지적 측면뿐 아니라 정서적 측면에서의 요구도 함께 고려해야 한다는 것을 의미한다. 이 같은 결과는 피어튜터링 프로그램이 의대생의 인지적, 정서적 요구에 대처해야 한다는 결과를 지지한다(Scha˜er et al., 1990).또한 피어튜터링의 긍정적인 효과가 학업성취와 같은 특정 영역에한정되어 있기보다는 정서적 영역을 포함한 여러 영역에 걸쳐 있으며 튜터와 튜티가 튜터링으로 인해 다양한 혜택을 얻을 수 있다는결과와도 일치한다(이은준과 김태형, 2011). 본 연구에서 정서적 측면이 성과로 인식된 결과는 학업스트레스와 성적에 대한 경쟁의식으로 인해 정서적 불안과 인간관계에서 소외감이나 외로움을 경험하는 학생들이 1:1 관계로 만나는 튜터링 활동을 통해서 의과대학학생으로서의 어려움을 서로 공감하면서 정서적인 지지나 안정감을 경험하기 때문인 것으로 이해된다.


    둘째, 일반적으로 피어튜터링은 학업성적이 우수한 선배나 동료가 후배에게 학습내용을 가르치는 것에 초점이 맞추어져 있지만본 연구에서 피어튜터링을 경험한 학생들이 피어튜터링에서 도움받은 점으로 가장 많은 응답을 한 내용은 학습방법 습득에 대한 부분이었다. 세부적인 반응 내용을 살펴보면 다음과 같다. ‘학습방법노하우를 알게 되었다,’ ‘시간관리방법을 배웠다,’ ‘시험문제 출제유형을 파악하는데 도움이 되었다,’ ‘학습방향(법)을 설정하는데 도움받았다’, ‘시험요령을 터득했다,’ ‘시험 시 주의사항을 알려주어 도움받았다’ 등의 반응들이 나타났다. 이 같은 결과는 의과대학에서피어튜터링이 교과목 내용 중심으로 가르치기보다는 공부하는 방법, 시간관리방법, 과목별 시험 보는 요령과 같은 학습방법적인 측면에 초점화되어 있다는 것을 시사한다. 이는 두 가지 측면으로 이해할 수 있다. 하나는 암기 중심, 시험전략 중심의 의대공부 특성이반영된 결과일 수 있으며 다른 하나는 본 연구의 피험자 특성이 반영된 결과로 이해할 수 있다. 즉, 본 연구에서 튜티로 참여한 학생들의 95%는 의전원 및 군위탁생으로 구성되어 있다. 과정상으로 볼때 이들은 대학원생들이지만 의대공부를 새롭게 접하는 대상들로의대문화와 학습방법에 적응하기에는 시간이 필요한 학생들이다.따라서 이들이 학교에 빨리 적응하기 위해서는 의과대학의 특성을빨리 이해하고 과목별 학습방법이나 시험전략과 같은 정보를 알려줄 대상이 필요하다. 그러나 의전원생들과 군위탁생들의 경우 선배와의 상호작용 기회가 거의 없기 때문에 선배와 직접적으로 매칭해주는 튜터링 프로그램에 적극 참여하여 필요한 도움을 받는 것으로 이해된다. 추후 연구에서는 의예과 학생들의 튜터링 프로그램참여율이 저조한 이유, 튜터링의 내용이 공부방법에 치우치는 이유등에 대한 추가적인 연구가 필요하다.


    셋째, 피어튜터링 과정 변인과 튜티의 학업성취와의 관련성을 알아보기 위해 상관분석을 실시하였다. 사례 수가 충분하지 않기 때문에 본 연구를 일반화하는데 통계적인 한계는 있지만 의과대학피어튜터링과 관련된 선행연구가 빈약한 상태에서 본 연구결과는하나의 참조치로 활용가치가 있을 것으로 기대한다. 연구결과를 좀더 살펴보면, 튜터링 횟수(한 학기 평균 6-8시간)는 튜터링 과정 변인 및 튜티의 학업성취와 상관이 없는 것으로 나타났다. 일반적으로대학에서는 한 학기당 30시간(황은영, 2008), 학기 중 2시간 10회이상(한안나, 2011), 한 학기 18시간 이상(고려대학교, 2009) 등 다양한 튜터링 운영시간을 제안하고 있다. 대학마다 자체적인 운영방침을 세우고 튜터링 횟수 및 시간을 제한하고 있는데 이 같은 제한이 튜터에게 봉사시간과 비용을 지불하기 위한 기준인지, 아니면튜터링의 실제적인 성과와 관련이 있기 때문인지는 명확하지 않다.그러나 선행연구에서 피어튜터링 효과성을 결정짓는 요인에 대한연구에서 튜터링 프로그램의 특징(예: 튜터 훈련, 튜터링 시간)이튜터링 효과를 결정짓는다고 보고한 바 있다(Robinson et al., 2005).본 연구는 선행연구와 상반된 결과로 이 같은 결과가 무엇을 의미하는지는 추가적인 연구가 진행되어야 할 필요가 있지만 튜터와 튜티가 시간과 노력을 낭비하지 않고 튜터링이 좀 더 효과적으로 운영되기 위해서는 적절한 튜터링 횟수를 제안할 필요가 있겠다.


    한편, 튜티는 튜터링이 도움이 됐다고 지각하는 정도가 높을수록, 튜터와의 관계만족도가 높을수록 다음 학기에 참여하고자 하는 동기가 높은 것으로 나타났다. 튜티와 튜터 간 형성된 관계는 튜터링의 중요한 성공 요인 중 하나라는 연구결과(Malik, 2000)가 보여주듯이 튜티가 튜터와 형성한 관계를 긍정적으로 지각하는 것은이후 튜터링 활동과 지속적인 참여에 영향을 미친다는 것을 시사하는 결과이다. 따라서 튜터링 오리엔테이션에서 튜터들에게 학습지원자로서의 역할뿐 아니라 관계형성을 위해 노력해야 한다는 점을 인지시켜야 할 것이다. 반면 튜터는 튜터링 활동이 튜터의 성장에 도움이 된 정도가 높을수록 다음 학기 참여의사가 높은 것으로나타났다. 이 같은 결과는 튜터링 프로그램이 지속적으로 유지되고 활성화되기 위해서는 튜티뿐 아니라 튜터들 또한 함께 성장하고도움받는 경험이 중요하다는 것을 시사한다. 튜티와 튜터의 지속적인 피어튜터링 참여는 경험과 노하우를 축적한다는 측면에서도 중요하지만 튜티가 진급 후 튜터로서의 활동으로 연장될 때 성장지향적인 경험을 할 수 있다는 것에 의미가 있다. 따라서 튜터링 운영에서 간과해서는 안되는 부분이 학생들이 지속적인 참여를 할 수 있도록 교육적 환경을 마련해주어야 한다는 것이다.


    넷째, 튜티의 학업성취와 가장 관련이 있는 튜터링 과정 변인은튜터가 지각하는 목표달성 정도인 것으로 나타났다. 즉 튜터가 목표로 하는 학업성적 향상, 선후배 유대관계형성, 학교생활적응이높다고 지각할수록 튜티의 학업성취도가 높다는 것이다. 이 같은결과는 튜터가 튜터링의 목표를 정확히 지각한 후 튜터링 활동에참여하는 것이 중요함을 의미한다. 또한 오리엔테이션을 통해 튜터링의 목표설정이 매우 중요함을 강조할 필요가 있으며 중간, 기말평가과정을 통해서 튜터링의 목표가 잘 설정되었는지, 목표대로 진행되고 있는지를 모니터링하는 과정이 필요함을 시사한다. 많은 경우 피어튜터링은 ‘학업부진학생’을 지원하기 위한 프로그램으로진행되는 것이 대부분이다(김지현, 2011). 의과대학의 경우 고등학교까지 매우 우수한 성적으로 학교생활을 해왔던 학생들이라 자신이 낮은 성적으로 인해 누군가로부터 도움을 받는다는 것을 편하게 생각하지 않는다. 자신이 공부를 못하는 학생이라는 낙인이 찍힐까 봐, 아니면 도움을 요청할 용기가 없어 튜터링 프로그램에 적극적으로 지원을 하지 않는 경우가 대부분이다. 튜터링이 성공적이기 위해서는 그것이 학업적 도움이건, 개인적 도움이건 간에 도움을 받을 필요가 있다는 것을 인식할 수 있어야 한다(Scha˜er et al.,1990). 튜터링이 학업부진학생만을 대상으로 하지 않는다는 것을적극적으로 홍보할 필요가 있으며 어려움을 경험할 때 도움을 요청하거나 도움을 받는 것이 부끄러운 일이 아니라는 인식의 전환이필요하다. 현재 연세 의과대학의 피어튜터링은 학교생활적응과 학업지원에 초점이 맞춰져 있지만 외국의 사례처럼 임상실습과 술기교육으로까지 피어튜터링 프로그램을 확대 적용할 필요가 있겠다.


    본 연구는 선행연구가 빈약한 상태에서 튜터링 프로그램 운영기간이 한 학기였고 사례 수도 충분하지 않았기 때문에 연구결과를일반화하는데 한계가 있다. 그러나 본 연구는 미래 피어튜터링 연구를 위한 하나의 단초를 제공할 수 있을 것으로 기대된다.







    This study was conducted to analyze the effects of peer tutoring programs and to introduce the peer tutoring

    program in medical College. Forty medical students participated in the study and data were collected through

    surveys and interviews. Through the interviews, we investigated the peer tutoring experiences of tutors and tutees,

    and what they perceived that they accomplished. Correlation analysis was conducted to investigate the tutoring

    process variables that affect the academic achievement of tutees. It was found that tutors and tutees reported

    achievements in the schoolwork, relationships, and emotional aspects.


    Keywords: Peer tutoring, Academic achievement

    의대 학생상담의 ABC

    ABC of medical student counseling

    이수현

    연세대학교 의과대학 의학교육학과

    Lee Su Hyun

    Department of Medical Education, Yonsei University College of Medicine





    최근 들어 학생상담에 대한 교수들의 역할이 강조되면서 교수들의 학생상담을 의무화하는 대학이 늘고 있다. 학생상담의 필요성에 대해서는 동의하지만 막상 상담을 하려고 할 때 무엇을,어떻게 해야 하는지 막막하다는 얘기를 간혹 듣는다. 초, 중, 고등학교 교사들은 교사가 되기 위한 준비 과정에서 교직과목으로학생상담 및 생활지도 과목을 필수로 수강하지만 대학의 교원들은 그러한 기회가 없기 때문에 대학 교수들을 대상으로 하는 학생상담 교육은 그 필요성이 더욱 클 수밖에 없다(강혜영, 이제경, 2009).


    교수들에 대한 학생상담교육의 필요성은 의대 교수들도 예외는 아니다. 의과대학 학생들은 다른 대학생들과 공통적인 특성을 보이기도 하지만 의대생들만의 독특한 문제를 가지기도 한다. 예를 들어, 적성에 대한 고민이 많고 의사가 되려는 동기가결여되어 있고 공부에 재미를 못 느끼고(암기식, 분량과다), 여가가 없다고 한다(박정한, 김경환, 김광우, 전혜리, 1999). 또한 친구사귀기에 인색하고 배타성이 크며 수동적인 공부(족보 중심의 공부)를 하며 교양 공부가 부족하여 세상 물정이 어둡다고 한다(민성길, 구민성, 1999). 이 같은 학생 특성으로 볼 때 의대 교수들도 학생상담을 피해갈 수 없는 상황이며 오히려 더 많은 관심과 지원이 필요한 상황일 수 있다.


    따라서 의대 교수들은 교과교육 뿐 아니라 학생들의 고민과문제를 인식하고 그들을 돕고자 하는 상담자로서의 역할을 수행할 준비가 되어 있어야 한다. 의대 교수들에게 상담자로서의 역할을 기대한다는 것이 또 다른 업무 부담으로 느껴질 수도 있다.그러나 학교교육은 교과교육과 생활지도라는 두 가지 활동을 중심으로 이루어지기 때문에 교과지도 뿐 아니라 생활지도를 위한상담활동을 하는 것은 교원의 매우 중요한 교육활동중의 하나다. 따라서 교수들은 학생상담에 대한 기본적인 지식과 방법을숙지하고 있어야 한다.


    그렇다면 학생상담은 왜 필요한 것인가? 학생상담은 두 가지측면에서 그 필요성이 강조된다. 

    • 첫째는 교육적 측면으로 학생들이 학교생활에 잘 적응하고 나아가 사회에서 접하게 되는 다양한 문제들을 해결해 나갈 수 있도록 돕는데 필요하다. 
    • 둘째는개인적인 측면으로 학생의 잠재력을 발휘하도록 도와 개인의 발전과 행복은 물론 사회와 국가의 발전에 기여하도록 지도하는데필요하다.


    학생상담의 중요성과 필요성을 공유했다면 이제는 학생들이주로 어떤 고민을 하는지에 대한 이해가 필요하다. 학생들이 고민하는 문제 영역은 학업영역, 진로영역, 개인사회적 영역으로구분할 수 있다. 의대에 입학할 정도의 학생 수준이라면 학업과관련해서는 고민이 없을 것이라고 생각하겠지만 생각보다 학업문제로 고민하는 학생들이 많다. 중고등학교 때의 공부와 달리대학공부는 자기 주도적 학습이 매우 중요하며 더욱이 의대공부는 짧은 시간에 방대한 양의 지식을 습득해야 하기 때문에 학습에 있어 전략적인 접근이 필요하다. 따라서 학업상담에서는 학습전략 습득, 학습방법 지도, 교육정보제공 등이 포함된다. 교수들은 학업에 어려움을 호소하고 성적이 낮은 학생의 고민을 노력 하지 않아서 생기는 결과라고 단정 짓기보다는 학생의 문제가 어디서 기인했는지 확인하고 해결방법을 함께 모색하려는 자세가 필요하다.


    진로상담은 자신에 대한 이해를 바탕으로 적절한 직업을 선택하도록 돕고 나아가 직장생활에 잘 적응하도록 도와주는 것을목적으로 한다. 따라서 단순히 학생과 직업을 기계적으로 짝짓는 작업이 아니기 때문에 효과적인 진로상담을 위해서 학생에대한 이해가 필수적이다. 대부분의 학생들은 의사가 되기 위해의대에 입학했을 것이다. 그러나 어떤 학생들은 부모나 주변의권유에 의해 의대에 입학하는 경우도 있다. 이렇게 입학을 했더라도 다행히 의대공부가 적성에 맞으면 괜찮지만 생각보다 재미없고 자신의 적성과 맞지 않다는 생각이 들 때 진로에 대한 고민이 시작된다. 실제로 의과대학 재학생들에게 다시 고등학교 3학년이 된다면 의과대학에 진학 할 것인가에 대한 질문에서 50.3%의 학생만이 진학의사를 밝히고 지금 전과가 가능하다면 전과를하겠다는 학생들이 28.1%에 달했다. 또한 의과대학 졸업 후 진로나 전공 선택을 심각하게 고민하고 있다고 응답한 학생이36%로 조사되었다(전우택, 양은배, 김은경, 2006, 재인용). 이미진로결정을 하고 왔더라도 의과대 안에서 세부적으로 관심있는직업군(임상가, 연구자, 내과, 이비인후과 등...)을 찾아야 하기때문에 진로에 대한 고민은 한 순간에 해결되는 문제가 아니다.따라서 진로고민을 하는 학생의 경우 적성에 대한 고민인지, 정보부족으로 인한 고민인지, 의사결정의 문제인지를 먼저 구별하고 문제에 따라 세부적으로 필요한 도움을 줄 수 있어야 한다.


    마지막으로 개인, 사회적 영역으로 정서와 성격문제, 대인관계, 건강과 여가지도 등이 포함된다. 학생들이 가지고 있는 능력을 효과적으로 발휘하기 위해서는 정서적인 문제를 함께 살펴야한다. 자신의 감정을 처리하는 능력이 미숙하거나 인간관계에서지속적인 문제가 발생하여 학교 적응에 어려움을 겪는 학생들이있다. 또한 심인성 질환으로 학교생활을 성실히 하지 못하는 경우도 있으며 시간관리가 안되거나 스트레스를 적절히 해소하지못하는 학생들이 있다. 이 같은 문제를 경험하는 학생들은 남들도 다 그런다고 생각하거나 사소한 문제라고 생각해서 문제를 해결하기 위해 적극적으로 노력하지 않으면서 계속 악순환하는 모습을 보이기도 한다. 교수는 학생들이 이 같은 문제들을 간과하고 진로나 학업문제로 상담을 요청할 경우 여러 문제들이 어떻게상호작용하고 있는지를 살펴야하며 더 근본적인 문제를 먼저 다루어주는 것이 필요하다. 만약 성격적인 문제나 정서적인 문제가만성화되어 우울이나 불안을 통제할 수 없는 상태로 보일 경우,전문가에게 연결(refer)하는 것도 매우 중요하다.


    지금부터는 학교상담의 특징과 주의해야 할 몇 가지 사항을살펴보고자 한다. 학교에서 이루어지는 상담의 가장 큰 특징 중하나는 비자발적인 학생들이 상담의 대상이 된다는 것이다. 이는 초, 중, 고등학교에서 나타나는 현상이지만 대학 상담에서도유급 대상자들이나 제적 위험에 있는 학생들, 학교에 적응하지못하는 학생들을 지도교수나 담임교수가 학생을 불러서 상담하는 경우가 종종 있다. 그러나 비자발적인 학생들은 상담에 대한동기가 현저히 낮기 때문에 변화를 위한 노력을 하지 않을 수 있다. 더불어 변화되지 않는 원인을 상담이나 상담하는 선생님의탓으로 귀인 할 수 있기 때문에 상담의 효과를 기대하기 어려운부분이 있다. 그렇지만 의대는 학생들의 학업관리가 매우 중요하기 때문에 유급생이나 제적 위험생들의 자발성만 기대해서는문제의 해결점을 찾기 어렵다. 따라서 학생상담에 대한 적극적인 홍보와 함께 학생들이 상담을 편안하게 받을 수 있는 분위기와 환경이 먼저 조성되어야 할 필요가 있다. 학생의 입장에서 볼때, 유급이나 제적문제로 상담 받는 것 자체가 학생에게는 낙인찍는 효과가 있기 때문에 도움 받는 것에 대한 반감이나 저항이생길 수 있다. 따라서 학생들의 도움추구 행동이 적극적이지 않은 이유를 상담에 대한 거부나 변화에 대한 욕구가 없는 것으로생각하는 하는 것은 위험하다. 상담 받는 학생에 대한 낙인찍기나 학생에 대한 편견, 평가적인 태도만 개선되어도 상담에 대한거부감은 상당부분 해결할 수 있다.


    두 번째 학교 상담의 특징은 이중관계와 관련된다. 이중관계란 상담자가 내담자와 둘 또는 그 이상의 역할을 동시에 또는 연속적으로 가지는 관계를 말한다. 즉 교수는 이미 학생과 사제관계이면서 상담자 역할을 하게 되니 이중관계라 할 수 있다. 물론전문적 관계에서 이중관계는 불가피하고 반드시 해로운 것만은아니지만 논란의 여지가 있다. 왜냐하면 이중관계가 야기하는 몇가지 문제들이 있기 때문이다. 

    • 첫째는 객관성과 중립성의 상실이상담자의 판단에 오류를 발생시킬 가능성이 높다. 이중관계에 들어갈 경우 학생에 대한 객관성을 유지하기 어렵기 때문에 상담자로서 적절하지 못한 판단을 할 가능성이 높아진다. 
    • 둘째, 학생과교사, 상담자와 내담자 사이에는 힘의 차이가 존재하기 때문에착취와 위해의 가능성 커진다는 점이다. 학생을 착취로부터 보호하고 발생 가능한 문제 상황을 점검하는 것은 교수의 의무라고할 수 있다. 따라서 교수나 상담자로서의 역할을 수행할 때 학생에게 도움이 되는 판단과 행동을 하는지에 대한 윤리적 의사결정과정이 매순간 필요하다. 
    • 마지막으로 상담이라는 맥락이 또 다른평가의 장이 될 수 있다. 즉 상담 받으러 온 학생을 공부 못하고부적절한 학생으로 낙인찍기를 할 수 있다. 이는 교수의 학생에대한 기본적인 태도와 관련이 되는데 학생을 상담할 때 문제에초점을 둘 것이 아니라 문제를 가진 사람을 대상으로 상담한다는것을 잊지 말아야 한다. 교수의 학생에 대한 인간적인 존중, 배려, 신뢰가 매우 중요하며 이는 상담의 효과에도 매우 큰 영향을미치게 된다. 따라서 학생상담을 효과적으로 하기 위해서는 상담자 역할을 하는 교수의 인간적 자질이 매우 중요하다.


    그렇다면 효과적인 상담을 하려면 어떻게 해야 할까? 

    • 첫째, 비록 학생이 문제를 가지고 있다 하더라도 문제보다 장점 찾는 것에 초점을 두어야 한다. 여기에는 두 가지 의미가 있는데 하나는학생이 변화될 것이라는 인간적인 신뢰이며 또 하나는 문제를해결하기 위해서는 학생의 긍정적 자원을 활용해야 하기 때문이다. 따라서 학생이 가져온 문제에 너무 몰두하지 말고 학생에게어떤 능력이 있는지를 먼저 볼 수 있도록 노력해야 한다. 
    • 둘째,학생이 하고 싶은 말을 편안하게 할 수 있도록 도와줘야 한다. 심리적 문제는 하고 싶은 말을 하지 못할 때 발생하는 경우가 많다.어떤 얘기에도 평가하거나 판단하지 않는다는 믿음만 학생에게있다면 학생은 솔직하게 하고 싶은 얘기를 할 수 있을 것이다
    • 마지막으로 학생의 입장에서 얘기를 잘 들어주어야 한다. 학생을이해하기 위해서는 기본적으로 경청의 자세가 필요하다. 경청하기만 잘 된다면 머릿속으로 판단하거나 평가하는 것은 자연히줄어들게 될 것이다.

    이렇게 노력했음에도 불구하고 상담성과가 생각만큼 나오지않는다고 해서 실망할 이유는 없다. 상담 성과는 금방 나타날 수도 있지만 시간이 지나면서 뒤 늦게 나타나는 경우도 있기 때문에 당장의 가시적인 결과로 상담자로서의 노력을 평가절하 할필요는 없다. 가장 효과적인 상담이 학생에 대한 존중과 긍정적인 관심임을 잊지 않는다면 학생에게 존중받는 스승이자 상담자가 될 수 있을 것이다.







    의학교육에서 고려해야 할 신세대 학생의 특징

    Characteristics of the Current Student Generation and Considerations for Medical Education

    김은경

    숭실대학교 베어드학부대학

    Eunkyung Kim

    Baird University College, Soongsil University, Seoul, Korea






    서 론


    대학생은 청년기에 해당하며 성인기를 준비하는 시기이다. 주로20대인 청년기는 청소년기의 급격한 신체적, 심리적, 사회적 발달의경험을 성숙시키고 독립된 성인으로서의 생활을 준비하는 시기로자아정체감을 확립하고 관심 있는 분야를 찾으며 대인관계의 폭을넓히면서 교육적, 직업적 진로를 선택하는 등 다양한 발달 과업을수행해야 하는 시기이다. 누구나 청년기를 보내지만 개개인의 각기다른 배경, 성격, 경험에 따라 각자 다른 대학생활을 하게 된다. Sanford(1967)는 대학생의 발달에서 격려와 도전이 중요함을 주장하면서 너무 많은 도전은 학생을 압도당하게 하고, 너무 많은 격려는나약한 학생을 만들게 된다고 했다. 따라서 대학은 학생의 특성과잠재력을 고려하여 적절한 격려와 도전을 제공해야 하며, 현재 대학생의 특성에 대한 정확한 인식과 대처는 대학교육을 성공적으로하기 위해 필수적인 과정이라고 할 수 있다.


    2000년대 이후 대학에 입학하는 학생들은 과거의 기성세대와는상당히 다른 가치관, 사고방식, 생활패턴을 가지고 있는 것으로 알려져 있고 이들을 부르는 호칭은 다양하다. 1980년대에서 90년대중반에 출생한 세대로 X세대의 뒤를 잇는다는 의미에서 Y세대라고 불리기도 하고, 21세기에 진입하는 시기에 태어났다고 해서 밀레니엄 세대라고도 불린다. 또한, 1990년대 후반부터는 N세대(netgeneration)로도 불려왔다. 매체와 컴퓨터의 급속한 보급으로 이들은 기성세대와 세대 간 차이를 현격하게 넓히며 다른 행동, 태도, 가치관을 나타내고 있다.


    이러한 신세대 학생들 중에서 매우 학업적으로 우수한 학생들이의과대학과 의학전문대학원에 입학한다. 우수한 자연계열 학생들의 의학계열 선호현상은 이미 널리 알려져 있는데, 정욱(2006)에 따르면 2006학년도 자연계열 수능점수 상위 30개 학과 중 26개가 의학계열(의예과, 치의예과, 한의예과)이라고 밝혔다. 이러한 현상은2005년 이후 의학전문대학원 제도가 도입된 이후 더욱 심화되고있고, 자연계열 우수 고등학생 중에서도 가장 성적이 좋은 학생들이 의학계열 전공을 희망하고 있는 것으로 나타났다(김은경, 2010).학업우수학생들이 입학한 의과대학과 의학전문대학원은 소수의우수한 학생들끼리의 경쟁과 협력을 통해 살아가야 하는 특별한환경에 노출된다. 따라서 의과대학과 의학전문대학원의 학생은 새로운 세대의 특징과 소규모의 우수학생집단에 소속된 학업우수학생의 특징을 공유하면서 그들만의 사고와 행동방식을 나타낼 것으로 볼 수 있다.


    그러나 이러한 의과대학 학생들의 특징에 대한 연구는 거의 찾아보기 어려우며, 새로운 세대의 학생들을 고려하는 변화가 의학교육에서 이루어지고 있는가에 대해서는 의문이다. 따라서 본 연구에서는 밀레니엄 세대와 학업우수학생의 특성을 살펴봄으로써 최근 의예과와 의학전문대학원에 입학하는 대학생의 특징을 탐색하고 새로운 세대의 학생들을 위한 의학교육에서의 시사점을 제공하고자 한다.


    밀레니엄 세대


    밀레니엄 세대의 학생들은 그 이전에 대학에 다니던 세대와 여러가지 측면에서 다르다. 이들은 즉각적인 피드백을 원하고, 비판적인사고능력이 부족하며, 특권의식과 비현실적인 기대감을 가지고 있다. 이들은 높은 수준의 부모의 관여와 학교 안팎에서 성공하는 방법에 대한 가이드를 기대하며, 적은 노력과 시간을 투자하여 성공하기를 원한다(Monaco &Martin, 2007). Durden (2005)은 밀레니엄 세대들이 의미 있는 노력 없이 단지 참여하는 것만으로 좋은 성적을 기대하고 안전하고 편안함을 추구하며 자신을 소비자로서 생각하므로 서비스 받기를 원한다고 하였다. 권위에 대한 존경은 거의 없고 건설적인 비판에 직면했을 때 방어적이며 미래에 대해 낙관한다. 이제 앞으로의 대학생들은 사실보다는 제약을 받지 않고편집되지 않은 즉각적인 자기주장을 중요하게 생각하며, 무슨 수를써서라도 자존감을 지키려고 한다.


    Howe &Strauss (1993)와 Howe &Strauss (2003)는 밀레니엄 세대의 학생들이 사회적 규준을 준수하지만 성적과 결과물에 집착하고, 수업 외 활동을 열심히 하며 안전하고 정돈된 환경을 원한다고 하면서 이들의 특징을 7가지로 정리하여 제시하고 있다. 

    • 첫째, 밀레니엄 세대는 자신이 특별하다고 느낀다. 부모와 친밀한 관계를 맺으면서 많은 칭찬과 격려 속에 자랐고, 이전 세대보다 고등교육을더 많이 받으면서 특별함을 느껴왔다. 
    • 둘째, 보호받는 세대이다. 부모가 주도하는 시간관리와 생활패턴에 익숙한 이들은 자유시간을많이 가지지 못하며 자랐다. 독립적인 창의적 사고와 의사결정능력의 기회를 많이 갖지 못한 것이 밀레니엄 세대의 교육에서 당면한과제이기도 하다. 
    • 셋째, 팀워크 지향적이다. 독립적으로 일하는 것은 실패에 대한 위험을 감수해야 하므로 밀레니엄 세대는 혼자서 일하는 것에 자신감이 부족하다. 함께 협력적으로 일하는 것을 선호한다.
    •  넷째, 자신감이 있고 매우 낙관적이다. 구체적으로 성공하는 방법에 대해 알지는 못하지만 큰 꿈을 꾸고, 성공할 것이라고 자신한다. 이러한 자신감은 고등학교 때까지 적은 노력으로 좋은 성적을 쉽게 얻은 것에 기인하는 것처럼 보인다. 인터넷을 통한 다양한 형태의 정보를 쉽게 접하면서 한꺼번에 여러 일을 처리하는 것이 익숙해지면서 이러한 자신감을 증진시킨다. 따라서 이들이 대학에 진학했을 때 A나 B학점을 받지 못하게 되면 당황하고 좌절하기쉽다. 
    • 다섯째, 이들은 압박감을 느낀다. 그렇기 때문에 빠른 피드백을 받기 원하고, 피드백과 방향제시가 없이는 무력해지기도 한다.
    • 여섯째, 성취하고자 하는 강한 욕구를 가지고 있다. 어린 시절부터특별하다는 이야기를 듣고 성장해온 이들은 위대한 것을 성취하고자 기대한다. 
    • 일곱째, 밀레니엄 세대는 관습적이다. 그들은 문화적,사회적 다양성을 존중하고 받아들인다.



    디지털 세대


    신세대의 여러 가지 특징 중에서 가장 주목해야 할 특징은 이들이 디지털 세대라는 것이다. 김재복(2002)은 신세대가 영상매체와사이버 친화적 사고와 행동양식을 가지고 있으며, 소비적 성향이강하고 대중문화 편향적이라고 하였다. 어려서부터 TV, 비디오, 전자게임, 컴퓨터 등을 접촉하면서 성장한 신세대는 영상이미지에 친숙하므로 감각적이고 감성적인 경향이 있다. 사이버세계가 익숙한이들은 인터넷의 활용으로 많은 정보를 접하므로 더욱 개방적 세계로 나갈 수 있다. 이로 인하여 신세대가 극도의 개성과 다양성을 추구함으로써 변화의 자신감을 표출하기도 하지만, 이는 자기중심성과 이기주의로 나타날 수도 있다. 이러한 특징은 기성세대의 논리적, 합리적 사고와 기존의 가치에 입각한 권위적 행동양식, 사회 전체 속에서 자신의 역할을 중시하는 공동체 의식과 크게 차이가 있어서 기성세대와 구별된다.


    Palfrey &Gasser (2008)는 정보화시대를 전후한 세대를 디지털이주민(digital imigrant), 디지털 정착민(digital settler), 디지털 원주민(digital native)으로 구분하였다. 

      • 아날로그 시대에 출생하여 본인의 의지와 상관없이 정보화시대에 노출되었기에 정보화에 준비가 되어 있지 않은 디지털 이주민, 
      • 정보화 이전 시대에 출생했으나디지털 환경에 적응하여 정보화 환경을 잘 이용하는, 그렇지만 여전히 사고방식은 아날로그적인 디지털 정착민, 
      • 인터넷이 대중화된1980년대 이후에 출생하여 디지털 환경에서 자란 밀레니엄 세대를 디지털 원주민으로 설명하고 있다. 

    디지털 원주민은 기존 세대와달리 거의 모든 일상생활을 디지털 환경에 의존하고, 컴퓨터 없이글을 쓴다거나 인터넷 없이 자료를 조사하는 것을 상상하지 못한다. 온라인에서 친구를 사귀는 것이 당연한 이들은 생활과 사고방식, 사회문화적 관습에서 아날로그적 사고방식을 가진 기존 세대와 분리된다. 단방향 미디어인 TV에 익숙한 베이비부머 세대는 미디어가 제공하는 정보를 수동적으로 받아들이는 데 익숙했다. 그러나 가정용 PC와 게임기가 등장한 이후 출생한 디지털 원주민 세대는 일대일 방식의 의사소통이 아닌 많은 사람들과의 동시다발적상호작용과 온라인 관계를 중시한다. 따라서 위계적, 폐쇄적, 획일적인 사고보다는 자유로운 사고방식과 행동양식을 갖게 된다.



    학업우수학생


    최근 의과대학이나 의학전문대학원에 입학하는 학생들은 경쟁을 통해 선발된 소수의 학업우수학생들이다. 이들은 이제까지 자신이 속한 집단에서 최고의 평가를 받아왔으며, 공부를 잘한다는자신감이 강한 학생들이다.


    학업우수학생들이 우수한 학생들끼리 모여 학업적 동질집단을형성하면 경쟁을 통해 학습동기가 향상되고 더 많은 시간을 공부에 투자하여 학업능력이 향상되는 긍정적 효과가 있다. 우수집단에 속한 학생들은 일반집단에 속한 우수학생들보다 학교생활의 만족도가 높다고 알려져 있다(Jin &Moon, 2006). 그러나 우수집단에 속한 학생들은 과도한 성취 경쟁과 처음 경험하는 성적 하락을동시에 경험하게 된다. 고등학교까지 우수한 학업성적에 근거해서자아정체감을 형성하였다가, 대학에 와서 자신의 상대적 위치가 달라지면 정체감의 혼란과 심리적 어려움을 경험할 수 있다. 학업우수학생들은 학업적 실패에 대해서는 높은 내성을 갖고 있지만 감정요인에 있어서는 일반학생들보다 더 취약하다(김영빈, 2011; 윤여홍, 2000; Webb et al., 1982). Zeidner &Schleyer (1999)는 학업우수학생들로만 구성된 집단의 학생들은 다양한 성적의 학생들로 구성된 일반집단의 학업우수학생들에 비해 시험불안이 높고, 사회적적응에 어려움이 있을 때 더 쉽게 상처를 받는다고 하였다. 선행연구에서는 학업우수학생들 중 15-20%는 심리적 어려움을 경험하는 것으로 나타났고, 그 주된 이유는 성공에 대한 압박감과 스트레스에 의한 것이었다(Kaiser &Berndt, 1985).


    학업우수학생들은 스스로 기준을 높게 세울 뿐 아니라 주변에서도 그러한 성과를 당연하게 여기므로 무엇이든 잘해야 한다는완벽주의에 빠지기 쉽다. 완벽주의는 긍정적 또는 부정적으로 작용하는데, Parker &Mills (1996)는 학문적 영재들이 또래보다 완벽을 추구하는 경향이 있어서 이들이 탁월함을 추구하게 된다고 하였고, LoCiero &Ashby (2000)는 영재가 적응적 완벽주의 성향을가지고 있어서 인지적 성숙도가 높고 과거의 많은 성공경험을 바탕으로 실패를 융통성 있게 수용한다고 하였다. 그러나 이러한 완벽주의는 자기 비판적 성향이 강하고 자신에 대해 엄격하게 평가하므로 실패를 경험하는 것이 자긍심에 심각한 타격을 준다고도 한다(Davis &Rimm, 1998).


    우리나라의 학업우수학생들의 부모는 강한 교육열로 인해 높은기대를 갖고 다양한 방식으로 자녀의 학업에 개입한다. 일반적으로부모의 관여와 높은 기대는 학생들에게 외적인 동기요소로 작용하여 학업에 긍정적 영향을 미칠 수 있지만 동시에 학업에 관련된 스트레스를 높이는 부정적 요소로 작용하는 것으로 알려져 있다. 그러나 우리나라 학생들을 대상으로 한 연구에서 성적이 우수한 학생은 부모의 높은 기대와 관여에도 낮은 스트레스 수준을 유지하는 것으로 나타나서, 우리나라 학업우수학생들에게 부모와의 친밀한 관계와 높은 교육열은 학생이 학업에 집중하도록 하는 긍정적인영향을 하는 것으로 설명된다(김종백과 김준엽, 2009). 그러나 우수한 학업적 동질집단에서 학업성적이 부진한 경우에는 부모와의친밀한 관계가 학업스트레스를 높이는 부정적인 영향을 미칠 수있다.



    결 론


    고등교육의 패러다임이 교수자 중심의 수업에서 학습자 중심의수업으로 옮겨가고 있음에 따라 학생들의 특성을 고려한 교육이대학에서 이루어져야 한다. 이제까지 새로운 세대에 대한 진술들은 이러한 세대인 학생들이 기존의 의학교육의 전통적 학문활동과기대에 잘 맞지 않을 것임을 시사한다. 이들 특성을 기반으로 의과대학과 의학전문대학원에서 신세대가 원하는 교육의 방향을 다음의 세 가지로 제언할 수 있다.


    첫째, 밀레니엄 세대의 개인차를 배려하는 다양한 교수방법이활용되는 교육이 이루어져야 한다. 모두에게 일방적으로 정보를 제공하는 방법으로 진행되는 수업은 새로운 세대의 학생들이 공부에흥미를 잃게 한다. 이들은 획일화가 아닌 다양화를 추구하며, 관습이나 권위적 행동양식에 무조건 따르는 것이 아니라 자기주장과 자존감이 강하므로 남이 걷지 않은 길을 택해 새로운 패러다임을 만들어 나갈 수 있을 것이다. 따라서 그들이 가진 개성과 새로움을 추구함으로써 창의성을 개발하는 교육을 제공하여 개인으로 하여금자아를 실현하고 유용한 지적 가치를 창출하고 활용하는 기회를줄 수 있어야 한다. 권위적이고 통제적인 분위기의 수업이 아니라학습자가 중심이 되어 선택하고 활동하면서 폭넓은 지식을 바탕으로 자유로운 사고와 의술을 연결하게 해주는 교육이 필요하다. 표 1의 Howe와 Strauss가 제시한 밀레니엄 세대의 7가지 특성에 따라Monaco &Martin (2007)이 제시한 수업적용방안은 우리나라의의학교육에도 구체적인 시사점을 줄 것이다.


    둘째, 디지털 기술을 통해서 학습이 이루어지는 것이 필요하다.밀레니엄 세대의 학생들은 인터넷과 모바일을 주된 의사소통 및 정보공유의 수단으로 활용하는 디지털 세대이며, 온라인과 오프라인에서 협력적으로 상호작용하는데 익숙하다. 기존 교실 중심의 교육에서 인터넷과 같은 디지털 매체를 기반으로 한 사이버 공간을적극 활용하여 학생들이 학습장소와 시간의 제약을 벗어나 자기주도적으로 학습하도록 하는 것이 필요하다. 정보의 바다에서 정보를 취사선택하고 고급 정보를 실생활과 밀접히 관련하여 이해하도록 하고, 교실에서는 학생들이 참여하고 상호작용하는 교육을 강화할 때 디지털 세대의 학습역량을 최대로 성장시킬 수 있을 것이다.


    셋째, 학생들의 정서적 발달과 스트레스에 관심을 기울여야 한다. 의과대학에 입학하기 전에는 작은 연못 속의 큰 물고기(a bigfish in a little pond)였던 학생들은 우수한 학생들끼리의 경쟁 속에서 실패를 경험하게 되고 자신이 더 이상 큰 물고기가 아님을 인식할 때 자존감과 그들의 완벽주의에 상처를 입고 좌절하기 쉽다. 따라서 이들이 의과대학에서 경험할 수 있는 심리적 어려움, 실패가능성에 대한 스트레스에 잘 대처할 수 있도록 돕는 정서적 지원이필요하다. 또한, 이들은 친밀한 부모 자녀관계를 형성해왔고 부모의보호와 제제를 받으며 성장했다. 부모들의 과잉보호적인 태도와 자녀의 삶에 대한 직접적인 관여는 학생들을 계속해서 청소년기에 머물도록 만들고 독립적이지 못하게 만든다. 부모의 관여와 보호 속에서 자란 이들에게는 독립적인 역할 수행과 의사결정에 대한 교육을 보다 관심을 가지고 제공해야 한다. 부모의 성적에 대한 과도한 기대는 의학공부에서 우수한 성적을 받지 못하는 학생들의 학업스트레스를 높이는 부정적 역할을 하게 된다. 학생들의 삶과 성적에 계속 관여하려는 부모와 학생의 발달 사이에서 대학은 균형을 잡아나가는 것이 필요할 것이다.


    본 연구는 밀레니엄 세대와 학업우수학생들의 특징을 근거로 의과대학생의 특징을 살펴보고자 하였다. 그러나 현재 의과대학에입학하는 학생들을 대상으로 설문이나 인터뷰 등을 통한 경험적인연구가 함께 이루어지지 못했다는 제한점이 있다. 앞으로 이들의특징을 보다 구체적으로 살펴보기 위한 추후 연구가 이루어질 필요가 있다.






    Current medical students are a distinct new generation who can be distinguished from the previous generation.

    Therefore, a clear understanding of their characteristics is vital in developing an appropriate educational

    program for them. The purpose of this article is to explore the characteristics of the current generation of high

    achieving medical students. Notable characteristics that define this generation include the following: they feel

    they are special, and they are sheltered, confident, highly optimistic, pressured, conventional, and have a strong

    desire to achieve. They are the digital generation, who can obtain information through various forms of technology.

    Furthermore, they are high achieving students in highly competitive educational environments. It has

    been suggested that various teaching methods be used in the medical school classroom. Using digital methods

    could be crucial in providing high-quality medical education. Educators should pay more attention to students’

    psychosocial development and help them to effectively cope with their academic stress.

    Keywords: Millennial student, New generation, High achievement student.

    외국 의과대학에서의 진로탐색 및 진로지도 프로그램

    The Career Counseling Program in Medical Schools outside Korea

    정은경

    전남대학교 의과대학ㆍ의학전문대학원 의학교육학교실

    Eun-Kyung Chung

    Department of Medical Education, Chonnam National University Medical School, Gwangju, Korea



    서 론

    흔히 의과대학에 입학하면 진로가 결정되었다고 생각할 수 있지만 4년 또는 6년간의 의학공부를 마친 후 의과대학생들은 구체적인 진로를 선택하여야 한다. 예를 들어, 기초의학을 할지 임상의학을 할지 그리고 세부적인 전공 분야도 결정해야 한다. 이 외에도 의학전문기자, 의료소송 전문변호사, 해외의료봉사활동, 의학연구원등 다양한 진로를 선택할 수 있다(Hur &Lee, 2012). 자신에게 적합한 진로를 선택하는 것은 의과대학생들에게는 중요한 의사결정 중의 하나이며, 진로 미결정은 학생들에게 불안과 스트레스를 야기하는 주요한 원인이 될 수 있다(Savickas et al., 1985). 더욱이 최근진로탐색의 기회가 되었던 인턴제도의 폐지가 논의되면서 의과대학에서 졸업 후 진로지도교육이 필요하다는 인식이 확산되고 있다.


    의과대학생의 진로탐색에 대한 국내 연구에서 84.8%의 의과대학생이 대학에서의 진로지도가 필요하다고 답하였으며, 특히 자신의 적성에 맞는 전문과목 선택방법이 진로지도의 중요한 교육내용이 되어야 한다고 하였다(Chung et al., 2001). 졸업 후 선택하게 될진로에 대한 선호도를 조사한 연구들(Kang et al., 2000; Lee et al.,2009)도 있었는데, 입학 학생들의 특성에 관계없이 내과에 대한 선호도가 가장 높음을 알 수 있었다. 또한 진로선택변인에 대한 연구들의 결과를 보면, 의과대학생들은 의과대학생활에서의 경험, 성격적 요인, 교육과정에서 배운 내용, 임상실습에서의 경험, 주변 사람들(가족, 친지, 교수 등)의 권유, 자신이나 가까운 사람의 질병 등 다양한 이유로 전공을 선택하게 된다(Jung et al., 1999; Lee et al., 2009;Lim &Cho, 2002). 이처럼 국내에서는 지금까지 일부 의과대학생들을 대상으로 의과대학생들이 졸업 후 선택하게 될 진로에 대한선호도나 진로선택변인에 대한 연구들이 주로 이루어져 왔을 뿐,구체적인 진로탐색 및 지도 프로그램이나 교육과정을 제시하지 못한 한계가 있다.


    반면 국외 연구에서는 의과대학생들의 진로에 대한 선호도나 선택과 관련된 요인에 대해 국가 단위의 설문조사를 통해 조사하거나(Lefevre et al., 2010), 시간에 따른 변화를 파악하는 등(Svirko etal., 2013) 다양하게 접근한 연구들이 많았다. 또한 미국의과대학협회(Association of American Medical Colleges)에서는 Careers in Medicine (CiM)이라는 프로그램을 제공하고 있으며, 미국의 의과대학들은 CiM을 각 대학의 특성에 맞춰 운영하고 그 결과를 발표하고 있다(Sweeney et al., 2012; Zink et al., 2007). 이 연구에서는 외국 의과대학에서의 진로탐색 및 진로지도 프로그램을 고찰하여 향후 우리나라 의과대학에서의 졸업 후 진로교육을 위한 유용한 기초자료를 제공하고자 하였다.


    미국의과대학협회의 Careers in Medicine과 운영사례


    미국의과대학협회에서 CiM이라는 프로그램을 제공하고 있는데, 이는 의과대학생들에게 자신에게 적합한 전공 분야를 선택하기 위한 기술, 정보, 자원들을 제공하는 포괄적인 진로지도 프로그램이다. CiM에서는 의과대학생들에게 적합한 전공 분야를 결정하기 위한 4년에 걸친, 4단계의 진로계획과정(career-planning process)을 제시하고 있다(Figure 1). 좀 더 구체적으로 CiM은 전공선택을 효과적으로 결정하기 위해 의과대학생 자신의 개인적 특징및 선호도를 평가할 수 있도록 하고 전공 분야에 대한 정보를 제공하여 최종적으로 자신에게 적합한 전공 분야를 선택하게 하는 것으로 각 단계별 개요는 Table 1과 같다.






    진로계획과정의 첫 번째 단계는 자기 자신 이해하기이다. 이 단계에서는 전공 분야 미결정(specialty indecision), 흥미, 가치관, 성격등에 대한 자가평가활동들이 제공된다.

    • 먼저 전공 분야 미결정을 자가평가하는 specialty indecision scale는 35개의 문항으로 구성된측정도구로서 자신이 의사결정과정 단계 중 어디에 위치하는지를보여주고 전공 분야 선택에 대한 걱정이나 문제를 극복하는 방법을찾을 수 있도록 돕는다. 
    • 흥미(interest)는 medical specialty preferenceinventory를 통해 측정되는데, 이는 의과대학생들이 향후 선택할 가능성이 있는 16개의 주요 전공 분야에 대한 선호도를 제시한다. 
    • 가치관(values)은 physician values in practice scale (PVIPS)를 통해 측정되며, PVIPS는 6개의 핵심 가치, 즉 자율성(autonomy),관리(management), 명성(prestige), 봉사(service), 생활방식(lifestyle), 학문적 추구(scholarly pursuits)에 대한 60개의 문항으로 구성되어 있다. 
    • 마지막으로 성격(personality)은 Myers-Briggs type indicator나 Keirsey Temperament Sorter 중 한 가지를 이용하여 자가평가된다. 각 개인의 전공 분야 미결정, 흥미, 가치관, 그리고 성격 등에 대한 기록에 의거하여 가장 적합한 전공 분야를 제시하며, 이러한 기록에는 당사자만 접근할 수 있도록 되어 있다.


    두 번째 단계는 전공 분야에 대한 탐색과정이다. 이 단계에서는전공 분야에 대한 정보를 수집할 수 있는 방법을 알려주고, 주요 전공 분야의 수련과정뿐만 아니라 임상의사 이외의 다른 진로에 대해서도 소개한다. 특히 CiM에서는 125개 이상의 전공 분야에 대해각각 전공 분야의 업무, 전공의 프로그램, 경쟁률, 월급 등 상세한정보를 제공한다. 전공 분야에 대한 탐색과정을 통해 의과대학생들로 하여금 선택하고자 하는 전공 분야의 폭을 좁히고, 관심 전공 분야에 대해 좀 더 알아볼 수 있는 선택실습 등의 경험을 갖도록 한다.


    세 번째 단계는 첫 번째와 두 번째 단계를 통해 얻은 자기 자신과관심 전공 분야에 대한 정보를 통합하여 전공 분야를 선택하는 것이다. CiM에서는 의과대학생에게 가장 적합한 전공 분야 선택을돕기 위해 의과대학생이 고려하고 있는 몇 개의 전공 분야를 비교한 의사결정과정모형을 제공한다.


    마지막 단계는 전공의 과정을 준비하는 것이다. 이 과정은 의과대학생들에게 전공의 지원에 필요한 정보를 제공하는데, 구체적으로는 전공의 지원할 때 필요한 이력서, 자기소개서, 추천서 작성방법 그리고 면접 및 매칭 시스템에 대해 소개한다.


    CiM은 각 의과대학의 교수나 교직원들이 의과대학생들의 진로탐색 및 지도를 돕는 프로그램을 시행할 수 있도록 필요한 자원들을 제공하고 있으며, 미국의 의과대학들은 CiM을 각 대학의 특성에 맞춰 운영하고 있다.



    1. 미시건 의과대학의 진로개발 프로그램(Career Development Program)


    미시건 의과대학의 Career Development Program (CDP)은 미국의과대학협회의 CiM에서 제시하고 있는 4단계 프로그램에 맞춰평가(assessment), 진로탐색(career exploration), 진로선택(careerselection), 시행(implementation)의 4단계로 구성되어 있다

    • 제1단계에서는 의학과 1학년과 2학년 학생들에게 전문 상담사의 지도하에 자가평가를 실시한다. 
    • 제2단계인 전공 분야 탐색과정으로서 의학과 1학년부터 2학년 학생들을 대상으로 점심시간을 이용하여 전공 분야에 대한 세미나를 개최하고, 의학과 2학년부터 4학년 학생들에게는 전공 분야의 전문가와 함께 그 분야에 대해 좀 더 체험할수 있는 섀도우 프로그램(shadow program)과 연구참여 프로그램을 운영한다.
    •  제3단계에서는 의학과 3학년 학생들을 대상으로 진로지도교수(faculty career advisor)와 함께 자신에게 적합한 전공분야를 선택하는 의사결정과정을 제공한다. 
    • 제4단계에서는 특히전공의 지원과 관련하여 어려움을 가진 학생들을 지원하는 위험군관리(at-risk plan) 프로그램을 제공한다. 


    CDP의 다양한 프로그램중 진로선택, 전공의 과정 지원준비, 면접 그리고 CDP 프로그램 평가에는 의과대학생들이 반드시 참여하도록 의무화하고 있다. Zinket al. (2007)은 졸업생 설문조사로부터 미시건 의과대학생들이 교수의 멘토링, 진로와 관련된 자가평가활동, 전공 분야에 대한 정보등 CDP에 대한 만족도가 점점 증가하고 있고, 다른 의과대학의 만족도 점수보다 통계적으로 유의하게 높다고 보고하고 있다.



    2. 벤더빌트 의과대학의 Careers in Medicine


    벤더빌트 의과대학의 CiM은 의과대학생들이 4년에 걸쳐 전공분야를 탐색할 수 있는 구조를 구축한다는 목적으로 2005년에 도입되었다. 이 CiM의 특징 중 하나는 전공탐색 기회를 의학과 1학년때부터 제공한다는 것이다. 의학과 1학년 학생들은 학기 초에 전공분야와의 만남(specialty speed dating)을 통해 각 전공 분야의 교수 및 전공의들과 5분 내외의 짧은 만남을 가짐으로써 조기에 다양한 전공 분야를 경험할 수 있게 한다. 또한 전공 분야 박람회(specialtyfair)에서는 각 전공과의 의사, 전공의, 의학과 4학년 학생들이 전공과목에 대해 홍보하고, 의학과 2학년 학생들은 관심 전공분야에 대해 좀 더 알아볼 수 있을 뿐만 아니라 의학과 3학년 임상실습 시 어떤 과목을 등록할지 결정하게 된다. 백스테이지 패스 프로그램(backstage pass elective)은 다양한 전공 분야의 의사 패널과 토의할 수 있는 수업과 전공 분야의 의사와 함께 그 분야에 대해좀 더 체험할 수 있는 섀도우 프로그램으로 구성되며, 학기당 30명정도의 의학과 1학년과 2학년 학생들을 대상으로 운영되고 있다.또 다른 벤더빌트 의과대학 CiM의 특징은 학생 대표들을 CiM 운영위원회의 위원으로 포함시켜 CiM 개발부터 평가에 이르기까지학생들의 의견을 적극적으로 수용하고, 관심 전공 분야에 대해 학생 스스로 조사하고 정보를 공유할 수 있는 학생자조모임을 활성화시키는 등 학생 참여를 최대화시켰다는 것이다. Sweeney et al.(2012)은 벤더빌트 의과대학에 CiM이 도입된 2005년 이후 졸업생들의 진로지도 프로그램에 대한 전반적인 만족도와 유용성 평가점수가 다른 의과대학보다 통계적으로 유의하게 높다고 보고하였다.


    의과대학 자체 개발 진로탐색 프로그램의 운영사례


    1. 메이오 의과대학과 브리티시 콜롬비아 대학의 진로탐색을위한 선택실습과정


    메이오 클리닉(Mayo Clinic)에서는 의학과 1학년과 2학년 학생들을 대상으로 전공탐색, 연구, 그리고 사회봉사를 할 수 있는 선택실습과정을 운영하고 있다. Keating et al. (2013)은 전공 분야를 선택하는 데 있어서 임상의학교육과정을 받기 전의 경험이 중요하다고 강조하고, 소아과에서 일주일 동안 소아과에 대한 소개, 소아과학과 관련된 이슈에 대한 워크숍, 패널 토의, 그리고 섀도우 프로그램 등을 운영한 경험을 소개하였다. 의학과 1학년과 2학년 학생들 중25% 이상이 자발적으로 참여하였으며, 참여한 학생들은 소아과에대한 지식을 넓힐 수 있었고 흥미를 높일 수 있었다고 평가하였다.


    브리티시 콜롬비아 대학(University of British Columbia)에서는의학과 3학년 학생들을 대상으로 2주간의 선택임상실습과정을 운영하고 있는데, 이는 정규임상실습과정과는 달리 외래 또는 지역사회에서 전공 분야와 관련된 경험을 하도록 권장하고 있다. Mihalynuket al. (2006)은 선택임상실습과정이 전공 분야를 결정하는 데영향을 미친다고 보고하였다.



    2. UCLA 의과대학의 진로탐색 칼리지 프로그램


    의학과 4학년 때의 교육과정이 다소 느슨하다는 지적에 따라UCLA에서는 의학과 4학년 학생들을 대상으로 하는 칼리지 프로그램(college program)이라는 멘토링 프로그램을 운영하고 있다.칼리지(college)란 교육과정 운영 및 실행을 위해 관련성이 있거나비슷한 특성을 갖는 전공 분야들끼리 묶는 구조를 말하며, UCLA에서는 6개로 구성하였다. 학생들은 의학과 4학년 교육과정이 시작될 때 1주일 동안 6개의 칼리지에 대해 다양한 방법으로 소개받고서브인턴십에 대비하게 된다. 그 후 칼리지 책임교수는 멘토로서 지속적으로 저녁식사시간을 이용하여, 최신 의학지식 및 술기 관련교육부터 전공의 지원과정 설명까지 다양한 주제의 세미나를 개최한다. Coates et al. (2008)은 학생들이 의학과 4학년 때의 자유가 침범된다고 불평할 것으로 생각했으나, 학생들이 멘토와 좋은 관계를발전시키고 유지할 수 있었고 전공의 과정을 준비하는 데도 도움이되었다고 응답한 결과를 보고하였다.



    결 론


    자신에게 적합한 진로를 선택하는 것은 의과대학생들에게는 중요한 의사결정 중의 하나이다. 최근 진로탐색의 기회가 되었던 인턴제도의 폐지가 논의되면서 의과대학에서 졸업 후 진로지도교육이필요하다는 인식이 확산되고 있다. 그러나 국내에서는 지금까지 일부 의과대학 학생들을 대상으로 의과대학생들이 졸업 후 선택하게될 진로에 대한 선호도나 진로선택 변인에 대한 연구들이 주로 이루어져 왔을 뿐, 구체적인 진로탐색 및 지도 프로그램이나 교육과정을 제시하지 못한 한계가 있다. 이에 이 연구에서는 의과대학생을 위한 진로탐색 및 진로지도에 관한 국외 사례를 고찰하여 향후의과대학에서 졸업 후 진로지도를 위한 유용한 기초자료를 제공하고자 하였다.


    미국의과대학협회의 CiM은 의과대학생들에게 자신에게 적합한 전공 분야를 선택하기 위한 기술, 정보, 자원들을 제공하는 포괄적인 진로지도 프로그램이다. CiM에서는 4년에 걸친, 4단계의 진로탐색 및 지도과정을 제시하고 있는데, 좀 더 구체적으로 CiM은 전공선택에 대해 효과적으로 결정하기 위해 의과대학생 자신의 개인적 특징 및 선호도를 평가할 수 있도록 하고 전공 분야에 대한 정보를 제공하여 최종적으로 자신에게 적합한 전공 분야를 선택하게하는 것이다. 미국의 의과대학들은 CiM을 각 대학의 특성에 맞춰운영하고 있다. 특히 미시건 의과대학에서는 전공의 지원과 관련하여 어려움을 가진 의학과 4학년 학생들을 지원하는 위험군 관리(at-risk plan) 프로그램을 제공하고 있고, 벤더빌트 의과대학에서는 학생 대표들을 CiM 운영위원회의 위원을 포함시켜 CiM 개발부터 평가에 이르기까지 학생들의 의견을 적극적으로 수용하고, 관심 전공 분야에 대해 학생 스스로 조사하고 정보를 공유할 수 있는학생 자조 모임을 활성화시키는 등 학생 참여를 최대화시켰다. 그외에도 메이오 클리닉과 브리티시 콜롬비아 대학의 선택 실습 과정이나 UCLA의 멘토링 프로그램 등도 의과대학생의 진로탐색 및 선택에 도움이 되었다고 보고되었다.


    향후 미국의과대학협회의 CiM에서 보듯이 의과대학 혹은 의학전문대학원을 졸업한 학생들이 자신에게 적합한 전공 분야를 미리발견하기 위해서는 전 학년에 걸친 체계적인 진로지도 시스템이 도입되어야 할 것이다. 즉, 학년에 따라 학생들에게 다각적인 진로정보가 제공되고, 다양한 진로와 관련된 경험을 할 수 있도록 시스템이 체계화되어야 할 필요가 있다. 또한 의과대학생을 위한 진로지도 시스템에 대한 양적 및 질적 평가가 이루어지고, 그 결과에 의거하여 그 경험이 공유되고 확산되어야 할 것이다.





    Medical students can choose to pursue any of a large number of specialties. This diversity reflects exciting opportunities,

    yet it also present significant challenges, such as providing medical students with adequate resources

    and guidance to help them to make informed career decisions. Additionally, because the medical internship

    will be abolished in the near future, many Korean medical schools have recently focused on implementing

    a career planning and advising program. This paper describes the Careers in Medicine (CiM) program

    offered by the Association of American Medical Colleges as a framework for other schools to adopt or adapt as

    they consider the best ways to address the career counseling needs of their own students. CiM is a comprehensive

    career planning program that provides students with the skills, information, and resources to choose a specialty

    and residency program that meets their career goals. CiM follows a four-year, four-step career planning

    process including self-understanding, exploring a variety of medical careers, and finally choosing a specialty.

    The CiM program has been evaluated as successful because of widespread participation and positive feedback

    from medical students. The information in this study can be used to develop a formal career advising program

    throughout the four years of medical school.

    Keywords: Career choice, Vocational guidance, Medical school



    의과대학 학생의 소진에 대한 지도사례 분석

    A Case Study on Approaches to Supporting Medical Students with Burnout Experience

    정은경ㆍ최기오ㆍ한의령

    전남대학교 의과대학ㆍ의학전문대학원 의학교육학교실

    Eun-Kyung ChungㆍKee-Oh ChayㆍEui-Ryoung Han

    Department of Medical Education, Chonnam National University Medical School, Gwangju, Korea







    서 론


    소진이란 지나친 업무부담으로 인한 정서적 탈진상태로 정신적․사회적․ 신체적 문제를 불러일으키고 일에 대한 열정이 감소되어자신이 하고 있는 일이 더는 의미 있게 느껴지지 않는 것을 말한다(Maslach et al., 1996). 의과대학 학생들에게는 과도한 학업으로인해 피로감, 좌절감, 학업에 대한 거리감, 스트레스, 정신적 소모감,무력감, 냉소적인 태도 등을 나타내는 학업소진이 중요한 문제로여겨져 왔다(Kim & Jeon, 2008). 학업소진상태가 오래 동안 지속되면 학습된 무기력을 경험하고, 학습된 무기력은 실패경험이 누적되어 지속적으로 학습자에게 부정적인 영향을 미치게 되기 때문이다.따라서 의과대학 학생들의 소진에 관한 연구가 주로 유급제도와연관되어 왔다. 즉 유급생 실태 및 예측요인에 대한 연구(Han etal., 2012; Kim & Jeon, 2008; Park et al., 2009)나 유급생을 대상으로 시행한 상담 및 교육지원 프로그램 및 운영 혹은 그 효과에 대한연구(Kim et al., 2013; Park et al., 2014) 등이 그 예이다. 이 연구들은 전국 대부분 의과대학에서 겪고 있는 유급제도에 대한 교육적인관심을 불러일으키고 유급생에 대한 기초자료와 학습 및 상담시스템과 같은 제도적 개선방안을 제시해주었다는 점에서 그 의미를 찾을수 있다.


    그러나 의과대학 학생들에게 소진의 결과가 유급과 같은 학습적문제뿐만 아니라 정서적 및 사회적 문제들을 유발할 수 있다. 유급이라는 학습결과가 이미 발생된 후보다는 소진을 경험하고 있는 학생들을 조기에 발견하고 학습과 생활에 관한 적극적인 지원을 하는것이 중요하다. Sayer et al. (2002)도 학업실패가 학생 개인의 학습방법이나 경제적인 문제 그리고 사회적, 정서적 문제 등 다양한 원인의종합적인 결과로 발생되며 학생 개인별 원인을 찾아 그에 맞는 맞춤형 지원이 필요함을 강조하였다.


    이 연구는 2010년부터 2013년까지 전남대학교 의과대학․ 의학전문대학원의 학생상담실의 상담을 통해, 혹은 지도교수 및 학부모의 요청으로 학생지도위원회에서 신체적․정서적․정신적 고갈상태인 소진으로 판단된 45명의 학생들을 대상으로 하였다. 학생지도위원회는 정기적인 모임을 통해 담당 학생에 대한 정보를 공유하여공동으로 지도방향 및 내용을 결정하고 학생지원 노력에 대한 효과판정 및 문제에 대한 대처방안을 논의하였다. 이 연구에서는 의과대학 학생들의 소진의 원인 및 지도사례를 분석하여 향후 의과대학학생들의 학교적응 및 정신건강을 위한 적극적인 지원이나 체계를마련하는 데 유용한 기초자료를 제공하고자 하였다.


    의과대학 학생의 소진 원인 및 지도사례 분석


    연구 대상들은 다양한 문제들을 복합적으로 가지고 있는 경우가많았으나, 학생지도위원회의 담당 교수가 작성한 학생상담기록표를토대로 의과대학 학생의 소진의 주요 원인에 따라 대표적인 유형을나누고 지도내용을 정리하였다(Table 1).



    1. 학교에서의 사회적 관계망 결여


    유급과 휴학으로 인해 동기들보다 몇 년이 늦어진데다 심리적위축감 및 자신감 결여로 의과대학 내 친구, 동기 관계에 어려움이있어 의과대학 생활에 흥미를 잃고 학업실패를 반복한다. 담당교수는 유급을 경험한 학생들이 주로 경험하는 동기들과 학년 주류 사회로부터의 분리, 불안에 대한 해결책으로 가장 효과적인 방법은 단짝친구 만들기임을 제시하였다. 단짝 친구를 통해서 대개 비슷한 경험해 본 친구 간에 동병상련의 연대관계가 성립되고 수업 및 식사나휴식시간 등 거의 모든 학교생활을 함께 하며 서로 지지하고 격려하여 학교생활에 잘 적응해 나갈 수 있도록 하였다. 사회성이 많이결여된 사례의 경우 담당교수는 같은 어려움을 겪었던 학생들 중도움을 줄 수 있는 학생들을 선별하여 멘토 역할을 하면서 함께잘 지내고 학습과 생활에 있어서 감시와 돌봄을 제공하도록 하였다.반면에 사회성에 문제가 없는 경우에는 먼저 성격이 비슷하여 말투나 행동에서 불편함이 없는 친구를 골라서 말 걸기, 식사 제안하기,과제물에 대해서 물어보기 등으로 접근하여 가까워지도록 유도하였다. 단짝 친구대상 학생이 이미 다른 사회적 관계망에 속해 있을경우에는 대상 친구에게 개인적으로 접근하여 친해지기보다는 사회적 관계망을 형성하고 있는 그룹의 친구들에게도 접근하여 그 그룹에 새로운 구성원으로 끼어들 수 있도록 지도하였다. 표준화 성격진단검사와 에니어그램을 통해 성격진단검사를 시행하여 학생 자신의성격에 대해 이해하고 특히 친구관계에서 주의할 점에 대해 이야기하였다. 친구관계 및 학교생활 그리고 성적 등에 대해 정기적으로확인하고 학생을 격려하였다.


    2. 정체성 미확립


    의과대학생이기 이전에 20대 또는 30대 초중반의 성인이지만이제까지 별다른 자아정체성 형성 계기를 갖지 못하다가 학업에어려움을 겪으면서 자신감 상실 및 열등감을 느끼게 된다. 부모로부터 독립할 나이가 지났음에도 자신의 일을 스스로 책임지고 노력하는 모습, 즉 삶의 주체의식이 부족하여 학교생활 부적응 및 성적부진을 초래하기도 한다. 일부 학생들은 별다른 이유 없이 정체성 확립이상당히 늦어지는 경우도 관찰되었다. 만약 부모가 강력한 통제력을가진 반면, 학생은 비교적 유순한 경우 부모의 통제에 대해 매우민감하지만 대개 수동적인 공격성과 의도적 태만으로 대항하였고학생의 성격이 강하고 적극적인 경우는 가출 등으로 부모에게 대항하였다. 이 경우 학생은 계속 학업성적에 좌절을 하고 부모도 그때문에 더욱더 학생에 대한 신뢰를 가지지 못해 통제를 더 강화하는쪽으로 행동하며 이에 대해 학생은 더욱더 태만이나 반항으로 대처함으로써 악순환의 고리를 형성하고 있음을 관찰할 수 있었다.특별한 이유 없이 자아정체성 형성이 늦어지는 경우 멘토를 지정하여 멘토와의 정기적인 만남 등을 통해 정체성 확립을 도울 수있다. 또한 남학생들의 경우 군대 입대를 권유하기도 하는데 학습관련 스트레스 상황에서 벗어나 규칙적인 신체활동을 통해 정신적성숙을 꾀할 수 있었다. 또한 군 복무기간에 정신적으로 성숙할시간을 가짐으로써 책임감 있고 현실 분별력 있는 학생이 되어 돌아오는 것으로 생각되었다. 반면에 강력한 통제력을 가진 부모로 인해자아정체성 확립이 되지 않은 경우 담당교수는 부모와의 전화 및면담을 통해 부모가 사사건건 간섭하고, 모든 정보를 통제하는 것을학생이 답답해하고 있음을 알리고, 학생이 스스로 할 수 있도록인정하는 태도를 보여 줄 것을 요청하였다. 또한 관계개선 및 문제해결을 위해 전문가와 함께 가족상담을 받아볼 것을 권유하였다. 자아정체성 확립이 되지 않은 경우 유급과 휴학을 반복하는 경우가 많은데, 이때 부모의 걱정과 학생 자신의 불안도 커져가고 있지만 지금직면한 문제들을 스스로 해결하여 정리하고 학업을 계속할 준비자세를 갖추는 것이 더 중요함을 정기적인 만남을 통해 이야기하였다.


    3. 취약한 정신건강상태


    대인관계에 취약한 정신건강상태, 즉 관계에 대한 피해의식 및원망의 마음이 있고 현실에 대한 왜곡 인지 및 남에게 자신의 문제를투사하는 경향이 있어 동기 및 친구들과의 잦은 문제를 만들고,이로 인한 스트레스로 학업에 집중하기가 어렵다고 호소하는 예이다. 또한 실패경험이 누적되어 학습된 무기력을 경험하고 이는 후속학습수행에서 학습하고자 하는 동기를 상실시키며 정서적인 손상과수행의 저하로 인해 자신 스스로 아주 무능하고 왜소한 존재로 인식하면서 심한 우울과 정서불안에 빠지게 된다.대인관계에 취약한 정신건강상태를 보인 경우 학생뿐만 아니라주변인에 대해서도 면담을 실시하여 인격장애 등이 있는지를 의심해보아야 한다. 이 경우 전문가에게 의뢰하여 학생이 자기 자신에대해 인지하게 되었고 자신의 문제로 인해 대인관계가 어려웠음을인정하게 되었다. 또한 학생 및 가족들에 의해 통제 불가능한 우울상태일 때는 조기에 인지하여 전문가에 의해 치료 및 상담을 받게하였다.


    4. 의과대학 진학의 내적 동기 결여 및 적성 문제


    본인의 의지와는 무관하게 성적이 좋아서 그리고 부모 등 주위의기대나 권유 등의 이유로 의과대학에 온 경우 의과대학 공부에 대한자발적인 동기가 결여되어 있다. 또한 의과대학의 학습과 평가방법이 학생 자신의 적성에 맞지 않다며 의과대학에 들어와서 의과대학공부에 집중하기보다 자신의 관심사를 지속적으로 유지시키거나거기에 시간을 많이 투입하는 경우이다. 이 경우 의과대학의 과도한공부량을 소화하기 위해 통용되는 암기위주의 학습유형에 적응하지못하고 학교의 정책이나 시스템에 대한 저항도 크다. 이런 경우자신의 태만한 태도에서 오는 문제를 회피하며 다른 주제나 학습에대한 자신의 열정이나 학교의 시스템에 대한 문제로 투사하는 경우가 많다. 담당교수는 학생에게 의과대학 진학의 내적 동기가 결여되어 있는지, 적성이 맞지 않아 다른 분야에 관심을 갖는 것인지, 아니면 현재의 스트레스 상황에 대한 회피적 자기변명인지를 자기탐색을통해 명료화하고 회피하거나 합리화하는 심리작용을 직면하게 하였다.


    5. 학업 외의 개인적인 위기


    학업 외의 이성문제와 결혼 및 출산, 그리고 경제적 어려움 등으로공부에 몰두하기가 어렵고 학교생활을 제대로 하지 못하는 경우도있다. 의과대학 과정에서 성적부진 이외의 이유로 일정 기간을 휴학한다는 것이 쉽지 않은 결정이지만 학업을 지속할 수 없다고 생각되는 개인적인 위기상황이라고 판단되는 경우 휴학을 하고 문제를해결할 수 있도록 하며, 학교 측에서 학생에게 지원해 줄 수 있는사항이 있는지 확인한다. 그리고 휴학기간 주로 이메일을 통해 근황을 확인하고 만나서 복학하여 의과대학 생활을 다시 시작할 수 있는지 서로 점검하였다.



    결 론


    이 연구는 비록 학생 상담 전문가는 아니지만 학생을 가장 가까운곳에서 도와줄 수 있는 교수들이 모여 공동으로 학생문제를 진단하고 개인 맞춤형 지원방안을 실행하고 있다는 점에서 의의가 있다.특히 단짝 친구 만들기를 통해 학교생활에 적응할 수 있도록 유도하였는데, 이는 아주 작고 미미한 사회정신적개입도 학교에 대한학생들의 생각과 감정을 변화시키고(Yeager & Walton, 2011), 단짝친구의 지지와 도움은 정서적 안정감과 학교에 대한 긍정적 태도를갖게 한다(Ladd & Kochenderfer, 1996)는 기존의 연구결과와도일치한다.


    어떤 학생들은 학교나 교수의 관심대상이 되는 것에 대한 부담감을 느끼거나 자신의 문제가 노출되는 것을 꺼려하여 자신의 문제들을 드러내지 않는 경우도 있다(Chew-Graham et al., 2003).Redwood & Pollak (2007)에 의하면 신체적 및 정신적으로 스트레스에 취약한 시기나 학생지원 및 상담 등을 필요로 하는 시기에는학생을 이해하기 위한 절차와 함께 학생의 자발적 참여로 이루어진소그룹 규모의 스트레스관리 프로그램을 같이 시행하는 것이 학생스스로 문제가 있는 학생으로 낙인찍힌다고 느끼는 것을 방지할수 있다고 한다. 따라서 이 연구에서는 과중한 학업량과 치열한경쟁으로 인해 학업에 대한 심한 부담감을 느끼고 스트레스를 많이받는 의학과 1학년 모든 학생들을 대상으로 학생탐색을 위한 상담뿐만 아니라 성격검사를 먼저 실시하였다. 그리고 개인 및 그룹상담을통해 혹은 가족상담을 통해 학습과 생활에서 어려움을 겪고 있다고판단되는 학생들을 조기에 발견하고 학생지도위원회에서 꾸준한상담과 지도를 시행하였다.


    의과대학 교수들은 학생들을 가르치는 일 이외의 학습지원 및학생상담에 관해서는 경험이 많지 않으므로 이에 대한 교수개발이중요하다(Vogan et al., 2014). 이 연구에서는 교수들이 모여 학생및 지도내용에 대한 정보를 공유하고, 어려운 사례에 대해서는 협력하면서 학생지원 및 상담의 중요성과 기술을 익히게 되었으며,Steinert et al. (2010)도 이 방법이 효과적이라고 하였다. 최근 각의과대학에서는 학생지도 및 지원을 위한 멘토 양성 프로그램에관심이 많은데 일회성의 세미나나 워크숍 등을 통한 교수개발 프로그램보다는 주기적인 만남 및 실제적인 사례관리를 통한 교수개발프로그램이 활성화되어야 할 것이다. Dyrbye et al. (2010)은 교수와의 의미 있는 상호작용은 의과대학 학생들의 소진으로부터의 회복력을 증가시킬 수 있다고 보고하였다. 긍정적인 변화는 학생들뿐만아니라 교수들에게도 나타나는데, 참여한 교수들은 소진을 경험한학생들의 지도에 있어서 교수와 학생 간의 동반자적 상호작용이중요하다는 것을 인식하게 되었고 학생들이 머물지 않고 계속 성장하고 성숙하고 있다는 것을 느끼게 되었다. 향후에는 학습지원 및학생상담과정에 참여한 교수들의 인식 변화 및 학생들에게 미치는영향 등에 관한 연구가 필요할 것이다.


    이 연구는 짧은 기간의 많지 않은 상담 및 지도사례를 제시한것이지만, 학교생활 및 학습에 있어서 어려움을 경험하고 있는 학생들에게 관심이나 학습적인 지원뿐만 아니라 신체적, 정신적, 사회적등 다차원적 접근을 하고자 노력했다는 점과 공통의 관심을 갖고있는 교수들의 주기적인 만남을 통해 실제적인 학생관리 및 지원과교수개발이 동시에 이루어졌다는 점이 긍정적으로 평가될 수 있을것이다.






    Medical students often need high levels of specialized institutional and personal support to recover from

    a burnout experience and to prevent student failure. This study aimed to present approaches to supporting

    medical students who had experienced burnout using case studies. The study subjects were 45 burnout

    cases who had been supported by the student support system and committee in one medical school. The

    factors associated with burnout include poor social network: the lack of belongingness, immature sense

    of identity, poor mental health, a lack of intrinsic motivation and aptitude, or a major personal crisis. We

    summarized some tips according to the factors associated with burnout. These tips should provide guidance

    to organizations and individual support providers to encourage the best quality support for medical students

    with burnout experiences.

    Keywords: Students, Medical, Burnout, Case reports


    의과대학ㆍ의학전문대학원생의 학업소진 양상과 관련 변인들과의 관계

    Relationship between Academic Burnout of Medical and Graduate Students and Related Variables

    천경희

    영남대학교 의과대학 의료인문학교실

    Kyung Hee Chun

    Department of Medical Humanities, Yeungnam University College of Medicine, Daegu, Korea



    서 론


    대학생들의 주요 발달과업은 독립을 위한 학업성취와 취업준비이며, 이 시기의 대학생들은 사회인으로서의 교양과 전공학문에 대한 기본적인 지식뿐만 아니라 첫 직업선택을 위한 다양한 사회 ․경제적 경험과 기회를 가져야 한다. 또한 대학생활 경험을 통해새로운 문화 및 생활양식을 정립해야 하고, 자기 자신이 당면한심리적, 사회적, 개인적 문제를 자력으로 대처해 나갈 수 있는 능력을습득해야 한다(Baker & Siryk, 1984). 그러나 우리나라의 대학생들은 그러한 발달과업 달성과정에서 심한 학업스트레스와 취업스트레스를 경험하게 되며, 사회적, 환경적 요인들에 의한 과도한 경쟁과취업난은 학업 및 취업스트레스를 가중시키고 대학생들의 정신건강을 위협하기도 한다. 의과대학생(이후 의대생으로 표기)과 의학전문대학원생(이후 의전원생으로 표기)들은 진학과 전문직 종사에 대한목적이 비교적 명확하여 취업스트레스나 진로에 대한 불확실성이다소 적지만, 고성취자 집단에서 발생하는 상대적 실패경험이나 학업부담 및 과다경쟁에 의한 스트레스를 경험하게 된다. Lee et al.(2013)에 따르면 의전원생들은 학업수행에 있어 과도한 학습량, 잦은 시험 및 실습스트레스 노출에 의한 우울을 쉽게 경험하게 되며,Lee et al. (2012)은 1개 대학의 의대생 중 14.1%가 경도우울증을보였으며, 고학년 학생들의 우울 정도가 심각한 것으로 보고한 바있다.


    의대생과 의전원생들이 주로 겪게 되는 학업에 의한 스트레스는학교에서의 공부나 성적으로 인해 학업이 매우 힘들거나 싫고 심지어 귀찮다고 생각하여 겪게 되는 정신적 부담, 긴장, 근심, 공포,우울, 초조감 등의 편치 않은 심리상태와 함께 정신적․신체적 증상을 유발하는 것으로 알려져 있다(Lee, 2012). 대부분의 학생들은이러한 학업스트레스를 나름의 방식으로 대처하게 되는데, 주로 스트레스를 유발하는 개인의 행동을 변화시키거나 환경자극 자체를변화시키려고 노력하는 문제중심적 대처와 스트레스 자극으로 유발된 부정적 정서반응을 조절하는 정서중심적 대처를 하게 된다(Yoo& Shin, 2013). 그러한 학업스트레스에 대하여 적응적으로 대처하지 못하여 부적응적인 결과들이 만성화 될 경우 신체적으로 지치고,학업에 대해 무능감과 냉담을 느끼며, 학업수행에 있어서의 문제를경험하게 될 수 있는데 이러한 현상을 학업소진(academic burnout)이라 한다. 즉 학업소진은 과도한 학업으로 인해 피로감, 좌절감,학업에 대한 거리감, 스트레스, 정신적 소모감, 무력감, 냉소적인태도 등을 나타내는 신체적․ 정서적 ․ 정신적 고갈상태이다(Go,2012; Lee et al., 2009; Park & Kim, 2008).


    이러한 학업소진의 하위요인은 일에서의 소진에 기반하여 3개요인으로 정의된다. 최초로 소진 척도를 개발한 Maslach & Jackson(1981)은 소진의 3개 하위요인을 정서적 고갈(emotionalexhaustion), 비인간화(depersonalization), 개인적 성취감의 저하(diminished personal accomplishment)로 정의하였다. 

      • 정서적 고갈은 업무에 대한 과도한 심리․정서적 요구로 인해 에너지의 손실과좌절, 긴장을 느끼는 상태이며, 
      • 비인간화는 관계로부터 분리되고정서적으로 무감각해지며 자신이나 동료에 둔하거나 냉소적인 태도를 보이게 되는 상태이고, 
      • 마지막으로 개인적 성취감의 저하란 자신에 대해 부정적으로 평가하고 자신감이 결여되어 무능감이 증가하는상태이다(Yang, 2004). 

    이러한 소진의 선행변수는 직무과부하를포함하는 직무부담과 사회적 지원을 포함하는 직무자원이 대표적인것으로 알려져 있으며, 직무소진의 결과는 이직, 결근, 직무몰입저하, 직무불만족 등 조직 생산성을 낮추는 직무관련 문제들로 나타난다(Shin, 2003). 학업에서의 소진도 산업 및 조직현장에서의 소진과 마찬가지로 정서적 고갈(emotional exhaustion), 냉소성(cynicism) 그리고 효능감(efficacy)의 저하 즉 무능력감의 3개 요인으로 구성된다(Schaufeli et al., 2002). 학업소진이 지속되면 학습된무기력과 동기 상실, 정서 손상, 수행 저하, 우울을 일으키는 것으로알려져 있다. 또한 학업소진의 결과는 성취감의 저하를 넘어 유급,자퇴와 같은 학교생활 부적응뿐만 아니라 우울 및 자살과 같은 심리․정서적 문제를 유발하며(Koeske & Koeske, 1991), 따라서 학업성취의 하락이나 학업중단 및 학생들의 정신건강을 위하여 학업 스트레스의 관리 및 학업소진을 예방하는 노력이 요구된다.


    학업소진과 관련한 연구들은 학업소진의 양상과 원인들을 주로탐색하고 있으며, 학업소진은 인구통계학적 원인, 환경적 원인, 그리고 성격적 원인에 의해 일어나는 것으로 알려져 있다(Lee, 2012).국외에서 이루어진 대표적인 학업소진 척도의 개발 연구와 관련변인들과의 관계연구는 그 대상이 주로 대학생이었던 반면 국내학업소진 연구는 초 ․ 중․ 고등학생을 대상으로 이루어져 왔다(Hong, 2010; Jo & Lee, 2010; Lee, 2012; Park et al., 2010).이들 연구 결과에 따르면 다수의 연구에서 남학생보다 여학생이학업소진을 더 경험하는 것으로 보고하고 있으며, 성적 및 학업부담이 높을수록 학업소진이 더 잘 일어나는 것으로 알려져 있다(Kimet al., 2010; Salmela-Arol et al., 2008). 또한 국내 초․중․고등학생대상 연구에서는 연령 증가 혹은 학년 증가에 따라 학업소진이 증가하는 것으로 나타났다(Lee, 2012; Park et al., 2010; Park & Kim,2008).


    학업소진에 영향을 미치는 성격요인에 대한 연구들에 따르면 내외통제와 대처전략의 유연성(Gan et al., 2007), 완벽주의(Jo & Lee,2010), 실패내성(Go, 2012)뿐만 아니라 외향성, 성실성, 정서적 안정성(Park et al., 2010)과 같은 성격특성도 학업소진에 영향을 미치는 것으로 나타난다. 또한 소진의 발생과정과 관련하여 Leiter (1989)는 과도한 직무부담과 사회적 지지의 저하와 같은 환경적 요인에의해 정서적 고갈이 발생하고, 이에 따른 스트레스 대처전략으로서의 비인격화 즉 냉소적 태도가 증가하게 되며, 이러한 태도가 효능감과 성취감을 저하시킨다고 주장한 바 있다(Lee, 2012). 최근에는국내에서도 대학생들을 중심으로 한 학업소진 연구가 다수 진행되었으나(Go, 2012; Jung, 2013; Yoo & Shin, 2013), 의대생 및 의전원생을 대상으로 한 학업소진 연구는 매우 드물다. 최근 382명의 의대생을 대상으로 Maslach burnout inventory students survey(MBI-SS)의 국내 타당화 연구를 시행한 Lee & Lee (2013)의 연구가대표적이다. 그 연구에서는 고갈, 냉소성, 무능력감의 3개 요인 구조를 확인하였으며, 우울, 학업적 자기효능감 척도를 활용한 공인 타당도 검증결과, 학업소진을 높게 지각하는 학생들의 우울감이 높고,학업적 효능감이 낮은 것으로 나타났다.


    본 연구는 학업소진의 정의와 원인 및 결과에 대한 연구들에 기반하여 의대생과 의전원생들의 학업소진 경험 및 학업소진과 관련변인들과의 관계를 탐색해보고자 하였다. 이를 위하여 양적 연구와질적 연구를 함께 진행하는 병렬적 통합설계(parallel mixeddesigns)방식의 통합연구방법(mixed method research)을 적용하였다. 특히 의대생과 의전원생의 학업소진에 영향을 미칠 수 있는인구통계학적 특성으로 성별과 연령, 학년을 살펴보았으며, 환경적특성으로는 학업부담 정도를, 성격특성으로는 완벽주의와 실패내성을 살펴보았다. 또한 14명의 학생들을 대상으로 포커스 그룹 인터뷰(focus group interview, FGI)를 시행하여 의대생과 의전원생의 학업소진 경험과 그 시기별 특성에 대하여 탐색해 보았다.


    본 연구의 연구과제는 다음과 같다. 1) 연구과제 1: 의대생과의전원생의 인구통계학적 특성(성별, 연령, 소속, 학년)에 따른 학업소진의 정도는 어떠한가? 2) 연구과제 2: 의대생과 의전원생의 학업소진에 대하여 환경적 요인(학업부담)과 성격적 요인(완벽주의 성향, 학업적 실패내성)의 영향력은 어떠한가? 3) 연구과제 3: 의대생과 의전원생이 경험하는 학업스트레스와 학업소진의 양상 및 특성은어떠한가?


    연구대상 및 방법


    1. 연구대상자


    본 연구는 대구소재 영남대학교 의과대학․의학전문대학원 학생들을 대상으로 이루어졌으며, 재학생 311명이 학업소진 설문에, 14명이 FGI에 참여하였다. 조사대상자의 63.78%가 남학생이고36.21%는 여학생이었으며, 조사대상자의 연령 평균은 만 25.49세였다. 조사대상자의 50% (151명)는 의대생이었으며, 나머지 50%(151명)는 의전원생이었다. 조사대상자의 성별, 학년별 빈도는Table 1과 같다. FGI에 참여한 학생들은 의학과 2학년 학생 7명(여학생 3명, 남학생 4명)과 4학년 학생 7명(여학생 2명, 남학생 5명)등 총 14명의 학생들이었으며, 학생대표의 추천 후 인터뷰에 사전동의한 학생들로 연령평균이 각각 25.83세와 26.42세였다.


    2. 연구도구


    FGI 참석자들의 기본 정보를 수집하기 위한 개별조사표와 전체인터뷰 진행 및 질문항목에 대한 가이드라인을 사전 제작하였다.인터뷰는 연구목적 및 인터뷰 소개, 연구 참여 여부 확인 및 녹음동의 확인, 의과대학 진학동기 및 진학 후 경험, 학업소진 및 유사경험의 원인과 결과, 소진 시 경험한 정서와 극복과정, 이후 적응및 예방방안 등의 순으로 진행되었다. 이러한 가이드라인 제작을위해 2명의 교육학 전문가와 1명의 심리측정 전문가가 참여하였으며, 1회의 시뮬레이션을 통한 수정 후 실제 FGI가 진행되었다.


    양적 연구를 위하여 학업소진 척도, 학업부담 척도, 완벽주의척도, 실패내성 척도가 사용되었으며, 조사 참여자의 유급 여부 및기타 인구통계학적 특성도 수집하였다. 먼저 학업소진을 측정하기위하여 Schaufeli et al. (2002)의 연구에서 사용된 MBI-SS에 기반하여 Lee & Lee (2013), Jung (2013)의 연구에서 사용된 문항들을재검토하여 2명의 교육학 전문가와 1명의 영문학 전공자, 1명의심리측정 전문가가 협의를 통하여 문항들을 수정하였다. MBI-SS척도는 고갈, 냉소성, 무능력감의 3개 하위요인, 총 15개 문항 7점Likert 척도로 구성된다. 척도의 신뢰도는 Table 2에 제시된다.


        • 완벽주의 성향을 측정하기 위하여 다차원적 완벽성 척도(Han,1993)를 사용하였다. 이 척도는 자기지향, 타인지향, 사회부과의3개 완벽주의 차원으로 구성되며, 총 45개 문항 5점 Likert 척도로구성된다. 자기지향 완벽주의란 스스로에게 높은 기준을 부여하고완벽해지기 위해 노력하는 성향을, 타인지향 완벽주의란 중요하다고 여기는 타인에게 완벽함을 기대하는 성향을, 사회부과 완벽주의는 타인이 자신에게 완벽하기를 요구한다고 여기고 이를 만족시켜야한다고 믿는 성향을 의미한다(Shin & Lee, 2010). 문항 제외 시신뢰도가 높아지는 7개 문항이 제외되었으며, 본 연구에서 사용된완벽주의 척도의 신뢰도는 Table 2에 제시된다.
        • 의대생의 학업적 실패내성 정도를 측정하기 위하여 Kim (2002)이 개발한 학업적 실패내성 척도를 사용하였으며, 이 척도는 감정(8문항), 행동(8문항), 과제수준선호(6문항)의 3개 요인, 총 22개 문항과 6점 Likert 척도로 구성된다. 
        • 학업적 실패내성은 학습자가 학업에서의 실패경험에 대하여 건설적인 태도로 반응하는 경향성(Clifford,1984; Kim &e Clifford, 1988)을 의미하며, 각 하위요인은 실패경험후에 보이는 감정적 반응, 실패를 만회하기 위한 계획수립 및 극복방안을 강구하는 경향성과 실패극복을 위해 어떤 행동을 취하는지,그리고 실패 가능성에도 불구하고 어려운 과제를 선호하는지의 여부를 나타낸다(Chun & Song, 2011). 문항 제외 시 신뢰도가 높아지는5개 문항이 제외되었으며, 본 연구에서 사용된 학업적 실패내성척도의 신뢰도는 Table 2에 제시된다.


    3. 자료수집 및 분석방법


    본 연구를 위한 FGI와 설문은 2014년 4-5월에 시행되었으며,FGI는 약 2시간, 설문소요시간은 약 30분이었다. FGI의 전 과정은녹음 후 전사하였으며, 주요 내용에 코드를 부여한 후, 주제어 기반으로 유목화 하는 1차 내용분석을 시행하였으며, 이를 다시 의미 중심으로 재유목화하여 각 유목별 대표반응을 결과에 기술하였다. 또한전 학년 학생들을 대상으로 지필설문을 진행하였으며, 불성실하게응답한 9명의 자료는 최종분석에서 제외하였다. 수집된 자료는 SASver. 8.2 (SAS Institute Inc., Cary, NC, USA)로 분석하였다. 각집단 간 학업소진 정도의 차이를 알아보기 위하여 변량분석을 실시하였으며, 학업소진에 영향을 미치는 학업부담 및 비인지적 특성들의 영향력을 알아보기 위하여 다중회귀분석(multiple regressionanalysis)을 실시하였다.





    결 과


    1. 의대생 및 의전원생의 인구통계학적 특성과 학업소진


    먼저 학업소진에서 성별 차이가 있는지를 알아본 결과, 학업소진전체와 하위요인 모두에서 통계적으로 유의한 차이가 없는 것으로나타났다. 조사 대상자의 연령을 23세 이하, 24-27세 이하, 28세이상의 3개 집단으로 구분하여 이들 집단 간 학업소진 및 하위요인에서의 변량분석을 실시하였으며 그 결과는 Table 3과 같다.학업소진 총점에서 3개 집단 간 유의한 차이가 있었으며, Scheffe검증결과 23세 이하 학생들이 24-27세 이하 집단과 28세 이상 집단보다 통계적으로 유의하게 소진을 더 경험하는 것으로 나타났다.하위요인인 냉소성과 무능력감에서도 연령 집단 간 유의한 차이가있었다. 또한 조사대상자의 연령과 학업소진 및 하위요인의 상관분석을 실시해본 결과, 학생들의 연령이 학업소진 총점(r=-0.17,p<0.01), 냉소성(r=-0.17, p<0.01), 무능력감(r=-0.14, p<0.05)과 통계적으로 유의한 역상관을 보이는 것으로 나타났다.


    의대생과 의전원생의 학년별, 소속별 학업소진 및 그 하위 요인의평균±표준편차는 Table 4와 Figure 1과 같으며, 의대생과 의전원생의 학년별 학업소진 차이를 알아보기 위한 변량분석결과는 Table5와 같다. 변량분석결과 학업소진의 하위요인에서 학년과 집단 간상호작용효과는 없었다. 의대생과 의전원생 간 학업소진 총점에 유의한 차이가 있었으며(F=13.15, p<0.001), 의대생이 의전원생보다통계적으로 유의하게 높은 학업소진을 보이는 것으로 나타났다. 또한 학업소진 총점에 있어서 학년별로도 유의한 차이가 있었으며(F=4.53, p<0.01), Scheffe 검증결과 2학년 학생들이 타 학년에비하여 학업소진 정도가 유의하게 높은 것으로 나타났다. 고갈요인에 있어는 집단 간 차이는 없었으나 학년 간 유의한 차이가 있었으며(F=10.09, p<0.001), Scheffe 검증결과, 2학년 학생들이 3학년과4학년 학생들보다 유의하게 높았다. 냉소성에 있어서도 의대생들이의전원생들보다 통계적으로 유의하게 높은 것으로 나타났으며(F=22.45, p<0.001), 학년에 따른 차이도 유의한 것으로 나타났다(F=3.21, p<0.05). 무능력감에 있어서도 의대생들이 의전원생들보다 유의하게 높은 것으로 나타났다(F=9.17, p<0.01).





    2. 학업부담, 완벽주의 성향, 학업적 실패내성과 학업소진


    환경적 요인으로서의 학업부담과 성격특성요인인 완벽주의 성향과 학업적 실패내성이 의대생과 의전원생의 학업소진에 어떤 영향을미치는지 알아보기 위하여 학업소진 및 각 하위요인들을 종속변인으로 하여 다중회귀분석을 실시하였다. 회귀분석결과는 Table 6과같다. 먼저 학업소진 총점에 유의한 정적 영향을 미치는 요인으로는학업부담(p<0.001), 사회부과 완벽주의(p<0.05)였으며, 유의한 부적 영향을 미치는 요인으로는 자기부과 완벽주의(p<0.001)와 학업적 실패내성 중 감정(p<0.05)과 행동(p<0.01)요인들인 것으로 나타났다. 이들 요인들은 학업소진 총점 변량의 약 30%를 설명하였다.학업소진의 하위요인 중 고갈에는 학업부담이 정적 영향을 미치는 것으로 나타났으며(p<0.001), 학업적 실패내성 중 과제수준 선호(p<0.01)와 감정(p<0.05)요인들이 유의한 부적 영향을 미치는것으로 나타났다. 이들 요인들은 고갈 변량의 약 48%를 설명하였다.학업소진의 하위요인인 냉소성의 경우에도 학업부담이 유의한 정적영향을 미치는 것으로 나타났으며(p<0.001), 자기지향 완벽주의(p<0.01)와 학업적 실패내성 중 감정요인(p<0.05)이 유의한 부적영향을 미치는 것으로 나타났다. 이들 요인들은 냉소성 변량의 약11%를 설명하였다. 마지막으로 학업소진의 하위요인인 무능력감에는 사회부과 완벽주의(p<0.001)가 통계적으로 유의한 정적 영향을미치는 것으로 나타났으며, 자기지향 완벽주의(p<0.001)와 학업적실패내성의 과제난이도 선호(p<0.05)와 행동(p<0.001)요인들이유의한 부적 영향을 미치는 것으로 나타났다. 이들 요인들이 무능력감 변량의 약 18%를 설명하고 있었다.


    3. 학업스트레스 및 학업소진과 관련한 Focus Group Interview결과


    FGI에 참여한 학생 14명 전원이 고등학교 3학년 시기의 학습량과학업스트레스보다 의과대학 및 의전원 진학 후 의학과 1학년과 2학년수학 중 받은 스트레스가 더 크다고 보고하였다. 학업준비 시기에따른 스트레스 정도를 질문해본 결과, 2명의 의전원 2학년 학생이의학과 1학년, 의학과 2학년, 의전원 준비기간, 대입 준비기간의순으로 학업부담 및 스트레스를 받았다고 응답하였으며, 나머지 5명의 의전원 2학년 학생들은 의학과 2학년, 의학과 1학년, 의전원준비기간, 대입 준비기간의 순으로 학업부담과 스트레스를 받았다고응답하였다. 4학년 학생들 중 4명은 의학과 2학년, 의학과 1학년,대입 및 의전원 준비기간의 순으로 학업부담과 스트레스를 받았다고응답하였다. 관련 내용에 대한 학생들의 대표적인 반응은 다음과같다.


    [학생 1의 진술]: 생활이 계속 쳇바퀴를 도는 것 같은 느낌이었어요. 큐 시험(쿼터별 시험) 한 달 전부터는 밥도 제대로 안먹고, 수업 듣고는 바로 자율학습실 가서 공부하고… 새벽 3-4시까지 공부를 하고 하루 2-3시간씩 잠을 자니까 너무 힘들고,수명이 깎이는 느낌이었어요. 그래서 방학되자마자 외국여행을 갔었어요. 그렇게 여행하고 와서는 또 힐링이 되는 것 같더라구요. 갇혀 있는 것이 너무 싫고, 대구가 싫고, 학교가 너무싫기도 했어요. 그런데도 수업에 빠진 적은 없었어요. 수업 들어가서 멍하니 앉아 있을 때도 많았구요. 큐 시험이 끝나면 늘방학 같은 느낌이에요. 그렇게 본 1을 끝내고는 내가 할 수있는 만큼만 하자라는 생각을 하게 되었어요(의전원 2학년,27세 여학생).


    [학생 2의 진술]: 본과 1학년 때는 해부학을 좋아했습니다.책을 펼치면 궁금한 것에 대한 해답이 다 나와 있고, 책에 있는것이 카데바에서도 딱딱 찾아지고 그랬었어요. 그래서 하면되는구나라는 생각을 했었습니다. 본과 2학년 와서는 병명이주어지고, 증상 10개, 진단 10개, 치료방법 10개 등등이 계속나열되는 수업을 들으면서 왜 이것이 이런지, 왜 이것을 해야하는지 모르겠고… 그래서 지금 현재 상황은 공부하는 방법을아직 못 찾은 느낌입니다. 주어진 대로 그냥 억지로 하고 있다는생각도… 전체를 이해해서 내 것으로 만들어야 되는 성격인데다 암기를 잘 못하는 스타일인데 요즘 와서는 수업이 이해가안 되면 듣는 걸 포기하는 경우가 종종 생깁니다. 그렇다고수업에 빠지진 않아요. (중간생략). 고3 때의 스트레스가 4라면,의전원 준비할 때는 3, 본과 1학년 때는 5-6, 본과 2학년 때는7-8 정도 스트레스를 받는 것 같아요(의전원 2학년, 25세 남학생).


    의학과 4학년 학생들에게 학업스트레스의 근본적인 원인과 소진경험에 대하여 질문해 본 결과, 4명의 학생들이 가장 스트레스를받은 시기는 의학과 1학년이었으며, 매우 지쳐 학업소진의 느낌이든 것은 의학과 2학년이었다고 보고하였다. 또한 FGI에 참여한 14명의 학생 모두가 학습동기문제나 학습행위 자체로 스트레스를 경험하는 것이 아니라 동료들과의 비교나 유급, 재시험과 같은 실패경험에대한 불안 등으로 인한 심리적 스트레스와 과도한 공부량에 따른수면부족과 시험 전 극도의 피곤함 등에 의한 신체적 스트레스를받는다고 보고하였다. 또한 4명의 학생은 학업스트레스가 단순히학업을 원인으로 발생하기보다는 가족 및 교우관계나 이성교재에서의 문제 등 인간관계에서의 문제가 복합적으로 작용하여 스트레스를가중시키고 소진하는 느낌을 받았다고 보고하였으며, 14명의 학생중 10명의 학생이 학자금 대출과 학업과 관련한 경제적 부담에 대해서도 언급하였다.


    학업스트레스 및 소진의 경험과 극복에 대해서는 4학년 학생7명이 모두 동일하게 자신과의 협상 및 적절한 적응적 선택을 하였다고 보고하였다. 이러한 협상 및 적응은 빠를 경우 의학과 1학년첫 시험과 두 번째 시험경험 이후에 이루어지거나(9명), 혹은 학업분량이 극도로 많아지는 의학과 2학년 임상통합교과목 수업을 수강하면서 이루어진다(4명)고 보고하였으며, 고성취군에 속하는 2학년학생 1명은 현재 가장 높은 스트레스와 공부방법 및 적응에 대한고민을 하고 있다고 보고하였다. 또한 인터뷰대상자 모두가 의과대학에서의 공부는 시험을 중심으로 이루어진다고 생각하고 있었으며,학업과부화와 암기 위주의 학습을 통해서도 좋은 의사가 될 수 있는지 의문을 가져보았다고 응답하였다. 10명의 학생들이 스트레스가극도에 달했을 때나 소진되었다고 느꼈을 때 견딜 수 있었던 것은주로 의과대학 및 의전원 동료와 의과대학과 의전원 밖 친구들의도움이었다고 응답하였으며, 2명의 학생은 오히려 동료들로 인해스트레스를 더 받았다고 응답하였다. 또한 나머지 2명의 학생은결국 본인이 해결해야 하는 것이며, 스스로 해결책을 찾아야 한다고생각하고 있었다. 이와 관련한 학생들의 대표적인 반응은 다음과같다.


    [학생 3의 진술]: 공부에 대한 스트레스는 유급이나 재시 때문에 받아요. 스트레스를 받으면 휴학하고 싶다는 생각도 들어요.시험 전날 새벽에 스트레스가 팍 오르고 그건 매 큐 시험 때마다나타나는 현상이죠. 시험에 대해 오히려 적응하게 된 건 한학기 정도 지나서였습니다. 선배들이 유급 안하는 정도에 관한노하우를 이야기 해주기도 하는데 불안해서 그렇게 하지는 못했어요. 결국은 스스로 터득하는 것 같아요. 그리고 저 같은경우는 공부도 공부지만 인간관계에서 더 많은 스트레스를 받았습니다. 동아리활동을 하다가 책임을 맡으면서 방학이 없어졌고, 책임지고 후배나 동기들을 불러 연습을 시키고 공연도해야 하는데… 여기서 오는 스트레스가 엄청 컸어요. 못하면선배들로부터 엄청난 질책을 받았고 발표회 1주 전에는 스트레스가 최고조로 올라갔어요. 오히려 학기 중에는 동아리활동이없어서 편안해진다는 느낌을 받았을 정도예요. 개학이 오히려방학의 느낌이었고… 그래서 제대로 공부를 못했습니다. 출석에 대한 강박 때문에 학교는 어쩔 수 없이 갔지만 멍하니 그냥앉아 있을 때가 많았어요. 그렇게 몇 번의 방학을 거치면서모두를 이끌고 갈 수는 없구나… 포기하는 것이 낫다는 걸 알게되면서 오히려 극복되기 시작했어요. 그러는 동안 주변 동료의지지와 비슷한 경험을 겪은 동아리 선배들의 지지로 지금은괜찮아졌습니다. 공부도 공부지만 동아리활동으로 인해 번 아웃을 겪은 것 같아요(의과대학 4학년, 24세 남학생).


    [학생 4의 진술]: 학부 때만 해도 저는 공부를 할 때 1-100까지다 봐야 하는 성격이었습니다. 그런데 의전원에서는 그렇게되질 않았죠. 그것이 스트레스였고 그런 스트레스로 인해 주변사람들에게 소홀해지고 싫어지고 신경도 덜 쓰게 되더라구요.친구들과의 관계도 소원해지고 가족들과는 눈을 아예 마주치지않고 지낸 적도 있어요. 고3도 아닌데 말이에요. 그냥 집에가면 잠만 자고… 어느 순간 극복을 못하면 살 수 없을 것이란생각이 들었어요. 더 웃긴 것은 번 아웃에 대한 경험이 공부못지않게 술 때문에도 왔었는데… 저는 종교 때문에도 그렇고실제로 술을 전혀 못 먹어요. 그런데 동문 선배들은 저에게바라는 게 있으셨어요. 1학년 때 결정적인 사건은 어느 선배결혼식에서 있었는데 처음으로 폭탄주 7잔을 마시고… (중간생략) 계속 먹기를 강요했었죠… 그때는 정말 다른 학교로 가야되는가하는 고민에 빠졌었어요. 공부도 공부였지만 진짜 술이더 힘들었어요. 그러다 2학년 중반 정도에 완전 번 아웃의 느낌이 왔고 그때 다 놨어요. 그런데 그 다음부터는 오히려 좀 좋아졌어요. 봐야할 것만 보고 외워야 할 것만 외우니 스트레스가낮아졌고 선배들에게 술 못 먹겠다고 선언해 버렸거든요(의전원 4학년, 26세 남학생).


    [학생 5의 진술]: 지금은 솔직히 좀 지쳤어요. 친구들은 취업을하고 사회생활을 하고 있는데 나는 아직도 공부를 하고 있으니돈은 돈대로 아무튼… 공부 양도 많고 학년이 올라갈수록 공부스트레스가 아니라 그냥 이런 처지에 대한 스트레스가 커지는것 같아요. 다행히 저는 타협을 잘 하는 스타입이에요. 처음의전원와서 주변 친구들이 100 중에서 80-90의 노력을 매일하니까 나도 따라가려고 나름대로 열심히 노력을 했어요. 그러면서 그들만큼 하지 못하는 내가 정상인지 비정상인지를 고민하고 스트레스를 받으면서 폭음도 하고 담배를 피우기 시작했었어요. 그러다가 3학년부터 실습을 나가면서 오히려 숨통이트이기 시작했어요. 그래서 지금은 스트레스가 그때만큼 그렇게 큰 것 같지는 않아요(의전원 4학년, 28세 남학생).


    [학생 6의 진술]: 본과 1학년에서 2학년 넘어갈 때 정말 스트레스가 심했어요. 유급에 대한 스트레스도 엄청났지만 스스로에대한 열등감으로 힘들었어요. 주변의 친한 친구들이 유독 공부를 잘해서 상대적으로 많이 위축이 되었어요. 그런데 번 아웃느낌은 재수를 하고 나서 겪었던 것도 같아요. 그 여파로 예과2년 동안은 성적에 관심이 없었고 학년만 넘기면 된다는 생각을했었습니다. 재수 후에 정말이지 공부가 너무 하기 싫었었어요.필수 전공과목들도 재미가 없었고… 본과 1학년 들어와서는중요 과목이라는 생각에 공부를 한다고 했는데 그 동안 너무놀아서 그런지 진짜 성적이 나오지 않았어요. 다행이 유급은안했지만 스스로에 대한 불만이 엄청 컸었어요. 그래서인지본과 1학년 때는 주변 사람들에게 신경 쓸 틈이 없었어요. 공부와 성적에 엄청 집착을 했었고 인간관계에 어려움을 느낄 정도였어요. 공부에 집착하면서 여유를 잃게 되었고 그러면서 주변사람들에게서 변했다는 소릴 듣기도 했어요. 그러면서 트러블이 생기고 방황을 했어요. 다행인건 학교의 몇몇 친구들 때문에견뎠어요. 그 친구들이 없었다면 분명 유급했을 것 같아요. 친구들의 지지가 나를 견디게 해주었고… 그런데 아직 완전히 극복한 건 아니예요(의학과 4학년, 24세 여학생).



    고 찰


    본 연구는 대구지역의 1개 의과대학․의학전문대학원에 재학 중인 학생들을 대상으로 인구통계학적 특성, 환경적 특성, 심리 성격적특성들이 학업소진에 어떤 영향을 미치는지에 대한 양적 연구와학업스트레스와 학업소진의 경험과 적응에 대한 포커스 그룹 인터뷰를 포함하는 질적 연구가 함께 진행된 통합연구이다.


    먼저 인구통계학적 특성에 따른 학업소진 정도의 차이를 알아본결과 성별에 따른 차이는 없었으나, 연령, 소속, 학년에 따른 차이는통계적으로 유의한 것으로 나타났다. 의과대학과 의전원생 중 연령이 낮은 학생들이 학업소진을 더 많이 경험하는 것으로 나타났으며이는 연령이 낮은 학생들이 학업과 관련되는 환경과 학업 자체에대하여 더 냉소적이며 학업에 대한 효능감이 더 낮음을 의미한다.반면 학업소진의 하위요인인 정서적 고갈에서 연령집단에 따른 유의한 차이가 나지 않는 이유는 전 연령집단에서 모두 높은 고갈을보고하였기 때문인 것으로 생각된다. 이는 FGI에서도 확인되는 바로의과대학 및 의전원 수학 중 경험하는 심리적 신체적 피로나 정서적고갈은 전 연령에서 모두 높게 보고되었다. 또한 의대생인지 의전원생인지의 여부에 따라 학업소진 정도가 유의하게 다른 것으로 나타났으며, 연령과 마찬가지로 정서적 고갈을 제외한 냉소성과 무능력감 모두 의대생이 의전원생보다 유의하게 높은 것으로 나타났다.이는 연령 증가에 따른 사전경험과 성숙에 의한 적응적 기제의 발달을 그 원인으로 예측해 볼 수 있으나, 국내 초․중․고등학생들 대상으로 한 연구에서 연령 증가에 따른 학업소진의 증가 추세(Lee, 2012)와 일반 대학생들을 대상으로 한 Yoo & Shin (2013)의 연구결과에서와 같이 연령이 높을수록 학업소진 수준이 높은 것으로 알려져있는 기존 연구결과들과 비교하여 어떤 심리적, 환경적 경험의 차이가 이러한 변화를 가져오는지에 대한 심층 연구가 요구된다.


    학년에 따른 학업소진의 차이는 매우 흥미롭다. 6개 학년의 의대생을 대상으로 한 Lee et al. (2012)의 연구에서는 학년별 학업스트레스 정도와 우울이 의학과 1학년에서 가장 높았고, 2학년, 3학년,4학년의 순으로 점차 감소하였으나 의예과 학생들에 비해서는 유의하게 높은 것으로 보고하였다. 특히 본 연구에서는 의학과 2학년생들의 학업소진 총점과 정서적 고갈 정도가 유의하게 높았다. 이는영남대학교 의과대학 및 의전원의 교육과정 편성과 운영에 있어서의학과 2학년의 학습과정과 학습량, 즉 학업부담이 매우 높아지는데기인하는 것으로 볼 수 있다. 영남대학교 의과대학 및 의전원의2013학년도 교육과정을 기초 및 임상의학기준으로 살펴보면 의학과 1학년에서 5개 기초의학 교과와 2개의 임상의학 교과가 운영되는반면, 의학과 2학년에서는 총 14개 임상의학 교과가 운영되어 실제교과별 학습량과 시험부담이 급격하게 가중되고 있으며, 이러한 학업부담 증가는 FGI에서도 일관되게 보고되었다. 이에 대하여 한성적우수 학생은 다음과 같이 보고하였다.


    1학년 때는 실습과 강의가 함께 있고, 그 분량도 이해해서 시험칠 수 있을 정도로 준비가 가능했어요. 그런데 2학년 때는 달랐어요. 2시간 수업을 마치면 그것을 학습하는데 4시간 정도 걸린다고 저는 예측했어요. 한 쿼터당 수업시간이 192시간이니까384시간이 필요한 거죠. 이건 매일 공부한다고 해서 다 볼 수있는 양이 아닌 거예요. 그때부터 일부 버리는 과목이나 내용이생기게 되고… 선택과 집중이랄까? 버릴 것은 버리고 볼 것만보자… 그렇게 되었죠(의전원 4학년, 26세 남학생).


    특히 FGI에 참여한 학생들의 보고에 따르면 의과대학 및 의전원에서의 학업부담에 의한 스트레스에 대처하기 위해서 스스로 학습전략이나 학업에 대한 태도를 전환 혹은 조절하며, 의학과 1학년 중시행되는 1-2회의 시험을 통해 학습의 강도나 분량 정도를 가늠하게된다고 하였다. 보다 적극적으로는 쿼터별로 시행되는 시험의 학습분량을 고려한 학업계획서가 일부 학생들에 의해 공유되고 있었다.즉 의과대학 및 의전원에서의 수학과정에 적응하기 위한 노력으로학습전략과 학업스트레스의 조정노력이 이루어진다는 사실이 확인되었으며, 추후 종단적 연구를 통해 이러한 학업에 대한 전략적자기협상과 메타인지의 전환시기 및 양상, 그리고 그 결과를 심도있게 연구해 볼 필요가 있다.


    학업부담은 학업소진의 주된 원인요인으로 본 연구에서도 그 영향력이 학업소진 총점과 그 하위요인 모두에서 확인되었다. Kimet al. (2010)의 연구에서는 이러한 학업부담과 학업소진의 관계에있어 통제감이 억제요인으로 작용한다는 점을 발견한바 있으며, 본연구의 FGI에서도 의학과 1학년 및 2학년의 학업부담 정도가 통제가능성을 넘었다고 지각될 때 불안과 스트레스를 경험하게 되며,의대생과 의전원생들이 경험하는 이러한 학업부담은 대학입시나의과대학 입학을 위한 시험준비기간에 경험하는 부담보다 훨씬 컸다고 보고하였다. 따라서 이후 연구에서는 학생들이 지각하는 학업부담의 정도와 그 역치를 학습전략의 전환과 연계하여 양적, 질적연구를 통해 탐색해볼 필요가 있으며, 의대 및 의전원에서는 이러한학업부담이 특정 학년에 과도하게 주어지는지의 여부와 자기관리및 스트레스 대처와 관련하여 어떤 형태로 교육 혹은 지원을 해야하는지에 대해 검토할 필요가 있다.


    학업소진의 개인차를 고려하여 성격특성으로서의 완벽주의 성향과 학업적 실패내성의 영향도 살펴보았다. 완벽주의는 영재의 대표적인 특성으로 높은 기준 설정과 자아실현과 관련되기도 하며(Chun& Song, 2012), 반면 자기 자신과 타인의 기대에 미치지 못할 경우실패에 대한 두려움으로 행위 착수나 실행을 지연하게 되거나 과도한 몰입으로 인한 균형감을 상실하도록 만들 수도 있다(Adderholt-Elliot, 1989). 의과대학생을 대상으로 한 Chun & Song(2012)의 연구에 따르면, 자기지향 완벽주의와 사회부과 완벽주의가 학업적 자기효능감에 유의한 정적 영향을 미치며, 특히 자기지향완벽주의는 학업성취도에도 유의한 정적 영향을 미치는 것으로 나타났다. 본 연구에서는 자기지향 완벽주의와 사회부과 완벽주의가 학업소진 총점에 영향을 미쳤으며, 이들의 영향력은 각기 다른 것으로나타났다. 특히 자기지향 완벽주의 성향은 냉소성과 무능력감에도부적 영향을 미치며, 이는 스스로 높은 기준을 부여하고 완벽해지기위한 성향을 가진 학생들이 학업에 대한 냉소성과 무능력감을 덜느끼고 학업소진 점수도 낮음을 의미한다. 반면, 타인이 자신에게기대하는 완벽을 만족시켜야 한다고 믿는 학생들은 학업소진을 높게경험하고, 특히 무능력감 또한 높은 것으로 나타났다. 이러한 결과는사회부과 완벽주의의 부적응적 특성에 대한 기존의 연구결과(Ha& Jang, 2011; Kim & Son, 2006)를 지지한다.


    학업적 실패내성은 학업상황에서의 실패경험에 대한 개인의 감정적 반응과 행위적 대처, 그리고 실패 가능성에도 불구하고 어려운과제를 선호하는지에 대한 것으로, Lee (2006)의 연구에서는 학업우수 학생들이 그렇지 않은 학생들보다 학업적 실패내성이 높은 것으로 나타났으며, 의과대학생을 대상으로 한 Chun & Song (2011)의연구에서는 과제수준 선호가 높고, 행동수준이 높은 학생 집단의학생들이 숙달접근적 학습동기를 가지는 것으로 나타난 바 있다.본 연구에서는 학업적 실패내성 중 과제수준 선호요인이 학습소진의하위요인인 정서적 고갈, 냉소성 및 무능력감 모두에 부적 영향을미치는 것으로 나타났다. 또한 감정요인은 정서적 고갈과 냉소성에부적 영향을 미치는 것으로 나타났으며 이는 실패에 대한 부정적감정이 높아 건설적으로 대처하는 능력이 낮은 학생들이 학습소진을더 경험하는 것을 의미한다. 또한 본 연구에서는 행동요인이 학업소진의 하위요인인 무능력감에 부적 영향을 미치는 것으로 나타났으며, Kim (2002)는 행동요인이 실패경험 후 실패를 만회하기 위한계획을 수립하고 해결방안을 강구하는 등 앞으로의 실패에 대한대비와 적극적이고 구체적인 극복행동을 보이는 것을 의미한다고하였다. 따라서 학업적 실패에 대한 인지적, 정서적, 행동적 측면의내성이 있는 학생들이 학업소진에 대하여 적응적일 수 있음을 알수 있었다.


    무엇보다도 본 연구에 참여한 의대생과 의전원생의 학업소진 하위요인 중 정서적 고갈의 평균(4.22±1.29)이 Lee & Lee (2013)의연구에서 보고된 국내 의대생 집단의 평균(3.4±1.5)과 Galan etal. (2011)의 연구에서 보고된 스페인 의대생의 평균(2.1±1.0)과영국 대학생의 평균(2.5±1.5)보다 높았으며, 냉소성(3.09±1.21)또한 국내 의대생 집단(3.0±1.5), 스페인 의대생(1.0±1.0), 그리고영국 대학생(2.1±1.6)보다 높았다. 전반적으로 본 연구에 참여한조사대상자들의 학업소진 총점이 이전 연구에 비하여 높으며, 따라서 이러한 차이가 국내 대학 간 그리고 국외 대학과의 비교를 통해어떤 원인에 의한 것인지를 탐색해볼 필요가 있다.


    본 연구의 한계점은 다음과 같다. 의대와 의전원 체제를 모두가지고 있는 1개 대학의 학생들에 대한 연구였으며, 이 의대․의전원의 교육환경 및 교육과정 특성에 의해 학업소진의 양상이 다를 수있음을 고려할 필요가 있다. 또한 탐색적 의미의 FGI를 실시한 바,이후 연구에서는 심층 인터뷰 및 장기간의 관찰 연구 등을 통해의대생과 의전원생의 학습소진 및 학습전략의 변화 양상 등을 보다구체적으로 분석해볼 필요가 있다.


    그럼에도 불구하고 본 연구의 시사점은 다음과 같다. 먼저 학업스트레스의 만성적 결과로서 학습소진이 발생한다는 점과 본 연구결과검증된 연령 및 학년에 따른 학습소진의 차이를 고려하여볼 때,의대와 의학전문대학원 내의 교육과정과 교육풍토에 대한 검토 및교육기관별 학업스트레스와 학습소진에 대한 인식과 대처방안들이모색되어야 할 것이다. 이는 학생상담과 같은 개별수준의 서비스지원뿐만 아니라 학생의 학업적 적응을 고려한 교육과정 편성 및교육평가의 계획과 실행을 대학 및 대학원 수준에서 고려해야 함을의미하며, 학생들의 학습 적응을 높여주기 위한 학습 오리엔테이션과 학습캠프, 학습 멘토링 시스템과 스트레스관리를 위한 다양한프로그램을 지원할 필요가 있다. 또한 학생들과의 인터뷰에서 확인된 바와 같이 학업소진을 경험하고 있는 학생들의 정서적 고갈과냉소성이 교수진의 노력과 상담, 그리고 다양한 프로그램의 운영을의례적인 행위로 인식할 수 있음을 이해할 필요가 있다. 이에 우수인재들의 건강하고 행복한 대학생활을 위해 대학과 개별 교수들이제공할 수 있는 지원과 이에 따른 학생들의 변화에 대한 후속연구가필요함을 제안하는 바이다.






    The purpose of this study was to investigate the influence of demographic data, academic demand, perfectionism,

    and academic failure tolerance on academic burnout of medical and graduate students at Yeungnam University.

    A mixed method study was conducted. Fourteen students participated in a focus group interview and 302

    students, including 151 medical students and 151 graduate students, completed a survey, which addressed

    the factors of academic burnout, academic demand, perfectionism, and academic failure tolerance. There

    were significant differences in the academic burnout by age and year in school. The 2nd year medical

    & graduate students experienced significantly higher academic burnout than the other students. Multiple

    regression analysis revealed that academic burnout of medical and graduate students was significantly affected

    by academic demand (p<0.001), self-oriented and socially-prescribed perfectionism (p<0.001, p<0.01),

    and feeling and behavior, which were sub-factors of academic failure tolerance (p<0.05, p<0.01). It is

    suggested that comparative studies with other student groups, a longitudinal study of medical and graduate

    students, and a prospective study of coping styles and methods of preventing academic burnout need

    to be conducted.

    Keywords: Academic burnout, Academic demand, Perfectionism, Academic failure tolerance

    교수학습의 의미와 일곱 가지 질문 전략

    Seven Strategies for Effective Questioning

    양은배

    Eunbae B. Yang

    연세대학교 의과대학 의학교육학과

    Department of Medical Education, Yonsei University College of Medicine, Seoul, Korea





    교수학습의 의미와 질문


    가르친다는 것은 무엇을 의미하는 것일까? 많은 의과대학 교수들이 곧잘 이런 질문을 던진다. 우리는 흔히 가르치는 일과 배우는 일을 분리해서 생각한다. 그래서 많은 사람들이 어떻게 가르칠 것인가에 대해 고민하고, 학생들이 어떻게 배우는가에 대해서는 별다른 고민을 하지 않는다. 일반적으로 교수(teaching)는 교육적 의도를 가지고 수행하는 일체의 활동으로 가르치는 행위로 정의된다. 그러므로 가르친다는 것은 목표로 하는 행동을 나타내도록 환경을 계획적으로 조직하고 전달하는 과정이라고 할 수 있다. 반면, 학습(learning)은 개인과 환경과의 상호작용에 의해서 나타나는 인간행동의 지속적인 변화를 의미한다. 이러한 학습은 교수(teaching)가 없는 상태에서도 일어날 수 있는 반면, 가르친다고 해도 학습이 일어나지 않을 수 있다. 


    그러나 가르치는 것은 결코 그 자체가 목적이 될 수 없다. Dressel & Marcus [1]는 ‘얼마나 잘 가르치느냐 하는 것의 정의는 학습자가 무엇을 어느 만큼이나 어떻게 배우느냐의 맥락에서만 정의될 수 있다’고 보았다. 교수(teaching)와 학습(learning)은 수단과 목적으로서 상보적이고 복합적인 상호작용 관계에 놓여 있는 것이므로 교수와 학습은 분리해서 생각할 수 없는 동시발생적인 교호작용이라고 할 수 있다. 이러한 맥락에서 학생들에게 장기간 지속될 수 있는 의미있는 학습을 어떻게 제공할 수 있을 것인가에 대한 질문은 의학교육에서 중요한 관심사가 되어 왔다. 의과대학 학생들이 의학적 지식을 완벽하게 암기하고, 그것을 학습한 모습 그대로 재생해 낸다고 할지라도 그러한 지식을 환자에게 적용하는 능력에는 항상 비판적 시각이 존재한다[2]. 학생들은 개별적인 지식을 완벽하게 암기하고 재생하지만, 각각의 지식이 어떠한 인과관계로 하나의 개념구조로 통합될 수 있는지 이해하지 못하고 있다[3]. 따라서 어떤 교수방법을 선택하여 사용할 때 유의미한 학습을 극대화 할 수 있을 것인가의 문제는 중요한 탐구 영역이었다. Lee [4]는 모든 상황에서 똑같은 효과를 거둘 수 있는 교수학습 방법은 없다고 전제하면서, 개개인의 교수자가 최적의 교수학습 방법을 스스로 창출해 내는 것이 무엇보다 중요하다는 점을 지적하였다. 아울러, 유의미 학습의 촉진을 위해서는 교수자와 학습자 간에 상호작용을 어떻게 촉진할 것인가가 중요한 영역이라는 점을 지적하였는데, 상호작용 교수법으로 질문교수법의 유용성을 제안하였다. 유의미 학습을 위한 질문교수법의 유용성과 관련하여 Sachdeva [5]는 학생들과 질문을 교환하는 것이 학생들의 학습을 강화한다는 점을 지적하였으며, Michael [2]은 혼자 공부하는 것보다는 동료 상호 간에 그룹으로 공부하는 것이 더 큰 학습 효과를 보이며, 혼자말로 되뇌는 것이나 학습자 스스로 질문하고 답변하는 것이 도움이 된다고 지적한바 있다. 질문은 학습자들이 수업에 집중할 수 있도록 하는 효과적인 방법이며, 학습자의 지적인 자극을 불러일으키는 중요한 교육 방법이다. 교수자들이 어떤 유형의 질문을 하는가에 따라 학습자들의 학습 효과는 달라진다. 고등질문기법을 사용하는 교수와 그렇지 않은 교수자 사이에는 어느 정도 학습 효과에서 뚜렷한 차이를 보인다. Redfield & Rousseau [6]은 고등질문기법을 사용한 경우 학생들의 평균 학습 효과는 27% 상승한다고 지적한바 있다.


    질문의 유형은 B.S. Bloom의 교육목표 분류학에 따라 지식, 이해, 적용, 분석, 종합 및 평가 등으로 구분할 수 있다. 또는 수렴적 사고를 요구하는 질문과 확산적 사고를 요구하는 질문으로도 구분할 수 있다. 예를 들어 X-선이 언제 발견되었는지를 질문하는 것과 A치료법과 B치료법을 비교해 보라고 하는 질문은 요구하는 사고능력이 다르다. 오늘날 많은 의과대학 교수들이 주로 어떤 유형의 질문을 사용하고 있는가 생각해 보라. 강의실을 들여다보면 질문은 거의 일어나지 않으며, 질문이 있는 경우에도 낮은 수준의 사고를 요구하는 질문들이 주류를 이루고 있는 것이 현실이다. 다음은 의과대학 강의실 및 임상실습 과정에서 학생들의 사고를 촉진하기 위해 교수자들이 사용할 수 있는 일곱 가지 질문 전략을 소개한다.



    일곱 가지 질문 전략


    1. 수업 전에 학생들에게 제시할 질문을 만들어라

    의과대학 강의를 관찰해 보면, 많은 교수들이 수업이 끝날 즈음에 ‘질문 없어요?’하고는 금세 강의를 끝내는 모습을 본다. 아마도 수업 시간 중에는 강의할 내용이 많기 때문에 질문할 시간을 배정하기 어렵기 때문일 것이다. 회진 시에도 비슷한 상황은 연출된다. 실제 환자를 대상으로 이루지는 현실성, 전공의 및 인턴과 함께 이루어지는 병동의 상황에서 학생들에게 질문하는 것은 거의 불가능하다. 그러나 문제는 학생에게 질문을 던져서 사고를 촉진시키려는 의도적 계획이 없었다는 점이다. 질문은 학생들에게 문제의식을 불러일으키게 하거나, 사고의 수준을 높이는 효과적인 수단이다. 즉흥적으로 학생들에게 던져지는 낮은 수준의 질문들은 효과가 없다. 따라서 교수자들이 강의 또는 회진하기 전에 학생들에게 제시할 질문을 준비하는 것은 무엇보다 중요하다. 어떤 목적으로, 어떤 질문을 할 것인지 생각해 보아야 한다.


    2. 질문의 효과를 극대화할 수 있는 시점과 질문의 유형을 선택하라

    수업이 끝날 즈음에 학생들에게 던지는 질문은 어떤 효과가 있을까? 한마디로 거의 효과가 없다. 벌써 학생들의 인지구조는 정리단계에 들어갔기 때문에 정말 중요하고 의미있는 질문이 아닌 다음에는 관심을 유도하기 어렵다. 오히려 학생들 사이에는 질문하는 학생에 대한 집단심리가 작용하는 경우가 흔하다. 학생들은 질문을 통해 수업이 끝나는 시간이 늦어지는 것을 원하지 않는다. 더욱이 수업이 끝날 때 나온 질문이 시시하거나 중요하게 받아들여지지 않으면 더욱 그렇다. 강의의 흐름에 맞지 않은 질문은 수업을 방해하는 요인이 된다. 수업의 흐름 선상에 있지 않은 갑작스러운 질문은 학생들을 혼란스럽게 하거나 학습의 흐름을 단절시킨다. 그러므로 수업의 단계에 따라 적절한 질문의 유형을 선택하는 것이 중요하다. 예를 들어 수업이 시작되는 도입 단계에서는 선행학습 수준 파악과 학습동기를 유발하는 질문이면 충분하다. 특히, 임상에서의 환자사례라던가, 사회적 이슈를 중심으로 학생들의 인지를 흔들어 놓는 것이 필요하다. 수업이 진행되는 과정에서는 수업의 흐름을 촉진하거나 학생들이 잘 학습하고 있는지를 확인하는 질문들이 필요하다. 또는 수업에서 다루어지는 중요한 내용을 질문의 형태로 강조할 수도 있다. 수업의 마무리 단계에서는 수업 내용을 질문의 형태로 요약하는 것도 좋은 방법이다.


    3. 질문 후 학생들이 생각하고 대답할 수 있을 만큼 충분히 기다려라

    교수자들이 학생들에게 질문을 하는 경우 학생들이 대답할 때까지 얼마나 기다릴까? 질문을 하는 교수자들을 관찰해 보면 평균 1초 이상을 기다리지 못하는 경우들이 많다. 개념이나 사실적 지식에 대한 질문은 대답을 듣기까지 많은 시간이 필요하지 않을 것이다. 그러나 조금이라도 내용에 대한 이해, 내용에 대한 분석과 비교를 요구하는 질문이라고 한다면 즉각적인 대답을 얻기 어렵다. 질문 후 기다리는 시간은 질문 그 자체만큼 중요하다. Rowe [7]는 교수자들의 질문 후 기다리는 시간을 ‘대기시간 I’, ‘대기시간 II’로 구분하여 제시하였다. 대기시간 I교수자가 첫 번째 질문을 하고 나서 학생이 반응할 때까지 기다려주는 시간이며, 대기시간 II는 학생의 첫 번째 응답 후 생기는 시간으로 교수자 또는 다른 학생이 이전 학생의 대답을 긍정 또는 부정하거나 교수자의 후속 활동이 일어나기 전까지의 시간을 의미한다. 두 가지 모두 의미있는 대기시간이 필요한데, 일반적으로 3초에서 최대 15초 동안 학생의 답변을 기다리는 것이 적절하다. Cooper et al. [8]은 교수자들이 질문 후 기다리는 시간이 늘어날수록 학생들은 보다 길고 자세한 대답을 하는 것으로 보고하였다.


    4. 질문에 대답하지 않은 학생들에게 힌트를 제공하지 마라

    교수자들이 학생들에게 질문할 때 학생들로부터 대답을 듣지 못하는 경우가 많다. 학생들은 묵묵히 고개를 숙이고 있거나 시간이 지나가기를 기다린다. 때로는 짧은 대답으로 상황이 끝나기를 바란다. 교수자들이 이러한 상황에 직면하면 스스로 질문에 대답하거나 다른 학생에게 질문을 돌려야 하는 일이 발생한다. 질문에 대답하지 않는 학생들을 어떻게 다루어야 할까? 상황에 따라 다르겠지만 학생들에게 어떠한 힌트도 제공하지 않는 것이 답이다. 만약 교수자가 질문에 대한 직접적인 답을 제공하거나 근접한 힌트를 제공하는 것은 학생들의 사고 확장 기회를 차단한다. 그리고 학생들은 금방 대답을 하지 않고 기다린다면 더 많은 힌트가 제공되거나 교수자가 정답을 이야기할 것이라는 상황을 학습하게 된다. 이러한 상황이 학습되는 순간 어떠한 문제 인식이나 사고의 확장을 가져오는 것은 불가능하게 된다. 중요한 것은 학생들이 분명히 질문에 대답해야 한다는 것이고, 질문에 대답할 때까지 충분히 기다려야 한다는 점이다. 끝까지 질문에 답하지 못하는 학습자에게는 좀 더 충분한 시간을 줄 수도 있다. 동일한 질문을 다른 학생에게 다시 제시하고, 다른 학생의 대답을 들은 후 다시 처음 질문을 제시했던 학생에 자신의 답을 말하도록 하는 것도 방법이다. 학생들은 금방 이러한 학습상황에 익숙해지고 교수자가 던지는 질문에 대해 생각하기 시작할 것이고, 어떤 형태로든지 대답을 준비할 것이다.


    5. 추가 질문을 하라

    한 번의 질문을 통해서 학생들로부터 얻고자 하는 대답을 끌어내거나 깊이있는 사고를 하도록 만들기는 어렵다. 이럴 때 사용하는 것이 추가 질문이이다. 일반적으로 탐침질문(probing question)이라고 한다. 이러한 질문은 개방형 질문을 한 다음 특정 초점에 맞추어 더 심도있는 정보를 얻어내기 위하여 사고의 폭을 확장하는 좋은 기법이다. 탐침질문은 목적에 따라 대답의 전환(redirection), 촉진(prompting), 정당화(justification), 명확화(clarification), 확장(extension) 질문으로 구분된다. 


        • 전환은 학생들의 대답이 특정 방향에 치우쳐져 있는 경우 다양한 의견을 도출하기 위해 사용하며, 
        • 촉진은 정확한 대답을 유도하기 위해 제시되는 질문이다. 
        • 정당화는 학생들이 제시한 대답에 대한 근거를 요구할 때 사용되고, 
        • 명확화는 학생들의 대답이 모호한 경우 분명한 대답을 유도하기 위해 사용한다. 
        • 확장은 현재의 대답을 구체화하거나 심화하기 위해 제시되는 질문이다. 


    실제 교육현장에서 개방형 질문, 개방형 질문에 기초한 탐침질문을 여러 번 사용하여 학생들의 대답 수준을 한 차원 높여주는 것이 중요하다. 대부분의 경우 일회성의 질문은 개념과 사실적 지식의 확인 수준에 머무르지만, 탐침질문은 학생들을 사고를 깊이 있게 만든다. 건설적이고 긍정적인 분위기에서 이루어지는 탐침질문은 교수자와 학습자의 관계를 신뢰롭게 만들기도 한다.


    6. 교수자가 질문하기보다 학습자들이 질문하게 하라

    수업에서 교수자만이 질문을 할 수 있는 유일한 사람이 되어서는 안 된다. 수업의 또 다른 주체자로서 학습자 또한 적극적으로 질문을 하도록 자극해야 한다. 학습자의 질문은 학습자 스스로의 동기 유발과 참여 촉진은 물론 후속 질문과 토의 및 탐구를 위한 디딤돌이 될 수 있다. 특히 구성주의 수업 환경에서 학습자의 질문은 좋은 수업 내용이 될 수 있기 때문에, 학습자가 질문을 하도록 적극 독려해 주어야 한다. Ann et al. [9]은 의과대학 학생들을 대상으로 질문의 주체에 따른 학습효과를 연구하였다. 이 연구는 교수자가 질문을 한 경우보다 학생들이 질문을 한 집단의 학업성취도가 의미 있게 높았으며, 6개월 후에 이루어진 반복측정에서도 기억하는 정보량이 많았다고 보고하였다. 학생들에게 질문을 만들 수 있는 기회를 제공하고, 학생들의 질문에 교수자들이 대답을 하는 것은 학생들의 유의미학습을 촉진하는 좋은 전략이 된다.


    7. 자신의 질문 패턴을 점검하기 위해 체크리스트를 활용하라

    교수자가 수업을 하기 전 질문에 대한 계획을 세운다고 하더라도 복합적이고 다면적인 수업 현장의 특성상 교수자가 의도한대로 질문 행동을 하는 것은 쉬운 일이 아니다. 대부분의 교수자들은 자신만의 질문 패턴을 갖고 있다. 오랜 시간 형성된 이러한 질문패턴은 쉽게 바뀌지 않는다. 사전에 수립한 질문 계획이 한두 번 실패로 끝나는 경우 대부분 자신의 처음 질문 패턴으로 돌아간다. 수업에서 교수자 스스로 질문 패턴을 파악하여 보완하기 위한 노력은 교육적 실천의 중요한 부분이다. 이러한 실천을 위해 일곱 가지 질문 전략을 체크리스트로 만들어서 활용하는 것이 도움이 된다. 필요하다면 여기에 몇 가지 항목을 추가해도 좋다. 질문의 간결성과 명료성, 긍정적 질문, 학생의 대답에 대한 경청, 건설적 피드백, 긍정적인 질문 분위기 형성 등과 같은 항목을 체크리스트에 추가해서 사용할 수 있다.














    Korean J Med Educ > Volume 24(3); 2012 > Article


    학생 발표 수업 활용하기

    아주대학교 의과대학 인문사회의학교실

    채수진






    최근 많은 대학들은 사회가 요구하는 인재상을 반영하여 대학의 교육목표를 이루기 위해서 강의식 이외의 다양한 학습방법을 도입하고 있다. 문제바탕학습, 팀바탕학습 등 발표와 토론중심의 교수-학습 방법이 대표적이라고 할 수 있다. 의과대학도 예외는 아니다. 의과대학 평가인증 기준에 의하면[1] 의료인문학 과정 학습성과에 도달하기 위해서 전체 과정의 1/3 이상은 수업 시간의 50% 이상을 단순식 강의 방법 이외 토론, 발표, 질의응답 등의 수업을 진행해야 한다.



    의과대학 강의실에서 학생들이 발표 수업을 진행하는 모습은 더 이상 새로운 풍경이 아니다. 교수들은 수업에서 학생들의 발표를 활용하고 있으며 그 결과를 성취도 평가에 반영하기도 한다. 교수들은 이러한 학생 발표를 통해서 학생들이 주도적으로 내용을 탐구하고 그 과정에서 학생들 간의 상호작용이 활성화되며 나아가 학습의 과정을 성찰하기를 바란다. 그러나 이러한 교수의 기대가 늘 충족되는 것은 아니며 때로는 의도와는 전혀 다른 난처한 상황이 발생하기도 한다.



    가장 먼저 학생들에게 발표를 시키는 목적이 무엇인지를 고민해야 한다. 

      • 학생들이 탐구를 하고 그 결과를 말 그대로 나와서 말하면 되는 것인지 아니면
        특정 주제를 담당한 학생이 실제로 교수 대신 수업을 하기를 원하는 것인지에 따라 준비 과정 및 평가 방법이 달라져야 하기 때문이다. 
      • 결과를 공유하기 위한 발표는 과제 자체에 대한 점수를 부여할 때 발표를 한 가지 요소로서 반영할 수 있으며,
        수업진행으로서의 발표는 발표 자체만으로 평가가 가능하다.



    발표 수업은 학생들이 주도적으로 발표를 준비하고 진행하므로 수동적으로 앉아서 교수의 강의를 듣는 것보다 더 효과적인 학습이 될 수 있다. 교수는 이러한 학생 발표 수업을 충분히 활용하여 교육의 목표를 달성하기 위해서는 발표 수업을 진행할 때 다음과 같은 사항을 고려할 필요가 있다[2].



    1) 발표의 의미를 학생들에게 안내한다.

    이 발표를 통해 여러 학생들이 무엇을 얻어가야 하는지, 무엇을 생각해 보아야 하는지를 수업 시작 시 환기시켜준다.



    2) 발표가 진행되는 동안 교수자도 여타 학생들과 같은 자세로 임한다.

    교수가 이미 알고 있는 내용에 대해서 학생이 수업을 진행하는 상황이 아니다. 학생들이 어떤 과정을 거쳐 어떤 산출물을 만들어왔는지 호기심을 보여주는 것이 바람직하다.



    3) 발표를 마친 후 팀 간 동료평가를 진행한다.

    이때 사용되는 평가 루브릭은 과제를 부여하는 시점에 이미 공유된 것이어야 한다. 초기에는 문제의 창의적 해결이 매우 중요하다고 강조했는데 막상 동료 평가지를 보니 창의성이 아닌 다른 평가 항목이 가득하다면 곤란하다.



    4) 각 발표가 끝날 때마다 토론 시간을 가진다.

    이를 통해 발표를 듣는 학생들이 수동적으로 임하는 것을 방지할 수 있을 뿐 아니라 결과물에 대한 좋은 피드백을 주고 받을 수 있다. 과제 수행 결과를 공유하기 위해 발표를 하는 경우 대부분의 학생들은 다른 팀의 발표에 귀를 기울인다. 자신들도 같은 과정을 거쳐 나름대로의 결과를 도출하였으니 남들은 어떻게 했는지 궁금하지 않을 수 없다. 따라서 이 경우는 토론이 활발하게 일어날 수 있도록 시간을 줄 필요가 있다.



    5) 발표를 마친 후 발표자에게 피드백을 준다.

    발표 수업에서 가장 중요한 것은 교수자의 피드백이다. 열심히 준비해서 나름대로 긴장하여 발표를 했는데 교수가 이에 대해 아무런 코멘트도 하지 않고 바로 다음 발표 순서로 넘어가버리게 되면 발표자도 발표를 듣는 학생들도 유의미한 경험으로 기억하지 못할 것이다.







    Korean J Med Educ > Volume 25(1); 2013 > Article

    © The Korean Society of Medical Education. All rights reserved.

    + Recent posts